Sei sulla pagina 1di 365

o

n

C
h
i
Mc lc
Li ni u iii
Cc k hiu v khi nim vii
Bi tp
1 S thc 3
1.1 Cn trn ng v cn d-i ng ca tp cc s thc. Lin
phn s . . . . . . . . . . . . . . . . . . . . . . . . . . . . . 6
1.2 Mt s bt ng thc s cp . . . . . . . . . . . . . . . . . . 11
2 Dy s thc 19
2.1 Dy n iu . . . . . . . . . . . . . . . . . . . . . . . . . . 23
2.2 Gii hn. Tnh cht ca dy hi t . . . . . . . . . . . . . . 30
2.3 nh l Toeplitz, nh l Stolz v ng dng . . . . . . . . . 37
2.4 im gii hn. Gii hn trn v gii hn d-i . . . . . . . . 42
2.5 Cc bi ton hn hp . . . . . . . . . . . . . . . . . . . . . . 48
3 Chui s thc 63
3.1 Tng ca chui . . . . . . . . . . . . . . . . . . . . . . . . . 67
3.2 Chui d-ng . . . . . . . . . . . . . . . . . . . . . . . . . . 75
3.3 Du hiu tch phn . . . . . . . . . . . . . . . . . . . . . . . 90
3.4 Hi t tuyt i. nh l Leibniz . . . . . . . . . . . . . . . 93
3.5 Tiu chun Dirichlet v tiu chun Abel . . . . . . . . . . . . 99
i

n

C
h
i
ii Mc lc
3.6 Tch Cauchy ca cc chui v hn . . . . . . . . . . . . . . 102
3.7 Sp xp li chui. Chui kp . . . . . . . . . . . . . . . . . . 104
3.8 Tch v hn . . . . . . . . . . . . . . . . . . . . . . . . . . . 111
Li gii
1 S thc 121
1.1 Cn trn ng v cn d-i ng ca tp cc s thc. Lin
phn s . . . . . . . . . . . . . . . . . . . . . . . . . . . . . 121
1.2 Mt s bt ng thc s cp . . . . . . . . . . . . . . . . . . 131
2 Dy s thc 145
2.1 Dy n iu . . . . . . . . . . . . . . . . . . . . . . . . . . 145
2.2 Gii hn. Tnh cht ca dy hi t . . . . . . . . . . . . . . 156
2.3 nh l Toeplitz, nh l Stolz v ng dng . . . . . . . . . . 173
2.4 im gii hn. Gii hn trn v gii hn d-i . . . . . . . . 181
2.5 Cc bi ton hn hp . . . . . . . . . . . . . . . . . . . . . . 199
3 Chui s thc 231
3.1 Tng ca chui . . . . . . . . . . . . . . . . . . . . . . . . . 231
3.2 Chui d-ng . . . . . . . . . . . . . . . . . . . . . . . . . . 253
3.3 Du hiu tch phn . . . . . . . . . . . . . . . . . . . . . . . 285
3.4 Hi t tuyt i. nh l Leibniz . . . . . . . . . . . . . . . 291
3.5 Tiu chun Dirichlet v tiu chun Abel . . . . . . . . . . . . 304
3.6 Tch Cauchy ca cc chui v hn . . . . . . . . . . . . . . 313
3.7 Sp xp li chui. Chui kp . . . . . . . . . . . . . . . . . . 321
3.8 Tch v hn . . . . . . . . . . . . . . . . . . . . . . . . . . . 338
Ti liu tham kho 354

n

C
h
i
Li ni u
Bn ang c trong tay tp I ca mt trong nhng sch bi tp gii tch
(theo chng ti) hay nht th gii .
Tr-c y, hu ht nhng ng-i lm ton ca Vit Nam th-ng s dng
hai cun sch ni ting sau (bng ting Nga v -c dch ra ting Vit):
1. "Bi tp gii tch ton hc"ca Demidovich (B. P. Demidovich;
1969, Sbornik Zadach i Uprazhnenii po Matematicheskomu Analizu,
Izdatelp1stvo "Nauka", Moskva)
v
2. "Gii tch ton hc, cc v d vbi tp"ca Ljaszko, Bojachuk,
Gai, Golovach (I. I. Lyashko, A. K. Boyachuk, YA. G. Gai, G. P.
Golobach; 1975, Matematicheski Analiz v Primerakh i Zadachakh,
Tom 1, 2, Izdatelp1stvo Vishaya Shkola).
ging dy hoc hc gii tch.
Cn ch rng, cun th nht ch c bi tp v p s. Cun th hai
cho li gii chi tit i vi phn ln bi tp ca cun th nht v mt s
bi ton khc.
Ln ny chng ti chn cun sch (bng ting Ba Lan v -c dch
ra ting Anh):
3. "Bi tp gii tch. Tp I: S thc, Dy s vChui s"(W. J.
Kaczkor, M. T. Nowak, Zadania z Analizy Matematycznej, Cze s c Pier-
wsza, Liczby Rzeczywiste, Ciagi i Szeregi Liczbowe, Wydawnictwo
Universytetu Marii Curie - Sklodowskiej, Lublin, 1996),
4. "Bi tp gii tch. Tp II: Lin tc vVi phn "(W. J. Kaczkor, M.
T. Nowak, Zadania z Analizy Matematycznej, Cze s c Druga, Funkcje
iii

n

C
h
i
iv Li ni u
Jednej Zmiennej{Rachunek R ozniczowy, Wydawnictwo Universytetu
Marii Curie - Sklodowskiej, Lublin, 1998).
bin dch nhm cung cp thm mt ti liu tt gip bn c hc v dy
gii tch. Khi bin dch, chng ti tham kho bn ting Anh:
3*. W. J. Kaczkor, M. T. Nowak, Problems in Mathematical Analy-
sis I, Real Numbers, Sequences and Series, AMS, 2000.
4*. W. J. Kaczkor, M. T. Nowak, Problems in Mathematical Analy-
sis II, Continuity and Differentiation, AMS, 2001.
Sch ny c cc -u im sau:
Cc bi tp -c xp xp t d cho ti kh v c nhiu bi tp hay.
Li gii kh y v chi tit.
Kt hp -c nhng t-ng hay gia ton hc s cp v ton hc
hin i. Nhiu bi tp c ly t cc tp ch ni ting nh-, Ame ri-
can Mathe matical Monthly (ti ng Anh), Mathe matics Today (ti ng
Nga), De lta (ti ng Balan). V th, sch ny c th dng lm ti liu
cho cc hc sinh ph thng cc lp chuyn cng nh- cho cc sinh
vin i hc ngnh ton.
Cc kin thc c bn gii cc bi tp trong sch ny c th tm trong
5. Nguyn Duy Tin, Bi Ging Gii Tch, Tp I, NX B i Hc Quc
Gia H Ni, 2000.
6. W. Rudin, Principles of Mathematical Analysis, McGraw -Hil
Book Company, New York, 1964.
Tuy vy, tr-c mi ch-ng chng ti trnh by tm tt l thuyt gip
bn c nh li cc kin thc c bn cn thit khi gii bi tp trong ch-ng
t-ng ng.
Tp I v II ca sch ch bn n hm s mt bin s (tr phn khng
gian metric trong tp II). Kaczkor, Nowak chc s cn vit Bi Tp Gii
Tch cho hm nhiu bin v php tnh tch phn.
Chng ti ang bin dch tp II, sp ti s xut bn.

n

C
h
i
Li ni u v
Chng ti rt bit n :
- Gio s- Phm X un Y m (Php) gi cho chng ti bn gc ting
Anh tp I ca sch ny,
- Gio s- Nguyn Hu Vit H-ng (Vit Nam) gi cho chng ti bn
gc ting Anh tp II ca sch ny,
- Gio s- Spencer Shaw (M) gi cho chng ti bn gc ting Anh
cun sch ni ting ca W. Rudin (ni trn), xut bn ln th ba, 1976,
- TS D-ng Tt Thng c v v to iu kin chng ti bin dch
cun sch ny.
Chng ti chn thnh cm n tp th sinh vin Ton - L K5 H o
To C Nhn Khoa Hc Ti Nng, Tr-ng HKHTN, HQGHN, c
k bn tho v sa nhiu li ch bn ca bn nh my u tin.
Chng ti hy vng rng cun sch ny s -c ng o bn c n
nhn v gp nhiu kin qu bu v phn bin dch v trnh by. Rt mong
nhn -c s ch gio ca qu v bn c, nhng kin gp xin gi v:
Chi on cn b, Khoa Ton C Tin hc, tr-ng i hc Khoa
hc T nhin, i hc Quc gia HNi, 334 Nguyn Tri, Thanh
Xun, HNi.
X in chn thnh cm n.
H Ni, X un 2002.
Nhm bin dch
on Chi

n

C
h
i

n

C
h
i
Cc k hiu v khi nim
R - tp cc s thc
R
+
- tp cc s thc d-ng
Z - tp cc s nguyn
N - tp cc s nguyn d-ng hay cc s t nhin
Q - tp cc s hu t
(a, b) - khong m c hai u mt l a v b
[a, b] - on (khong ng) c hai u mt l a v b
[x] - phn nguyn ca s thc x
Vi x R, hm du ca x l
sgn x =
_

_
1 vi x > 0,
1 vi x < 0,
0 vi x = 0.
Vi x N,
n! = 1 2 3 ... n,
(2n)!! = 2 4 6 ... (2n 2) (2n),
(2n 1)!! = 1 3 5 ... (2n 3) (2n 1).
K hiu
_
n
k
_
=
n!
k!(nk)!
, n, k N, n k, l h s ca khai trin nh
thc Newton.
vii

n

C
h
i
viii Cc k hi u v khi ni m
Nu A R khc rng v b chn trn th ta k hiu sup A l cn
trn ng ca n, nu n khng b chn trn th ta quy -c rng
sup A = +.
Nu A R khc rng v b chn d-i th ta k hiu inf A l cn
d-i ng ca n, nu n khng b chn d-i th ta quy -c rng
inf A = .
Dy {a
n
} cc s thc -c gi l n iu tng (t-ng ng n iu
gim) nu a
n+1
a
n
(t-ng ng nu a
n+1
a
n
) vi mi n N. Lp
cc dy n iu cha cc dy tng v gim.
S thc c -c gi l im gii hn ca dy {a
n
} nu tn ti mt dy
con {a
n
k
} ca {a
n
} hi t v c.
Cho S l tp cc im t ca dy {a
n
}. Cn d-i ng v cn trn
ng ca dy , k hiu ln l-t l lim
n
a
n
v lim
n
a
n
-c xc nh
nh- sau
lim
n
a
n
=
_

_
+ nu {a
n
} khng b chn trn,
nu {a
n
} b chn trn v S = ,
sup S nu {a
n
} b chn trn v S = ,
lim
n
a
n
=
_

_
nu {a
n
} khng b chn d-i,
+ nu {a
n
} b chn d-i v S = ,
inf S nu {a
n
} b chn d-i v S = ,
Tch v hn

n=1
a
n
hi t nu tn ti n
0
N sao cho a
n
= 0 vi
n n
0
v dy {a
n
0
a
n
0
+1
... a
n
0
+n
} hi t khi n ti mt gii
hn P
0
= 0. S P = a
n
0
a
n
0
+1
... a
n
0
+n
P
0
-c gi l gi tr ca
tch v hn.
Trong phn ln cc sch ton n-c ta t tr-c n nay, cc hm
tang v ctang cng nh- cc hm ng-c ca chng -c k hiu
l tg x, cotg x, arctg x, arccotg x theo cch k hiu ca cc sch c
ngun gc t Php v Nga, tuy nhin trong cc sch ton ca M
v phn ln cc n-c chu u, chng -c k hiu t-ng t l
tan x, cot x, arctan x, arccot x. Trong cun sch ny chng ti s
s dng nhng k hiu ny bn c lm quen vi nhng k hiu
-c chun ho trn th gii.

n

C
h
i
Bi tp

n

C
h
i

n

C
h
i
Ch-ng 1
S thc
Tm tt l thuyt
Cho A l tp con khng rng ca tp cc s thc R = (, ).
S thc x R -c gi l mt cn trn ca A nu
a x, x A.
Tp A -c gi l b chn trn nu A c t nht mt cn trn.
S thc x R -c gi l mt cn d-i ca A nu
a x, a A.
Tp A -c gi l b chn d-i nu A c t nht mt cn d-i.
Tp A -c gi l b chn nu A va b chn trn v va b chn d-i.
R rng A b chn khi v ch khi tn ti x > 0 sao cho
|a| x, a A.
Cho A l tp con khng rng ca tp cc s thc R = (, ).
S thc x R -c gi l gi tr ln nht ca A nu
x A, a x, a A.
Khi , ta vit
x = max{a : a A} = max a
aA
.
3

n

C
h
i
4 Ch-ng 1. S thc
S thc x R -c gi l gi tr b nht ca A nu
x A, a x, a A.
Khi , ta vit
x = min{a : a A} = mina
aA
.
Cho A l tp con khng rng ca tp cc s thc R = (, ). Gi
s A b chn trn.
S thc x R -c gi l cn trn ng ca A, nu x l mt cn
trn ca A v l cn trn b nht trong tp cc cn trn ca A. Tc l,
a x, a A,
> o, a

A, a

> x .
Khi , ta vit
x = sup{a : a A} = sup a
aA
.
Cho A l tp con khng rng ca tp cc s thc R = (, ). Gi
s A b chn d-i.
S thc x R -c gi l cn d-i ng ca A, nu x l mt cn
d-i ca A v l cn trn ln nht trong tp cc cn d-i ca A. Tc l,
a x, a A,
> o, a

A, a

< x +.
Khi , ta vit
x = inf{a : a A} = inf a
aA
.
Tin v cn trn ng ni rng nu A l tp con khng rng,
b chn trn ca tp cc s thc, th A c cn trn ng (duy nht).
Tin trn t-ng -ng vi: nu A l tp con khng rng, b chn
d-i ca tp cc s thc, th A c cn d-i ng (duy nht).
T suy ra rng A l tp con khng rng, b chn ca tp cc s thc,
th A c cn trn ng, v c cn d-i ng.
Nu tp A khng b chn trn, th ta qui -c sup A = +; Nu tp
A khng b chn d-i, th ta qui -c inf A = ;

n

C
h
i
Tm tt l thuy t 5
Cho hai s nguyn a, b. Ta ni rng b chia ht cho a hoc a chia b,
nu tn ti s nguyn c, sao cho b = a.c. Trong tr-ng hp ta ni a l
-c ca b (hoc b l bi ca a) v vit a|b.
Cho hai s nguyn a
1
, a
2
. S nguyn m -c gi l -c chung ca
a
1
, a
2
nu m|a
1
, m|a
2
. S nguyn m -c gi l bi chung ca a
1
, a
2
nu a
1
|m, a
2
|m.
c chung m 0 ca a
1
, a
2
c tnh cht l chia ht cho bt k -c
chung no ca a
1
, a
2
) -c gi l -c chung ln nht ca a
1
, a
2
v
uc k hiu l (a
1
, a
2
).
Bi chung m 0 ca a
1
, a
2
c tnh cht l -c ca bt k bi chung
no ca a
1
, a
2
-c gi l bi chung nh nht ca a
1
, a
2
v uc k
hiu l [a
1
, a
2
].
Nu (a, b) = 1 th ta ni a, b nguyn t cng nhau.
S nguyn d-ng p N -c gi l s nguyn t, nu p ch c hai
-c (tm th-ng) l 1 v p.
Ga s m l s nguyn d-ng. Hai s nguyn a, b -c gi l ng d-
theo modulo m, nu m|(a b). Trong tr-ng hp ta vit
a = b (mod m).
Ta gi r l s hu t (hay phn s), nu tn ti p, q Z sao cho
r = p/q. Phn s ny l ti gin nu (p, q) = 1.
S v t l s thc nh-ng khng phi l s v t. Tp hp cc s
hu t tr mt trong tp cc s thc, tc l, gia hai s thc khc
nhau bt k (a < b) tn ti t nht mt s hu t (r: a < r < b).
Phn nguyn ca s thc x, -c k hiu l [x], l s nguyn (duy
nht) sao cho x 1 < [x] x. Phn l ca s thc x, -c k hiu l
{x}, l s thc xc nh theo cng thc {x} = x [x].
Cc hm s s cp a
x
, log
a
x, sinx, cos x, arcsinx, arccos x -c nh
ngha theo cch thng th-ng. Tuy nhin, cn ch rng, ti liu ny dng
cc k hiu tiu chun quc t sau
tan x = sinx/ cos x, cot x = cos x/ sinx,
cosh x =
e
x
+e
x
2
, sinhx =
e
x
e
x
2
,
tanhx = sinhx/ cosh x, cothx = cosh x/ sinh x.
T-ng t ta c cc k hiu v hm ng-c arctanx, arccot x.

n

C
h
i
6 Ch-ng 1. S thc
1.1 Cn trn ng v cn d-i ng ca tp cc
s thc. Lin phn s
1.1.1. Chng minh rng
sup{x Q : x > 0, x
2
< 2} =

2.
1.1.2. Cho A R khc rng. nh ngha A = {x : x A}. Chng
minh rng
sup(A) = inf A,
inf(A) = supA.
1.1.3. Cho A, B R l khng rng. nh ngha
A+B = {z = x +y : x A, y B} ,
AB = {z = x y : x A, y B} .
Chng minh rng
sup(A+B) = sup A+ sup B,
sup(AB) = sup Ainf B.
Thit lp nhng cng thc t-ng t cho inf(A+B) v inf(AB).
1.1.4. Cho cc tp khng rng A v B nhng s thc d-ng, nh ngha
A B = {z = x y : x A, y B} ,
1
A
=
_
z =
1
x
: x A
_
.
Chng minh rng
sup(A B) = sup A sup B,
v nu inf A > 0 th
sup
_
1
A
_
=
1
inf A
,
khi inf A = 0 th sup
_
1
A
_
= +. Hn na nu A v B l cc tp s thc b
chn th
sup(A B)
= max {supA supB, sup A inf B, inf A sup B, inf A inf B} .

n

C
h
i
1.1. Cn tr n ng v cn d-i ng. Li n phn s 7
1.1.5. Cho A v B l nhng tp con khc rng cc s thc. Chng minh rng
sup(A B) = max {sup A, sup B}
v
inf(A B) = min{inf A, inf B} .
1.1.6. Tm cn trn ng v cn d-i ng ca A
1
, A
2
xc nh bi
A
1
=
_
2(1)
n+1
+ (1)
n(n+1)
2
_
2 +
3
n
_
: n N
_
,
A
2
=
_
n 1
n + 1
cos
2n
3
: n N
_
.
1.1.7. Tm cn trn ng v cn d-i ng ca cc tp A v B, trong
A = {0, 2; 0, 22; 0, 222; . . . } v B l tp cc phn s thp phn gia 0 v 1
m ch gm cc ch s 0 v 1.
1.1.8. Tm cn d-i ng v cn trn ng ca tp cc s
(n+1)
2
2
n
, trong
n N.
1.1.9. Tm cn trn ng v cn d-i ng ca tp cc s
(n+m)
2
2
nm
, trong
n, m N.
1.1.10. Xc nh cn trn ng v cn d-i ng ca cc tp sau:
A =
_
m
n
: m, n N, m < 2n
_
, (a)
B =
_
n [

n] : n N
_
. (b)
1.1.11. Hy tm
sup
_
x R : x
2
+x + 1 > 0
_
, (a)
inf
_
z = x +x
1
: x > 0
_
, (b)
inf
_
z = 2
x
+ 2
1
x
> 0
_
. (c)

n

C
h
i
8 Ch-ng 1. S thc
1.1.12. Tm cn trn ng v cn d-i ng ca nhng tp sau:
A =
_
m
n
+
4n
m
: m, n N
_
, (a)
B =
_
mn
4m
2
+n
2
: m Z, n N
_
, (b)
C =
_
m
m+n
: m, n N
_
, (c)
D =
_
m
|m| +n
: m Z, n N
_
, (d)
E =
_
mn
1 +m+n
: m, n N
_
. (e)
1.1.13. Cho n 3, n N. Xt tt c dy d-ng hu hn (a
1
, . . . , a
n
), hy
tm cn trn ng v cn d-i ng ca tp cc s
n

k=1
a
k
a
k
+a
k+1
+a
k+2
,
trong a
n+1
= a
1
, a
n+2
= a
2
.
1.1.14. Chng minh rng vi mi s v t v vi mi n N tn ti mt s
nguyn d-ng q
n
v mt s nguyn p
n
sao cho


p
n
q
n

<
1
nq
n
.
ng thi c th chn dy {p
n
} v {q
n
} sao cho


p
n
q
n

<
1
q
n
2
.
1.1.15. Cho l s v t. Chng minh rng A = {m+n : m, n Z} l
tr mt trong R, tc l trong bt k khong m no u c t nht mt phn t
ca A.
1.1.16. Chng minh rng {cos n : n N} l tr mt trong on [1, 1].
1.1.17. Cho x R \ Z v dy {x
n
} -c xc nh bi
x = [x] +
1
x
1
, x
1
= [x
1
] +
1
x
2
, . . . , x
n1
= [x
n1
] +
1
x
n
.

n

C
h
i
1.1. Cn tr n ng v cn d-i ng. Li n phn s 9
khi
x = [x] +
1
[x
1
] +
1
[x
2
] +
1
.
.
. +
1
[x
n1
] +
1
x
n
.
Chng minh rng x l s hu t khi v ch khi tn ti n N sao cho x
n
l mt
s nguyn.
Ch . Ta gi biu din trn ca x l mt lin phn s hu hn. Biu thc
a
0
+
1
a
1
+
1
a
2
+
1
.
.
. +
1
a
n1
+
1
a
n
-c vit gn thnh
a
0
+
1|
|a
1
+
1|
|a
2
+. . . +
1|
|a
n
.
1.1.18. Cho cc s thc d-ng a
1
, a
2
, . . . , a
n
, t
p
0
= a
0
, q
0
= 1,
p
1
= a
0
a
1
+ 1, q
1
= a
1
,
p
k
= p
k1
a
k
+p
k2
, q
k
= q
k1
a
k
+q
k2
, vi k = 2, 3, . . . , n,
v nh ngha
R
0
= a
0
, R
k
= a
0
+
1|
|a
1
+
1|
|a
2
+. . . +
1|
|a
k
, k = 1, 2, . . . , n.
_
R
k
-c gi l phn t hi t th k n a
0
+
1|
|a
1
+
1|
|a
2
+. . . +
1|
|an
_
.
Chng minh rng
R
k
=
p
k
q
k
vi k = 0, 1, . . . , n.
1.1.19. Chng minh rng nu p
k
, q
k
-c nh ngha nh- trong bi ton trn
v a
0
, a
1
, . . . , a
n
l cc s nguyn th
p
k1
q
k
q
k1
p
k
= (1)
k
vi k = 0, 1, . . . , n.
S dng ng thc trn kt lun rng p
k
v q
k
l nguyn t cng nhau.

n

C
h
i
10 Ch-ng 1. S thc
1.1.20. Cho x l mt s v t, ta nh ngha dy {x
n
} nh- sau:
x
1
=
1
x [x]
, x
2
=
1
x
1
[x
1
]
, . . . , x
n
=
1
x
n1
[x
n1
]
, . . . .
Ngoi ra, chng ta cho t a
0
= [x], a
n
= [x
n
], n = 1, 2, . . . , v
R
n
= a
0
+
1|
|a
1
+
1|
|a
2
+. . . +
1|
|a
k
.
Chng minh rng lch gia s x v phn t hi t th n ca n -c cho bi
cng thc
x R
n
=
(1)
n
(q
n
x
n+1
+q
n1
)q
n
,
trong p
n
, q
n
l -c nh ngha trong 1.1.18. T hy suy ra rng x nm
gia hai phn t hi t lin tip ca n.
1.1.21. Chng minh rng tp {sinn : n N} l tr mt trong [1, 1].
1.1.22. S dng kt qu trong bi 1.1.20 chng minh rng vi mi s v t x
tn ti dy
_
pn
qn
_
cc s hu t, vi q
n
l, sao cho

x
p
n
q
n

<
1
q
2
n
.
(So snh vi 1.1.14.)
1.1.23. Kim tra cng thc sau v hiu s gia hai phn t hi t lin tip:
R
n+1
R
n
=
(1)
n
q
n
q
n+1
.
1.1.24. Cho x l s v t. Chng minh rng phn t hi t R
n
nh ngha
trong 1.1.20 tin ti x sao cho
|x R
n+1
| < |x R
n
| , n = 0, 1, 2, . . . .
1.1.25. Chng minh rng phn t hi t R
n
= p
n
/q
n
l -c l-ng tt nht
ca x trong tt c cc phn s hu t vi mu s q
n
hoc nh hn. Tc l:
nu r/s l mt s hu t vi mu s d-ng c dng |x r/s| < |x R
n
| th
s > q
n
.
1.1.26. Khai trin mi biu thc sau thnh cc lin phn s v hn:

2,

51
2
.
1.1.27. Cho s nguyn d-ng k, biu din ca

k
2
+k thnh lin phn s v
hn.
1.1.28. Tm tt c cc s x (0, 1) m s biu din lin tc v hn c a
1
(xem
1.1.20) t-ng ng vi s nguyn d-ng n cho tr-c.

n

C
h
i
1.2. Mt s bt ng thc s cp 11
1.2 Mt s bt ng thc s cp
1.2.1. Chng minh rng nu a
k
> 1, k = 1, . . . , n l cc s cng d-ng
hoc cng m th
(1 +a
1
) (1 +a
2
) . . . (1 +a
n
) 1 +a
1
+a
2
+. . . +a
n
.
Ch . Nu a
1
= a
2
= . . . = a
n
= a th ta c bt ng thc Bernoulli:
(1 +a)
n
1 +na, a > 1.
1.2.2. S dng php qui np, hy chng minh kt qu sau: Nu a
1
, a
2
, . . . , a
n
l cc s thc d-ng sao cho a
1
a
2
. . . a
n
= 1 th a
1
+a
2
+. . . +a
n
n.
1.2.3. K hiu A
n
, G
n
v H
n
ln l-t l trung bnh cng, trung bnh nhn v
trung bnh iu ho ca n s thc d-ng a
1
, a
2
, . . . , a
n
, tc l
A
n
=
a
1
+a
2
+. . . +a
n
n
,
G
n
=
n

a
1
a
2
. . . a
n
,
H
n
=
n
1
a
1
+
1
a
2
+. . . +
1
an
.
Chng minh rng A
n
G
n
H
n
.
1.2.4. S dng kt qu G
n
A
n
trong bi ton tr-c kim tra bt ng thc
Bernoulli
(1 +x)
n
1 +nx vi x > 0.
1.2.5. Cho n N, hy kim tra cc khng nh sau:
1
n
+
1
n + 1
+
1
n + 1
+. . .
1
2n
>
2
3
, (a)
1
n + 1
+
1
n + 2
+
1
n + 3
+. . . +
1
3n + 1
> 1, (b)
1
2
<
1
3n + 1
+
1
3n + 2
+. . . +
1
5n
+
1
5n + 1
<
2
3
, (c)
(n
n

n + 1 1) < 1 +
1
2
+. . . +
1
n
(d)
< n
_
1
1
n

n + 1
+
1
n + 1
_
, n > 1.

n

C
h
i
12 Ch-ng 1. S thc
1.2.6. Chng minh rng vi mi x > 0 v n N ta c
x
n
1 +x +x
2
+x
3
+. . . +x
2n

1
2n + 1
.
1.2.7. Cho {a
n
} l mt cp s cng vi cc s hng d-ng. Chng minh rng

a
1
a
n

n

a
1
a
2
. . . a
n

a
1
+a
n
2
.
1.2.8. Chng minh rng

n
n

n!
n + 1
2
, n N.
1.2.9. Cho a
k
, k = 1, 2, . . . , n, l cc s d-ng tho mn iu kin
n

k=1
a
k
1.
Chng minh rng
n

k=1
1
a
k
n
2
.
1.2.10. Cho a
k
> 0, k = 1, 2, . . . , n (n > 1) v t s =
n

k=1
a
k
. Hy kim
tra cc khng nh sau:
n
_
n

k=1
a
k
s a
k
_
1
n 1
1
n
n

k=1
s a
k
a
k
, (a)
n

k=1
s
s a
k

n
2
n 1
, (b)
n
_
n

k=1
a
k
s +a
k
_
1
n + 1. (c)
1.2.11. Chng minh rng nu a
k
> 0, k = 1, . . . , n v a
1
a
2
. . . a
n
= 1
th
(1 +a
1
) (1 +a
2
) . . . (1 +a
n
) 2
n
.
1.2.12. Chng minh bt ng thc Cauchy
(1)
:
_
n

k=1
a
k
b
k
_
2

k=1
a
2
k
n

k=1
b
2
k
.
(1)
Cn gi l bt ng thc Buniakovskii- Cauchy - Schwarz

n

C
h
i
1.2. Mt s bt ng thc s cp 13
1.2.13. Chng minh rng
_
_
_
n

k=1
a
k
_
2
+
_
n

k=1
b
k
_
2
_
_
1
2

k=1
_
a
2
k
+b
2
k
_1
2
.
1.2.14. Chng minh rng nu
n

k=1
a
2
k
=
n

k=1
b
2
k
= 1 th

k=1
a
k
b
k

1.
1.2.15. Cho a
k
> 0, k = 1, 2, . . . , n, hy kim tra nhng khng nh sau
n

k=1
a
k
n

k=1
1
a
k
n
2
, (a)
n

k=1
a
k
n

k=1
1 a
k
a
k
n
n

k=1
(1 a
k
), (b)
(log
a
a
1
)
2
+ (log
a
a
2
)
2
+. . . + (log
a
a
n
)
2

1
n
, (c)
vi iu kin a
1
a
2
. . . a
n
= a = 1.
1.2.16. Cho > 0, chng minh rng

k=1
a
k
b
k

k=1
a
2
k
+

4
n

k=1
b
2
k
.
1.2.17. Chng minh cc bt ng thc sau:
n

k=1
|a
k
|

n
_
n

k=1
a
2
k
_1
2

n
n

k=1
|a
k
|.
1.2.18. Chng minh rng
_
n

k=1
a
k
b
k
_
2

k=1
ka
2
k
n

k=1
b
2
k
k
, (a)
_
n

k=1
a
k
k
_
2

k=1
k
3
a
2
k
n

k=1
1
k
5
. (b)

n

C
h
i
14 Ch-ng 1. S thc
1.2.19. Chng minh rng
_
n

k=1
a
p
k
_
2

k=1
a
p+q
k
n

k=1
a
pq
k
,
vi mi p, q v mi b s d-ng a
1
, a
2
, . . . , a
n
.
1.2.20. Tm gi tr nh nht ca tng
n

k=1
a
2
k
vi iu kin
n

k=1
a
k
= 1.
1.2.21. Cho p
1
, p
2
, . . . , p
n
l cc s d-ng. Tm gi tr nh nht ca tng
n

k=1
p
k
a
2
k
vi iu kin
n

k=1
a
k
= 1.
1.2.22. Chng minh rng
_
n

k=1
a
k
_
2
(n 1)
_
n

k=1
a
2
k
+ 2a
1
a
2
_
.
1.2.23. Chng minh cc bt ng thc sau:
_
n

k=1
(a
k
+b
k
)
2
_1
2

_
n

k=1
a
2
k
_1
2
+
_
n

k=1
b
2
k
_1
2
, (a)

_
n

k=1
a
2
k
_1
2

_
n

k=1
b
2
k
_1
2

k=1
|a
k
b
k
|. (b)
1.2.24. Cho p
1
, p
2
, . . . , p
n
l cc s d-ng. Tm gi tr nh nht ca
n

k=1
a
2
k
+
_
n

k=1
a
k
_
2
vi iu kin
n

k=1
p
k
a
k
= 1.
1.2.25. Chng minh bt ng thc Chebyshev.
Nu
a
1
a
2
. . . a
n
v b
1
b
2
. . . b
n
,
hoc
a
1
a
2
. . . a
n
v b
1
b
2
. . . b
n
,

n

C
h
i
1.2. Mt s bt ng thc s cp 15
th
n

k=1
a
k
n

k=1
b
k
n
n

k=1
a
k
b
k
.
1.2.26. Gi s a
k
0, k = 1, 2, . . . , n v p N, chng minh rng
_
1
n
n

k=1
a
k
_
p

1
n
n

k=1
a
p
k
.
1.2.27. Chng minh bt ng thc
(a +b)
2
(1 +c)a
2
+
_
1 +
1
c
_
b
2
vi s d-ng c v s thc a, b bt k.
1.2.28. Chng minh rng

a
2
+b
2

a
2
+c
2

|b c|.
1.2.29. Cho cc s d-ng a, b, c, kim tra cc khng nh sau:
bc
a
+
ac
b
+
ab
c
(a +b +c), (a)
1
a
+
1
b
+
1
c

1

bc
+
1

ca
+
1

ab
, (b)
2
b +c
+
2
a +c
+
2
a +b

9
(a +b +c)
, (c)
b
2
a
2
c +a
+
c
2
b
2
a +b
+
a
2
c
2
b +c
0, (d)
1
8
(a b)
2
a

a +b
2

ab
1
8
(a b)
2
b
vi b a. (e)
1.2.30. Cho a
k
R, b
k
> 0, k = 1, 2, . . . , n, t
m = min
_
a
k
b
k
: k = 1, 2, . . . , n
_
v
M = max
_
a
k
b
k
: k = 1, 2, . . . , n
_
.
Chng minh rng
m
a
1
+a
2
+. . . +a
n
b
1
+b
2
+. . . +b
n
M

n

C
h
i
16 Ch-ng 1. S thc
1.2.31. Chng minh rng nu 0 <
1
<
2
< . . . <
n
<

2
, n > 1 th
tan
1
<
sin
1
+ sin
2
+. . . + sin
n
cos
1
+ cos
2
+. . . + cos
n
< tan
n
.
1.2.32. Cho c
1
, c
2
, . . . , c
n
d-ng v k
1
, k
2
, . . . , k
n
N, t
S = max {
k
1

c
1
,
k
2

c
2
, . . . ,
kn

c
n
} ,
s = min{
k
1

c
1
,
k
2

c
2
, . . . ,
kn

c
n
} .
Chng minh rng
s (a
1
+a
2
+. . . +a
n
)
1
k
1
+k
2
+...+kn
S.
1.2.33. Cho a
k
> 0, b
k
> 0, k = 1, 2, . . . , n, t
M = max
_
a
k
b
k
: k = 1, 2, . . . , n
_
.
Chng minh rng
a
1
+a
2
2
+. . . +a
n
n
b
1
+Mb
2
2
+. . . +M
n1
b
n
n
M.
1.2.34. Chng minh rng nu x l mt s thc ln hn cc s a
1
, a
2
, . . . , a
n
th
1
x a
1
+
1
x a
2
+. . . +
1
x a
n

n
x
a
1
+a
2
+...+an
n
.
1.2.35. t c
k
=
_
n
k
_
, k = 0, 1, 2, . . . , n. Chng minh bt ng thc

c
1
+

c
2
+. . . +

c
n

_
n(2
n
1).
1.2.36. Cho n 2, chng minh rng
n

k=0
_
n
k
_

_
2
n
2
n 1
_
n1
.
1.2.37. Cho a
k
> 0, k = 1, 2, . . . , n v k hiu A
n
l trung bnh cng ca
chng. Chng minh rng
n

k=1
A
p
k

p
p 1
n

k=1
A
p1
k
a
k
vi mi s nguyn p > 1.

n

C
h
i
1.2. Mt s bt ng thc s cp 17
1.2.38. Cho a
k
> 0, k = 1, 2, . . . , n, t a = a
1
+a
2
+. . . +a
n
. Hy chng
minh rng
n1

k=1
a
k
a
k+1

a
2
4
.
1.2.39. Chng minh rng vi mi hon v b
1
, b
2
, . . . , b
n
ca cc s d-ng
a
1
, a
2
, . . . , a
n
ta u c
a
1
b
1
+
a
2
b
2
+. . . +
a
n
b
n
n.
1.2.40. Chng minh bt ng thc Weierstrass.
Nu 0 < a
k
< 1, k = 1, 2, . . . , n v a
1
+a
2
+. . . +a
n
< 1 th
1 +
n

k=1
a
k
<
n

k=1
(1 +a
k
) <
1
1
n

k=1
a
k
, (a)
1
n

k=1
a
k
<
n

k=1
(1 a
k
) <
1
1 +
n

k=1
a
k
. (b)
1.2.41. Gi s 0 < a
k
< 1, k = 1, 2, . . . , n, t a
1
+ a
2
+ . . . + a
n
= a.
Chng minh rng
n

k=1
a
k
1 a
k

na
n a
.
1.2.42. Cho 0 < a
k
1, k = 1, 2, . . . , n v n 2. Kim tra bt ng thc
sau:
n

k=1
1
1 +a
k

n
n

k=1
a
k
n

k=1
a
k
+n
n

k=1
a
k
.
1.2.43. Cho a
k
, k = 1, 2, . . . , n khng m sao cho a
1
+a
2
+ . . . + a
n
= 1,
chng minh rng
n

k=1
(1 +a
k
) (n + 1)
n
n

k=1
a
k
, (a)
n

k=1
(1 a
k
) (n 1)
n
n

k=1
a
k
. (b)

n

C
h
i
18 Ch-ng 1. S thc
1.2.44. Chng minh rng nu a
k
> 0, k = 1, 2, . . . , n v
n

k=1
1
1+an
= n 1
th
n

k=1
1
a
k
(n 1)
n
.
1.2.45. Chng minh rng vi gi thit cho trong bi 1.2.43 ta c
n

k=1
(1 +a
k
)
(n + 1)
n

n

k=1
(1 a
k
)
(n 1)
n
, n > 1.
1.2.46. Cho a
1
, a
2
, . . . , a
n
l cc s d-ng, chng minh rng
a
1
a
2
+a
3
+
a
2
a
3
+a
4
+. . . +
a
n2
a
n1
+a
n
+
a
n1
a
n
+a
1
+
a
n
a
1
+a
2

n
4
.
1.2.47. Cho t v a
1
, a
2
, . . . , a
n
l cc s thc bt k. Chng minh bt ng
thc
n

k=1
_
|a
k
t|
2
k

n

k=2
_
|a
k
a
1
|
2
k
.
1.2.48. Cho a
1
, a
2
, . . . , a
n
v b
1
, b
2
, . . . , b
n
l cc s d-ng, chng minh rng
n
_
(a
1
+b
1
)(a
2
+b
2
) . . . (a
n
+b
n
)
n

a
1
a
2
. . . a
n
+
n
_
b
1
b
2
. . . b
n
.
1.2.49. Gi s rng 0 < a
1
< a
2
< . . . < a
n
v p
1
, p
2
, . . . , p
n
l cc s
khng m m
n

k=1
p
k
= 1. Chng minh bt ng thc
_
n

k=1
p
k
a
k
__
n

k=1
p
k
1
a
k
_

A
2
G
2
,
trong A =
1
2
(a
1
+a
n
) v G =

a
1
a
n
.
1.2.50. Cho s nguyn d-ng n, t t-ng ng (n) v (n) l tng cc -c
s d-ng ca n v s cc -c s . Chng minh rng
(n)
(n)


n.

n

C
h
i
Ch-ng 2
Dy s thc
Tm tt l thuyt
Dy s l mt nh x t tp cc s t nhin (hoc cc s nguyn khng
m) vo tp cc s thc
f : N R.
t a
n
= f(n), n N, v dng k hiu {a
n
} ch dy s.
Dy s {a
n
} -c gi l
- d-ng (m) nu a
n
> 0 (a
n
< 0) vi mi n;
- khng m (khng d-ng) nu a
n
0 (a
n
0) vi mi n;
- n iu tng (gim) nu a
n+1
a
n
(a
n+1
a
n
) vi mi n;
- tng (gim) ngt nu a
n+1
> a
n
(a
n+1
< a
n
) vi mi n;
- hi t ti a R (hoc c gii hn hu hn l a), nu vi mi s > 0
cho tr-c b ty , tn ti n

N sao cho
|a
n
a| < , n n

.
Trong tr-ng hp nh- th, ta ni dy {a
n
} hi t, v gi a l gii hn ca
dy {a
n
} v vit
lim
n
a
n
= a;
- phn k ra +, nu vi mi s > 0 cho tr-c ln ty , tn ti
n

N sao cho
a
n
> , n n

.
19

n

C
h
i
20 Ch-ng 2. Dy s thc
Trong tr-ng hp nh- th, ta vit
lim
n
a
n
= +;
- phn k ra , nu vi mi s > 0 cho tr-c ln ty , tn ti
n

N sao cho
a
n
< , n n

.
Trong tr-ng hp nh- th, ta vit
lim
n
a
n
= ;
- dy Cauchy (hoc dy c bn), nu vi mi s > 0 cho tr-c b ty
, tn ti n

N sao cho
|a
m
a
n
| < , m, n n

.
nh l hi t n iu ni rng dy s n iu (tng hoc gim)
v b chn c gii hn hu hn.
Tiu chun Cauchy ni rng dy s hi t khi v ch khi n l dy
Cauchy.
Cc tnh cht c bn ca gii hn l
- Mt dy hi t th b chn.
- Bo ton cc php tnh s hc, tc l, nu
lim
n
a
n
= a, lim
n
b
n
= b,
th
lim
n
(a
n
b
n
) = a b, , R;
lim
n
(a
n
b
n
) = ab; lim
n
(a
n
/b
n
) = a/b vi b = 0.
- Bo ton th t theo ngha sau: nu
lim
n
a
n
= a, lim
n
b
n
= b, a
n
b
n
; vi n n
0
no ,
th a b.
- nh l kp: Cho ba dy s thc {a
n
}, {b
n
}, {c
n
}. Nu
lim
n
a
n
= a, lim
n
b
n
= a, a
n
c
n
b
n
, vi n n
0
no

n

C
h
i
Tm tt l thuy t 21
th lim
n
c
n
= a.
Cho {a
n
} l dy s thc v {n
k
} l dy cac s t nhin tng ngt, tc
l n
1
< n
2
< < a
k
< a
k+1
< . Khi , ta gi {a
n
k
} l mt dy
con ca dy {a
n
}. S thc a -c gi l gii hn ring hay lim
gii hn ca {a
n
}, nu tn ti mt dy con {a
n
k
} hi t ti a, tc l,
lim
k
a
n
k
= a.
nh l Bolzano - Weierstrass khng nh rng, mi dy s thc
b chn c t nht mt im gii hn.
Tp cc gii hn ring ca mt dy s thc b chn {a
n
} c gi tr ln
nht. Gi tr ny -c gi l gii hn trn ca dy {a
n
} v -c k hiu
l
lim
n
a
n
.
Tp cc gii hn ring ca mt dy s thc b chn {a
n
} c gi tr b
nht. Gi tr ny -c gi l gii hn d-i ca dy {a
n
} v -c k hiu
l
lim
n
a
n
.
Ni rng {a
n
} l dy truy hi cp h nu
a
n
= f(a
n1
, ..., a
nh
), n h,
trong f l hm s thc no .
Ni rng {a
n
} l cp s cng nu n c dng
a
n
= a
0
+nd,
(a
0
l s hng u, d l cng sai).
Ni rng {a
n
} l cp s nhn nu n c dng
a
n
= a
0
q
n
,
(a
0
l s hng u, q l cng bi).
Cc k hiu ca Landau. Cho hai dy {a
n
} v {b
n
}. Ta ni rng
- Dy {b
n
} chn dy {a
n
}, nu tn ti hng s C > 0 v tn ti s
n
0
N sao cho
|a
n
| C|b
n
|, n n
0
.

n

C
h
i
22 Ch-ng 2. Dy s thc
Trong tr-ng hp ta vit
a
n
= O(b
n
).
- Dy {a
n
} khng ng k so vi {b
n
}, nu vi mi > 0 tn ti s
n

N sao cho
|a
n
| |b
n
|, n n

,
tc l
lim
n
a
n
b
n
= 0.
Trong tr-ng hp ta vit
a
n
= (b
n
).
- Dy {a
n
} t-ng -ng vi {b
n
}, nu
a
n
b
n
= (b
n
),
tc l
lim
n
a
n
b
n
= 1.
Trong tr-ng hp ta vit
a
n
b
n
.

n

C
h
i
2.1. Dy n i u 23
2.1 Dy n iu
2.1.1. Chng minh rng:
(a) Nu {a
n
} l dy n iu tng th lim
n
a
n
= sup {a
n
: n N},
(b) Nu {a
n
} l dy n iu gim th lim
n
a
n
= inf {a
n
: n N} .
2.1.2. Gi s a
1
, a
2
, ..., a
p
l nhng s d-ng c nh. Xt cc dy sau:
s
n
=
a
n
1
+a
n
2
+... +a
n
p
p
v x
n
=
n

s
n
, n N.
Chng minh rng {x
n
} l dy n iu tng.
Gi . Tr-c tin xt tnh n iu ca dy
_
sn
s
n1
_
, n 2.
2.1.3. Chng minh rng dy {a
n
}, vi a
n
=
n
2
n
, n > 1, l dy gim ngt v
tm gii hn ca dy.
2.1.4. Cho {a
n
} l dy b chn tho mn iu kin a
n+1
a
n

1
2
n
, n N.
Chng minh rng dy {a
n
} hi t.
Gi . Xt dy
_
a
n

1
2
n1
_
.
2.1.5. Chng minh s hi t ca cc dy sau:
a
n
= 2

n +
_
1

1
+
1

2
+... +
1

n
_
; (a)
b
n
= 2

n + 1 +
_
1

1
+
1

2
+... +
1

n
_
. (b)
Gi . Tr-c tin thit lp bt ng thc:
2(

n + 1 1) <
1

1
+
1

2
+ ... +
1

n
< 2

n, n N.
2.1.6. Chng minh rng dy {a
n
} -c xc nh theo cng thc truy hi
a
1
=
3
2
, a
n
=
_
3a
n1
2, vi n 2
hi t v tm gii hn ca n.

n

C
h
i
24 Ch-ng 2. Dy s thc
2.1.7. Cho c > 2, xt dy {a
n
} -c xc nh theo cng thc truy hi
a
1
= c
2
, a
n+1
= (a
n
c)
2
, n 1.
Chng minh dy {a
n
} tng ngt.
2.1.8. Gi s dy {a
n
} tho mn iu kin
0 < a
n
< 1, a
n
(1 a
n+1
) >
1
4
vi n N.
Thit lp s hi t ca dy v tm gii hn ca n.
2.1.9. Thit lp s hi t v tm gii hn ca dy -c xc nh theo biu thc
a
1
= 0, a
n+1
=

6 +a
n
vi n 1.
2.1.10. Chng minh dy -c cho bi
a
1
= 0, a
2
=
1
2
, a
n+1
=
1
3
(1 +a
n
+a
3
n1
) vi n > 1
hi t v xc nh gii hn ca n.
2.1.11. Kho st tnh n iu ca dy
a
n
=
n!
(2n + 1)!!
, n 1,
v xc nh gii hn ca n.
2.1.12. Hy xc nh tnh hi t hay phn k ca dy
a
n
=
(2n)!!
(2n + 1)!!
, n 1.
2.1.13. Chng minh s hi t ca cc dy sau
a
n
= 1 +
1
2
2
+
1
3
2
+ ...
1
n
2
, n N. (a)
a
n
= 1 +
1
2
2
+
1
3
3
+ ...
1
n
n
, n N. (b)
2.1.14. Cho dy {a
n
} c s hng tng qut
a
n
=
1
_
n(n + 1)
+
1
_
(n + 1)(n + 2)
+... +
1
_
(2n 1)2n
, n N.
Chng minh rng dy hi t.

n

C
h
i
2.1. Dy n i u 25
2.1.15. Cho p N, a > 0 v a
1
> 0, nh ngha dy {a
n
} bi
a
n+1
=
1
p
_
(p 1)a
n
+
a
a
p1
n
_
, n N.
Tm lim
n
a
n
.
2.1.16. Dy {a
n
} -c cho theo cng thc truy hi
a
1
=

2, a
n+1
=
_
2 +

a
n
vi n 1.
Chng minh dy {a
n
} hi t v tm gii hn ca n.
2.1.17. Dy {a
n
} -c xc nh theo cng thc truy hi
a
1
= 1, a
n+1
=
2(2a
n
+ 1)
a
n
+ 3
vi n N.
Thit lp s hi t v tm gii hn ca dy {a
n
}.
2.1.18. Tm cc hng s c > 0 sao cho dy {a
n
} -c nh ngha bi cng thc
truy hi
a
1
=
c
2
, a
n+1
=
1
2
(c +a
2
n
) vi n N
l hi t. Trong tr-ng hp hi t hy tm lim
n
a
n
.
2.1.19. Cho a > 0 c nh, xt dy {a
n
} -c xc nh nh- sau
a
1
> 0, a
n+1
= a
n
a
2
n
+ 3a
3a
2
n
+a
vi n N.
Tm tt c cc s a
1
sao cho dy trn hi t v trong nhng tr-ng hp hy
tm gii hn ca dy.
2.1.20. Cho dy {a
n
} nh ngha truy hi bi
a
n+1
=
1
4 3a
n
vi n 1.
Tm cc gi tr ca a
1
dy trn hi t v trong cc tr-ng hp hy tm
gii hn ca dy.

n

C
h
i
26 Ch-ng 2. Dy s thc
2.1.21. Cho a l mt s c nh bt k v ta nh ngha {a
n
} nh- sau:
a
1
R v a
n+1
= a
2
n
+ (1 2a)a
n
+a
2
vi n N.
Xc nh a
1
sao cho dy trn hi t v trong tr-ng hp nh- th tm gii hn
ca n.
2.1.22. Cho c > 0 v b > a > 0, ta nh ngha dy {a
n
} nh- sau:
a
1
= c, a
n+1
=
a
2
n
+ab
a +b
vi n N.
Vi nhng gi tr ca a, b v c dy trn s hi t ? Trong cc tr-ng hp hy
xc nh gii hn ca dy.
2.1.23. Chng minh rng dy {a
n
} -c nh ngha bi cng thc
a
1
> 0, a
n+1
= 6
1 +a
n
7 +a
n
, n N
hi t v tm gii hn ca n.
2.1.24. Cho c 0 xt {a
n
} -c cho hi cng thc
a
1
= 0, a
n+1
=

c +a
n
, n N.
Chng minh rng dy hi t v tm gii hn ca n.
2.1.25. Kho st s hi t ca dy -c cho bi cng thc
a
1
=

2, a
n+1
=

2a
n
, n N.
2.1.26. Cho k N, kho st s hi t ca dy {a
n
} -c cho bi cng thc
truy hi sau
a
1
=
k

5, a
n+1
=
k

5a
n
, n N.
2.1.27. Kho st s hi t ca dy {a
n
} sau
1 a
1
2, a
2
n+1
= 3a
n
2, n N.
2.1.28. Vi c > 1, nh ngha dy {a
n
} v {b
n
} nh- sau:
a
1
=
_
c(c 1), a
n+1
=
_
c(c 1) +a
n
, n 1, (a)
b
1
=

c, b
n+1
=
_
cb
n
, n 1. (b)
Chng minh rng c hai dy u c gii hn l c.

n

C
h
i
2.1. Dy n i u 27
2.1.29. Cho a > 0 v b > 0, nh ngha dy {a
n
} bi
0 < a
1
< b, a
n+1
=
_
ab
2
+a
2
n
a + 1
vi n 1.
Tm lim
n
a
n
.
2.1.30. Chng minh s hi t ca dy {a
n
} -c cho bi cng thc truy hi
a
1
= 2, a
n+1
= 2 +
1
3 +
1
an
vi n 1
v tm gii hn ca n.
2.1.31. Dy {a
n
} -c cho bi
a
1
= 1, a
2
= 2, a
n+1
=

a
n1
+

a
n
, vi n 2.
Chng minh dy trn b chn v tng ngt. Hy tm gii hn ca dy ny.
2.1.32. Dy {a
n
} -c xc nh theo cng thc truy hi
a
1
= 9, a
2
= 6, a
n+1
=

a
n1
+

a
n
, vi n 2.
Chng minh rng dy trn b chn v gim ngt. Tm gii hn ca dy ny.
2.1.33. Dy {a
n
} v {b
n
} -c cho bi cng thc
0 < b
1
< a
1
, a
n+1
=
a
n
+b
n
2
v b
n+1
=
_
a
n
b
n
vi n N.
Chng minh rng {a
n
} v {b
n
} cng tin ti mt gii hn. (Gii hn ny -c
gi l trung bnh cng - nhn ca a
1
v b
1
).
2.1.34. Chng minh rng c hai dy {a
n
} v {b
n
} xc nh theo cng thc
0 < b
1
< a
1
, a
n+1
=
a
2
n
+b
2
n
a
n
+b
n
v b
n+1
=
a
n
+b
n
2
vi n N
u n iu v c cng gii hn.
2.1.35. Hai dy truy hi {a
n
} v {b
n
} -c cho bi cng thc
0 < b
1
< a
1
, a
n+1
=
a
n
+b
n
2
v b
n+1
=
2a
n
b
n
a
n
+b
n
vi n N.
Chng minh tnh n iu ca hai dy trn v ch ra rng c hai dy u tin
ti trung bnh cng - nhn ca a
1
v b
1
. (Xem bi ton 2.1.33).

n

C
h
i
28 Ch-ng 2. Dy s thc
2.1.36. Chng minh s hi t v tm gii hn ca dy {a
n
} vi
a
n
=
n + 1
2
n+1
_
2
1
+
2
2
2
+... +
2
n
n
_
vi n N.
2.1.37. Gi s c mt dy b chn {a
n
} tho mn
a
n+2

1
3
a
n+1
+
2
3
a
n
vi n 1.
Chng minh rng dy trn hi t.
2.1.38. Cho {a
n
} v {b
n
} nh ngha bi:
a
n
=
_
1 +
1
n
_
n
, b
n
=
_
1 +
1
n
_
n+1
vi n N.
S dng bt ng thc lin h gia trung bnh cng, nhn v iu ho chng
minh rng
(a) a
n
< b
n
vi n N.
(b) dy {a
n
} tng ngt,
(c) dy {b
n
} gim ngt,
Chng minh rng {a
n
} v {b
n
} c cng gii hn, -c gi l s e ca Euler.
2.1.39. Cho
a
n
=
_
1 +
x
n
_
n
vi n N.
(a) Chng t rng nu x > 0 th dy {a
n
} b chn v tng ngt.
(b) Gi s x l mt s thc tu . Chng minh rng dy {a
n
} b chn v tng
ngt vi n > x.
e
x
-c nh ngha l gii hn ca dy ny.
2.1.40. Gi s c x > 0, l N v l > x. Chng minh rng dy {b
n
} vi
b
n
=
_
1 +
x
n
_
l+n
vi n N,
l dy gim ngt.

n

C
h
i
2.1. Dy n i u 29
2.1.41. Thit lp tnh n iu ca cc dy {a
n
} v {b
n
}, vi
a
n
= 1 +
1
2
+... +
1
n 1
lnn vi n N,
b
n
= 1 +
1
2
+... +
1
n 1
+
1
n
lnn vi n N.
Chng minh rng c hai dy trn cng tin n cng mt gii hn , gi l
hng s Euler.
Gi . S dng bt ng thc (1 +
1
n
)
n
< e < (1 +
1
n
)
n+1
, (suy ra t 2.1.38).
2.1.42. Cho x > 0 v t a
n
=
2
n

x, n N. Chng t rng dy {a
n
} b
chn. ng thi chng minh rng dy ny tng ngt nu x < 1 v gim ngt
nu x > 1. Tnh lim
n
a
n
.
Hn na, t
c
n
= 2
n
(a
n
1) v d
n
= 2
n
_
1
1
a
n
_
vi n N.
Chng minh rng {c
n
} l dy gim, cn {d
n
} l dy tng v c hai dy cng
c chung gii hn.

n

C
h
i
30 Ch-ng 2. Dy s thc
2.2 Gii hn. Tnh cht ca dy hi t
2.2.1. Tnh:
lim
n
n

1
2
+ 2
2
+... +n
2
, (a)
lim
n
n + sinn
2
n + cos n
, (b)
lim
n
1 2 + 3 4 +... + (2n)

n
2
+ 1
, (c)
lim
n
(

2
3

2)(

2
5

2)...(

2
2n+1

2), (d)
lim
n
n
2

n
, (e)
lim
n
n!
2
n
2
, (f)
lim
n
1

n
_
1

1 +

3
+
1

3 +

5
+... +
1

2n 1 +

2n + 1
_
, (g)
lim
n
_
1
n
2
+ 1
+
2
n
2
+ 2
+... +
n
n
2
+n
_
, (h)
lim
n
_
n
n
3
+ 1
+
2n
n
3
+ 2
+... +
nn
n
3
+n
_
. (i)
2.2.2. Cho s > 0 v p > 0. Chng minh rng
lim
n
n
s
(1 +p)
n
= 0.
2.2.3. Cho (0, 1), tnh
lim
n
((n + 1)

).
2.2.4. Cho Q, hy tnh
lim
n
sin(n!).
2.2.5. Chng minh rng khng tn ti lim
n
sinn.
2.2.6. Chng minh rng vi mi s v t , lim
n
sinn khng tn ti.

n

C
h
i
2.2. Gii hn. Tnh cht ca dy hi t 31
2.2.7. Vi R, hy tnh
lim
n
1
n
_
_
a +
1
n
_
2
+
_
a +
2
n
_
2
+... +
_
a +
n 1
n
_
2
_
.
2.2.8. Gi s a
n
= 1 vi mi n v lim
n
a
n
= 1. Cho k nguyn d-ng, hy
tnh
lim
n
a
n
+a
2
n
+... +a
k
n
k
a
n
1
.
2.2.9. Tnh
lim
n
_
1
1.2.3
+
1
2.3.4
+... +
1
n.(n + 1)(n + 2)
_
.
2.2.10. Tnh
lim
n
n

k=2
k
3
1
k
3
+ 1
.
2.2.11. Tnh
lim
n
n

i=1
i

j=1
j
n
3
.
2.2.12. Tnh
lim
n
_
1
2
2.3
__
1
2
3.4
_
...
_
1
2
(n + 1).(n + 2)
_
.
2.2.13. Tnh
lim
n
n

k=1
k
3
+ 6k
2
+ 11k + 5
(k + 3)!
.
2.2.14. Cho x = 1 v x = 1, hy tnh
lim
n
n

k=1
x
2
k1
1 x
2
k
.
2.2.15. Vi gi tr x R no th gii hn
lim
n
n

k=0
(1 +x
2
k
).
tn ti v tm gi tr ca gii hn ny.

n

C
h
i
32 Ch-ng 2. Dy s thc
2.2.16. Tm tt c x R sao cho gii hn
lim
n
n

k=0
_
1 +
2
x
2
k
+x
2
k
_
.
tn ti v tm gi tr ca gii hn ny.
2.2.17. Vi gi tr x R no thi gii hn
lim
n
n

k=1
(1 +x
3
k
+x
2.3
k
).
tn ti v tm gi tr ca gii hn ny.
2.2.18. Tnh
lim
n
1.1! + 2.2! +... +n.n!
(n + 1)!
.
2.2.19. Vi x R no sao cho ng thc sau
lim
n
n
1999
n
x
(n 1)
x
=
1
2000
-c thc hin
2.2.20. Cho a v b sao cho a b > 0, nh ngha dy {a
n
} nh- sau:
a
1
= a +b, a
n
= a
1

ab
a
n1
, n 2.
Hy xc nh s hng th n ca dy v tnh lim
n
a
n
.
2.2.21. nh ngha dy {a
n
} bi
a
1
= 0, a
2
= 1 v a
n+1
2a
n
+a
n1
= 2 vi n 2.
Hy xc nh s hng th n ca dy v tnh lim
n
a
n
.
2.2.22. Cho a > 0, b > 0, xt dy {a
n
} cho bi
a
1
=
ab

a
2
+b
2
v
a
n
=
aa
n1
_
a
2
+a
2
n1
, n 2.
Tm s hng th n ca dy v tnh lim
n
a
n
.

n

C
h
i
2.2. Gii hn. Tnh cht ca dy hi t 33
2.2.23. Cho dy truy hi {a
n
} nh ngha bi
a
1
= 0, a
n
=
a
n1
+ 3
4
, n 2.
Tm s hng th n v gii hn ca dy.
2.2.24. Hy xt tnh hi t ca dy cho bi
a
1
= a, a
n
= 1 +ba
n1
, n 2.
2.2.25. Ta inh ngha dy Fibonacci {a
n
} nh- sau:
a
1
= a
2
= 1, a
n+2
= a
n
+a
n+1
, n 1.
Chng minh rng
a
n
=

n

n

,
trong v l nghim ca ph-ng trnh x
2
= x + 1. Tnh lim
n
n

a
n
.
2.2.26. Cho hai dy {a
n
} v {b
n
} theo cng thc sau:
a
1
= a, b
1
= b,
a
n+1
=
a
n
+b
n
2
, b
n+1
=
a
n+1
+b
n
2
.
Chng minh rng lim
n
a
n
= lim
n
b
n
.
2.2.27. Cho a {1, 2, ..., 9}, hy tnh
lim
n
a +aa +... +
n s hng
..
aa...a
10
n
.
2.2.28. Tnh
lim
n
_
n

n 1
_
n
.
2.2.29. Gi s rng dy {a
n
} hi t ti 0. Hy tm lim
n
a
n
n
.
2.2.30. Cho p
1
, p
2
, ..., p
k
v a
1
, a
2
, ..., a
k
l cc s d-ng, tnh
lim
n
p
1
a
n+1
1
+p
2
a
n+1
2
+... +p
k
a
n+1
k
p
1
a
n
1
+p
2
a
n
2
+... +p
k
a
n
k
.

n

C
h
i
34 Ch-ng 2. Dy s thc
2.2.31. Gi s rng lim
n

a
n+1
an

= q. Chng minh rng:


(a) Nu q < 1 th lim
n
a
n
= 0,
(b) Nu q > 1 th lim
n
|a
n
| = .
2.2.32. Gi s c lim
n
n
_
|a
n
| = q. Chng minh rng:
(a) Nu q < 1 th lim
n
a
n
= 0,
(b) Nu q > 1 th lim
n
|a
n
| = .
2.2.33. Cho l mt s thc v x (0, 1), hy tnh
lim
n
n

x
n
.
2.2.34. Tnh
lim
n
m(m1) ... (mn + 1)
n!
x
n
, vi m N v |x| < 1.
2.2.35. Gi s lim
n
a
n
= 0 v {b
n
} mt dy b chn. Chng minh rng
lim
n
a
n
b
n
= 0.
2.2.36. Chng minh rng nu lim
n
a
n
= a v lim
n
b
n
= b th
lim
n
max {a
n
, b
n
} = max {a, b} .
2.2.37. Cho a
n
1 vi n N v lim
n
a
n
= 0. Cho p N, hy tm
lim
n
p

1 +a
n
.
2.2.38. Gi s c dy d-ng {a
n
} hi t ti 0. Cho s t nhin p 2, hy xc
nh
lim
n
p

1 +a
n
1
a
n
.

n

C
h
i
2.2. Gii hn. Tnh cht ca dy hi t 35
2.2.39. Cho cc s d-ng a
1
, a
2
, ..., a
p
, hy tnh
lim
n
_
p
_
(n +a
1
)(n +a
2
)...(n +a
p
) n
_
.
2.2.40. Tnh
lim
n
_
1

n
2
+ 1
+
1

n
2
+ 2
+... +
1

n
2
+n + 1
_
.
2.2.41. Cho a
1
, a
2
, ..., a
p
l cc s d-ng, hy tm
lim
n
n

a
n
1
+a
n
2
+... +a
n
p
p
.
2.2.42. Tnh
lim
n
n
_
2sin
2
n
1999
n + 1
+ cos
2
n
1999
n + 1
.
2.2.43. Tnh
lim
n
(n + 1 +ncos n)
1
2n+n sinn
.
2.2.44. Tnh
lim
n
n

k=1
_
_
1 +
k
n
2
1
_
.
2.2.45. Hy xc nh
lim
n
n

k=1
_
3
_
1 +
k
2
n
3
1
_
.
2.2.46. Cho cc s d-ng a
k
, k = 1, 2, ..., p, hy tnh
lim
n
_
1
p
p

k=1
n

a
k
_
p
.
2.2.47. Cho (0, 1). Hy tnh
lim
n
n1

k=0
_
+
1
n
_
k
.

n

C
h
i
36 Ch-ng 2. Dy s thc
2.2.48. Cho s thc x 1, hy chng t rng
lim
n
(2
n

x 1)
n
= x
2
.
2.2.49. Chng minh rng
lim
n
(2
n

n 1)
n
n
2
= 1.
2.2.50. Trong nhng dy d-i y, dy no l dy Cauchy ?
a
n
=
tan 1
2
+
tan 2
2
2
+... +
tan n
2
n
, (a)
a
n
= 1 +
1
4
+
2
2
4
2
+... +
n
2
4
n
, (b)
a
n
= 1 +
1
2
+
1
3
+... +
1
n
, (c)
a
n
=
1
1.2

1
2.3
+... + (1)
n1
1
n(n + 1)
, (d)
a
n
=
1
q
1
+
2
q
2
+... +
n
q
n
, (e)
vi |q| < 1, |
k
| M, k = 1, 2, ...,
a
n
=
1
2
2
+
2
3
2
+... +
n
(n + 1)
2
. (f)
2.2.51. Cho dy {a
n
} tho mn iu kin
|a
n+1
a
n+2
| < |a
n
a
n+1
|.
vi (0, 1). Chng minh rng {a
n
} hi t .
2.2.52. Cho dy {a
n
} cc s nguyn d-ng, nh ngha
S
n
=
1
a
1
+
1
a
2
+... +
1
a
n
v

n
=
_
1 +
1
a
1
__
1 +
1
a
2
_
...
_
1 +
1
a
n
_
.
Chng minh rng nu {S
n
} hi t th {ln
n
} cng hi t.
2.2.53. Chng minh rng nu dy {R
n
} hi t n mt s v t x (nh ngha
trong bi ton 1.1.20) th n l dy Cauchy.

n

C
h
i
2.3. nh l Toe plitz, nh l Stolz 37
2.2.54. Cho mt dy cp s cng {a
n
} vi cc s hng khc 0, hy tnh
lim
n
_
1
a
1
a
2
+
1
a
2
a
3
+... +
1
a
n
a
n+1
_
.
2.2.55. Cho mt dy cp s cng {a
n
} vi cc s hng d-ng, hy tnh
lim
n
1

n
_
1

a
1
+

a
2
+
1

a
2
+

a
3
+... +
1

a
n
+

a
n+1
_
.
2.2.56. Tnh
(a) lim
n
n(
n

e 1), (b) lim


n
e
1
n
+e
2
n
+... +e
n
n
n
.
2.2.57. Cho dy {a
n
} nh ngha nh- sau:
a
1
= a, a
2
= b, a
n+1
= pa
n1
+ (1 p)a
n
, n = 2, 3, ...
Xc nh xem vi gi tr a, b v p no th dy trn hi t.
2.2.58. Cho {a
n
} v {b
n
} nh ngha bi
a
1
= 3, b
1
= 2, a
n+1
= a
n
+ 2b
n
v b
n+1
= a
n
+b
n
.
Hn na cho
c
n
=
a
n
b
n
, n N.
Chng t rng |c
n+1

2| <
1
2
|c
n

2|, n N. (a)
Tnh lim
n
c
n
. (b)
2.3 nh l Toeplitz, nh l Stolz v ng dng
2.3.1. Chng minh nh l Toeplitz sau v php bin i chnh qui t dy sang
dy.

n

C
h
i
38 Ch-ng 2. Dy s thc
Cho {c
n,k
: 1 k n, n 1} l mt bng cc s thc tho mn:
c
n,k

n
0 vi mi k N, (i)
n

k=1
c
n,k

n
1, (ii)
tn ti hng s C > 0 sao cho vi mi s nguyn d-ng n th (iii)
n

k=1
|c
n,k
| C.
Khi vi mi dy hi t {a
n
} th dy bin i {b
n
} -c cho bi cng thc
b
n
=
n

k=1
c
n,k
a
k
, n 1, cng hi t v lim
n
b
n
= lim
n
a
n
.
2.3.2. Chng minh rng nu lim
n
a
n
= a th
lim
n
a
1
+a
2
+... +a
n
n
= a.
2.3.3.
(a) Chng minh rng gi thit (iii) trong nh l Toeplitz (bi ton 2.3.1) c
th b qua nu tt c c
n,k
l khng m.
(b) Cho {b
n
} l dy -c nh ngha trong nh l Toeplitz (xem bi 2.3.1) vi
c
n,k
> 0, 1 k n, n 1. Chng minh rng nu lim
n
a
n
= + th
lim
n
b
n
= +.
2.3.4. Chng minh rng nu lim
n
a
n
= + th
lim
n
a
1
+a
2
+... +a
n
n
= +.
2.3.5. Chng minh rng nu lim
n
a
n
= a th
lim
n
na
1
+ (n 1)a
2
+... + 1.a
n
n
2
=
a
2
.
2.3.6. Chng minh rng nu dy d-ng {a
n
} hi t ti a th
lim
n
n

a
1
...a
n
= a.

n

C
h
i
2.3. nh l Toe plitz, nh l Stolz 39
2.3.7. Cho dy d-ng {a
n
}, chng minh rng nu lim
n
a
n+1
an
= a th
lim
n
n

a
n
= a.
2.3.8. Cho lim
n
a
n
= a v lim
n
b
n
= b. Chng minh rng
lim
n
a
1
b
n
+a
2
b
n1
+... +a
n
b
1
n
= ab.
2.3.9. Cho {a
n
} v {b
n
} l hai dy tho mn
b
n
> 0, n N, v lim
n
(b
1
+b
2
+... +b
n
) = +, (i)
lim
n
a
n
b
n
= g. (ii)
Chng minh rng
lim
n
a
1
+a
2
+... +a
n
b
1
+b
2
+... +b
n
= g.
2.3.10. Cho {a
n
} v {b
n
} l hai dy tho mn
b
n
> 0, n N, v lim
n
(b
1
+b
2
+... +b
n
) = +, (i)
lim
n
a
n
= a. (ii)
Chng minh rng
lim
n
a
1
b
1
+a
2
b
2
+... +a
n
b
n
b
1
+b
2
+... +b
n
= a.
2.3.11. S dng cc kt qu ca bi tr-c, hy chng minh nh l Stolz.
Cho {x
n
} , {y
n
} l hai dy tho mn:
{y
n
} tng thc s ti +, (i)
lim
n
x
n
x
n1
y
n
y
n1
= g. (ii)
Khi
lim
n
x
n
y
n
= g.

n

C
h
i
40 Ch-ng 2. Dy s thc
2.3.12. Tnh
lim
n
1

n
_
1 +
1

2
+... +
1

n
_
, (a)
lim
n
n
a
n+1
_
a +
a
2
2
+... +
a
n
n
_
, a > 1, (b)
lim
n
1
n
k+1
_
k! +
(k + 1)!
1!
+... +
(k +n)!
n!
_
, k N, (c)
lim
n
1

n
_
1

n
+
1

n + 1
... +
1

2n
_
, (d)
lim
n
1
k
+ 2
k
+... +n
k
n
k+1
, k N, (e)
lim
n
1 + 1.a + 2.a
2
...na
n
na
n+1
, a > 1, (f)
lim
n
_
1
n
k
(1
k
+ 2
k
+... +n
k
)
n
k + 1
_
, k N. (g)
2.3.13. Gi s rng lim
n
a
n
= a. Tm
lim
n
1

n
_
a
1
+
a
2

2
+
a
3

3
+... +
a
n

n
_
.
2.3.14. Chng minh rng nu dy{a
n
} tho mn
lim
n
(a
n+1
a
n
) = a,
th
lim
n
a
n
n
= a.
2.3.15. Cho lim
n
a
n
= a. Hy tnh
lim
n
_
a
n
1
+
a
n1
2
+... +
a
1
2
n1
_
.
2.3.16. Gi s rng lim
n
a
n
= a. Hy tnh
lim
n
_
a
n
1.2
+
a
n1
2.3
+... +
a
1
n.(n + 1)
_
, (a)
lim
n
_
a
n
1

a
n1
2
1
+... + (1)
n1
a
1
2
n1
_
. (b)

n

C
h
i
2.3. nh l Toe plitz, nh l Stolz 41
2.3.17. Cho k l mt s t nhin c nh bt k ln hn 1. Hy tnh
lim
n
n

_
nk
n
_
.
2.3.18. Cho mt cp s cng d-ng {a
n
}, tnh
lim
n
n(a
1
...a
n
)
1
n
a
1
+a
2
+... +a
n
.
2.3.19. Cho dy {a
n
} sao cho dy {b
n
} vi b
n
= 2a
n
+ a
n1
, n 2, hi t
ti b. Hy xt tnh hi t ca {a
n
} .
2.3.20. Cho dy {a
n
} tho mn lim
n
n
x
a
n
= a vi s thc x no . Chng
minh rng
lim
n
n
x
(a
1
.a
2
...a
n
)
1
n
= ae
x
.
2.3.21. Tnh
lim
n
1 +
1
2
+... +
1
n1
+
1
n
lnn
, (a)
lim
n
1 +
1
3
+
1
5
+... +
1
2n1
lnn
. (b)
2.3.22. Gi s {a
n
} tin ti a. Chng minh rng
lim
n
1
lnn
_
a
1
1
+
a
2
2
+... +
a
n
n
_
= a.
2.3.23. Tnh
(a) lim
n
_
n!
n
n
e
n
_1
n
, (b) lim
n
_
(n!)
3
n
3n
e
n
_1
n
,
(c) lim
n
_
(n!)
2
n
2n
_1
n
, (d) lim
n
_
n
3n
(n!)
3
_1
n
,
(e) lim
n
k

n
n

n!
, k N.

n

C
h
i
42 Ch-ng 2. Dy s thc
2.3.24. Chng minh rng nu lim
n
a
n
= a th
lim
n
1
lnn
n

k=1
a
k
k
= a.
2.3.25. Cho dy {a
n
}, xt dy {A
n
} cc trung bnh cng A
n
=
a
1
+a
2
+...+an
n
.
Chng minh rng nu lim
n
A
n
= A th
lim
n
1
lnn
n

k=1
a
k
k
= A.
2.3.26. Chng minh iu ng-c li ca nh l Toeplitz trong 2.3.1.
Cho {c
n,k
: 1 k n, n 1} l mt bng s thc bt k. Nu vi mi
dy {a
n
} hi t bt k, dy bin i {b
n
} cho bi cng thc
b
n
=
n

k=1
c
n,k
a
k
, n 1
cng hi t n cng mt gii hn th
c
n,k

n
0 vi mi k N, (i)
n

k=1
c
n,k

n
1, (ii)
tn ti hng s C > 0 sao cho vi mi s nguyn d-ng n, ta c (iii)
n

k=1
|c
n,k
| C.
2.4 im gii hn. Gii hn trn v gii hn
d-i
2.4.1. Cho {a
n
} l dy tho mn {a
2k
} , {a
2k+1
} v {a
3k
} hi t.
(a) Chng minh rng dy {a
n
} cng hi t.
(b) Liu t s hi t ca hai trong ba dy con trn c suy ra -c s hi t ca
{a
n
}?

n

C
h
i
2.4. i m gii hn. Gii hn tr n v gii hn d-i 43
2.4.2. T s hi t ca tt c cc dy con ca dy {a
n
} d-i dng {a
s.n
} , s >
1, c suy ra -c s hi t ca {a
n
}?
2.4.3. Cho {a
pn
} , {a
qn
} , . . . , {a
sn
} l cc dy con ca dy {a
n
} sao cho
{p
n
} , {q
n
} , . . . , {s
n
} ri nhau tng cp v hp thnh dy {n}. Chng minh
rng nu S, S
p
, S
q
, . . . , S
s
t-ng ng l cc tp cc im gii hn
(1)
ca cc
dy {a
n
} , {a
pn
} , {a
qn
} , . . . , {a
sn
} th
S = S
p
S
q
... S
s
.
Chng minh rng nu mi dy con {a
pn
} , {a
qn
} , ..., {a
sn
} hi t ti a th dy
{a
n
} cng hi t ti a.
2.4.4. nh l trn (bi ton 2.4.3) c ng trong tr-ng hp s l-ng cc dy
con l v hn khng ?
2.4.5. Chng minh rng, nu mi dy con {a
n
k
} ca dy {a
n
} u cha mt
dy con
_
a
n
k
l
_
hi t ti a th dy {a
n
} cng hi t ti a.
2.4.6. Xc nh tp cc im gii hn ca dy {a
n
}, vi
a
n
=
_
4
(1)
n
+ 2, (a)
a
n
=
1
2
_
n 2 3
_
n 1
3
___
n 3 3
_
n 1
3
__
, (b)
a
n
=
(1 (1)
n
)2
n
+ 1
2
n
+ 3
, (c)
a
n
=
(1 + cos n) ln3n + lnn
ln2n
, (d)
a
n
=
_
cos
n
3
_
n
, (e)
a
n
=
2n
2
7

_
2n
2
7
_
. (f)
2.4.7. Tm tp hp cc im gii hn ca dy {a
n
} cho bi cng thc
a
n
= n [n], Q, (a)
a
n
= n [n], Q, (b)
a
n
= sinn, Q, (c)
a
n
= sinn, Q. (d)
(1)
Cn gi l cc gii hn ring hay cc im t ca dy.

n

C
h
i
44 Ch-ng 2. Dy s thc
2.4.8. Cho {a
k
} l mt dy sinh ra t cch nh s mt-mt bt k cc phn
t ca ma trn {
3

n
3

m} , n, m N. Chng minh rng mi s thc u


l im gii hn ca dy ny.
2.4.9. Gi s {a
n
} l dy b chn. Chng minh rng tp cc im gii hn
ca n l ng v b chn.
2.4.10. Xc nh lim
n
a
n
v lim
n
a
n
vi:
a
n
=
2n
2
7

_
2n
2
7
_
, (a)
a
n
=
n 1
n + 1
cos
n
3
, (b)
a
n
= (1)
n
n, (c)
a
n
= n
(1)
n
n
, (d)
a
n
= 1 +nsin
n
2
, (e)
a
n
=
_
1 +
1
n
_
n
(1)
n
+ sin
n
4
, (f)
a
n
=
n
_
1 + 2
n(1)
n
, (g)
a
n
=
_
2 cos
2n
3
_
n
, (h)
a
n
=
lnn (1 + cos n)n
ln2n
. (i)
2.4.11. Tm gii hn trn v gii hn d-i ca cc dy sau:
a
n
= n [n], Q, (a)
a
n
= n [n], Q, (b)
a
n
= sinn, Q, (c)
a
n
= sinn, Q. (d)
2.4.12. Vi dy {a
n
} bt k chng minh rng:
(a) nu tn ti k N sao cho vi mi n > k, bt ng thc a
n
A lun ng
th lim
n
a
n
A,
(b) nu vi mi k N tn ti n
k
> k a
n
k
A th lim
n
a
n
A,

n

C
h
i
2.4. i m gii hn. Gii hn tr n v gii hn d-i 45
(c) nu tn ti k N sao cho bt ng thc a
n
a ng vi mi n > k th
lim
n
a
n
a,
(d) nu vi mi k N tn ti n
k
> k sao cho a
n
k
a th lim
n
a
n
a.
2.4.13. Gi s dy {a
n
} tn ti gii hn trn v gii hn d-i hu hn. Chng
minh rng
(a) L = lim
n
a
n
khi v ch khi
(i) Vi mi > 0 tn ti k N sao cho a
n
< L + nu n > k v
(ii) Vi mi > 0 v k N tn ti n
k
> k sao cho L < a
n
k
(b) l = lim
n
a
n
khi v ch khi
(i) Vi mi > 0 tn ti k N sao cho a
n
> l nu n > k v
(ii) Vi mi > 0 v k N tn ti n
k
> k sao cho a
n
k
< l +
Hy pht biu nhng khng ng t-ng ng cho gii hn trn v gii hn trong
tr-ng hp v hn.
2.4.14. Gi s tn ti mt s nguyn n
0
sao cho vi n n
0
, a
n
b
n
. Chng
minh rng
lim
n
a
n
lim
n
b
n
, (a)
lim
n
a
n
lim
n
b
n
. (b)
2.4.15. Chng minh cc bt ng thc sau (tr tr-ng hp bt nh +
v +):
lim
n
a
n
+ lim
n
b
n
lim
n
(a
n
+b
n
) lim
n
a
n
+ lim
n
b
n
lim
n
(a
n
+b
n
) lim
n
a
n
+ lim
n
b
n
.
Hy -a ra mt s v d sao cho du trong cc bt ng thc trn -c
thay bng du < .
2.4.16. Cc bt ng thc sau
lim
n
a
n
+ lim
n
b
n
lim
n
(a
n
+b
n
),
lim
n
(a
n
+b
n
) lim
n
a
n
+ lim
n
b
n
.

n

C
h
i
46 Ch-ng 2. Dy s thc
c ng trong tr-ng hp c v hn dy khng ?
2.4.17. Ly {a
n
} v {b
n
} l cc dy s khng m. Chng minh rng (tr
tr-ng hp 0.(+) v (+).0) cc bt ng thc sau:
lim
n
a
n
lim
n
b
n
lim
n
(a
n
b
n
) lim
n
a
n
lim
n
b
n
lim
n
(a
n
b
n
) lim
n
a
n
lim
n
b
n
.
Hy -a ra mt s v d sao cho du trong cc bt ng thc trn -c
thay bng du < .
2.4.18. Chng minh rng iu kin cn v mt dy {a
n
} hi t l c
gii hn trn v gii hn d-i hu hn v
lim
n
a
n
= lim
n
a
n
.
Chng minh rng bi ton vn ng cho tr-ng hp cc dy phn k ti
v +.
2.4.19. Chng minh rng nu lim
n
a
n
= a, a R th
lim
n
(a
n
+b
n
) = a + lim
n
b
n
,
lim
n
(a
n
+b
n
) = a + lim
n
b
n
,
2.4.20. Chng minh rng nu lim
n
a
n
= a, a R, a > 0, v tn ti mt s
nguyn d-ng n
0
sao cho b
n
0 vi n n
0
, khi
lim
n
(a
n
.b
n
) = a. lim
n
b
n
,
lim
n
(a
n
.b
n
) = a. lim
n
b
n
,
2.4.21. Chng minh rng
lim
n
(a
n
) = lim
n
a
n
, lim
n
(a
n
) = lim
n
a
n
.
2.4.22. Chng minh rng vi dy s d-ng {a
n
} ta c
lim
n
_
1
a
n
_
=
1
lim
n
a
n
,

n

C
h
i
2.4. i m gii hn. Gii hn tr n v gii hn d-i 47
lim
n
_
1
a
n
_
=
1
lim
n
(a
n
)
.
( y
1
+
= 0,
1
0
+
= +.)
2.4.23. Chng minh rng nu dy {a
n
} l dy s d-ng tho mn
lim
n
(a
n
) lim
n
_
1
a
n
_
= 1,
th dy {a
n
} hi t.
2.4.24. Chng minh rng nu {a
n
} l dy tho mn vi bt k dy {b
n
} ,
lim
n
(a
n
+b
n
) = lim
n
a
n
+ lim
n
b
n
,
v
lim
n
(a
n
+b
n
) = lim
n
a
n
+ lim
n
b
n
.
th dy {a
n
} hi t.
2.4.25. Chng minh rng, nu {a
n
} l mt dy d-ng tho mn vi bt k dy
d-ng {b
n
},
lim
n
(a
n
b
n
) = lim
n
a
n
lim
n
b
n
.
hoc
lim
n
(a
n
.b
n
) = lim
n
a
n
lim
n
b
n
,
v vy {a
n
} hi t.
2.4.26. Chng minh rng vi bt k dy d-ng {a
n
},
lim
n
a
n+1
a
n
lim
n
n

a
n
lim
n
n

a
n
lim
n
a
n+1
a
n
.
2.4.27. Cho dy {a
n
} , ly dy {b
n
} xc nh nh- sau
b
n
=
1
n
(a
1
+a
2
+... +a
n
), n N.
Chng minh rng
lim
n
a
n
lim
n
b
n
lim
n
b
n
lim
n
a
n
.

n

C
h
i
48 Ch-ng 2. Dy s thc
2.4.28. Chng minh rng
lim
n
(max {a
n
, b
n
}) = max
_
lim
n
a
n
, lim
n
b
n
_
, (a)
lim
n
(min{a
n
, b
n
}) = min
_
lim
n
a
n
, lim
n
b
n
_
, (b)
Kim tra cc bt ng thc sau:
lim
n
(min{a
n
, b
n
}) = min
_
lim
n
a
n
, lim
n
b
n
_
, (a)
lim
n
(max {a
n
, b
n
}) = max
_
lim
n
a
n
, lim
n
b
n
_
(d)
c ng khng?
2.4.29. Chng minh rng mi dy s thc u cha mt dy con n iu.
2.4.30. S dng kt qu bi tr-c chng minh nh l Bolzano-Weierstrass:
Mi dy s thc b chn u cha mt dy con hi t.
2.4.31. Chng minh rng vi mi dy s d-ng {a
n
},
lim
n
a
1
+a
2
+... +a
n
+a
n+1
a
n
4.
Chng minh rng 4 l nh gi tt nht.
2.5 Cc bi ton hn hp
2.5.1. Chng minh rng nu lim
n
a
n
= + hay lim
n
a
n
= th
lim
n
_
1 +
1
a
n
_
an
= e.
2.5.2. Vi x R chng minh rng
lim
n
_
1 +
x
n
_
n
= e
x
.

n

C
h
i
2.5. Cc bi ton hn hp 49
2.5.3. Vi x > 0 hy kim chng bt ng thc
x
x + 2
< ln(x + 1) < x.
(S dng o hm ) chng minh rng bt ng thc tri c th mnh hn nh-
sau
x
x + 2
<
2x
x + 2
< ln(x + 1), x > 0.
2.5.4. Chng minh rng
lim
n
n(
n

a 1) = lna, a > 0, (a)


lim
n
n(
n

n 1) = +. (b)
2.5.5. Ly {a
n
} l dy s d-ng vi cc s hng khc 1, chng minh rng nu
lim
n
a
n
= 1 th
lim
n
lna
n
a
n
1
= 1.
2.5.6. Ly
a
n
= 1 +
1
1!
+
1
2!
+... +
1
n!
, n N.
Chng minh rng
lim
n
a
n
= e v 0 < e a
n
<
1
nn!
.
2.5.7. Chng minh rng
lim
n
_
1 +
x
1!
+
x
2
2!
+... +
x
n
n!
_
= e
x
.
2.5.8. Chng minh rng
lim
n
_
1
n
+
1
n + 1
+... +
1
2n
_
= ln2,
(a)
lim
n
_
1
_
n(n + 1)
+
1
_
(n + 1)(n + 2)
+... +
1
_
2n(2n + 1)
_
= ln2.
(b)

n

C
h
i
50 Ch-ng 2. Dy s thc
2.5.9. Tm gii hn ca dy {a
n
} ,trong
a
n
=
_
1 +
1
n
2
__
1 +
2
n
2
_
...
_
1 +
n
n
2
_
, n N.
2.5.10. Ly {a
n
} l dy -c xc nh qui np nh- sau
a
1
= 1, a
n
= n(a
n1
+ 1) vi n = 2, 3, ...
Tnh
lim
n
n

k=1
_
1 +
1
a
k
_
.
2.5.11. Chng minh rng lim
n
(n!e [n!e]) = 0.
2.5.12. Cho cc s d-ng a v b, chng minh rng
lim
n
_
n

a +
n

b
2
_
n
=

ab.
2.5.13. Cho {a
n
} v {b
n
} l cc dy d-ng tha mn
lim
n
a
n
n
= a, lim
n
b
n
n
= b, trong a, b > 0,
v gi s cc s d-ng p, q tha mn p +q = 1. Chng minh rng
lim
n
(pa
n
+qb
n
)
n
= a
p
b
q
.
2.5.14. Cho hai s thc a v b, xc nh dy {a
n
} nh- sau
a
1
= a, a
2
= b, a
n+1
=
n 1
n
a
n
+
1
n
a
n1
, n 2.
Tm lim
n
a
n
.
2.5.15. Cho {a
n
} l mt dy -c xc nh nh- sau
a
1
= 1, a
2
= 2, a
n+1
= n(a
n
+a
n1
), n 2.
Tm cng thc hin ca cc s hng tng qut ca dy.

n

C
h
i
2.5. Cc bi ton hn hp 51
2.5.16. Cho a v b xc nh {a
n
} nh- sau
a
1
= a, a
2
= b, a
n+1
=
1
2n
a
n1
+
2n 1
2n
a
n
, n 2.
Tm lim
n
a
n
.
2.5.17. Cho
a
n
= 3
n

k=1
1
k(k + 1)(k + 1)!
, n N.
(a) Chng minh rng lim
n
a
n
= e.
(b) Chng minh rng 0 < a
n
e <
1
(n+1)(n+1)!
.
2.5.18. Tnh lim
n
nsin(2n!e).
2.5.19. Gi s {a
n
} l dy tho mn a
n
< n, n = 1, 2, ..., v lim
n
a
n
= +.
Hy xt tnh hi t ca dy
_
1
a
n
n
_
n
, n = 1, 2, ....
2.5.20. Gi s dy {b
n
} d-ng hi t ti +. Xt tnh hi t ca dy
_
1 +
b
n
n
_
n
, n = 1, 2, ....
2.5.21. Cho dy truy hi {a
n
} nh ngha nh- sau
0 < a
1
< 1, a
n+1
= a
n
(1 a
n
), n 1,
chng minh rng
lim
n
na
n
= 1, (a)
lim
n
n(1 a
n
)
lnn
= 1, (b)
2.5.22. Xt dy truy hi {a
n
} nh- sau
0 < a
1
< , , a
n+1
= sina
n
, n 1.
Chng minh rng lim
n

na
n
=

3.

n

C
h
i
52 Ch-ng 2. Dy s thc
2.5.23. Cho
a
1
= 1, a
n+1
= a
n
+
1
n

k=1
a
k
, n 1.
Chng minh rng
lim
n
a
n

2 lnn
= 1.
2.5.24. Cho {a
n
} nh- sau
a
1
> 0, a
n+1
= arctan a
n
, n 1,
tnh lim
n
a
n
.
2.5.25. Chng minh rng dy qui
0 < a
1
< 1, a
n+1
= cos a
n
, n 1,
hi t ti nghim duy nht ca ph-ng trnh x = cos x.
2.5.26. nh ngha dy {a
n
} nh- sau
a
1
= 0, a
n+1
= 1 sin(a
n
1), n 1
Tnh
lim
n
1
n
n

k=1
a
k
.
2.5.27. Cho {a
n
} l dy cc nghim lin tip ca ph-ng trnh tanx =
x, x > 0. Tm lim
n
(a
n+1
a
n
).
2.5.28. Cho |a|

2
v a
1
R. Nghin cu tnh hi t ca dy {a
n
} cho bi
cng thc sau:
a
n+1
= a sina
n
, n 1.
2.5.29. Cho a
1
> 0, xt dy {a
n
} cho bi
a
n+1
= ln(1 +a
n
), n 1.
Chng minh rng
lim
n
na
n
= 2, (a)
lim
n
n(na
n
2)
lnn
=
2
3
. (b)

n

C
h
i
2.5. Cc bi ton hn hp 53
2.5.30. Cho dy {a
n
} nh- sau
a
1
= 0 v a
n+1
=
_
1
4
_
an
, n 1.
Hy nghin cu tnh hi t ca dy.
2.5.31. Cho a
1
> 0, nh ngha dy {a
n
} nh- sau:
a
n+1
= 2
1an
, n 1.
Kho st tnh hi t ca dy.
2.5.32. Tm gii hn ca dy cho bi
a
1
=

2, a
n+1
= 2
an
2
, n 1.
2.5.33. Chng minh rng nu lim
n
(a
n
a
n2
) = 0 th
lim
n
a
n
a
n1
n
= 0.
2.5.34. Chng minh rng nu vi dy d-ng {a
n
} bt k tho mn
lim
n
n
_
1
a
n+1
a
n
_
tn ti (hu hn hoc v hn) th
lim
n
ln
1
an
lnn
cng tn ti v c hai gii hn bng nhau.
2.5.35. Cho a
1
, b
1
(0, 1), Chng minh rng dy {a
n
} v {b
n
} cho bi cng
thc
a
n+1
= a
1
(1 a
n
b
n
) +a
n
, b
n+1
= b
1
(1 a
n
b
n
) +b
n
, n 1
hi t v tm gii hn ca chng.
2.5.36. Cho a v a
1
d-ng, xt dy {a
n
} nh- sau
a
n+1
= a
n
(2 na
n
), n = 1, 2, ...
Kho st s hi t ca dy.

n

C
h
i
54 Ch-ng 2. Dy s thc
2.5.37. Chng minh rng nu a
1
v a
2
l hai s d-ng v
a
n+2
=

a
n
+

a
n+1
, n = 1, 2, ...
th dy {a
n
} hi t. Tm gii hn ca dy.
2.5.38. Gi s f : R
k
+
R

l mt hm tng vi mi bin v tn ti a > 0


sao cho
f(x, x, ..., x) > x vi 0 < x < a,
f(x, x, ..., x) < x vi x > a.
Cho cc s d-ng a
1
, a
2
, . . . , a
k
, nh ngha dy truy hi {a
n
} nh- sau:
a
n
= f(a
n1
, a
n2
, ..., a
nk
), vi n > k.
Chng minh rng lim
n
a
n
= a.
2.5.39. Cho a
1
v a
2
l hai s d-ng. Xt tnh hi t ca dy {a
n
} -c nh
ngha truy hi nh- sau
a
n+1
= a
n
e
ana
n1
vi n 1.
2.5.40. Cho a > 1 v x > 0, nh ngha {a
n
} bi a
1
= a
x
, a
n+1
=
a
an
, n N. Hy xt tnh hi t ca dy.
2.5.41. Chng minh rng
_
2 +
_
2 +... +

2
. .
n - cn
= 2 cos

2
n+1
.
S dng kt qu trn tnh gii hn ca dy truy hi cho bi
a
1
=

2, a
n+1
=

2 +a
n
, n 1.
2.5.42. Cho {
n
} l dy sao cho cc s hng ch nhn mt trong ba gi tr
1, 0, 1. Thit lp cng thc

1
_
2 +
2
_
2 + +
n

2 = 2 sin
_

4
n

k=1

2
...
k
2
k1
_
, n N.
v chng t rng dy
a
n
=
1
_
2 +
2
_
2 + +
n

2
hi t.

n

C
h
i
2.5. Cc bi ton hn hp 55
2.5.43. Tnh
lim
n
_
arctan
1
2
+ arctan
1
2.2
2
+... + arctan
1
2n
2
_
.
2.5.44. Tnh lim
n
sin(

n
2
+n).
2.5.45. Xt tnh hi t ca dy truy hi d-i y
a
1
=

2, a
2
=
_
2 +

3, a
n+2
=
_
2 +

3 +a
n
vi n 1.
2.5.46. Chng minh rng
lim
n

_
1 + 2

1 + 3
_
1 +...
_
1 + (n 1)

1 +n = 3.
2.5.47. Cho a > 0, cho dy {a
n
} bi
a
1
< 0, a
n+1
=
a
a
n
1 vi n N.
Chng minh rng dy trn hi t ti nghim m ca ph-ng trnh x
2
+x = a.
2.5.48. Cho a > 0, xt dy {a
n
} :
a
1
> 0, a
n+1
=
a
a
n
+ 1
vi n N.
Chng minh rng dy hi t ti nghim d-ng ca ph-ng trnh x
2
+x = a.
2.5.49. Cho {a
n
} l dy truy hi cho bi cng thc sau
a
1
= 1, a
n+1
=
2 +a
n
1 +a
n
vi n N.
Chng minh rng {a
n
} l dy Cauchy v tm gii hn ca n.
2.5.50. Chng minh rng dy nh ngha bi
a
1
> 0, a
n+1
= 2 +
1
a
n
, n N,
l dy Cauchy v tm gii hn ca dy.

n

C
h
i
56 Ch-ng 2. Dy s thc
2.5.51. Cho a > 0, nh ngha {a
n
} nh- sau:
a
1
= 0 a
n+1
=
a
2 +a
n
vi n N.
Hy xt tnh hi t ca dy {a
n
} .
2.5.52. Gi s rng a
1
R v a
n+1
= |a
n
2
1n
| vi n N. Hy xt tnh
hi t ca dy v trong tr-ng hp hi t hy tm gii hn .
2.5.53. Chng minh rng
(a) Nu 0 < a < 1 th
lim
n
n1

j=1
ja
j
n j
= 0,
(b) Nu 0 < a < 1 th
lim
n
na
n
n

j=1
1
ja
j
=
1
1 a
,
(c) Nu b > 1 th
lim
n
n
b
n
n

j=1
b
j1
j
=
1
b 1
.
2.5.54. Tnh
lim
n
_
sin

n + 1
+ sin

n + 2
+... + sin

2n
_
.
2.5.55. Tnh
lim
n
n

k=1
_
1 +
k
2
cn
3
_
, vi c > 0, (a)
lim
n
n

k=1
_
1
k
2
cn
3
_
, vi c > 1. (b)
2.5.56. Xc nh
lim
n

n
3n
n!
n

k=1
sin
k
n

n
.

n

C
h
i
2.5. Cc bi ton hn hp 57
2.5.57. Cho dy {a
n
} nh ngha theo cng thc sau:
a
n
=
n

k=0
_
n
k
_
1
, n 1,
Chng minh rng lim
n
a
n
= 2.
2.5.58. Tm gi tr sao cho dy
a
n
=
_
1
_
1
n
_

__
1
_
2
n
_

_
...
_
1
_
n 1
n
_

_
, n 2,
hi t.
2.5.59. Vi x R, nh ngha {x} = x [x]. Tnh lim
n
_
(2 +

3)
n
_
.
2.5.60. Cho {a
n
} l mt dy d-ng v t S
n
= a
1
+ a
2
+ ... + a
n
, n 1.
Gi s ta c
a
n+1

1
S
n+1
((S
n
1)a
n
+a
n1
), n > 1.
Hy tnh lim
n
a
n
.
2.5.61. Cho {a
n
} l dy d-ng tho mn
lim
n
a
n
n
= 0, lim
n
a
1
+a
2
+... +a
n
n
< .
Tnh
lim
n
a
2
1
+a
2
2
+... +a
2
n
n
2
.
2.5.62. Xt hai dy d-ng {a
n
} v {b
n
} tho mn
lim
n
a
n
a
1
+ a
2
+... +a
n
= 0 lim
n
b
n
b
1
+b
2
+... +b
n
= 0.
nh ngha dy {c
n
} nh- sau:
c
n
= a
1
b
n
+a
2
b
n1
+... +a
n
b
1
, n N.
Chng minh rng
lim
n
c
n
c
1
+c
2
+... +c
n
= 0.

n

C
h
i
58 Ch-ng 2. Dy s thc
2.5.63. Tnh
lim
n
_
1 +
1
n
_
n
2
e
n
.
2.5.64. Gi s dy {a
n
} b chn trn v tho mn iu kin
a
n+1
a
n
>
1
n
2
, n N.
Hy thit lp s hi t ca dy {a
n
} .
2.5.65. Gi s dy {a
n
} b chn tho mn iu kin
a
n+1
2
n

2 a
n
, n N.
Hy thit lp s hi t ca dy {a
n
} .
2.5.66. K hiu l v L t-ng ng l gii hn d-i v gii hn trn ca dy
{a
n
} . Chng minh rng nu lim
n
(a
n+1
a
n
) = 0 th mi im trong khong
m (l, L) l im gii hn ca {a
n
} .
2.5.67. K hiu l v L t-ng ng l gii hn d-i v gii hn trn ca dy
{a
n
} . Gi s rng vi mi n, a
n+1
a
n
>
n
, vi
n
> 0 v lim
n

n
= 0.
Chng minh rng mi im trong khong m (l, L) l im gii hn ca {a
n
} .
2.5.68. Cho {a
n
} l dy d-ng v n iu tng. Chng minh rng tp cc
im gii hn ca dy
a
n
n +a
n
, n N,
l mt khong, khong ny suy bin thnh mt im trong tr-ng hp hi t.
2.5.69. Cho a
1
R, xt dy {a
n
} nh- sau:
a
n+1
=
_
an
2
nu n chn,
1+an
2
nu n l.
Tm cc im gii hn ca dy trn.
2.5.70. Liu 0 c phi l mt im gii hn ca dy {

n sinn} ?
2.5.71. Chng minh rng vi dy d-ng {a
n
} ta c
lim
n
_
a
1
+a
n+1
a
n
_
n
e.

n

C
h
i
2.5. Cc bi ton hn hp 59
2.5.72. Chng minh kt qu tng qut ca bi ton trn: Cho s nguyn d-ng
p v dy d-ng {a
n
}, Chng minh rng
lim
n
_
a
1
+a
n+p
a
n
_
n
e
p
.
2.5.73. Chng minh vi dy d-ng {a
n
} ta c
lim
n
n
_
1 +a
n+1
a
n
1
_
1.
Chng minh 1 l hng s tt nht c th -c ca bt ng thc trn.
2.5.74. Cho
a
n
=
_
1 +
_
1 +... +

1
. .
n - cn
Tm lim
n
a
n
.
2.5.75. Cho {a
n
} l dy vi cc phn t ln hn 1 . Gi s ta c
lim
n
lnlna
n
n
= ,
Xt dy {b
n
} nh- sau:
b
n
=
_
a
1
+
_
a
2
+... +

a
n
, n N.
Chng minh rng nu < ln2 th {b
n
} hi t, ng-c li nu < ln2 th dy
phn k ti .
2.5.76. Gi s cc s hng ca dy ca dy {a
n
} tho mn iu kin
0 a
n+m
a
n
+a
m
vi n, m R.
Chng minh rng gii hn lim
n
an
n
tn ti.
2.5.77. Gi s cc s hng ca dy ca dy {a
n
} tho mn iu kin
0 a
n+m
a
n
a
m
vi n, m R.
Chng minh rng gii hn lim
n
n

a
n
tn ti.

n

C
h
i
60 Ch-ng 2. Dy s thc
2.5.78. Gi s cc s hng ca dy ca dy {a
n
} tho mn iu kin
|a
n
| 1,
a
n
+a
m
1 a
n+m
a
n
+a
m
+ 1
vi n, m N.
(a) Chng minh rng gii hn lim
n
an
n
tn ti.
(b) Chng minh rng nu gii hn lim
n
an
n
= g th
ng 1 a
n
ng + 1 vi n N.
2.5.79. Cho {a
n
} l dy d-ng v n iu tng tho mn iu kin
a
n.m
na
m
vi n, m N.
Chng minh rng nu sup
_
an
n
: n N
_
< + th dy
_
an
n
_
hi t.
2.5.80. Cho hai s d-ng a
1
v a
2
, chng minh dy truy hi {a
n
} cho bi
a
n+2
=
2
a
n+1
+a
n
vi n N
hi t.
2.5.81. Cho b
1
a
1
> 0, xt hai dy {a
n
} v {b
n
} cho bi cng thc truy
hi:
a
n+1
=
a
n
+b
n
2
, b
n+1
=
_
a
n+1
b
n
vi n N.
Chng minh rng c hai dy u hi t ti cng mt gii hn.
2.5.82. Cho a
k,n
, b
k,n
, n N, k = 1, 2, ..., n, l hai bng tam gic cc s thc
vi b
k,n
= 0. Gi s rng
a
k,n
b
k,n

n
1 u i vi k, c ngha l vi mi > 0,
lun tn ti mt s d-ng n
0
sao cho

a
k,n
b
k,n
1

<
vi mi n > n
0
v k = 1, 2, ..., n. Chng minh rng nu lim
n
n

k=1
b
k,n
tn ti
th
lim
n
n

k=1
a
k,n
= lim
n
n

k=1
b
k,n
.

n

C
h
i
2.5. Cc bi ton hn hp 61
2.5.83. Cho a = 0, tm
lim
n
n

k=1
sin
(2k 1)a
n
2
.
2.5.84. Vi a > 0, tnh
lim
n
n

k=1
_
a
k
n
2
1
_
.
2.5.85. Tnh
lim
n
n

k=1
_
1 +
k
n
2
_
.
2.5.86. Vi p = 0 v q > 0, hy tnh
lim
n
n

k=1
_
_
1 +
k
q1
n
q
_1
p
1
_
.
2.5.87. Cho cc s d-ng a, b v d vi b > a, tnh
lim
n
a(a +d)...(a +nd)
b(b +d)...(b +nd)
.

n

C
h
i

n

C
h
i
Ch-ng 3
Chui s thc
Tm tt l thuyt
Cho chui hnh thc

n=1
a
n
. (A)
a
n
-c gi l s hng th n hay s hng tng qut ca chui (A).
Dy cc tng ring ca chui (A) -c nh ngha l
s
n
=
n

k=1
a
k
, n N.
s
n
-c gi l tng ring th n ca chui (A).
Ni rng chui (A) hi t v c tng bng s, nu
lim
n
s
n
= s.
Trong tr-ng hp ny, phn d- ca chui (A) -c nh ngha l
r
n
= s s
n
=

k=n+1
a
k
, n N.
Ni rng chui (A) phn k, nu gii hn ni trn khng tn ti
63

n

C
h
i
64 Ch-ng 3. Chui s thc
iu kin cn chui (A) hi t l
lim
n
a
n
= 0.
iu kin cn v chui (A) hi t l: vi > cho tr-c, tn
ti n

N sao cho

n+p

k=n
a
k

< , n > n

, p N.
(A) -c gi l chui d-ng nu a
n
0 vi mi n.
Tiu chun so snh. Cho hai chui d-ng (A) v (B)

n=1
b
n
. (B)
Gi s
a
n
b
n
n N.
Khi ,
nu chui (B) hi t, th chui (A) cng hi t;
nu chui (A) phn k, th chui (B) cng phn k.
c bit, nu
lim
n
a
n
b
n
= k = 0,
th hai chui (A), (B) cng hi t hoc cng phn k.
Tiu chun t s (D'Alembert). Cho chui d-ng (A).
Nu
lim
n
a
n+1
a
n
< 1,
th chui (A) hi t.
Nu
lim
n
a
n+1
a
n
> 1,
th chui (A) phn k.

n

C
h
i
Tm tt l thuy t 65
c bit, gi s tn ti gii hn
a = lim
n
a
n+1
a
n
,
khi , nu a < 1 th chui (A) hi t; nu a > 1 th chui (A) phn k.
Tiu chun cn (Cachy). Cho chui d-ng (A). Gi s tn ti gii
hn
c = lim
n
n

a
n
,
khi , nu c < 1 th chui (A) hi t; nu c > 1 th chui (A) phn k.
Tiu chun Raabe. Cho chui d-ng (A).
Nu
lim
n
n
_
a
n
a
n+1
1
_
> 1,
th chui (A) hi t.
Nu
lim
n
n
_
a
n
a
n+1
1
_
< 1,
th chui (A) phn k.
c bit, gi s tn ti gii hn
r = lim
n
n(
a
n
a
n+1
1)
khi , nu r > 1 th chui (A) hi t; nu r < 1 th chui (A) phn k.
Ni rng chui (A) hi t tuyt i, nu chui (gm cc tr s
tuyt i)

n=1
|a
n
|
hi tu.
Chui hi t tuyt i th hi t. iu ng-c li, ni chung, khng ng.
Ni rng chui (A) hi t c iu kin hay bn hi t, nu
chui n hi t nh-ng khng hi t tuyt i.
Chui an du l chui c dng
b
1
b
2
+b
3
+ (1)
n1
+ , b
n
0.

n

C
h
i
66 Ch-ng 3. Chui s thc
Tiu chun Leibniz ni rng, nu dy s {b
n
} n iu gim v hi
t v 0 th chui an du hi t.
Php bin i Abel Cho hai chui bt k (A) v (B). t
A
n
=
n

k=1
a
k
, B
n
=
n

k=1
b
k
, C
n
=
n

k=1
a
k
b
k
.
Khi ta c
C
n
= a
n
B
n

n1

k=1
(a
k+1
a
k
)B
k
.
Tiu chun Abel. Cho hai chui bt k (A) v (B). X t chui (C)
nh- sau

n=1
a
n
b
n
. (C)
Nu chui (B) hi t v dy {a
n
} n iu v b chn th chui (C) hi t.
Tiu chun Dirichlet. Nu dy {A
n
} b chn, dy {b
n
} n iu
v c gii hn bng 0 th chui (C) hi t.

n

C
h
i
3.1. Tng ca chui 67
3.1 Tng ca chui
3.1.1. Tm cc chui v tng ca chng nu dy {S
n
} cc tng ring ca chng
-c cho nh- sau:
(a) S
n
=
n + 1
n
, n N, (b) S
n
=
2
n
1
2
n
, n N,
(c) S
n
= arctan n, n N, (d) S
n
=
(1)
n
n
, n N,
3.1.2. Tm tng ca cc chui
(a)

n=1
2n + 1
n
2
(n + 1)
2
, (b)

n=1
n
(2n 1)
2
(2n + 1)
2
,
(c)

n=1
n

n
2
1
_
n(n + 1)
, (d)

n=1
1
4n
2
1
,
(e)

n=1
1
(

n +

n + 1)
_
n(n + 1)
.
3.1.3. Tnh cc tng sau
ln
1
4
+

n=1
ln
(n + 1)(3n + 1)
n(3n + 4)
, (a)

n=1
ln
(2n + 1)n
(n + 1)(2n 1)
. (b)
3.1.4. Tm tng ca cc chui

n=1
1
n(n + 1) . . . (n +m)
, m N, (a)

n=1
1
n(n +m)
, m N, (b)

n=1
n
2
(n + 1)(n + 2)(n + 3)(n + 4)
. (c)
3.1.5. Tnh
(a)

n=1
sin
n!
720
, (b)

n=1
1
n
_
lnn
n lnn
_
.

n

C
h
i
68 Ch-ng 3. Chui s thc
3.1.6. Tnh

n=1
sin
1
2
n+1
cos
3
2
n+1
.
3.1.7. Tm

n=0
1
n!(n
4
+n
2
+ 1)
.
3.1.8. Chng minh rng

n=1
n
3 5 . . . (2n + 1)
=
1
2
.
3.1.9. Gi s {a
n
} l mt dy tho mn
lim
n
((a
1
+ 1)(a
2
+ 1) . . . (a
n
+ 1)) = g, 0 < g +.
Chng minh rng

n=1
a
n
(a
1
+ 1)(a
2
+ 1) . . . (a
n
+ 1)
= 1
1
g
.
( qui -c:
1

= 0).
3.1.10. Dng kt qu trong bi ton tr-c, tm tng ca cc chui

n=1
n 1
n!
, (a)

n=1
2n 1
2 4 6 . . . 2n
, (b)

n=2
1
n
2
_
1
1
2
2
_ _
1
1
3
2
_
...
_
1
1
n
2
_. (c)
3.1.11. Gi {a
n
} l dy cho bi
a
1
> 2, a
n+1
= a
2
n
2 vi n N.
Chng minh rng

n=1
1
a
1
a
2
. . . a
n
=
a
1

a
1
2
4
2
.

n

C
h
i
3.1. Tng ca chui 69
3.1.12. Vi b > 2, kim tra rng

n=1
n!
b(b + 1) . . . (b +n 1)
=
1
b 2
.
3.1.13. Cho a > 0 v b > a + 1, chng minh ng thc

n=1
a(a + 1) . . . (a +n 1)
b(b + 1) . . . (b +n 1)
=
a
b a 1
.
3.1.14. Cho a > 0 v b > a + 2, kim tra ng thc sau

n=1
a(a + 1) . . . (a +n 1)
b(b + 1) . . . (b +n 1)
=
a(b 1)
(b a 1)(b a 2)
.
3.1.15. Cho

n=1
1
an
l chui phn k vi cc s hng d-ng. Cho tr-c b > 0,
tm tng

n=1
a
1
a
2
. . . a
n
(a
2
+b)(a
3
+b) . . . (a
n+1
+b)
.
3.1.16. Tnh

n=0
(1)
n
cos
3
3
n
x
3
n
.
3.1.17. Cho cc hng s khc khng a, b v c, gi s cc hm f v g tho mn
iu kin f(x) = af(bx) +cg(x).
(a) Chng minh rng nu lim
n
a
n
f(b
n
x) = L(x) tn ti th

n=0
a
n
g(b
n
x) =
f(x) L(x)
c
.
(b) Chng minh rng nu lim
n
a
n
f(b
n
x) = M(x) tn ti th

n=0
a
n
g(b
n
x) =
M(x) af(bx)
c
.

n

C
h
i
70 Ch-ng 3. Chui s thc
3.1.18. Dng ng nht thc sinx = 3 sin
x
3
4 sin
3 x
3
, chng minh rng

n=0
3
n
sin
3
x
3
n+1
=
x sinx
4
, (a)

n=0
1
3
n
sin
3
x
3
n+1
=
3
4
sin
x
3
. (b)
3.1.19. Dng ng nht thc cot x = 2 cot(2x) + tanx vi x = k

2
, k Z,
chng minh rng

n=0
1
2
n
tan
x
2
n
=
1
x
2 cot(2x).
3.1.20. Dng ng nht thc arctan x = arctan(bx) + arctan
(1b)x
1+bx
2
, thit
lp cc cng thc sau:

n=0
arctan
(1 b)b
n
x
1 +b
2n+1
x
2
= arctan x vi 0 < b < 1, (a)

n=0
arctan
(b 1)b
n
x
1 +b
2n+1
x
2
= arccot x vi x = 0 v b > 1. (b)
3.1.21. Cho {a
n
} l dy Fibonacci -c xc nh bi
a
0
= a
1
= 1, a
n+1
= a
n
+a
n1
, n 1
v t S
n
=
n

k=0
a
2
k
. Tm

n=0
(1)
n
S
n
.
3.1.22. Vi dy Fibonacci {a
n
} trong bi trn, tnh

n=0
(1)
n
a
n
a
n+2
.
3.1.23. Vi dy Fibonacci {a
n
} trong bi trn, xc nh tng

n=1
arctan
1
a
2n
.

n

C
h
i
3.1. Tng ca chui 71
3.1.24. Tm tng
(a)

n=1
arctan
2
n
2
, (b)

n=1
arctan
1
n
2
+n + 1
,
(c)

n=1
arctan
8n
n
4
2n
2
+ 5
.
3.1.25. Cho {a
n
} l dy d-ng phn k ti v cng. Chng minh rng

n=1
arctan
a
n+1
a
n
1 +a
n
a
n+1
= arctan
1
a
1
.
3.1.26. Chng minh rng vi bt k hon v no ca cc s hng ca chui
d-ng, tng ca chui nhn -c khng thay i.
3.1.27. Chng minh ng nht thc

n=1
1
(2n 1)
2
=
3
4

n=1
1
n
2
.
3.1.28. Chng minh rng

n=1
1
n
2
=

2
6
, (a)

n=1
1
n
4
=

4
90
, (b)

n=0
(1)
n
1
2n + 1
=

4
(c)
3.1.29. Cho dy {a
n
} -c xc nh bi
a
1
= 2, a
n+1
= a
2
n
a
n
+ 1 vi n 1.
Tm

n=1
1
an
.
3.1.30. Cho dy {a
n
} -c xc nh nh- sau
a
1
> 0, a
n+1
= ln
e
an
1
a
n
vi n 1,
v t b
n
= a
1
a
2
. . . a
n
. Tm

n=1
b
n
.

n

C
h
i
72 Ch-ng 3. Chui s thc
3.1.31. Cho dy {a
n
} -c xc nh bi
a
1
= 1, a
n+1
=
1
a
1
+a
2
+. . . +a
n

2 vi n 1.
Tm tng ca chui

n=1
a
n
.
3.1.32. Tm tng ca cc chui sau

n=1
(1)
n1
1
n
, (a)

n=1
(1)
n1
2n + 1
n(n + 1)
, (b)

n=1
_
1
x + 2n 1
+
1
x + 2n

1
x +n
_
, x = 1, 2, . . . . (c)
3.1.33. Tnh

n=1
(1)
n1
ln
_
1 +
1
n
_
.
3.1.34. Tnh

n=1
(1)
n1
ln
_
1
1
(n + 1)
2
_
.
3.1.35. Xc nh tng ca cc chui

n=1
_
1
n
ln
_
1 +
1
n
__
.
3.1.36. Gi s hm f kh vi trn (0, +), sao cho o hm f

ca n n
iu trn mt khong con (a, +), v lim
x
f

(x) = 0. Chng minh rng gii


hn
lim
n+
_
1
2
f(1) +f(2) +f(3) +. . . +f(n 1) +
1
2
f(n)
_
n
1
f(x)dx
_
tn ti. Xt cc tr-ng hp c bit ca khi hm f(x) c dng f(x) =
1
x
v
f(x) = lnx.

n

C
h
i
3.1. Tng ca chui 73
3.1.37. Xc nh tng ca chui

n=1
(1)
n
lnn
n
.
3.1.38. Tm

n=1
_
nln
2n + 1
2n 1
1
_
.
3.1.39. Cho tr-c s nguyn k 2, chng minh rng chui

n=1
_
1
(n 1)k + 1
+
1
(n 1)k + 2
+. . . +
1
nk 1

x
nk
_
hi t i vi duy nht mt gi tr ca x. Tm gi tr ny v tng ca chui.
3.1.40. Cho dy {a
n
} -c xc nh bi
a
0
= 2, a
n+1
= a
n
+
3 + (1)
n
2
,
tnh

n=0
(1)
[
n+1
2
]
1
a
2
n
1
.
3.1.41. Chng minh rng tng ca cc chui
(a)

n=1
1
n!
, (b)

n=1
1
(n!)
2
l v t.
3.1.42. Cho {
n
} l dy vi
n
nhn hai gi tr 1 hoc 1. Chng minh rng
tng ca chui

n=1
n
n!
l s v t.
3.1.43. Chng minh rng vi mi s nguyn d-ng k , tng ca chui

n=1
(1)
n
(n!)
k
l v t.

n

C
h
i
74 Ch-ng 3. Chui s thc
3.1.44. Gi s rng {n
k
} l dy n iu tng cc s nguyn d-ng sao cho
lim
k
n
k
n
1
n
2
. . . n
k1
= +.
Chng minh rng

i=1
1
n
i
l v t.
3.1.45. Chng minh rng nu {n
k
} l dy cc s nguyn d-ng tho mn
lim
k
n
k
n
1
n
2
. . . n
k1
= + v lim
k
n
k
n
k1
> 1,
th

i=1
1
n
i
l v t.
3.1.46. Gi s rng {n
k
} l dy n iu tng cc s nguyn d-ng sao cho
lim
k
2
k

n
k
= . Chng minh rng

k=1
1
n
k
l v t.
3.1.47. Gi s chui

n=1
pn
qn
, p
n
, q
n
N l chui hi t v gi s
p
n
q
n
1

p
n+1
q
n+1
1

p
n
q
n
.
K hiu A l tp tt c cc s n sao cho bt ng thc trn c du > . Chng
minh rng

n=1
pn
qn
v t khi v ch khi A l v hn.
3.1.48. Chng minh rng vi mi dy tng ngt cc s nguyn d-ng {n
k
},
tng ca chui

k=1
2
n
k
n
k
!
l v t.

n

C
h
i
3.2. Chui d-ng 75
3.2 Chui d-ng
3.2.1. Cc chui sau hi t hay phn k
(a)

n=1
(

n
2
+ 1
3

n
3
+ 1), (b)

n=1
(
n
2
+ 1
n
2
+n + 1
)
n
2
,
(c)

n=2
(2n 3)!!
(2n 2)!!
, (d)

n=1
(
n
n + 1
)
n(n+1)
,
(e)

n=1
_
1 cos
1
n
_
, (f)

n=1
(
n

n 1)
n
,
(g)

n=1
(
n

a 1), a > 1.
3.2.2. Kim tra s hi t ca cc chui sau y
(a)

n=1
1
n
ln
_
1 +
1
n
_
, (b)

n=2
1

n
ln
n + 1
n 1
,
(c)

n=1
1
n
2
lnn
, (d)

n=2
1
(lnn)
ln n
,
(e)

n=2
1
(lnn)
lnlnn
.
3.2.3. Cho

n=1
a
n
,

n=1
b
n
l cc chui d-ng tho mn
a
n+1
a
n

b
n+1
b
n
vi n n
0
.
Chng minh rng nu

n=1
b
n
hi t, th

n=1
a
n
cng hi t.
3.2.4. Kim tra s hi t ca cc chui sau y
(a)

n=1
n
n2
e
n
n!
, (b)

n=1
n
n
e
n
n!
.

n

C
h
i
76 Ch-ng 3. Chui s thc
3.2.5. Tm gi tr ca cc chui sau hi t
(a)

n=1
_
n

a 1
_

, a > 1, (b)

n=1
_
n

n 1
_

,
(c)

n=1
_
_
1 +
1
n
_
n+1
e
_

, (d)

n=1
_
1 nsin
1
n
_

.
3.2.6. Chng minh rng nu chui d-ng

n=1
a
n
hi t th

n=1
(a
an
1) , a > 1
cng hi t.
3.2.7. Kho st s hi t ca cc chui sau
(a)

n=1
ln
_
cos
1
n
_
, (b)

n=1
e
alnn+b
c lnn+d
, a, b, c, d R
(c)

n=1
n
2n
(n +a)
(n+b)
(n +b)
(n+a)
, a, b > 0.
3.2.8. Gi s chui

n=1
a
n
vi cc s hng khng m hi t. Chng minh rng

n=1

a
n
a
n+1
cng hi t. Chng minh rng iu ng-c li l khng ng, tuy
nhin nu dy {a
n
} n iu gim th iu ng-c li ng.
3.2.9. Gi s rng chui d-ng

n=1
a
n
phn k. Nghin cu s hi t cc chui
sau y
(a)

n=1
a
n
1 +a
n
, (b)

n=1
a
n
1 +na
n
,
(c)

n=1
a
n
1 +n
2
a
n
, (d)

n=1
a
n
1 +a
2
n
.

n

C
h
i
3.2. Chui d-ng 77
3.2.10. Gi s chui d-ng

n=1
a
n
phn k, k hiu dy cc tng ring ca n
l {S
n
} . Chng minh rng

n=1
a
n
S
n
phn k,
v

n=1
a
n
S
2
n
hi t.
3.2.11. Chng minh rng vi cc gi thit nh- ca bi tr-c, chui

n=2
a
n
S
n
S

n1
hi t vi mi > 0.
3.2.12. Chng minh rng cc gi thit cho bi tp 3.2.10 , chui

n=2
a
n
S

n
hi t nu > 1 v phn k nu 1.
3.2.13. Cho chui

n=1
a
n
hi t, k hiu r
n
=

k=n+1
a
k
, n N l dy cc
phn d- ca n. Chng minh rng

n=2
a
n
r
n1
phn k, (a)

n=2
a
n

r
n1
hi t. (b)
3.2.14. Chng minh rng vi cc gi thit -c cho bi tr-c , chui

n=2
a
n
r

n1
hi t nu < 1 v phn k nu 1.

n

C
h
i
78 Ch-ng 3. Chui s thc
3.2.15. Chng minh rng vi gi thit nh- bi 3.2.13, chui

n=1
a
n+1
ln
2
r
n
hi t.
3.2.16. Cho chui d-ng

n=1
a
n
. Gi s rng
lim
n
nln
a
n
a
n+1
= g.
Chng minh rng

n=1
a
n
hi t nu g > 1 v phn k nu g < 1 (k c tr-ng
hp g = + v g = ). Hy -a v d chng t rng khi g = 1 th ta
khng th -a ra kt lun -c.
3.2.17. Nghin cu s hi t ca cc chui sau y
(a)

n=1
1
2

n
, (b)

n=1
1
2
lnn
,
(c)

n=1
1
3
ln n
, (d)

n=1
1
a
lnn
, a > 0,
(e)

n=2
1
a
lnlnn
, a > 0.
3.2.18. Kho st s hi t ca chui

n=1
a
1+
1
2
+...+
1
n
, a > 0.
3.2.19. Dng kt qu ca bi ton 3.2.16, chng minh dng gii hn ca Tiu
chun Raabe.
Cho a
n
> 0, n N, t
lim
n
n
_
a
n
a
n+1
1
_
= r.
Chng minh rng

n=1
a
n
hi t nu r > 1 v phn k nu r < 1.
3.2.20. Cho dy {a
n
} -c xc nh bi
a
1
= a
2
= 1, a
n+1
= a
n
+
1
n
2
a
n1
vi n 2.
Nghin cu s hi t ca chui

n=1
1
an
.

n

C
h
i
3.2. Chui d-ng 79
3.2.21. Cho a
1
v l cc s d-ng. Dy {a
n
} -c xc nh nh- sau
a
n+1
= a
n
e
a

n
, vi n = 1, 2, . . . .
Hy xc nh v chui

n=1
a

n
hi t.
3.2.22. Xc nh a chui

n=1
n!
(a + 1)(a + 2) . . . (a +n)
hi t.
3.2.23. Cho a l s d-ng tu v {b
n
} l dy s d-ng hi t ti b. Nghin
cu s hi t ca chui

n=1
n!a
n
(a +b
1
)(2a +b
2
) . . . (na +b
n
)
.
3.2.24. Chng minh rng nu dy cc s d-ng {a
n
} tho mn
a
n+1
a
n
= 1
1
n


n
nlnn
,
trong
n
> 1, th

n=1
a
n
hi t. Mt khc, nu
a
n+1
a
n
= 1
1
n


n
nlnn
,
trong
n
< 1, th

n=1
a
n
phn k. (Tiu chun Bertrand.)
3.2.25. Dng tiu chun Bertrand v Raabe chng minh tiu chun Gauss.
Nu dy cc s d-ng {a
n
} tho mn
a
n+1
a
n
= 1

n


n
n

,
trong > 1, v {
n
} l dy b chn, th

n=1
a
n
hi t khi > 1 v phn
k nu 1.

n

C
h
i
80 Ch-ng 3. Chui s thc
3.2.26. Kho st s hi t ca chui

n=1
( + 1) . . . ( +n 1)
n!

( + 1) . . . ( +n 1)
( + 1) . . . ( +n 1)
y , v l cc hng s d-ng.
3.2.27. Tm gi tr ca p chui

n=1
_
(2n 1)!!
(2n)!!
_
p
hi t.
3.2.28. Chng minh tiu chun c c ca Cauchy.
Cho {a
n
} l dy n iu gim cc s khng m. Chng minh rng chui

n=1
a
n
hi t khi v ch khi chui

n=1
2
n
a
2
n hi t.
3.2.29. Kim tra s hi t ca cc chui sau y
(a)

n=2
1
n(lnn)

, (b)

n=3
1
n lnn lnlnn
.
3.2.30. Chng minh nh l Schlomilch (suy rng ca nh l Cauchy, xem
bi tp 3.2.28).
Nu {g
k
} l dy tng ngt cc s nguyn d-ng sao cho vi c > 0 no v
vi mi k N, g
k+1
g
k
c(g
k
g
k1
) v vi dy d-ng {a
n
} gim ngt,
ta c

n=1
a
n
< khi v ch khi

n=1
(g
k+1
g
k
)a
g
k
< .
3.2.31. Cho {a
n
} l dy n iu gim cc s d-ng. Chng minh chui

n=1
a
n
hi t khi v ch khi cc chui sau hi t
(a)

n=1
3
n
a
3
n, (b)

n=1
na
n
2, (c)

n=1
n
2
a
n
3,
(d) S dng tiu chun trn hy nghin cu s hi t ca cc chui trong bi
tp 3.2.17.

n

C
h
i
3.2. Chui d-ng 81
3.2.32. Gi s {a
n
} l dy d-ng. Chng minh rng chui

n=1
a
n
hi t nu
lim
n
(a
n
)
1
lnn
<
1
e
.
3.2.33. Gi s {a
n
} l dy d-ng.Chng minh rng
lim
n
(na
n
)
1
lnlnn
<
1
e
ko theo s hi t ca

n=1
a
n
.
3.2.34. Cho {a
n
} l dy d-ng, n iu gim tho mn
2
n
a
2
n
a
n
g < 1.
Chng minh rng

n=1
a
n
hi t.
3.2.35. Cho {a
n
} l dy khng m, n iu gim. Chng minh rng nu

n=1
a
n
hi t, th lim
n
na
n
= 0. Chng minh rng y khng l iu kin
cho s hi t ca chui.
3.2.36. Hy nu mt v d chui d-ng hi t nh-ng iu kin lim
n
na
n
= 0
khng tho mn.
3.2.37. Gi s

n=1
a
n
l chui d-ng hi t. Tm iu kin cn v tn
ti dy d-ng {b
n
} sao cho cc chui

n=1
b
n
v

n=1
a
n
b
n
u hi t.
3.2.38. Tn ti hay khng mt dy d-ng {a
n
} sao cho cc chui

n=1
b
n
v

n=1
1
n
2
a
n
u hi t.

n

C
h
i
82 Ch-ng 3. Chui s thc
3.2.39. Chng minh rng

n=1
1
n

1 +a
n+1
a
n
phn k vi mi dy d-ng {a
n
}.
3.2.40. Gi s {a
n
} v {b
n
} n iu gim ti khng sao cho cc chui

n=1
a
n
v

n=1
b
n
phn k. C th ni g v s hi t ca chui

n=1
c
n
, trong c
n
=
min{a
n
, b
n
}?
3.2.41. Cho {a
n
} l dy n iu gim, khng m sao cho

n=1
an
n
phn k.
Gi s rng
b
n
= min
_
a
n
,
1
ln(n + 1)
_
.
Chng minh rng chui

n=1
bn
n
cng phn k.
3.2.42. Cho {a
n
} l dy d-ng , b chn v n iu tng. Chng minh rng

n=1
_
1
a
n
a
n+1
_
hi t.
3.2.43. Cho {a
n
} l dy d-ng, tng v phn k ra v cc. Chng minh rng
:

n=1
_
1
a
n
a
n+1
_
phn k.
3.2.44. Cho {a
n
} l dy d-ng n iu tng. Chng t rng vi mi > 0
ta c

n=1
_
a
n+1
a
n
a
n+1
a

n
_
hi t.
3.2.45. Chng t rng vi chui d-ng phn k

n=1
a
n
bt k, tn ti mt dy
{c
n
} n iu gim ti 0 sao cho

n=1
a
n
c
n
phn k.

n

C
h
i
3.2. Chui d-ng 83
3.2.46. Chng t rng vi chui d-ng hi t

n=1
a
n
bt k, tn ti mt dy
{c
n
} n iu tng ra v cc sao cho

n=1
a
n
c
n
hi t.
3.2.47. Cho

n=1
a
n
l mt chui d-ng hi t v k hiu {r
n
} l dy phn d-
ca n. Chng minh rng nu

n=1
r
n
hi t th
lim
n
na
n
= 0.
3.2.48. Cho {a
n
} l dy d-ng, phn k ra v cc. C th ni g v s hi t
ca cc chui sau:
(a)

n=1
1
a
n
n
, (b)

n=1
1
a
lnn
n
, (c)

n=1
1
a
lnlnn
n
?
3.2.49. Nghin cu s hi t ca chui

n=1
a
n
, y :
a
1
= 1, a
n+1
= cos a
n
vi n N.
3.2.50. Cho p l mt s khng m c nh. Nghin cu s hi t ca chui

n=1
a
n
, y :
a
1
= 1, a
n+1
= n
p
sina
n
vi n N.
3.2.51. Cho {a
n
} l dy cc nghim d-ng lin tip ca ph-ng trnh tan x =
x. Nghin cu s hi t ca chui

n=1
1
a
2
n
.
3.2.52. Cho {a
n
} l dy cc nghim d-ng lin tip ca ph-ng trnh
tan

x = x. Nghin cu s hi t ca chui

n=1
1
an
.
3.2.53. Cho a
1
l mt s d-ng tu v a
n+1
= ln(1 +a
n
) vi n 1. Nghin
cu s hi t ca chui

n=1
a
n
.
3.2.54. Cho dy d-ng n iu gim {a
n
} sao cho chui

n=1
a
n
phn k.
Chng minh rng:
lim
n
a
1
+a
3
+. . . +a
2n1
a
2
+a
4
+. . . +a
2n
= 1.

n

C
h
i
84 Ch-ng 3. Chui s thc
3.2.55. Cho S
k
= 1 +
1
2
+
1
3
+. . . +
1
k
v k hiu k
n
l s nguyn d-ng nh
nht S
k
n. Tm
lim
n
k
n+1
k
n
.
3.2.56. Cho A l tp tt c cc s nguyn d-ng sao cho trong biu din thp
phn ca chng khng cha ch s 0.
(a) Chng t rng

nA
1
n
hi t.
(b) Tm tt c cc gi tr sao cho

nA
1
n

hi t.
3.2.57. Cho

n=1
a
n
l mt chui s vi cc s hng d-ng v cho
lim
n
ln
1
an
lnn
= g.
Chng minh rng nu g > 1 th chui hi t, cn nu g < 1 th chui phn k
( y g c th bng ).
Cho v d chng t rng trong tr-ng hp g = 1 th ch-a th c kt lun g.
3.2.58. Chng t rng tiu chun Raabe (xem 3.2.19) v tiu chun cho trong
bi tp 3.2.16 l t-ng -ng. Hn na, chng t rng khng nh trong bi
tp trn l mnh hn cc tiu chun .
3.2.59. Nghin cu s hi t ca chui

n=1
a
n
vi cc s hng -c cho bi :
a
1
=

2, a
n
=

_
2
_
2 +
_
2 +. . . +

2
. .
(n 1) - cn
, n 2.
3.2.60. Cho {a
n
} l mt dy n iu gim ti 0. Chng t rng nu dy s
c s hng tng qut l
(a
1
a
n
) + (a
2
a
n
) +. . . + (a
n1
a
n
)
b chn th chui

n=1
a
n
phi hi t.

n

C
h
i
3.2. Chui d-ng 85
3.2.61. Tm chui s c s hng a
n
tho mn cc iu kin sau:
a
1
=
1
2
, a
n
= a
n+1
+a
n+2
+. . . vi n = 1, 2, 3, . . . .
3.2.62. Gi s cc s hng ca mt chui hi t

n=1
a
n
c tng S tho mn cc
iu kin sau:
a
1
a
2
a
3
. . . v 0 < a
n
a
n+1
+a
n+2
+. . . , n N.
Chng t rng c th biu din tt c cc s s bt k trong khong na ng
(0, S] bi mt tng hu hn cc s hng ca chui

n=1
a
n
hoc bi mt chui
con v hn

k=1
a
n
k
, y {a
n
k
} l mt dy con ca {a
n
}.
3.2.63. Gi s

n=1
a
n
l mt chui c cc s hng d-ng n iu gim. Chng
t rng nu mi s trong khong (0, S), S l tng chui, u c th biu din bi
mt tng hu hn cc s hng ca {a
n
} hoc bi mt chui con v hn

k=1
a
n
k
, trong {a
n
k
} l mt dy con ca {a
n
}, th bt ng thc sau ng:
a
n
a
n+1
+a
n+2
+. . . , vi mi n N.
3.2.64. Cho mt chui d-ng

n=1
a
n
phn k, gi thit rng lim
n
an
Sn
= 0,
trong S
n
= a
1
+a
2
+. . . +a
n
. Chng minh rng:
lim
n
a
1
S
1
1
+a
2
S
1
2
+. . . +a
n
S
1
n
lnS
n
= 1.
3.2.65. S dng bi tp trn chng minh rng
lim
n
1 +
1
2
+
1
3
+. . . +
1
n
lnn
= 1.
3.2.66. Cho

n=1
a
n
l mt chui hi t vi cc s hng d-ng. C th ni g v
s hi t ca

n=1
a
1
+a
2
+. . . +a
n
n
?

n

C
h
i
86 Ch-ng 3. Chui s thc
3.2.67. Chng minh rng nu {a
n
} l mt dy d-ng sao cho
1
n
n

k=1
a
k

2n

k=n+1
a
k
vi n N, th

n=1
a
n
2ea
1
.
3.2.68. Chng minh bt ng thc Carleman:
Nu {a
n
} l mt dy d-ng v chui

n=1
a
n
hi t th

n=1
n

a
1
. . . a
n
< e

n=1
a
n
.
3.2.69. Chng minh rng nu {a
n
} l dy s d-ng th vi mi s nguyn
d-ng k

n=1
n

a
1
a
2
. . . a
n

1
k

n=1
a
n
_
n +k
n
_
n
.
3.2.70. Cho {a
n
} l dy s d-ng. Chng minh rng t s hi t ca chui

n=1
1
an
suy ra s hi t ca chui

n=1
_
_
n
2
a
n
_
n

k=1
a
k
_
2
_
_
.
3.2.71. Cho {a
n
} l dy s d-ng n iu tng sao cho

n=1
1
an
phn k. Chng
minh rng chui

n=2
1
na
n
(n 1)a
n1
cng phn k.
3.2.72. Cho {p
n
} l dy tt c cc s nguyn t lin tip. Hy nghin cu s
hi t ca chui

n=1
1
pn
.
3.2.73. Nghin cu s hi t ca chui

n=2
1
np
n
(n 1)p
n1
,
trong p
n
l s nguyn t th n.

n

C
h
i
3.2. Chui d-ng 87
3.2.74. Hy nh gi gii hn
lim
n

k=2
1
k
n+1

k=2
1
k
n
.
3.2.75. Cho dy s {a
n
} tho mn iu kin:
0 a
n
1 vi mi n N v a
1
= 0.
t
S
n
= a
1
+a
2
+. . . +a
n
v T
n
= S
1
+S
2
+. . . +S
n
.
Hy xc nh cc gi tr > 0 sao cho chui

n=1
an
T

n
hi t.
3.2.76. Cho k l mt s nguyn d-ng tu . Gi s {a
n
} l dy s d-ng n
iu tng sao cho chui

n=1
1
an
hi t. Chng minh rng hai chui

n=1
ln
k
a
n
a
n
v

n=1
ln
k
n
a
n
cng hi t hoc cng phn k.
3.2.77. Gi s f : N (0, ) l hm gim v : N N l hm tng sao
cho (n) > n vi mi n N. Hy kim tra cc bt ng thc sau:
(n)1

k=1
f(k) <
(1)1

k=1
f(k) +
n1

k=1
f((k))((k + 1) (k)), (1)
(n)

k=(1)1
f(k) >
n

k=2
f((k))((k) (k 1)). (2)
3.2.78. Vi gi thit ca bi trn, chng minh rng nu tn ti s q sao cho
vi mi n N bt ng thc sau
f((n))((n + 1) (n))
f(n)
q < 1

n

C
h
i
88 Ch-ng 3. Chui s thc
ng th chui

n=1
f(n) hi t. Mt khc, nu
f((n))((n) (n 1))
f(n)
1, n N
th chui

n=1
f(n) phn k.
3.2.79. Suy ra t bi trn du hiu sau v s hi t v phn k ca chui s
d-ng.
Chui s d-ng

n=1
a
n
vi cc s hng n iu gim s hi t nu
lim
n
a
2n
a
n
= g <
1
2
v phn k nu
lim
n
a
2n
a
n
= g >
1
2
.
3.2.80. Suy ra t bi 3.2.78 du hiu sau v s hi t v phn k ca chui s
d-ng (so snh vi bi 3.2.34).
Chui s d-ng

n=1
a
n
vi cc s hng n iu gim s hi t nu
lim
n
2
n
a
2
n
a
n
= g < 1
v phn k nu
lim
n
2
n
a
2
n
a
n
> 2.
3.2.81. S dng bi 3.2.77, chng minh tiu chun trong bi 3.2.31.
3.2.82. Chng minh du hiu Kummer.
Cho {a
n
} l dy s d-ng.
(1) Nu tn ti mt dy d-ng {b
n
} v mt hng s d-ng c sao cho
b
n
a
n
a
n+1
b
n+1
c vi mi n N,
th chui

n=1
a
n
hi t.

n

C
h
i
3.2. Chui d-ng 89
(2) Nu tn ti mt dy d-ng {b
n
} sao cho chui

n=1
1
bn
phn k v
b
n
a
n
a
n+1
b
n+1
0 vi mi n N,
th chui

n=1
a
n
phn k.
3.2.83. Chng minh cc du hiu d'Alembert, Raabe (3.2.19) v Bertrand
(3.2.24) u l tr-ng hp ring ca du hiu Kummer (3.2.82).
3.2.84. Chng minh chiu ng-c li ca du hiu Kummer.
Cho {a
n
} l dy s d-ng.
(1) Nu chui

n=1
a
n
hi t th tn ti mt dy d-ng {b
n
} v mt hng s
d-ng c sao cho
b
n
a
n
a
n+1
b
n+1
c.
(2) Nu chui

n=1
a
n
phn k th tn ti mt dy d-ng {b
n
} sao cho chui

n=1
1
bn
phn k v
b
n
a
n
a
n+1
b
n+1
0.
3.2.85. Chng minh cc du hiu sau v s hi t v phn k ca chui s
d-ng.
(a) Cho k l mt s nguyn d-ng v lim
n
a
n+k
an
= g. Nu g < 1 th chui

n=1
a
n
hi t, v nu g > 1 th chui

n=1
a
n
phn k.
(b) Cho k l mt s nguyn d-ng v lim
n
n
_
an
a
n+k
1
_
= g. Nu g > k th
chui

n=1
a
n
hi t, v nu g < k th chui

n=1
a
n
phn k.
3.2.86. Cho hai dy s d-ng {a
n
} v {
n
}. Gi s rng
n
= O(
1
ln n
). Chng
minh rng nu chui

n=2
a
n
hi t th chui

n=2
a
1n
n
cng hi t.

n

C
h
i
90 Ch-ng 3. Chui s thc
3.3 Du hiu tch phn
3.3.1. Chng minh du hiu tch phn.
Gi s f l mt hm d-ng v n iu gim trn on [1, ). Khi chui

n=1
f(n) hi t khi v ch khi dy {I
n
} b chn, trong I
n
=
n
_
1
f(x)dx.
3.3.2. Cho f l hm d-ng v kh vi trn khong (0, ) sao cho f

n iu
gim ti 0. Chng minh rng hai chui

n=1
f

(n) v

n=1
f

(n)
f(n)
cng hi t hoc cng phn k.
3.3.3. Cho f l hm d-ng v n iu gim trn [1, ). t
S
N
=
N

n=1
f(n) v I
N
=
_
N
1
f(x)dx.
Chng minh rng dy {S
N
I
N
} n iu gim v c gii hn thuc vo on
[0, f(1)].
3.3.4. Chng minh rng gii hn ca cc dy sau
1 +
1
2
+. . . +
1
n
lnn, (a)
1 +
1
2

+. . . +
1
n


_
n
1
1
x

dx, 0 < < 1, (b)


u thuc vo khong (0, 1).
3.3.5. S dng du hiu tch phn, hy nghin cu s hi t ca chui cho
trong bi 3.2.29.
3.3.6. Cho

n=1
a
n
l mt chui d-ng phn k v S
n
= a
1
+a
2
+. . . +a
n
> 1
vi n 1. Hy kim tra cc kt qu sau:

n=1
a
n+1
S
n
lnS
n
phn k, (a)

n=1
a
n
S
n
ln
2
S
n
hi t. (b)

n

C
h
i
3.3. Du hi u tch phn 91
3.3.7. Cho f l hm d-ng v n iu gim trn [1, ). Gi s hm tng
ngt, kh vi v tho mn (x) > x vi x > 1. Chng minh rng nu tn ti
q < 1 sao cho

(x)f((x))
f(x)
q khi x ln th chui

n=1
f(n) hi t. Ng-c
li, nu

(x)f((x))
f(x)
1 khi x ln, th chui

n=1
f(n) phn k.
3.3.8. Cho f, g l cc hm d-ng, kh vi lin tc trn khong (0, ). Gi s
hm f n iu gim. Chng minh rng:
(a) Nu lim
x
_
g(x)
f

(x)
f(x)
g

(x)
_
> 0 th chui

n=1
f(n) hi t.
(b) Nu dy
_
n
_
1
1
g(x)
dx
_
khng b chn v g(x)
f

(x)
f(x)
g

(x) 0 khi x
ln th chui

n=1
f(n) phn k.
3.3.9. Cho f l hm d-ng, kh vi lin tc trn khong (0, ). Chng minh
rng:
(a) Nu lim
x
_

xf

(x)
f(x)
_
> 1 th chui

n=1
f(n) hi t.
(b) Nu
xf

(x)
f(x)
1 khi x ln th chui

n=1
f(n) phn k.
3.3.10. Cho f l hm d-ng, kh vi lin tc trn khong (0, ). Chng minh
rng:
(a) Nu lim
x
_

(x)
f(x)

1
x
_
xlnx > 1 th chui

n=1
f(n) hi t.
(b) Nu
_

(x)
f(x)

1
x
_
xlnx 1 khi x ln th chui

n=1
f(n) phn k.
3.3.11. Chng minh chiu ng-c li ca nh l cho trong bi 3.3.8.
Cho f l hm d-ng, n iu gim, kh vi lin tc trn khong (0, ).
(a) Nu chui

n=1
f(n) hi t th s tn ti mt hm g d-ng, kh vi lin tc
trn khong (0, ) sao cho
lim
x
_
g(x)
f

(x)
f(x)
g

(x)
_
> 0.

n

C
h
i
92 Ch-ng 3. Chui s thc
(b) Nu chui

n=1
f(n) phn k th s tn ti mt hm g d-ng, kh vi lin
tc trn khong (0, ) sao cho dy
_
n
_
1
1
g(x)
dx
_
khng b chn v khi x
ln th
g(x)
f

(x)
f(x)
g

(x) 0.
3.3.12. Vi 0, nghin cu s hi t ca chui

n=2
1
(lnn)
(lnn)

.
3.3.13. Nghin cu s hi t ca chui

n=3
1
n
1+
1
lnlnn
lnn
.
3.3.14. Cho {
n
} l dy s d-ng n iu tng v f l hm d-ng, n iu
tng tho mn iu kin

1
1
tf(t)
dt < .
Chng minh rng

n=1
_
1

n

n+1
_
1
f(
n
)
< .
3.3.15. Chng minh tiu chun tch phn suy rng.
Cho {
n
} l dy tng ngt ti v cng v f l hm d-ng, lin tc n iu
gim trn [
1
, ).
(a) Nu tn ti M > 0 sao cho
n+1

n
M vi n N v nu tch phn

1
f(t)dt hi t th chui

n=1
f(
n
) cng hi t.
(b) Nu tn ti M > 0 sao cho
n+1

n
M vi n N v nu tch phn

1
f(t)dt phn k th chui

n=1
f(
n
) cng phn k.
3.3.16. Gi s rng f : (0, ) R l hm d-ng, kh vi v c o hm
d-ng. Chng minh rng chui

n=1
1
f(n)
hi t khi v ch khi chui

n=1
f
1
(n)
n
2
hi t.

n

C
h
i
3.4. Hi t tuy t i. nh l Le ibniz 93
3.3.17. K hiu ln
1
x = lnx, ln
k
x = ln(ln
k1
x) vi k > 1 v x ln. Vi
mi n N, chn s nguyn d-ng (n) tho mn 1 ln
(n)
n < e. Khi
chui

n=3
1
n(ln
1
n)(ln
2
n) . . . (ln
(n)
n)
hi t hay phn k?
3.4 Hi t tuyt i. nh l Leibniz
3.4.1. Hy xt s hi t tuyt i, hi t c iu kin hoc phn k ca cc
chui sau theo a thuc min ch ra:

n=1
_
an
n + 1
_
n
, a R, (a)

n=2
(1)
n
(lnn)
a
n
, a R, (b)

n=1
(1)
n
sin
a
n
, a R, (c)

n=1
1
n + 1
_
a
2
4a 8
a
2
+ 6a 16
_
n
, a R\{8, 2}, (d)

n=1
n
n
a
n
2
, a = 0, (e)

n=1
(1)
n
(lnn)
ln n
n
a
, a > 0. (f)
3.4.2. Vi a R, nghin cu s hi t v hi t tuyt i ca chui

n=na
a
n1
na
n1
+ lnn
,
trong n
a
l mt ch s ph thuc vo a sao cho na
n1
+lnn = 0 vi n n
a
.
3.4.3. Gi s

n=1
a
n
l chui hi t vi cc s hng khc khng. Hy nghin
cu s hi t ca chui

n=1
_
1
sina
n
a
n
_
.

n

C
h
i
94 Ch-ng 3. Chui s thc
3.4.4. T iu kin lim
n
an
bn
= 1 c suy ra -c rng s hi t ca chui

n=1
a
n
t-ng -ng vi s hi t ca chui

n=1
b
n
khng?
3.4.5. Gi s rng chui

n=1
a
n
hi t c iu kin v t p
n
=
|an|+an
2
, q
n
=
|an|an
2
. Chng minh rng c hai chui

n=1
p
n
v

n=1
q
n
u phn k.
3.4.6. Gi s rng chui

n=1
a
n
hi t c iu kin. Gi {P
n
} v {Q
n
} ln
l-t l dy tng ring ca chui

n=1
p
n
v chui

n=1
q
n
nh ngha trong bi
trn. Chng minh rng
lim
n
P
n
Q
n
= 1.
3.4.7. Nghin cu s hi t v hi t tuyt i ca chui

n=1
(1)
[
n
3
]
n
.
3.4.8. Vi a R, xc nh khi no chui

n=1
(1)
[

n]
n
a
hi t tuyt i, hi t c iu kin hoc phn k.
3.4.9. Xc nh xem chui

n=1
(1)
[lnn]
n
hi t tuyt i, hi t c iu kin hay phn k.
3.4.10. t

n
=
_
+1 nu 2
2k
n < 2
2k+1
,
1 nu 2
2k+1
n < 2
2k+2
,
trong k = 0, 1, 2, . . . . Hy xt s hi t ca cc chui sau
(a)

n=1

n
n
, (b)

n=2

n
nlnn
.

n

C
h
i
3.4. Hi t tuy t i. nh l Le ibniz 95
3.4.11. Nghin cu s hi t ca chui

n=2
(1)
n

n
(1)
n
+

n
sin
1

n
.
3.4.12. Nghin cu s hi t (tuyt i, c iu kin) ca cc chui sau:

n=1
(1)
n
(
n

n 1)
n
, (a)

n=1
(1)
n
(
n

a 1), a > 1, (b)

n=1
(1)
n
(
n

n 1), (c)

n=1
(1)
n
_
e
_
1 +
1
n
_
n
_
, (d)

n=1
(1)
n
_
_
1 +
1
n
_
n+1
e
_
. (e)
3.4.13. Cho a, b > 0, hy xt s hi t ca cc chui sau:
(a)

n=1
(1)
n
(lnn)
a
n
b
, (b)

n=1
(1)
n
(lnn)
ln n
n
b
.
3.4.14. Cho

n=1
(1)
n1
a
n
l chui an du tho mn iu kin ca du hiu
Leibniz, tc l 0 < a
n+1
a
n
vi mi n v lim
n
a
n
= 0. t r
n
l phn d-
th n ca chui, r
n
=

k=n+1
(1)
k1
a
k
. Chng minh rng r
n
cng du vi s
hng (1)
n
a
n+1
v |r
n
| < a
n+1
.
3.4.15. Gi s rng dy {a
n
} dn ti 0. Chng minh rng hai chui sau

n=1
a
n
v

n=1
(a
n
+a
n+1
)
cng hi t hoc cng phn k.

n

C
h
i
96 Ch-ng 3. Chui s thc
3.4.16. Cho dy {a
n
} hi t n 0 v cc s a, b, c tho mn a + b + c = 0.
Chng minh rng hai chui

n=1
a
n
v

n=1
(aa
n
+ba
n+1
+ca
n+2
)
cng hi t hoc cng phn k.
3.4.17. Cho dy {a
n
} c cc s hng khc 0 v lim
n
a
n
= a = 0. Chng minh
rng hai chui

n=1
(a
n+1
a
n
) v

n=1
_
1
a
n+1

1
a
n
_
cng hi t tuyt i hoc cng khng hi t tuyt i.
3.4.18. Chng minh rng nu dy {na
n
} v chui

n=1
n(a
n
a
n+1
) hi t th
chui

n=1
a
n
cng hi t.
3.4.19. Cho dy {a
n
} n iu gim ti 0, hy nghin cu s hi t ca chui

n=1
(1)
n+1
a
1
+a
2
+. . . a
n
n
.
3.4.20. Tm cc gi tr ca a chui

n=1
(1)
n
n! sin a sin
a
2
. . . sin
a
n
hi t tuyt i v tm cc gi tr ca a chui phn k.
3.4.21. Cho a, b v c l cc s d-ng, nghin cu s hi t ca chui

n=1
_
n

a
n

b +
n

c
2
_
.
3.4.22. Hy nghin cu s hi t ca cc chui sau:
(a)

n=1
(cos n)
n
, (b)

n=1
(sinn)
n
.

n

C
h
i
3.4. Hi t tuy t i. nh l Le ibniz 97
3.4.23. Cho {a
n
} l dy s d-ng. Chng minh rng
(a) nu lim
n
n
_
an
a
n+1
1
_
> 0 th chui

n=1
(1)
n
a
n
hi t.
(b) nu n
_
an
a
n+1
1
_
0 th chui

n=1
(1)
n
a
n
phn k (c bit, nu
lim
n
n
_
an
a
n+1
1
_
< 0 th chui

n=1
(1)
n
a
n
phn k).
3.4.24. Cho {a
n
} l dy s d-ng. Gi s tn ti R, > 0 v mt dy
b chn {
n
} sao cho
a
n
a
n+1
= 1 +

n
+

n
n
1+
.
Chng minh rng chui

n=1
(1)
n
a
n
hi t vi > 0 v phn k vi 0.
3.4.25. Nghin cu s hi t ca chui

n=1
(1)
n
n!e
n
n
n+p
, p R.
3.4.26. Gi s rng chui

n=1
a
n
hi t v {p
n
} l dy d-ng n iu tng
n +. Chng minh rng
lim
n
a
1
p
1
+a
2
p
2
+. . . +a
n
p
n
p
n
= 0.
3.4.27. Cho {a
n
} l dy s d-ng n iu gim ti 0. Chng minh rng nu
chui

n=1
a
n
b
n
hi t th
lim
n
a
n
(b
1
+b
2
+. . . +b
n
) = 0.
3.4.28. Cho l mt s d-ng. Chng minh rng nu chui

n=1
an
n

hi t th
lim
n
a
1
+a
2
+. . . +a
n
n

= 0.

n

C
h
i
98 Ch-ng 3. Chui s thc
3.4.29. Cho {k
n
} l dy cc s t nhin tng ngt. Khi chui

n=1
a
kn
-c
gi l chui con ca chui

n=1
a
n
. Chng minh rng nu tt c cc chui con
ca mt chui hi t th chui hi t tuyt i.
3.4.30. Cho k, l l cc s nguyn sao cho k 1, l 2. Chui

n=1
a
n
c hi
t tuyt i khng nu tt c cc chui con c dng

n=1
a
k+(n1)l
u hi t?
3.4.31. Hy tm v d mt chui

n=1
a
n
hi t sao cho chui

n=1
a
3
n
phn k.
3.4.32. C tn ti hay khng chui

n=1
a
n
hi t sao cho tt c cc chui c
dng

n=1
a
k
n
, trong k N, k 2, u phn k?
3.4.33. Cho {a
n
} l dy n iu gim cc s d-ng sao cho chui

n=1
a
n
phn
k. Gi s rng chui

n=1

n
a
n
hi t, trong
n
bng 1 hoc 1. Chng minh
rng
lim
n

1
+
2
+. . . +
n
n
0 lim
n

1
+
2
+. . . +
n
n
.
3.4.34. Gi s {a
n
} l dy n iu gim cc s d-ng v chui

n=1

n
a
n
hi
t, trong
n
bng 1 hoc 1. Chng minh rng
lim
n
(
1
+
2
+. . . +
n
)a
n
= 0.
(Xem 3.2.35.)
3.4.35. Gi s rng chui

n=1
b
n
hi t v {p
n
} l dy n iu tng sao cho
lim
n
p
n
= + v

n=1
1
pn
= +. Chng minh rng
lim
n
p
1
b
1
+p
2
b
2
+. . . +p
n
b
n
n
0 lim
n
p
1
b
1
+p
2
b
2
+. . . +p
n
b
n
n
.

n

C
h
i
3.5. Ti u chun Dirichle t v ti u chun Abe l 99
3.4.36. Chng minh rng chui nhn -c t chui iu ho

n=1
1
n
bng cch
cho p s hng u mang du + , q s hng tip theo mang du , p s
hng tip theo mang du + . . . , hi t khi v ch khi p = q.
3.4.37. Chng minh nh l Toeplitz tng qut (xem 2.3.1 v 2.3.36).
Cho {c
n,k
: n, k N} l bng cc s thc. Khi vi mi dy hi t {a
n
},
dy {b
n
} xc nh bi
b
n
=

k=1
c
n,k
a
k
, n 1,
s hi t v c cng gii hn khi v ch khi ba iu kin sau tho mn:
(i) c
n,k

n
0 vi mi k N.
(ii)

k=1
c
n,k
= 1,
(iii) tn ti C > 0 sao cho vi mi s nguyn d-ng n u c

k=1
|c
n,k
| C.
3.5 Tiu chun Dirichlet v tiu chun Abel
3.5.1. S dng tiu chun Dirichlet v tiu chun Abel, hy nghin cu s hi
t ca cc chui sau:

n=1
(1)
n
sin
2
n
n
, (a)

n=1
sinn
n
_
1 +
1
2
+. . . +
1
n
_
, (b)

n=2
1
ln
2
n
cos
_

n
2
n + 1
_
, (c)

n=1
sin
n
4
n
a
+ sin
n
4
, a > 0. (d)

n

C
h
i
100 Ch-ng 3. Chui s thc
3.5.2. Chui

n=2
sin(n +
1
n
)
lnlnn
c hi t khng?
3.5.3. Vi a R, hy nghin cu s hi t ca cc chui

n=1
sin(na) sin(n
2
a)
n
, (a)

n=1
sin(na) cos(n
2
a)
n
. (b)
3.5.4. Chng minh rng chui

n=1
cos nsin(na)
n
hi t vi mi a R.
3.5.5. Tm a R sao cho chui

n=1
sin(na)
n
hi t tuyt i.
3.5.6. Chng minh rng vi a R v n N th

k=1
sin(ak)
k

< 2

.
3.5.7. Chng minh rng chui

n=1
(1)
n
arctann

n
hi t.
3.5.8. Vi x > 1, nghin cu s hi t ca chui

n=1
(1)
n
n

lnx
n
.
3.5.9. Chng minh b Kronecker.
Cho chui

n=1
a
n
hi t v {b
n
} l dy n iu tng tho mn lim
n
b
n
=
+. Khi
(a)

k=n
a
k
b
k
= o
_
1
b
n
_
, (b)
n

k=1
a
k
b
k
= o(b
n
),
trong o(b
n
) l v cng b ca b
n
, tc l lim
n
o(bn)
bn
= 0.

n

C
h
i
3.5. Ti u chun Dirichle t v ti u chun Abe l 101
3.5.10. Gi s chui

n=1
nc
n
hi t. Chng minh rng vi mi n N, chui

k=0
(k+1)c
n+k
cng hi t. Hn na, nu t
n
=

k=0
(k+1)c
n+k
th lim
n
t
n
= 0.
3.5.11. Gi s chui

n=1
a
n
c dy tng ring b chn. Chng minh rng nu
chui

n=1
|b
n
b
n+1
| hi t v lim
n
b
n
= 0 th vi mi s t nhin k, chui

n=1
a
n
b
k
n
cng hi t.
3.5.12. Chng minh rng nu chui

n=1
(b
n
b
n+1
) hi t tuyt i v chui

n=1
a
n
hi t th chui

n=1
a
n
b
n
cng hi t.
3.5.13. S dng tiu chun Abel, chng minh rng t s hi t ca chui

n=1
a
n
suy ra s hi t ca chui

n=1
a
n
x
n
vi |x| < 1.
3.5.14. Cho dy s {a
n
}. Chng minh rng nu chui Dirichlet

n=1
a
n
n
x
hi t vi x = x
0
th n s hi t vi mi x > x
0
.
3.5.15. Chng minh rng s hi t ca chui Dirichlet

n=1
an
n
x
cho ta s hi t
ca chui

n=1
n!a
n
x(x + 1) . . . (x +n)
, x = 0, 1, 2, . . . .
3.5.16. Chng minh rng nu chui

n=1
a
n
x
n
hi t vi |x| < 1 th

n=1
a
n
x
n
1x
n
cng hi t.
3.5.17. S hi t ca chui

n=1
a
n
c l tuyt i khng nu mi chui con ca
n c dng

n=1
a
kl
n, k 1, l 2,
u hi t?

n

C
h
i
102 Ch-ng 3. Chui s thc
3.6 Tch Cauchy ca cc chui v hn
3.6.1. Chng minh nh l Mertens.
Nu t nht mt trong hai chui hi t

n=0
a
n
v

n=0
b
n
hi t tuyt i th
tch Cauchy ca chng ( tc l chui

n=0
c
n
m c
n
= a
0
b
n
+a
1
b
n1
+. . .+a
n
b
0
)
hi t. Hn na nu

n=0
a
n
= A v

n=0
b
n
= B th

n=1
c
n
= AB.
3.6.2. Tm tng cc chui sau:

n=1
nx
n1
, |x| < 1, (a)

n=1
c
n
, vi c
n
=
n

k=0
x
k
y
nk
, |x| < 1, |y| < 1, (b)

n=1
c
n
, vi c
n
=
n

k=1
1
k(k + 1)(n k + 1)!
. (c)
3.6.3. Lp tch Cauchy ca cc chui cho v tnh cc tng ca chng:

n=0
2
n
n!
v

n=0
1
2
n
n!
, (a)

n=1
(1)
n
1
n
v

n=1
1
3
n
, (b)

n=0
(n + 1)x
n
v

n=0
(1)
n
(n + 1)x
n
. (c)
3.6.4. Gi s rng chui

n=0
a
n
hi t v t A
n
= a
0
+a
1
+. . . +a
n
. Chng
minh rng vi |x| < 1 chui

n=0
A
n
x
n
hi t v

n=0
a
n
x
n
= (1 x)

n=0
A
n
x
n
.
3.6.5. Tnh tch Cauchy ca chui

n=0
(1)
n x
2n
(n!)
2
, x R vi chnh n.
Gi . S dng ng thc

n=0
_
n
k
_
2
=
_
2n
n
_
.

n

C
h
i
3.6. Tch Cauchy ca cc chui v hn 103
3.6.6. Cho a > 0 v |x| < 1 hy chng t cc khng nh sau:
_
1
a
+
1
2
x
a + 2
+
1.3
2.4
x
2
a + 4
+. . . +
1.3. . . . (2n 1)
2.4. . . . .(2n)
x
n
a + 2n
+. . .
_

_
1 +
1
2
x +
1.3
2.4
x
2
+. . . +
1.3. . . . (2n 1)
2.4. . . . (2n)
x
n
+. . .
_
=
1
a
_
1 +
a + 1
a + 2
x +
(a + 1)(a + 3)
(a + 2)(a + 4)
x
2
+
+. . . +
(a + 1) . . . (a + 2n 1)
(a + 2) . . . (a + 2n)
x
n
+. . .
_
.
3.6.7. Chng minh nh l Abel.
Nu tch Cauchy

n=0
c
n
ca hai chui hi t

n=0
a
n
= A v

n=0
b
n
= B
cng hi t ti C th C = AB.
3.6.8. Chng minh rng chui

n=1
(1)
n1
2
n + 1
_
1 +
1
2
+. . . +
1
n
_
l tch Cauchy ca chui

n=1
(1)
n1 1
n
vi chnh n. Hy tm tng .
3.6.9. Nghin cu tnh hi t ca tch Cauchy ca chui

n=1
(1)
n1 1

n
vi
chnh n.
3.6.10. Chng minh rng nu t nht mt trong hai chui d-ng phn k th
tch Cauchy ca chng s phn k.
3.6.11. Tch Cauchy ca hai chui phn k c nht thit phn k khng ?
3.6.12. Chng minh rng tch Cauchy ca hai chui hi t

n=1
a
n
v

n=1
b
n
hi t khi v ch khi
lim
n
n

k=1
a
k
(b
n
+b
n1
+. . . +b
nk+1
) = 0.

n

C
h
i
104 Ch-ng 3. Chui s thc
3.6.13. Cho hai dy d-ng {a
n
} v {b
n
} gim n iu v 0. Chng minh
rng tch Cauchy ca cc chui

n=0
(1)
n
a
n
v

n=0
(1)
n
b
n
hi t khi v ch
khi
lim
n
a
n
(b
n
+b
n1
+. . . +b
0
) = 0 v lim
n
b
n
(a
n
+a
n1
+. . . +a
0
) = 0.
3.6.14. Chng minh rng tch Cauchy ca hai chui

n=1
(1)
n
n

n=1
(1)
n
n

, , > 0,
hi t khi v ch khi + > 1.
3.6.15. Gi s cc dy d-ng {a
n
} v {b
n
} n iu gim v 0. Chng minh
rng s hi t ca chui

n=0
a
n
b
n
l iu kin chui tch Cauchy ca
chui

n=0
(1)
n
a
n
v

n=1
(1)
n
b
n
hi t, v chng minh rng s hi t ca
chui

n=0
(a
n
b
n
)
1+
vi mi > 0 l mt iu kin cn cho s hi t ca chui
Cauchy ny.
3.7 Sp xp li chui. Chui kp
3.7.1. Cho {m
k
} l mt dy tng thc s cc s nguyn d-ng v t
b
1
= a
1
+a
2
+. . . +a
m
1
, b
2
= a
m
1
+1
+a
m
1
+2
+. . . +a
m
2
, . . . .
Chng minh rng nu chui

n=1
a
n
hi t th chui

n=1
b
n
cng hi t v hai
chui c tng bng nhau.
3.7.2. Xt chui
1
1
2

1
4
+
1
3

1
6

1
8
+
1
5
. . . ,
nhn -c bng cch thay i th t ca chui

n=1
(1)
n1
n
bng cch t hai
phn t m sau mi mt phn t d-ng, Hy tm tng ca chui .

n

C
h
i
3.7. Sp x p li chui. Chui k p 105
3.7.3. Ta thay i th t cc s hng ca chui

n=1
(1)
n1
n
sao cho khi
thnh phn d-ng ca chui -c xen k vi khi thnh phn m ca chui,
tc l
1 +
1
3
+. . . +
1
2 1

1
2

1
4
. . .
1
2
+
1
2 + 1
+
1
2 + 3
+. . . +
+
1
4 1

1
2 + 2

1
2 + 4
. . .
1
4
+. . . .
Hy tm tng chui va nhn -c.
3.7.4. Chng minh rng
1
1
2

1
4

1
6

1
8
+
1
3

1
10

1
12

1
14

1
16
+
1
5
. . . = 0.
3.7.5. Hy thay i th t cc s hng ca chui

n=1
(1)
n1
n
chui nhn
-c tng ln gp i chui ban u.
3.7.6. Hy thay i th t cc s hng ca chui

n=1
(1)
n1
n
nhn -c mt
chui phn k.
3.7.7. Nghin cu tnh hi t ca chui
1 +
1

2
+
1

5
+
1

4
+. . .
nhn -c bng cch t lin tip hai phn t d-ng v mt phn t m ca
chui

n=1
(1)
n1

n
.
3.7.8. Chng minh rng mi chui nhn -c bng cch i ch cc phn t
ca mt chui hi t tuyt i s hi t v c chung tng.
3.7.9. Gi s xt hm f : (0, +) (0, ), gim ti 0 khi x sao cho
dy {nf(n)} tng ti . t S l tng ca chui

n=1
(1)
n1
f(n). Cho tr-c
l, tm mt cch i th t chui trn chui nhn -c hi t v S +l.
3.7.10. Gi s hm f : (0, +) (0, ), gim ti 0 khi x tho
mn iu kin lim
n
nf(n) = g, g (0, +). t S l tng ca chui

n=1
(1)
n1
f(n). Cho tr-c l, tm mt cch i th t chui trn chui nhn
-c hi t v S + l.

n

C
h
i
106 Ch-ng 3. Chui s thc
3.7.11. Hy i ch cc phn t ca chui

n=1
(1)
n1 1
n
p
, p (0, 1) tng
gi tr ca tng chui ln l.
3.7.12. Cho tr-c s > 0, hy s dng kt qu bi 3.7.10, tm mt cch i
th t ca chui

n=1
(1)
n 1
n
t -c mt chui c tng bng ln2 +
1
2
ln.
3.7.13. Bng cch i ch cc s hng, c th lm nhanh phn k ca mt
chui phn k vi cc s hng d-ng v gim n iu -c khng?
3.7.14. Gi s chui

n=1
a
n
vi cc s hng d-ng phn k v lim
n
a
n
= 0.
Chng minh rng c th lm chm tc phn k mt cch tu bng cch i
ch cc phn t; tc l vi mi dy {Q
n
} tho mn
0 < Q
1
< Q
2
< . . . < Q
n
< . . . , lim
n
Q
n
= +.
tn ti mt s i ch

n=1
a
n
k
sao cho
a
n
1
+a
n
2
+. . . +a
nm
Q
m
, vi m N.
3.7.15. Cho {r
n
} v {s
n
} l hai dy s nguyn d-ng tng thc s khng c
phn t chung. Gi s rng mi s nguyn d-ng u xut hin mt trong hai
dy ny. Khi hai chui con

n=1
a
rn
v

n=1
a
sn
-c gi l hai chui con b
ca chui

n=1
a
n
. Ta ni rng s sp xp li dch chuyn hai chui con b nhau,
nu vi mi s nguyn d-ng m v n sao cho m < n s hng a
rm
xut hin
tr-c a
rn
v s hng a
sm
xut hin tr-c a
sn
. Chng minh rng, cc s hng
ca chui hi t c iu kin

n=1
a
n
c th sp xp li bng cch dch chuyn
hai chui con b nhau ca tt cc cc s hng m v s hng d-ng ca n
nhn -c mt chui hi t c tng l mt s nh du tu .
3.7.16. Cho

k=1
a
n
k
l mt s i ch ca mt chui hi t c iu kin

n=1
a
n
.
Chng minh rng nu {n
k
k} l mt dy b chn, th

k=1
a
n
k
=

n=1
a
n
. iu
g s xy ra nu dy {n
k
k} khng b chn?

n

C
h
i
3.7. Sp x p li chui. Chui k p 107
3.7.17. Cho

k=1
a
n
k
l mt s i ch ca mt chui hi c iu kin t

n=1
a
n
.
Chng minh rng

k=1
a
n
k
=

n=1
a
n
khi v ch khi tn ti mt s nguyn d-ng
N sao cho mi tp {n
k
: 1 k m} u l hp ca nhiu nht N khi ri
nhau ca cc s nguyn d-ng lin tip nhau.
3.7.18. T mt ma trn v hn {a
i,k
}, i = 1, 2, . . . , k = 1, 2, . . . ca cc
s thc, ta thit lp mt chui kp

i,k=1
a
i,k
. Ta ni rng chui kp hi t ti
S R nu vi > 0 cho tr-c, tn ti mt s n
0
N sao cho
|S
m,n
S| < vi m, n > n
0
,
trong
S
m,n
=
m

i=1
n

k=1
a
i,k
.
Khi ta vit
S = lim
m,n
S
m,n
=

i,k=1
a
i,k
.
Ni rng dy

i,k=1
a
i,k
hi t tuyt i nu

i,k=1
|a
i,k
| hi t. Ch rng cc s
hng ca mt ma trn v hn (a
i,k
)
i,k=1,2,...
c th -c xp th t thnh mt
dy {c
n
}, v khi chui

n=1
c
n
-c gi l s xp th t ca

i,k=1
a
i,k
thnh
mt chui n. Chng minh rng nu mt trong cc cch sp xp th t ca
chui kp hi t tuyt i th chui kp hi t (tuyti) ti cng tng.
3.7.19. Chng minh rng nu mt chui kp

i,k=1
a
i,k
hi t tuyt i, th mi
cch xp th t ca n

n=1
c
n
hi t v

i,k=1
a
i,k
=

n=1
c
n
.
3.7.20. Chng minh rng mi chui kp hi t tuyt i l hi t.

n

C
h
i
108 Ch-ng 3. Chui s thc
3.7.21. Ta gi mt chui lp

i=1
_

k=1
a
i,k
_
l hi t tuyt i nu chui

i=1
_

k=1
|a
i,k
|
_
hi t; nh ngha t-ng t cho chui

k=1
_

i=1
a
i,k
_
. Chng
minh rng mt chui lp hi t tuyt i l hi t.
3.7.22. Chng minh rng nu chui kp

i,k=1
a
i,k
hi t tuyt i th hai chui
lp

i=1
_

k=1
a
i,k
_
v

k=1
_

i=1
a
i,k
_
hi t tuyt i v

i=1
_

k=1
a
i,k
_
=

k=1
_

i=1
a
i,k
_
=

i,k=1
a
i,k
.
3.7.23. Chng minh rng nu mt trong bn chui

i,k=1
|a
i,k
|,

i=1
_

k=1
|a
i,k
|
_
,

k=1
_

i=1
|a
i,k
|
_
,

n=1
(|a
n,1
| +|a
n1,2
| +|a
n2,3
| +. . . +|a
1,n
|)
hi t th mi chui

i,k=1
a
i,k
,

i=1
_

k=1
a
i,k
_
,

k=1
_

i=1
a
i,k
_
,

n=1
(a
n,1
+a
n1,2
+a
n2,3
+. . . +a
1,n
)
u hi t ti cng mt tng.
3.7.24. Tnh

n,k=0
1
n!k!(n +k + 1)
.
3.7.25. Tnh

n,k=1
1
nk(n +k + 2)
.

n

C
h
i
3.7. Sp x p li chui. Chui k p 109
3.7.26. Chng minh rng

n,k=0
n!k!
(n +k + 2)!
=

2
6
.
3.7.27. Cho 0 < x < 1, xt ma trn v hn
_
_
_
_
x x
2
x
2
x
3
x
3

x(1 x) x
2
(1 x
2
) x
2
(1 x
2
) x
3
(1 x
3
) x
3
(1 x
3
)
x(1 x)
2
x
2
(1 x
2
)
2
x
2
(1 x
2
)
2
x
3
(1 x
3
)
2
x
3
(1 x
3
)
2

. . . . . . . . . . . . . . . . . . . . . . . . . . . . . . . . . . . . . . . . . . . . . . . . . . . . . . . . . . . . . . . . . .
_
_
_
_
Chng minh rng ch c mt chui lp t-ng ng vi ma trn ny hi t (khng
hi t tuyt i).
3.7.28. Nghin cu tnh hi t ca cc chui kp sau:

i,k=0
x
i
y
k
, vi |x|, |y| < 1. (a)

i,k=1
1
i

, vi , > 0. (b)

i,k=1
1
(i +k)
p
, vi p > 0. (c)
3.7.29. Tm tng ca cc chui kp sau:

i,k=2
1
(p +i)
k
, vi p > 1. (a)

i=2,k=1
1
(2k)
i
, (b)

i,k=1
1
(4i 1)
2k
. (c)
3.7.30. Cho mt ma trn v hn (b
i,k
)
i,k=1,2,...
, chng minh rng tn ti duy
nht mt chui kp

i,k=1
a
i,k
sao cho
S
m,n
=
m

i=1
n

k=1
a
i,k
= b
m,n
, m, n = 1, 2, . . . .

n

C
h
i
110 Ch-ng 3. Chui s thc
3.7.31. Ly
b
i,k
= (1)
i+k
_
1
2
i
+
1
2
k
_
, i, k = 1, 2, . . . ,
trong bi ton trn hy nghin cu tnh hi t ca chui kp t-ng ng

i,k=1
a
i,k
.
3.7.32. Chng minh rng vi |x| < 1, chui kp

i,k=1
x
ik
hi t tuyt i S
dng iu hy chng minh rng

i,k=1
x
ik
=

k=1
x
k
1 x
k
=

n=1
(n)x
n
= 2

n=1
x
n
2
1 x
n
+

n=1
x
n
2
,
trong (n) l cc -c t nhin ca n.
3.7.33. Chng minh rng vi |x| < 1 chui kp

i,k=1
ix
ik
hi t tuyt i. Hn
na, hy chng minh rng

i,k=1
ix
ik
=

i,k=1
kx
k
1 x
k
=

i,k=1
(n)x
n
,
trong (n) l tng cc -c t nhin ca n.
3.7.34. Cho (p) =

n=1
1
n
p
, p > 1, l hm zeta Riemann. t
S
p
=

n=2
1
n
p
= (p) 1, p > 1,
Chng minh rng
(a)

p=2
S
p
= 1, (b)

p=2
(1)
p
S
p
=
1
2
.
3.7.35. Chng minh nh l Goldbach.
Nu A = {k
m
; m, k = 2, 3, . . . } th

nA
1
n1
= 1.

n

C
h
i
3.8. Tch v hn 111
3.7.36. Gi s l hm zeta Riemann. Chng minh rng vi mi s nguyn
n 2,
(2)(2n 2) +(4)(2n 4) +. . . +(2n 2)(2) =
_
n +
1
2
_
(2n).
3.7.37. S dng kt qu ca bi tp trn hy tm tng ca cc chui

n=1
1
n
6
v

n=1
1
n
8
.
3.8 Tch v hn
3.8.1. Tnh:
(a)

n=2
_
1
1
n
2
_
, (b)

n=2
n
3
1
n
3
+ 1
,
(c)

n=1
cos
x
2
n
, x = 2
m
_

2
+k
_
, m N, k Z,
(d)

n=1
cosh
x
2
n
, x R, (e)

n=0
_
1 +x
2
n
_
, |x| < 1
(f)

n=1
_
1 +
1
n(n + 2)
_
, (g)

n=1
a
(1)
n
n
, a > 0
(h)

n=1
e
1
n
1 +
1
n
, (i)

n=1
9n
2
9n
2
1
,
3.8.2. Nghin cu tnh hi t ca tch v hn sau:

n=2
_
1 +
(1)
n
n
_
, (b)

n=1
_
1 +
1
n
_
, (a)

n=2
_
1
1
n
_
, (c)
3.8.3. Gi s a
n
0, n N. Chng minh rng tch v hn

n=1
(1 + a
n
) hi
t khi v ch khi chui

n=1
a
n
hi t.

n

C
h
i
112 Ch-ng 3. Chui s thc
3.8.4. Gi s a
n
0 v a
n
= 1 vi n N. Chng minh rng tch v hn

n=1
(1 a
n
) hi t khi v ch khi chui

n=1
a
n
hi t.
3.8.5. Cho
a
2n1
=
1

n
+
1
n
, a
2n
=
1

n
, n N.
Chng minh rng tch

n=1
(1 +a
n
) hi t mc d chui

n=1
a
n
phn k.
3.8.6. Nghin cu tnh hi t ca tch sau:

n=1
cos
1
n
, (b)

n=1
nsin
1
n
, (a)

n=1
tan
_

4
+
1
n
_
(d)

n=1
nln(1 +
1
n
) (c)

n=1
n

n, (f)

n=1
n
2

n (e)
3.8.7. Gi s rng chui

n=1
a
n
hi t. Chng minh rng tch

n=1
(1 +a
n
) hi
t khi v ch khi chui

n=1
a
2
n
hi t. Chng minh rng nu chui

n=1
a
2
n
phn
k th tch v hn

n=1
(1 +a
n
) phn k ti 0.
3.8.8. Gi s rng dy {a
n
} n iu gim v 0. Chng minh rng tch v
hn

n=1
(1 + (1)
n
a
n
) hi t khi v ch khi chui

n=1
a
2
n
hi t.
3.8.9. Chng minh rng tch v hn

n=1
_
1 + (1)
n+1
1

n
_
phn k, nh-ng chui

n=1
(1)
n+1 1

n
li hi t .
3.8.10. Chng minh rng nu hai chui

n=1
_
a
n

1
2
a
2
n
_
v

n=1
|a
n
|
3

n

C
h
i
3.8. Tch v hn 113
hi t th tch

n=1
(1 +a
n
) hi t.
3.8.11. S hi t ca tch

n=1
(1 +a
n
) c suy ra s hi t ca cc chui

n=1
a
2
n
v

n=1
a
n
-c khng?
Gi . Xt tch
_
1
1
2

__
1 +
1
2

+
1
2
2
__
1
1
3

__
1 +
1
3

+
1
3
2
_
. . . ,
trong
1
3
<
1
2
.
3.8.12. Chng minh kt qu tng qut ho ca bi 3.8.10. Cho k 2, nu hai
chui

n=1
_
a
n

1
2
a
2
n
+. . . +
(1)
k1
k
a
k
n
_
, v

n=1
|a
n
|
k+1
hi t th tch

n=1
(1 +a
n
) hi t.
3.8.13. Chng minh rng t s hi t ca tch

n=1
(1+a
n
) v ca chui

n=1
a
2
n
suy ra s hi t ca chui

n=1
a
n
.
3.8.14. Chng minh rng nu tch

n=1
(1 +a
n
) v tch

n=1
(1 a
n
) hi t th
cc chui

n=1
a
n
v

n=1
a
2
n
cng hi t.
3.8.15. Gi s dy {a
n
} gim n iu v 1. Tch v hn
a
1

1
a
2
a
3

1
a
4
a
5
. . .
c lun hi t khng?
3.8.16. Cho cc tch v hn hi t

n=1
a
n
v

n=1
b
n
vi cc tha s d-ng. Hy
nghin cu tnh hi t ca cc tch sau:

n=1
(a
n
+b
n
), (b)

n=1
a
2
n
, (a)

n=1
a
n
b
n
, (d)

n=1
a
n
b
n
. (c)

n

C
h
i
114 Ch-ng 3. Chui s thc
3.8.17. Chng minh rng vi x
n
(0,

2
), n N, tch

n=1
cos x
n
v

n=1
sinx
n
x
n
hi t khi v ch khi chui

n=1
x
2
n
hi t.
3.8.18. Cho

n=1
a
n
l chui d-ng hi t, t S
n
l tng ring th n ca chui.
Chng minh rng
a
1

n=2
_
1 +
a
n
S
n1
_
=

n=1
a
n
.
3.8.19. Chng minh rng nu tch v hn

n=1
(1 +a
n
), a
n
> 1 hi t v P
th chui

n=1
a
n
(1 +a
1
) (1 +a
2
) . . . (1 +a
n
)
cng hi t. Hn na nu S l tng ca n th S = 1
1
P
.
3.8.20. Gi s tch v hn

n=1
(1 +a
n
), vi a
n
> 0, n N,
phn k. Chng minh rng

n=1
a
n
(1 +a
1
) (1 +a
2
) . . . (1 +a
n
)
= 1.
3.8.21. Chng minh rng

n=1
x
n
(1 +x) (1 +x
2
) . . . (1 +x
n
)
= 1, vi x > 1.
3.8.22. Cho a
n
= 0 vi n N. Chng minh rng tch v hn

n=1
a
n
hi t
khi v ch khi n tho mn tiu chun Cauchy sau: Vi mi > 0 tn ti s
nguyn d-ng n
0
sao cho
|a
n
a
n+1
. . . a
n+k
1| <
vi mi n > n
0
v k N.

n

C
h
i
3.8. Tch v hn 115
3.8.23. Cho |x| < 1, hy kim tra ng thc sau:

n=1
(1 +x
n
) =
1

n=1
(1 x
2n1
)
.
3.8.24. Tch

n=1
(1 +a
n
) -c gi l hi t tuyt i nu

n=1
(1 +|a
n
|) hi t.
Chng minh rng tch

n=1
(1 +a
n
) hi t tuyt i khi v ch khi chui

n=1
a
n
hi t tuyt i.
3.8.25. Chng minh rng mi tch

n=1
(1 +a
n
) hi t tuyt i l hi t.
3.8.26. Chng minh rng nu tch

n=1
(1 +a
n
) hi t tuyt i th

n=1
(1 +a
n
) = 1 +

n=1
a
n
+

n
1
,n
2
=1
n
1
<n
2
a
n
1
a
n
2
+
+. . . +

n
1
,n
2
,... ,n
k
=1
n
1
<n
2
<...<n
k
a
n
1
a
n
2
. . . a
n
k
+. . .
3.8.27. Gi s tch v hn

n=1
(1 +a
n
) hi t tuyt i, chng minh rng tch
v hn

n=1
(1 +a
n
x) hi t tuyt i vi mi x R v c th khai trin thnh
mt chui hi t tuyt i theo h thc

n=1
(1 +a
n
x) = 1 +

k=1
A
k
x
k
,
trong
A
k
=

n
1
,n
2
,... ,n
k
=1
n
1
<n
2
<...<n
k
a
n
1
a
n
2
. . . a
n
k
.
3.8.28. Thit lp ng thc sau:

n=1
(1 +q
n
x) = 1 +

n=1
q
n(n+1)
2
(1 q)(1 q
2
) . . . (1 q
n
)
x
n
, |q| < 1.

n

C
h
i
116 Ch-ng 3. Chui s thc
3.8.29. Kim tra ng thc sau:

n=1
(1 +q
2n1
x) = 1 +

n=1
q
n
2
(1 q
2
)(1 q
4
) . . . (1 q
2n
)
x
n
, |q| < 1.
3.8.30. Gi s chui

n=1
a
n
hi t tuyt i. Chng minh rng nu x = 0 th

n=1
(1 +a
n
x)
_
1 +
a
n
x
_
= B
0
+

n=1
B
n
_
x
n
+
1
x
n
_
,
vi B
n
= A
n
+A
1
A
n+1
+ +A
2
A
n+2
. . . , n = 0, 1, 2, . . . , v

n=1
(1 +a
n
x) = A
0
+

k=1
A
k
x
k
( xem 3.8.27).
3.8.31. Cho |q| < 1 v x = 0 hy chng minh ng thc sau:

n=1
(1 q
2n
)

n=1
(1 +q
2n1
x)
_
1 +
q
2n1
x
_
= 1 +

n=1
q
n
2
_
x
n
+
1
x
n
_
.
3.8.32. Cho |q| < 1, hy kim tra cc ng thc sau:

n=1
(1 q
2n
)

n=1
(1 q
2n1
)
2
= 1 + 2

n=1
(1)
n
q
n
2
, (a)

n=1
(1 q
2n
)

n=1
(1 +q
2n1
)
2
= 1 + 2

n=1
q
n
2
, (b)

n=1
(1 q
2n
)

n=1
(1 +q
2n
)
2
= 1 +

n=1
q
n
2
+n
, (c)
3.8.33. Cho x > 0, xt dy {a
n
} nh- sau:
a
1
=
1
1 +x
, a
n
=
n
x +n
n1

k=1
x k
x +k
, n > 1.
Chng minh rng chui

n=1
a
n
hi t, tm tng ca n.

n

C
h
i
3.8. Tch v hn 117
3.8.34. Chng minh rng nu tch v hn

n=1
(1 + ca
n
) hi t vi hai gi tr
khc nhau ca hng s c R\{0} th n hi t vi mi c.
3.8.35. Chng minh rng nu chui

n=1
a
n
n

k=0
(x
2
k
2
)
hi t ti x = x
0
, x
0
Z th n hi t vi mi gi tr ca x.
3.8.36. Cho {p
n
} l mt dy cc s nguyn t lin tip ln hn 1.
(a) Chng minh cng thc tch Euler

n=1
_
1
1
p
x
n
_
1
=

n=1
1
n
x
vi x > 1.
(b) Chng minh rng chui

n=1
1
pn
phn k (hy so snh vi bi tp 3.2.72).
3.8.37. Hy dng quy tc DeMoivre chng minh cc khng nh sau:
sinx = x

n=1
_
1
x
2
n
2

2
_
, (a)
cos x =

n=1
_
1
4x
2
(2n 1)
2

2
_
. (b)
3.8.38. S dng kt qu cu trn hy chng minh cng thc Wallis
lim
n
(2n)!!
(2n 1)!!

n
=

.
3.8.39. Nghin cu s- hi t ca cc tch sau:

n=1
_
1 +
x
n
_
e

x
n
, x > 1, (a)

n=1
_
1 +
1
n
_
x
1 +
x
n
, x > 1. (b)
3.8.40. Chng minh rng tch v hn

n=1
(1 +a
n
) hi t tuyt i khi v ch
khi mi s i ch cc tha s ca n khng lm thay i gi tr ca n.

n

C
h
i
118 Ch-ng 3. Chui s thc
3.8.41. Tnh
_
1 +
1
2
__
1 +
1
4
_
. . .
_
1 +
1
2
_

_
1
1
3
_
. . .
_
1
1
2 + 1
__
1 +
1
2 + 2
_
. . .
tch ny l s i ch cc nhn t ca tch

n=2
_
1 +
(1)
n
n
_
bng cch t cc
khi gm tha s ln hn 1 v khi gm tha s ln hn 1 xen k nhau.
3.8.42. Chng minh rng t tch

n=1
(1 +a
n
), a
n
> 1 hi t nh-ng khng
hi t tuyt i ta c th i ch nhn -c mt tch c gi tr l mt s
d-ng bt k cho tr-c, hoc mt tch phn k v 0 hoc v cng. (So snh vi
bi tp 3.7.15).

n

C
h
i
Li gii

n

C
h
i

n

C
h
i
Ch-ng 1
S thc
1.1 Cn trn ng v cn d-i ng ca tp cc
s thc. Lin phn s
1.1.1. t A = {x Q : x > 0, x
2
< 2} v s = sup A, d thy s > 1. Ta
s ch ra rng vi s n nguyn d-ng bt k th
_
s
1
n
_
2
2
_
s +
1
n
_
2
. (1)
Tht vy, v s
1
n
khng l cn trn ca A nn tn ti x

A sao cho
s
1
n
< x

, suy ra
_
s
1
n
_
2
< (x

)
2
< 2.
Gi s rng
_
s +
1
n
_
2
< 2. Nu s l s hu t th s +
1
n
A v s +
1
n
> s,
tri vi gi thit s = supA. Nu s l s v t th w =
[(n+1)s]
n+1
+
1
n+1
l s hu
t tho mn s < w < s +
1
n
, do w
2
<
_
s +
1
n
_
2
< 2 tc l w A, mu
thun. Vy ta chng minh -c rng
_
s +
1
n
_
2
2. S dng v tri ca
(1) ta c s
2

2s
n
< s
2

2s
n
+
1
n
2
2, t suy ra
s
2
2
2s
<
1
n
. Cho n ,
ta nhn -c s
2
2 0. T-ng t, t bt ng thc v phi ca (1) suy ra
s
2
2
2s

1
n
, suy ra s
2
2 0. Do s
2
= 2.
1.1.2. Gi s A b chn d-i v t a = inf A, khi
121

n

C
h
i
122 Ch-ng 1. S thc
(1) x a vi mi x A,
(2) vi > 0, tn ti x

A sao cho x

< a +.
Nhn hai bt ng thc trong (1) v (2) vi 1, ta c
(1') x a vi mi x (A),
(2') vi > 0 bt k , tn ti x

(A) sao cho x

> a .
T suy ra a = sup(A). Nu A khng b chn d-i th A khng b
chn trn v do sup(A) = inf(A) = +. Cc ng thc cn li -c
chng minh t-ng t.
1.1.3. Gi s A v B b chn trn v t a = sup A v b = sup B, khi
a l mt cn trn ca A, b l mt cn trn ca B, suy ra a + b l mt cn
trn ca A+B. Hn na, vi mi > 0, tn ti x

A v y

B sao cho
x

> a

2
v y

> b

2
, do x

+y

> a+b. V z

= x

+y

A+B
suy ra a + b = sup(A+B). Nu A hoc B khng b chn trn, th A+B
khng b chn trn, t nh ngha ca cn trn ng ta suy ra sup(A+B) =
sup A+ sup B = +.
ng thc th hai l mt h qu trc tip ca ng thc th nht v ca
bi ton tr-c. Tht vy,
sup(AB) = sup(A+ (B)) = sup A+ sup(B) = sup Ainf B.
Lp lun t-ng t trn c th suy ra cc ng thc
inf(A+B) = inf A+ inf B,
inf(AB) = inf Asup B.
1.1.4. Gi s c hai tp b chn trn, t a = sup A v b = supB. V cc
phn t ca A v ca B l cc s d-ng nn xy ab vi x A v y B.
Ta s chng minh rng ab l cn trn b nht ca A B. Cho tr-c > 0,
tn ti x

A v y

B sao cho x

> a v y

> b . Khi
x

> ab (a + b ). V (a + b ) c th nh tu vi nh, ta
thy rng bt k s no nh hn ab khng th l cn trn ca A B. Do
ab = supA B. Nu A hoc B khng b chn trn th A B cng khng b
chn. Do sup(A B) = supA supB = +.
Cng vic by gi l chng minh sup
_
1
A
_
=
1
inf A
> 0 nu a

= inf A > 0.
Tht vy, vi mi x A, bt ng thc x a

t-ng -ng vi
1
x

1
a

nn

n

C
h
i
1.1. Cn tr n ng v cn d-i ng. Li n phn s 123
1
a

l cn trn ca
1
A
. Hn na, vi > 0 bt k, tn ti x

A sao cho
x

< a

+, do
1
x

>
1
a

+
=
1
a

(a

+)
.
V

a

(a

+)
nh tu nn
1
a

l cn trn nh nht ca
1
A
. Xt tr-ng hp a

= 0,
thy rng tp
1
A
l b chn (tht vy, vi > 0, tn ti x

1
A
sao cho x

>
1

).
Do , sup
1
A
= +.
By gi gi s rng A, B l cc tp b chn cc s thc bt k v t
a = sup A, b = sup B, a

= inf A, b

= inf B. Nu a

v b

l khng m th
s dng kt qu trn ta suy ra ng thc cn chng minh. Nu a

< 0 v
a, b

> 0 th xy ab vi bt k x A v y B. Chn > 0 nh


a > 0. Khi tn ti x

Asao cho x

> a. Hn na, tn ti y

B
sao cho y

> b . Do
x

> x

(b ) > (a )(b ) = ab (b +b ).
Vy trong tr-ng hp ny ta c sup(A B) = ab.
Xt tr-ng hp a

, b

< 0 v a, b > 0. Vi bt k x A v y B ta c
xy max{ab, a

}.
u tin xt tr-ng hp max {ab, a

} = a

. Theo nh ngha ca cn d-i


ng, vi > 0 nh tn ti x

A v y

B sao cho x

< a

+ < 0 v
y

< b

+ < 0, suy ra
x

> x

(b

+) > (a

+)(b

+) = a

+(a

+b

+ (a

+b

+).
T nhn xt rng a

+ b

+ l s m suy ra a

l cn trn b nht ca
A B. Trong tr-ng hp max {ab, a

} = ab lp lun t-ng t ta suy ra


sup(A B) = ab. Cc tr-ng hp cn li -c chng minh t-ng t.
1.1.5. Tr-c ht gi s A v B b chn trn, t a = sup A v b = sup B,
khng gim tng qut ta coi a b, th th vi mi x A B ta c x b.
Hn na, vi > 0, tn ti x

B sao cho x

> b . Hin nhin x

thuc
vo A B. Do ng thc th nht l ng. Nu A hoc B khng b chn
trn th A B cng khng b chn trn. V vy sup(A B) = +, v ta
qui -c rng max{+, c} = max{+, +} = + mi s thc c. Chng
minh ng thc th hai t-ng t.

n

C
h
i
124 Ch-ng 1. S thc
1.1.6. Ta c
A
1
=
_
3,
11
2
, 5
_

_
3
4k
,
3
4k + 1
, 4
3
4k + 2
, 4 +
3
4k + 3
; k N
_
,
A
2
=
_
3k 1
3k + 1
,
3k 2
6k
,
3k 3
2(3k 1)
; k N
_
.
Do inf A
1
=
11
2
, supA
1
= 5 v inf A
2
=
1
2
, supA
2
= 1.
1.1.7. supA =
2
9
, inf A =
1
5
, sup B =
1
9
, inf B = 0.
1.1.8. C th ch ra bng quy np rng vi n 11, 2
n
> (n + 1)
3
. Do
0 <
(n + 1)
2
2
n
<
(n + 1)
2
(n + 1)
3
=
1
n + 1
vi n 11,
do 0 l cn d-i ng ca tp hp ang xt. Bn cnh ta d dng chng
minh rng 2
n
> (n + 1)
2
vi n 6, do
(n+1)
2
2
n
< 1 vi n 6. Cc s
2,
9
4
,
25
16
,
36
32
(ln hn 1) cng nm trong tp ang xt, suy ra cn trn ng ca
tp l
9
4
.
1.1.9. T bi ton tr-c suy ra cn d-i ng ca tp ny bng 0. S dng
bt ng thc nu trong li gii bi tr-c ta -c 2
nm
> (nm + 1)
2
vi
nm 6. V nm+ 1 n +m vi n, m N, ta c
(n +m)
2
2
nm
<
(n +m)
2
(nm+ 1)
2

(n +m)
2
(n +m)
2
= 1 nu nm 6.
Vi nm < 6, cc phn t 1, 2,
9
4
,
25
16
,
36
32
cng thuc tp ang xt, do cn trn
b nht l
9
4
.
1.1.10.
(a) Hin nhin 2 l cn trn ca tp A, ta s ch ra rng n l cn trn ng
ca A. Tht vy, nu > 0 l mt s c nh bt k , th vi s nguyn
d-ng bt k n

>
_
2

, ta thu -c
2(n

1)
n

> 2 . Cn trn ng
ca A l 0, v
m
n
> 0 vi m, n N. Cho tr-c > 0, tn ti n sao cho
1
n
< .

n

C
h
i
1.1. Cn tr n ng v cn d-i ng. Li n phn s 125
(b) Hin nhin 0

n [

n] < 1. Chn n = k
2
, k N, ta thy rng
0 B, do inf B = 0. chng minh rng supB = 1 tr-c ht ta c
_
n
2
+ 2n

= n vi mi n nguyn d-ng, xt 0 < < 1, thc hin mt


s tnh ton ta -c bt ng thc

n
2
+ 2n
_

n
2
+ 2n
_
=
2
_
1 +
2
n
+ 1
> 1
tho mn vi bt k n >
(1)
2
2
.
1.1.11.
(a) sup{x R : x
2
+x + 1 > 0} = +,
(b) inf {z = x +x
1
: x > 0} = 2,
(c) inf
_
z = 2
x
+ 2
1
x
: x > 0
_
= 4.
Hai ng thc u tin d kim tra. chng minh ng thc th ba, ch
rng
a+b
2

ab vi a, b > 0, do
2
x
+ 2
1
x
2

_
2
1
x
+x

2
2
= 2,
du ng thc khi v ch khi x = 1. Ta -c iu phi chng minh.
1.1.12.
(a) S dng bt ng thc
a+b
2

ab vi a, b > 0, ta c
m
n
+
4n
m
4,
du ng thc xy ra khi m = 2n. Do inf A = 4. Ly m = 1, c th
ch ra rng tp A khng b chn trn. iu ny c ngha sup A = +.
(b) T-ng t ta c

1
4

mn
4m
2
+n
2

1
4
,
cc bt ng thc tr thnh ng thc khi ln l-t m = 2n v m = 2n.
Kt qu l inf B =
1
4
, supB =
1
4
.

n

C
h
i
126 Ch-ng 1. S thc
(c) Ta c inf C = 0 v sup C = 1. Thc vy, 0 <
m
m+n
< 1, v vi bt k
> 0, tn ti cc s nguyn d-ng n
1
v m
1
sao cho
1
n
1
< v
m
1
m
1
+ 1
> 1
(d) inf D = 1 v sup D = 1.
(e) Chn m = n th tp khng b chn trn. Do supE = +. Tri li, vi
bt k m, n N ta c
mn
1+m+n

1
3
, du ng thc xy ra khi m = n = 1,
tc l inf E =
1
3
.
1.1.13. t s = a
1
+a
2
+... +a
n
, ta c
a
k
s

a
k
a
k
+a
k+1
+a
k+2
1
a
k+1
s

a
k+2
s
,
suy ra
1
n

k=1
a
k
a
k
+a
k+1
+a
k+2
n 2.
By gi ta cn chng minh rng
inf
n

k=1
a
k
a
k
+a
k+1
+a
k+2
= 1 v sup
n

k=1
a
k
a
k
+a
k+1
+a
k+2
= n 2.
Chn a
k
= t
k
, t > 0 th
n

k=1
a
k
a
k
+a
k+1
+a
k+2
=
t
t +t
2
+t
3
+... +
t
n2
t
n2
+t
n1
+t
n
+
t
n1
t
n1
+t
n
+t
+
t
n
t
n
+t +t
2
= (n 2)
1
1 +t +t
2
+
t
n2
t
n1
+t
n2
+ 1
+
t
n1
t
n1
+t + 1
.
Cho t 0
+
, ta thy rng sup
n

k=1
a
k
a
k
+a
k+1
+a
k+2
= n 2, v cho t +,
ta thu -c inf
n

k=1
a
k
a
k
+a
k+1
+a
k+2
= 1.

n

C
h
i
1.1. Cn tr n ng v cn d-i ng. Li n phn s 127
1.1.14. C nh n N v xt n + 1 s thc thuc khong [0, 1)
0, [], 2 [2], ..., n [n].
V n khong
_
j
n
,
j+1
n
_
, j = 0, 1, ..., n 1, ph [0, 1) nn tn ti khong cha
t nht hai trong s nhng im ny, gi s nh- n
1
[n
1
] v n
2
[n
2
]
vi 0 n
1
< n
2
n. T suy ra
|n
2
[n
2
] n
1
+ [n
1
]| <
1
n
.
By gi ta xt q
n
= n
2
n
1
v p
n
= [n
2
] [n
1
], t nhn xt trn suy ra
q
n
n, tc l bt ng thc th hai ng.
1.1.15. Ta s ch ra rng trong bt k khong (p, q) tn ti t nht mt phn
t ca A. Gi 0 < = q p. T bi ton tr-c suy ra tn ti cc s p
n
v q
n
sao cho


p
n
q
n

<
1
q
2
n
.
V l v t nn lim
n
q
n
= +. Do
|q
n
p
n
| <
1
q
n
< .
vi hu ht n. t a = |q
n
p
n
| th t nht mt trong cc s ma, m Z,
thuc khong (p, q); tc l hoc mq
n
mp
n
hoc mq
n
+mp
n
thuc khong
ny.
1.1.16. Cho t [1, 1], khi tn ti x sao cho t = cos x. T kt qu
ca bi ton tr-c, tn ti cc dy s nguyn {m
n
} v {k
n
} sao cho x =
lim
n
(k
n
2 +m
n
). T nhn xt ny v tnh lin tc ca hm cos x ta suy ra
t = cos x = cos( lim
n
(k
n
2 +m
n
)) = lim
n
cos m
n
= lim
n
cos |m
n
|.
V vy, mi s trong [1, 1] l gii hn ca tp {cos n : n N} . Ta -c iu
phi chng minh.
1.1.17. Hin nhin, nu tn ti n sao cho x
n
l mt s nguyn, th x l hu
t. By gi gi s x =
p
q
vi p Z v q N. Nu x[x] = 0 th
p
q

_
p
q
_
=
l
q
vi l l mt s nguyn d-ng nh hn q. Do mu s ca x
1
=
q
l
nh hn
mu s ca x. iu ny c ngha x
1
, x
2
, ... l dy tng cht lin tip v khng
th to thnh mt dy v hn.

n

C
h
i
128 Ch-ng 1. S thc
1.1.18. Ta s chng minh bng quy np . D kim tra rng
R
k
=
p
k
q
k
vi k = 0, 1, 2.
Gi s vi m 2 bt k chn tr-c,
R
m
=
p
m
q
m
=
p
m1
a
m
+p
m2
q
m1
a
m
+q
m2
.
Ch l nu by gi thay a
m
trong R
m
bng a
m
+
1
a
m+1
, th ta thu -c phn
t hi t R
m+1
. Do
R
m+1
=
p
m1
_
a
m
+
1
a
m+1
_
+p
m2
q
m1
_
a
m
+
1
a
m+1
_
+q
m2
=
(p
m1
a
m
+p
m2
)a
m+1
+p
m1
(q
m1
a
m
+q
m2
)a
m+1
+q
m1
=
p
m+1
q
m+1
.
1.1.19. K hiu

k
= p
k1
q
k
q
k1
p
k
vi k = 1, 2, ..., n.
Khi vi k > 1,

k
= p
k1
(q
k1
a
k
+q
k2
) q
k1
(p
k1
a
k
+p
k2
)
= (p
k2
q
k1
q
k2
p
k1
) =
k1
.
V
1
= p
0
q
1
q
0
p
1
= a
0
a
1
(a
0
a
1
+1) = 1, ta thu -c
k
= (1)
k
, t
suy ra p
k
v q
k
nguyn t cng nhau.
1.1.20. Theo li gii ca 1.1.18 ta c vi n > 1
R
n
=
p
n
q
n
=
p
n1
a
n
+p
n2
q
n1
a
n
+q
n2
.
T-ng t
x =
p
n
x
n+1
+p
n1
q
n
x
n+1
+q
n1
vi n = 1, 2, ....
Do
x R
n
=
p
n
a
n+1
+p
n1
q
n
a
n+1
+q
n1

p
n
q
n
=
p
n1
q
n
q
n1
p
n
(q
n
x
n+1
+q
n1
)q
n
=
(1)
n
(q
n
x
n+1
+q
n1
)q
n
,

n

C
h
i
1.1. Cn tr n ng v cn d-i ng. Li n phn s 129
trong , ng thc cui cng suy ra t kt qu trong 1.1.19. Do
x R
n
_
> 0 vi n chn,
< 0 vi n l.
Do vy x nm gia hai dy hi t lin tip.
1.1.21. Tr-c ht ta chng minh rng nu l mt s v t d-ng, th tp
{n m : n, m N} l tr mt trong R
+
. lm iu , ly mt khong
(a, b), 0 < a < b. Ta s ch ra rng khong ny cha t nht mt phn t ca
tp cho. t = b a > 0, t kt qu bi ton tr-c, tn ti mt dy hi
t R
n
sao cho
0 < R
n
<
1
q
2
n
, (1)
Thc vy, ly mt s n l v ch rng
(q
n
x
n+1
+q
n1
)q
n
> q
2
n
.
V lim
n
q
n
= +, vi n ln ta c
1
qn
< . T iu ny v (1) suy ra
0 < p
n
q
n
< 1/q
n
< vi n ln, do tn ti n
0
N sao cho
n
0
(p
n
q
n
) (a, b). Cho t [1, 1], tn ti s nguyn x sao cho t = sinx.
T nhn xt trn suy ra tn ti mt dy cc s nguyn d-ng {m
n
} v {k
n
}
vi x = lim
n
(m
n
2k
n
) = +. S dng tnh lin tc ca hm sinx ta -c
t = sinx = sin( lim
n
(m
n
2k
n
)) = lim
n
sin m
n
).
Do vy, ta chng minh -c rng mi s thuc khong [1, 1] u l im
gii hn ca tp {sinn : n N}.
1.1.22. Gi p
n
v q
n
l cc s nguyn -c nh ngha trong 1.1.20. V x
n+1
=
a
n+1
+
1
x
n+2
> a
n+1
, ta c (q
n
x
n+1
+q
n1
)q
n
> (q
n
a
n+1
+q
n1
)q
n
= q
n+1
q
n
.
Do , theo 1.1.20,
|x R
n
| <
1
q
n
q
n+1
V q
n+1
= q
n
a
n+1
+q
n1
> q
n
a
n+1
> q
n
, ko theo bt ng thc mong mun.
Ta s ch ra rng dy {q
n
} cha v hn cc s l. Thc vy, theo kt qu trong
1.1.19, q
n
v q
n+1
khng th cng chn.
1.1.23. S dng bi 1.1.19.

n

C
h
i
130 Ch-ng 1. S thc
1.1.24. Tr-c ht ta c nhn xt rng dy {q
n
} tng ngt v q
n
n. Hn
na t bi 1.1.20 suy ra
|x R
n
| =
1
(q
n
x
n+1
+q
n1
)q
n
.
kt hp vi bt ng thc x
n+1
< a
n+1
+ 1 ta suy ra
|x R
n
| >
1
(q
n
(a
n+1
+ 1) +q
n1
)q
n
=
1
(q
n+1
+q
n
)q
n
.
V a
n+2
1 ta c
|x R
n+1
| <
1
(q
n+1
a
n+2
+q
n
)q
n+1
<
1
(q
n+1
+q
n
)q
n
.
T cc bt ng thc ny suy ra iu cn chng minh.
1.1.25. Gi s

x
r
s

< |x R
n
| < |x R
n1
|. V x nm gia R
n
v
R
n1
(xem bi ton 1.1.20), suy ra

r
s
R
n1

< |R
n1
R
n
| .
Do , theo kt qu ca 1.1.23,
|rq
n1
sp
n1
|
sq
n1
<
1
q
n1
q
n
.
Hn na, ta c
1
sq
n1
<
1
qnq
n1
v |rq
n1
sp
n1
| 1. Do s > q
n
.
1.1.26. Theo thut ton cho trong 1.1.20, ta c
a
0
=
_

2
_
= 1, x
1
=
1

2 1
=

2 + 1.
Do , a
1
= [x
1
] = 2. T-ng t,
x
2
=
1
_
2 + 1
_
2
v a
2
= a
1
= 2.
Bng quy np ,

2 = 1 +
1|
|2
+
1|
|2
+....
T-ng t

5 1
2
=
1|
|1
+
1|
|1
+
1|
|1
+....

n

C
h
i
1.2. Mt s bt ng thc s cp 131
1.1.27. V k <

k
2
+k < k +1, a
0
=
_
k
2
+k

= k nn x
1
=

k
2
+k+k
k
.
Vy, 2 < x
1
< 2 +
1
k
v a
1
= 2. Hn na,
x
2
=
1
1

k
2
+kk
2
= k +

k
2
+k.
Do 2k < x
2
< 2k + 1 v a
2
= 2k. T-ng t ta thu -c a
3
= 2. S dng
quy np ta suy ra

k
2
+k = k +
1|
|2
+
1|
|2k
+
1|
|2
+
1|
|2k
+....
1.1.28. V 0 < x < 1, suy ra a
0
= 0 v x
1
= 1/x. Do a
1
= n suy ra
[1/x] = n. T 1/x 1 < n 1/x suy ra 1/(n + 1) < x 1/n.
1.2 Mt s bt ng thc s cp
1.2.1. Ta s s dng quy np sau. Vi n = 1, bt ng thc l hin nhin.
Ly n nguyn d-ng bt k v gi s l
(1 +a
1
) (1 +a
2
) ... (1 +a
n
) 1 +a
1
+a
2
+... +a
n
.
Th
(1 +a
1
)(1 +a
2
) ... (1 +a
n
)(1 +a
n+1
)
(1 +a
1
+a
2
+... +a
n
)(1 +a
n+1
)
= 1 +a
1
+a
2
+... +a
n
+a
n+1
+a
n+1
(1 +a
1
+a
2
+... +a
n
)
(1 +a
1
+a
2
+... +a
n
+a
n+1
).
Ta suy ra iu phi chng minh.
1.2.2. Ta s dng quy np. Vi n = 1, iu khng nh l hin nhin. Ta
gi s rng khng nh ng vi n bt k chn tr-c. Khng mt tnh tng
qut, gi s a
1
, a
2
, ..., a
n
tho mn iu kin a
1
a
2
... a
n+1
= 1 -c
sp theo th t a
1
a
2
... a
n
a
n+1
, th th a
1
1 v a
n+1

1. V a
2
a
3
... a
n
(a
n+1
a
1
) = 1, s dng gi thit quy np suy ra
a
2
+a
3
+... +a
n
+ (a
n+1
a
1
) n, do
a
1
+a
2
+... +a
n
+a
n+1
n +a
n+1
+a
1
a
n+1
a
1
= n +a
n+1
(1 a
1
) +a
1
1 + 1
= n + 1 + (a
n+1
1)(1 a
1
) n + 1.

n

C
h
i
132 Ch-ng 1. S thc
1.2.3. S dng kt qu trong bi 1.2.2. suy ra bt ng thc cn chng minh.
Thc vy, thay a
j
bng
a
j
n

a
1
...an
, ta c A
n
G
n
, thay a
j
bng nghch o ca
n
1
a
j
trong bt ng thc ny ta suy ra bt ng thc G
n
H
n
.
1.2.4. S dng bt ng thc lin h gia trung bnh cng v trung bnh nhn
ca b s a
1
, ..., a
n
ta c
n
_
(1 +nx) 1 1... 1 1 +x (n nhn t).
1.2.5.
(a) S dng bt ng thc lin h gia trung bnh cng v trung bnh iu
ho
(1)
.
(b) S dng bt ng thc lin h gia trung bnh cng v trung bnh iu
ho.
(c) Bt ng thc v tri -c chng minh nh- trong (a) v (b). chng minh
bt ng thc v phi, ta ch rng
1
3n + 1
+
1
3n + 1
+... +
1
5n
+
1
5n + 1
<
1
3n + 1
+
2n
3n + 2
<
2
3
.
(a) S dng bt ng thc lin h gia trung bnh cng v trung bnh nhn
(2)
ta -c
2
1
+
3
2
+
4
3
+... +
n + 1
n
> n
n

n + 1,
do
1 + 1 + 1 +
1
2
+ 1 +
1
3
+... + 1 +
1
n
> n
n

n + 1
v
1 +
1
2
+
1
3
+... +
1
n
> n(
n

n + 1 1).
chng minh bt ng thc cn li, ta cng dng bt ng thc lin h
gia trung bnh cng v trung bnh nhn, c
1
2
+
2
3
+
3
4
+... +
n
n + 1
>
n
n

n + 1
,
suy ra
1 +
1
2
+
1
3
+... +
1
n
< n
_
1
1
n

n + 1
+
1
n + 1
_
.
(1)
Cn gi l bt ng thc trung bnh iu ho.
(2)
Cn gi l bt ng thc Cauchy tng qut.

n

C
h
i
1.2. Mt s bt ng thc s cp 133
1.2.6. T bt ng thc G
n
A
n
suy ra
x
n
=
2n+1

1 x ... x
2n

1 +... +x
2n
2n + 1
.
1.2.7. Bt ng thc v phi l h qu trc tip ca G
n
A
n
. C th
chng minh bt ng thc cn li bng quy np. R rng bt ng thc ng
vi n = 1, gi s n ng vi n, ta i chng minh bt ng thc ng vi
n +1. Tc l ta cn chng minh rng (a
1
a
n+1
)
n+1
(a
1
... a
n
a
n+1
)
n
bit
(a
1
a
n
)
n
(a
1
... a
n
)
2
. Qu vy, ta c
(a
1
a
n+1
)
n+1
a
1
a
n
(a
1
... a
n
)
2

_
a
n+1
a
n
_
n+1
.
Suy ra ta ch cn phi chng minh rng
a
1
a
n+1
n+1
a
n
n
a
2
n+1
.
Nh-ng bt ng thc cui cng c th vit d-i dng
a
1
_
1 +
d
a
1
+ (n 1)d
_
n1
a
1
+ (n 1)d,
trong a
n
= a
1
+ (n + 1)d, nn s dng quy np ta c th chng minh bt
ng thc ny d dng, t suy ra iu phi chng minh.
1.2.8. y l mt h qu trc tip ca kt qu tr-c.
1.2.9. S dng bt ng thc gia trung bnh cng v iu ho.
1.2.10.
(a) S dng bt ng thc gia trung bnh cng v iu ho ta -c
n
_
n

k=1
1
a
k
_
1

1
n
n

k=1
a
k
,
c th, ta c
n

k=1
1
a
k

n
2
s
.

n

C
h
i
134 Ch-ng 1. S thc
T-ng t nh- vy, t bt ng thc
n
_
n

k=1
1
s a
k
_
1

1
n
n

k=1
(s a
k
)
ta suy ra
n

k=1
1
s a
k

n
2
s(n 1)
.
S dng bt ng thc trn cng cc ng thc
n

k=1
a
k
s a
k
= s
n

k=1
1
s a
k
n v
n

k=1
s a
k
a
k
= s
n

k=1
1
a
k
n
suy ra iu phi chng minh.
(b) Xem li gii phn (a).
(c) Chng minh t-ng t cu (a).
1.2.11. S dng bt ng thc
1+a
k
2


a
k
.
1.2.12. Ta c
n

k=1
a
2
k
n

k=1
b
2
k

_
n

k=1
a
k
b
k
_
2
=
n

k,j=1
a
2
k
b
2
j

n

k,j=1
a
k
b
k
a
j
b
j
=
1
2
n

k,j=1
(a
k
b
j
b
k
a
j
)
2
0.
1.2.13. Bt ng thc ny t-ng -ng vi
n

k,j=1
(a
k
a
j
+b
k
b
j
)
n

k,j=1
_
a
2
k
+b
2
k
_1
2
_
a
2
j
+b
2
j
_1
2
,
y l mt h qu trc tip ca bt ng thc
(3)
hin nhin a
k
a
j
+ b
k
b
j

(a
2
k
+b
2
k
)
1
2
(a
2
j
+b
2
j
)
1
2
.
1.2.14. Suy ra t bt ng thc Cauchy.
(3)
Ta gi y l bt ng thc Buniakovskii - Cauchy - Schwarz.

n

C
h
i
1.2. Mt s bt ng thc s cp 135
1.2.15.
(a) Theo bt ng thc Cauchy ,
n

k=1
a
k
n

k=1
1
a
k

_
n

k=1
_
a
k
1
a
k
_
2
= n
2
.
(b) T (a) suy ra
n

k=1
a
k
n

k=1
1 a
k
a
k
=
n

k=1
a
k
n

k=1
1
a
k
n
n

k=1
a
k
n
2
n
n

k=1
a
k
= n
n

k=1
(1 a
k
).
(c) Theo gi thit ca ta th log
a
a
1
+log
a
a
2
+... +log
a
a
n
= 1. iu ny kt
hp vi bt ng thc Cauchy (bi 1.2.12) suy ra iu phi chng minh.
1.2.16. Bt ng thc cn chng minh t-ng -ng vi
0 4

k=1
a
k
b
k

+ 4
n

k=1
a
2
k
+
2
n

k=1
b
2
k
,
bt ng thc ny ng vi mi thc, bi v
= 16
_
n

k=1
a
k
b
k
_
2
16
n

k=1
a
2
k
n

k=1
b
2
k
0.
1.2.17. S dng bt ng thc Cauchy ta c
n

k=1
|a
k
| =
n

k=1
1|a
k
|

n
_
n

k=1
a
2
k
_1
2

n
n

k=1
|a
k
|.
1.2.18.
(a) S dng bt ng thc Cauchy ta c
n

k=1
(a
k
b
k
)
2
=
_
n

k=1

ka
k
b
k

k
_
2

k=1
ka
2
k
n

k=1
b
2
k
k
.

n

C
h
i
136 Ch-ng 1. S thc
(b) T-ng t
_
n

k=1
a
k
k
_
2
=
_
n

k=1
k
3
2
a
k
k
5
2
_
2

k=1
k
3
a
2
k
n

k=1
1
k
5
.
1.2.19. T bt ng thc Cauchy suy ra
_
n

k=1
a
p
k
_
2
=
_
n

k=1
a
p+q
2
k
a
pq
2
k
_
2

k=1
a
p+q
k
n

k=1
a
pq
k
.
1.2.20. S dng bt ng thc Cauchy ta -c
n

k=1
a
2
k
n =
n

k=1
a
2
k
n

k=1
1
_
n

k=1
a
k
_
2
= 1.
V
n

k=1
a
2
k

1
n
, ng thc vi a
k
=
1
n
, k = 1, 2, ..., n. Do gi tr b nht cn
tm l
1
n
.
1.2.21. Hon ton t-ng t nh- li gii ca bi ton trn ta c
1 =
_
n

k=1
a
k
_
2
=
_
n

k=1

p
k
a
k
1

p
k
_
2

k=1
p
k
a
2
k
n

k=1
1
p
k
.
Do vy,
n

k=1
p
k
a
2
k

1
n

k=1
1
p
k
du ng thc xy ra khi a
k
=
1
p
k
_
n

k=1
1
p
k
_
1
, suy ra gi tr nh nht cn tm
l
_
n

k=1
1
p
k
_
1
.
1.2.22. T li gii ca bi ton 1.2.20 suy ra
_
n

k=1
a
k
_
2
n
n

k=1
a
2
k
.

n

C
h
i
1.2. Mt s bt ng thc s cp 137
Do
_
n

k=1
a
k
_
2
=
_
(a
1
+a
2
) +
n

k=3
a
k
_
2
(n 1)
_
(a
1
+a
2
)
2
+
n

k=3
a
2
k
_
= (n 1)(
n

k=1
a
2
k
+ 2a
1
a
2
).
1.2.23.
(a) S dng bt ng thc Cauchy,
_
n

k=1
(a
k
+b
k
)
2
_1
2
=
_
n

k=1
_
a
2
k
+ 2a
k
b
k
+b
2
k
_
_1
2

_
_
n

k=1
a
2
k
+ 2
_
n

k=1
a
2
k
_1
2
_
n

k=1
b
2
k
_1
2
+
n

k=1
b
2
k
_
_
1
2
=
_
n

k=1
a
2
k
_1
2
+
_
n

k=1
b
2
k
_1
2
(b) S dng cu (a) ta -c
_
n

k=1
a
2
k
_1
2

_
n

k=1
b
2
k
_1
2

_
n

k=1
(a
k
b
k
)
2
_1
2
.
T bt ng thc trn cng vi bt ng thc nu trong bi 1.2.17 ta suy
ra
_
n

k=1
a
2
k
_1
2

_
n

k=1
b
2
k
_1
2

k=1
|a
k
b
k
|.
T-ng t
_
n

k=1
b
2
k
_1
2

_
n

k=1
a
2
k
_1
2

k=1
|a
k
b
k
|
v ta c iu phi chng minh.

n

C
h
i
138 Ch-ng 1. S thc
1.2.24. V
n

k=1
p
k
a
k
= 1 nn 1 =
n

k=1
p
k
a
k
=
n

k=1
(p
k
)a
k
+
n

k=1
a
k
vi
mi s thc . S dng bt ng thc Cauchy ta c
1
_
n

k=1
(p
k
)
2
+
2
_
_
_
n

k=1
a
2
k
+
_
n

k=1
a
k
_
2
_
_
,
do
n

k=1
a
2
k
+
_
n

k=1
a
k
_
2

_
n

k=1
(p
k
)
2
+
2
_
1
.
t =
1
n+1
n

k=1
p
k
, ta thu -c cn d-i ng. Do
n

k=1
a
2
k
+
_
n

k=1
a
k
_
2

n + 1
(n + 1)
n

k=1
p
2
k

_
n

k=1
p
k
_
2
,
bt ng thc tr thnh ng thc khi
a
k
=
(n + 1)p
k

n

k=1
p
k
(n + 1)
n

k=1
p
2
k

_
n

k=1
p
k
_
2
.
1.2.25. S dng quy np. Vi n = 1 ta c ng thc a
1
b
1
= a
1
b
1
. Hn na,
nu bt ng thc ng vi n, th
n+1

k=1
a
k
n+1

k=1
b
k
(n + 1)
n+1

k=1
a
k
b
k
a
n+1
n

k=1
b
k
+b
n+1
n

k=1
a
k
na
n+1
b
n+1

n

k=1
a
k
b
k
=
n

k=1
(b
n+1
b
k
)(a
k
a
n+1
) 0.
Theo gi thit quy np suy ra iu phi chng minh.

n

C
h
i
1.2. Mt s bt ng thc s cp 139
1.2.26. S dng quy np theo p. Vi p = 1, ng thc a
p
1
= a
p
1
lun ng.
Gi s bt ng thc ng vi p, ta s chng minh n ng vi p +1. R rng,
khng mt tnh tng qut, ta c th gi s rng cc s a
k
-c sp xp theo th
t sao cho a
1
a
2
... a
n
, theo gi thit quy np v kt qu ca bi ton
tr-c ta suy ra
_
1
n
n

k=1
a
k
_
p+1

1
n
2
n

k=1
a
p
k
n

k=1
a
k

1
n
n

k=1
a
p+1
k
.
1.2.27. Ta c
(1 +c)a
2
+
_
a +
1
c
_
b
2
= a
2
+b
2
+
_

ca
1

c
b
_
2
+ 2ab (a +b)
2
.
1.2.28. R rng

a
2
+b
2
+

a
2
+c
2
|b| +|c| |b +c|. Do ta suy ra
bt ng thc |b
2
c
2
| |b c|
_
a
2
+b
2
+

a
2
+c
2
_
v n t-ng -ng
vi bt ng thc cn chng minh.
1.2.29.
(a) Vi cc s thc a, b, c ta c a
2
+ b
2
+ c
2
ab + bc + ca. Do b
2
c
2
+
a
2
c
2
+a
2
b
2
abc(a +b +c), t-ng -ng vi kt lun ca ta.
(b) iu phi chng minh -c suy ra t bt ng thc a
2
+ b
2
+c
2
ab +
bc +ca theo cch hon ton t-ng t nh- trong (a).
(c) y l kt qu ca bt ng thc lin h gia trung bnh cng v trung
bnh iu ho.
(d) Ta c
b
2
a
2
c +a
=
b +a
c +a
(b a) =
b +a
c +a
((b +c) (c +a)).
t u = a +b, v = b +c v z = c +a ta thu -c
b
2
a
2
c +a
+
c
2
b
2
a +b
+
a
2
c
2
b +c
=
u
z
(v z) +
v
u
(z u) +
z
v
(u v)
=
u
2
v
2
+v
2
z
2
+z
2
u
2
(u
2
vz +v
2
uz +z
2
uv)
uvz
=
u
2
(v
2
+z
2
) +v
2
(u
2
+z
2
) +z
2
(u
2
+z
2
) 2(u
2
vz +v
2
uz +z
2
uv)
2uvz
0.

n

C
h
i
140 Ch-ng 1. S thc
(e) Vi a = b, bt ng thc l hin nhin. Gi s 0 < b < a, th th
a b
2

a
=
_

b
__

a +

b
_
2

a
<

b <
a b
2

b
v do
(a b)
2
4a
<
_

b
_
2
<
_

b
_
2
= a+b2

ab <
(a b)
2
4b
.
1.2.30. t m =
a
i
b
i
. Th th
m(b
1
+... +b
n
) =
a
i
b
i
(b
1
+b
2
+... +b
n
) =
a
i
b
i
b
1
+
a
i
b
i
b
2
+... +
a
i
b
i
b
n

a
1
b
1
b
1
+
a
2
b
2
b
2
+... +
a
n
b
n
b
n
= a
1
+a
2
+... +a
n
M(b
1
+b
2
+... +b
n
).
1.2.31. Bt ng thc suy ra t kt qu trong bi ton tr-c v t tnh n
iu ca hm tan x trn khong (0, /2).
1.2.32. p dng bt ng thc cho trong 1.2.30 vi a
i
= lnc
i
v b
i
= k
i
, i =
1, 2, ..., n.
1.2.33. Ch rng
a
1
b
1
M,
a
2
2
Mb
2
2
M, ...,
a
n
n
M
n1
b
n
n
M
v s dng bt ng thc -c chng minh trong 1.2.30 suy ra iu phi chng
minh.
1.2.34. S dng bt ng thc lin h gia trung bnh cng v trung bnh
iu ho (xem v d 1.2.3) ta -c
n
1
xa
1
+
1
xa
2
+... +
1
xan

(x a
1
) + (x a
2
) +... + (x a
n
)
n
=
nx (a
1
+a
2
+... +a
n
n
.
t suy ra kt qu cn chng minh rt d dng.
1.2.35. Ta c
1 +c
1
+c
2
+... +c
n
= (1 + 1)
n
= 2
n
,
p dng bt ng thc Cauchy (xem 1.2.12) vi a
k
= 1 v b
k
=

c
k
, k =
1, 2, ..., n.

n

C
h
i
1.2. Mt s bt ng thc s cp 141
1.2.36. V
n

k=0
_
n
k
_
=
n1

k=1
_
n
k
_
v 2
n
2 =
n1

k=1
_
n
k
_
,
s dng bt ng thc lin h gia trung bnh cng v trung bnh iu ho ta
suy ra iu phi chng minh (xem 1.2.3).
1.2.37. S dng bt ng thc lin h gia trung bnh cng v trung bnh
iu ho (xem 1.2.3) ta -c
A
p1
k
A
k1

(p 1)A
p
k
+A
p
k1
p
, k = 1, 2, ..., n
trong A
0
= 0. T suy ra
A
p
k

p
p 1
A
p1
k
a
k
= A
p
k

p
p 1
A
p1
k
(kA
k
(k 1)A
k1
)
= A
p
k
_
1
kp
p 1
_
+A
p1
k
A
k1
(k 1)p
p 1
A
p
k
_
1
kp
p 1
_
+
k 1
p 1
_
(p 1)A
p
k
+A
p
k1
_
=
1
p 1
_
(k 1)A
p
k1
kA
p
k
_
.
Cng cc bt ng thc ta -c iu cn chng minh.
1.2.38. Gi s a
i
= max {a
1
, a
2
, ..., a
n
}, khi ta c
n1

k=1
a
k
a
k+1
=
i1

k=1
a
k
a
k+1
+
n1

k=i
a
k
a
k+1
a
i
i1

k=1
a
k
+a
i
n1

k=i
a
k+1
= a
i
(a a
i
) =
a
2
4

_
a
2
a
i
_
2

a
2
4
.
1.2.39. p dng kt qu trong 1.2.2
1.2.40. Bt ng thc v tri suy ra t 1.2.1
(a) R rng
1 +a
k
=
1 a
2
k
1 a
k
<
1
1 a
k
.
Do
n

k=1
(1 +a
k
) <
_
n

k=1
(1 a
k
)
_
1
.

n

C
h
i
142 Ch-ng 1. S thc
V a
1
+a
2
+... +a
n
< 1, p dng mt ln na kt qu trong 1.2.1 ta c
n

k=1
(1 +a
k
) <
_
1
n

k=1
a
k
_
1
.
(b) Lp lun nh- cu (a).
1.2.41. p dng bt ng thc cho trong 1.2.15 (b), thay a
k
bng 1 a
k
.
1.2.42. V 0 < a
k
1 vi k = 1, 2, ..., n bt ng thc
n

k=1
a
k

n

k=1
a
k

n

k=1
1
a
k
(1)
ng vi n 2. By gi ta s dng bt ng thc 1.2.15 (b), trong ta thay
a
k
bi
a
k
1+a
k
, k = 1, 2, ..., n -c
n

k=1
1
a
k
_
n
n

k=1
1
1 +a
k
_
n
n

k=1
1
1 +a
k
.
Nhn c hai v ca bt ng thc ny vi
n

k=1
a
k
v s dng (1), ta -c kt qu
cn chng minh.
1.2.43.
(a) S dng bt ng thc lin h gia trung bnh cng v trung bnh iu ho
ta -c
n

k=1
(1 +a
k
)
(n + 1)
n
=
2a
1
+a
2
+... +a
n
n + 1

a
1
+ 2a
2
+... +a
n
n + 1
...
a
1
+a
2
+... + 2a
n
n + 1

n

k=1
a
k
.
(b) Chng minh ca phn ny ging nh- trong (a).
1.2.44. Ch rng nu
n

k=1
1
1+a
k
= n 1 th
n

k=1
a
k
1+a
k
= 1. nhn -c
kt qu cn chng minh, ta s dng bt ng thc trong 1.2.43 (b) vi a
k
-c
thay bi
a
k
1+a
k
.

n

C
h
i
1.2. Mt s bt ng thc s cp 143
1.2.45. [M. S. Klamkin, Amer. Math. Monthly 82 (1975), 741-742] Ta c
th gi s rng a
1
, a
2
, ..., a
n
-c sp xp sao cho a
1
= min{a
1
, a
2
, ..., a
n
}
v a
2
= max {a
1
, a
2
, ..., a
n
}, v gi s A
n
= 1/n l trung bnh cng ca
a
1
, ..., a
n
. Xt mt dy
_
a

k
_
bng cch t a

1
= A
n
, a

2
= a
1
+a
2
A
n
, a

i
=
a
i
vi 3 i n. Ta s chng minh rng
n

k=1
1 +a
k
1 a
k

k=1
1 +a

k
1 a

k
. (1)
T cch xc nh ca dy
_
a

k
_
ta suy ra (1) t-ng -ng vi
(1 +a
1
)(1 +a
2
)
(1 a
1
)(1 a
2
)

(1 +A
n
)(1 +a
1
+a
2
A
n
)
(1 A
n
)(1 a
1
a
2
+A
n
)
tc l t-ng -ng vi
(A
n
a
1
)(A
n
a
2
) 0.
Bt ng thc cui cng l mt h qu trc tip ca gi thit trn. Gi ra lp
li cc th tc trn cho dy
_
a

n
_
c dy
_
a

n
_
; C t nht hai hng t ca
dy
_
a

n
_
bng A
n
. Hn th, dy tho mn bt ng thc (1). Nu ta lp li
th tc ny nhiu nht n 1 ln, ta c dy hng m cc phn t u bng
A
n
. S dng bt ng thc (1) trong tr-ng hp ny ta -c
n

k=1
1 +a
k
1 a
k

k=1
1 +A
n
1 A
n
=
_
n + 1
n 1
_
n
.
1.2.46. Gi a
k
1
= max {a
1
, a
2
, ..., a
n
} . Tn ti mt phn s v tri ca bt
ng thc c t s bng a
k
1
. Mu s ca phn s ny c hai hng t. K hiu
hng t ln hn l a
k
2
. Ly phn s c t s l a
k
2
v k hiu a
k
3
l hng t ln
hn trong hai hng t trong mu s ca n , ... Ch rng
a
a
k
i
a
k
i
+1
+a
k
i
+2

a
k
i
2a
k
i+1
, i = 1, 2, .... (1)
T cch xy dng trn suy ra tn ti mt s l sao cho a
k
l+1
= a
k
1
. Tip
theo, r rng cc s a
k
i
v a
k
i+1
xut hin trong bt ng thc nh- l t s ca
hoc l hai phn s ln cn hoc l ca hai phn s cch nhau bi mt hng
t (ta gi s y rng hai phn s u v cui l cc phn s ln cn). Hn
na, a
k
i+1
xut hin nh- l t s ca phn s bn phi phn s c t s a
k
i
.

n

C
h
i
144 Ch-ng 1. S thc
chuyn t phn s c ch s a
k
1
n phn s c ch s a
k
l
+1
cn l b-c, trong
l
n
2
. Do , t (1) v bt ng thc lin h gia trung bnh cng v trung
bnh iu ho ta -c
a
k
1
2a
k
2
+
a
k
2
2a
k
3
+... +
a
k
l
2a
k
1
l
l
_
1
2
l

n
4
.
1.2.47. Ta c
n

k=1
_
|a
k
t|
2
k

_
1
2
2
+
1
2
3
+... +
1
2
n
_
_
|a
1
t| +
_
|a
2
t|
2
2
+... +
_
|a
n
t|
2
n

1
2
2
_
_
|a
1
t| +
_
|a
2
t|
_
+
1
2
3
_
_
|a
1
t|
+
_
|a
3
t|
_
+... +
1
2
n
_
_
|a
1
t| +
_
|a
n
t|
_
suy ra bt ng thc cn chng minh.
1.2.48. S dng bt ng thc lin h gia trung bnh nhn v trung bnh
cng ta -c
n
_
a
1
a
1
+b
1

a
2
a
2
+b
2
...
a
n
a
n
+b
n
+
n
_
b
1
a
1
+b
1

b
2
a
2
+b
2
...
b
n
a
n
+b
n

1
n
_
a
1
a
1
+b
1
+...
a
n
a
n
+b
n
+
b
1
a
1
+b
1
+... +
b
n
a
n
+b
n
_
= 1
1.2.49. [V. Ptak, Amer. Math. Monthly 102 (1995), 820-821] Tr-c ht ch
rng khi thay mi a
k
bng ca
k
vi c > 0 th c hai v tri v v phi ca bt
ng thc khng thay i, do ta c th gi thit rng G = 1, suy ra a
n
=
1
a
1
.
Hn na nu a
1
x
1
a
1
th x +
1
x
a
1
+
1
a
1
. Do
n

k=1
p
k
a
k
+
n

k=1
p
k
1
a
k
a
1
+
1
a
1
= 2A.
thu -c iu phi chng minh ta p dng bt ng thc lin h gia trung
bnh cng v trung bnh nhn.
1.2.50. Ta hy sp xp cc -c nguyn d-ng ca n thnh cc cp (k, l) theo
cch sao cho kl = n. S dng bt ng thc lin h gia trung bnh cng v
trung bnh nhn
(4)
suy ra
k+l
2

kl. Cng cc bt ng thc li ta -c


(n)
2

(n)
2

n.
(4)
y l bt ng thc Cauchy i vi k v l.

n

C
h
i
Ch-ng 2
Dy s thc
2.1 Dy n iu
2.1.1.
(a) Cho {a
n
} l dy n iu tng, b chn trn th
sup{a
n
: n N} = A < ,
suy ra vi mi n N, a
n
A. V vi mi > 0, s A khng phi l cn
trn ca tp {a
n
: n N} nn tn ti s a
n
0
sao cho a
n
0
> A. Do tnh n
iu ca dy s ta c : A a
n
> A vi n > n
0
. Do vy, lim
n
a
n
= A.
By gi ta gi s rng a
n
khng b chn trn, khi vi mi M, tn ti a
n
0
sao cho a
n
0
> M, theo tnh n iu ca dy s ta c, a
n
> M vi n > n
0
, v
vy, lim
n
a
n
= +.
(b) H qu ca (a).
2.1.2. Ta c
s
n
s
n1

s
n+1
s
n
vi n 2.
Thc vy, theo1.2.19,
s
2
n
s
n+1
s
n1
. (1)
Ta s chng minh rng {x
n
} l dy tng. Tht vy, t
_
p

i=1
a
k
_
2
p
p

i=1
a
2
k
suy ra x
1
x
2
(h qu ca 1.2.20). Gi s rng x
n1
x
n
th
s
n1
s
n1
n
n
,
145

n

C
h
i
146 Ch-ng 2. Dy s thc
do , theo (1) v (2),
x
n+1
=
n+1

s
n+1

n+1

s
2
n
s
n1

n+1

s
2
n
s
n1
n
n
= x
n
.
2.1.3. Ta c a
n+1
=
n+1
2n
a
n
< a
n
, n > 1. Do {a
n
} l dy gim ngt. T
tnh cht dy b chn d-i, chng hn bi 0, do tn ti lim
n
a
n
= g. Ta thy
g tho mn iu kin g =
1
2
g. Do , g = 0.
2.1.4. Cho b
n
= a
n

1
2
n1
. Ta c b
n+1
b
n
= a
n+1
a
n
+
1
2
n
0. Do
dy {b
n
} hi t, ko theo dy {a
n
} hi t.
2.1.5.
(a) Ta phi chng minh rng dy {a
n
} l dy n iu gim v b chn d-i.
Thc vy,
a
n+1
a
n
=
1

n + 1
_
n + 1 +

n
_
2
< 0.
Ngoi ra, theo bt ng thc -c cho trong phn h-ng dn (chng minh
bng quy np) ta c a
n
> 2(

n + 1

n 1) > 2.
(b) Chng minh t-ng t cu (a).
2.1.6. u tin, theo quy np, chng ta chng minh -c rng
3
2
a
n
2
vi n N v dy {a
n
} l tng ngt, suy ra n hi t. t g = lim
n
a
n
. Do
a
n
=

3a
n1
2 ta c g =

3g 2, suy ra g = 2.
2.1.7. Chng minh -c bng ph-ng php qui np rng a
n
> 2c. Ta c,
a
1
< a
2
. Ngoi ra nu a
n
> a
n1
, th
a
n+1
= (a
n
c)
2
> (a
n1
c)
2
= a
n
.
Bt ng thc cn li -c suy ra t tnh n iu ca hm s f(x) = x
2
trn
R
+
.
2.1.8. Theo bt ng thc Cauchy v gi thit ta c
a
n
+ (1 a
n+1
)
2

_
a
n
(1 a
n+1
) >
1
2
.
Suy ra a
n
a
n+1
> 0, do dy {a
n
} hi t ti g. T iu kin a
n
(1a
n+1
) >
1
4
suy ra g(1 g)
1
4
. Bt ng thc cui cng t-ng -ng vi bt ng thc
(2g 1)
2
0, t suy ra g =
1
2
.

n

C
h
i
2.1. Dy n i u 147
2.1.9. Hin nhin 0 a
n
< 3 vi n 1. Hn na, a
2
n+1
a
2
n
= a
2
n
+
a
n
+ 6 > 0 vi 0 a
n
< 3, do {a
n
} l dy tng b chn trn nn n hi t
. Theo nh ngha ca dy ta c lim
n
a
n
= 3.
2.1.10. Ta c 0 a
n
< 1 vi n 1. chng minh tnh n iu ca dy,
ta lm nh- sau:
W(n) ng vi mi s t nhin n nu hai iu kin sau tho mn:
(i) W(1) ng.
(ii) Nu W(k) ng vi1 k n ko theo W(n + 1) ng.
Gi s rng a
n1
a
n2
v a
n
a
n1
, th
a
n+1
a
n
=
1
3
(a
n
a
n1
+a
3
n1
a
3
n2
) 0.
Suy ra dy hi t. K hiu gii hn l g th ta c g =
1
3
(1 +g +g
3
), Vy
g = 1 hoc g =
1 +

5
2
hoc g =
1

5
2
.
Ch rng cc phn t ca dy l khng m v nh hn
1+

5
2
, do vy
lim
n
a
n
=
1+

5
2
.
2.1.11. Ta c a
n+1
=
n+1
2n+3
a
n
< a
n
, n 1. Theo phn 2.1.3 ta c g = 0.
2.1.12. T a
n+1
=
2n+2
2n+3
a
n
< a
n
, n 1 l dy n iu gim , n b chn
d-i b 0, do n hi t.

n

C
h
i
148 Ch-ng 2. Dy s thc
2.1.13.
(a) R rng {a
n
} l dy n iu tng. Ta cn chng minh rng n b chn
trn. Thc vy,
a
n
= 1 +
1
2
2
+
1
3
2
+. . . +
1
n
2
< 1 +
1
1 2
+
1
2 3
+. . . +
1
(n 1)n
= 1 +
_
1
1
2
_
+
_
1
2

2
3
_
+. . . +
_
1
n 1

1
n
_
= 2
1
n
< 2.
(b) Hin nhin {a
n
} l dy n iu tng. Hn na,
a
n
= 1 +
1
2
2
+
1
3
3
+. . . +
1
n
n
< 1 +
1
2
2
+
1
3
2
+. . . +
1
n
2
.
Nn theo cu (a) dy ny b chn trn.
2.1.14. Vi n 1, ta c
a
n+1
a
n
=
1
_
n(n + 1)
+
1
_
2n(2n + 1)
+
1
_
(2n + 1)(2n + 2)
< 0,
suy ra n l dy n iu gim b chn d-i, do n hi t.
2.1.15. T bt ng thc Cauchy chng ta c
a
n+1

p
_
a
p1
n
a
a
p1
n
=
p

a, n 1.
Do
a
n+1
a
n
=
a
n
p
+
a
pa
p1
n
=
a
p
n
a
pa
p1
n
0, n 2,
suy ra dy {a
n
} hi t v lim
n
a
n
=
p

a.
2.1.16. D dng thy rng, 0 a
n
< 2 vi n 1. Hn na, nu a
n
> a
n1
th
a
2
n+1
a
2
n
=

a
n

a
n1
> 0, khi a
n
> a
n1
.
Do dy hi t ti g no tho mn ph-ng trnh g =
_
2 +

g.
Ch . S dng cng thc Cardano v nghim thc ca a thc bc ba, c th
ch ra rng
g =
1
3
_
3
_
1
2
(79 + 3

249) +
3
_
1
2
(79 3

249) 1
_
.

n

C
h
i
2.1. Dy n i u 149
2.1.17. Ch rng a
n+1
= 2
_
2
5
an+3
_
, n 1. Bng ph-ng php qui
np ta c th ch ra rng 0 < a
n
< 2 vi n 1. Hn na,
a
n+1
a
n
=
(a
n
+ 1)(a
n
2)
a
n
+ 3
0.
Do dy hi t v lim
n
a
n
= 2.
2.1.18. Bng qui np c th ch ra rng dy {a
n
} tng ngt. Nu n b chn
trn th s tn ti s g tho mn g = lim
n
a
n
, hay g
2
2g+c = 0. Ph-ng trnh
ny c nghim thc khi v ch khi c 1, v vy nu gi s rng 0 < c 1.
Suy ra dy {a
n
} b chn trn bi 1

1 c, v lim
n
a
n
= 1

1 c.
Tr-ng hp c > 1 th dy tng ngt, v do phn k v +.
2.1.19. T
a
n+1
= a
n
_
1 2
a
2
n
a
3a
2
n
+a
_
, n 1,
ta c
Nu a
n
>

a th a
n+1
< a
n
,
Nu a
n
<

a th a
n+1
> a
n
,
Nu a
n
=

a th a
n+1
= a
n
.
Ta nhn thy rng
a
n
a
2
n
+ 3a
3a
2
n
+a
>

a khi v ch khi
_
a
n

a
_
3
> 0,
tc l a
n
>

a. Do ta c cc kt lun
Nu 0 < a
1
<

a th {a
n
} l dy tng v b chn trn bi

a,
Nu a
1
>

a th {a
n
} l dy gim v b chn d-i bi

a,
Nu a
1
=

a th {a
n
} l dy hng.
Trong mi tr-ng hp trn dy u hi t ti

a.

n

C
h
i
150 Ch-ng 2. Dy s thc
2.1.20. Theo qui np ta c
a
n
=
(3
n1
1) (3
n1
3)a
1
(3
n
1) (3
n
3)a
1
vi n = 1, 2, 3 . . . .
Do dy khng xc nh vi a
1
=
3
n+1
1
3
n+1
3
vi n N. Khi a
1
= 1 th a
n
= 1
vi n = 1, 2, 3 . . . . Vi cc gi tr khc ca a
1
, dy hi t ti
1
3
.
2.1.21. Ta c a
n+1
= (a
n
a)
2
+ a
n
a
n
vi n = 1, 2, 3 . . . . Do
dy l n iu tng. Hn na, nu dy hi t th lim
n
a
n
= a. Do nu
a
1
> a, th dy cho phn k. Trong tr-ng hp a 1 a
1
a, ta cng
c a 1 a
n
a vi n = 1, 2, 3 . . . , do dy hi t. Cui cng, nu
a
1
< a 1, th a
2
> a, suy ra dy phn k.
2.1.22. R rng nhn thy rng dy ch c th hi t ti a hoc b, xt cc
tr-ng hp sau.
1
0
c > b. Khi a
2
=
c
2
+ab
a+b
> c = a
1
, s dng qui np ta c a
n+1
> a
n
.
Do , lim
n
a
n
= +.
2
0
c = b. Hin nhin, a
n
= b vi n = 1, 2, 3 . . . .
3
0
a < c < b. Ta c th chng minh theo qui np rng dy {a
n
} l n
iu gim v b chn d-i bi a, do lim
n
a
n
= a.
4
0
c = a. D dng thy rng, a
n
= a vi n = 1, 2, 3 . . . .
5
0
0 < c < a. S dng qui np ta c th ch ra rng {a
n
} l n iu tng
v b chn trn bi a. Do lim
n
a
n
= a.
2.1.23. Ch rng a
n+1
= 6
_
1
6
an+7
_
vi n N. Do theo qui np ta
c
Nu a
1
< 2 th a
n
< 2, n N
Nu a
1
> 2 th a
n
> 2, n N.
Hn na
a
n+1
a
n
=
(a
n
+ 3)(a
n
2)
a
n
+ 7
.
Do

n

C
h
i
2.1. Dy n i u 151
1
0
Nu 0 < a
1
< 2 th {a
n
} l dy tng v b chn trn bi 2 v
lim
n
a
n
= 2,
2
0
Nu a
1
> 2 th {a
n
} l dy gim v b chn d-i bi 2 v lim
n
a
n
= 2,
3
0
Nu a
1
= 2 th a
n
= 2, vi n N.
2.1.24. T 0 = a
1
a
2
v a
2
n+1
a
2
n
= a
n
a
n1
, do theo qui np ta
thy rng a
n+1
a
n
vi n N. Mt khc dy b chn trn bi

1 + 4c. D
dng tnh -c rng lim
n
a
n
=
1+

1+4c
2
.
2.1.25. V a
2
=
_
2

2 >

2 = a
1
v a
2
n+1
a
2
n
= 2(a
n
a
n1
), theo qui
np ta nhn thy rng a
n+1
a
n
vi n N, ng thi dy b chn trn bi 2,
do lim
n
a
n
= 2.
2.1.26. Vi k = 1 ta c a
n
= 5
n
vi n N, do dy {a
n
} phn k.
Vi k > 1,
a
2
=
k
_
5
k

5 >
k

5 = a
1
v a
k
n+1
a
k
n
= 5(a
n
a
n1
).
Theo qui np {a
n
} tng ngt. Hn na a
n
<
k1

5, n N, t suy ra
lim
n
a
n
=
k1

5.
2.1.27. Theo qui np, ta thy 1 a
n
2, n N. Tnh n iu ca dy
s suy ra t ng thc a
2
n+1
a
2
n
= 3(a
n
a
n1
), do , vi 1 < a
1
< 2 dy
n iu tng v gii hn ca n bng 2, ngoi ra, nu a
1
= 1 hoc a
1
= 2 th
n l dy hng.
2.1.28.
(a) Ta c a
1
< a
2
v a
2
n+1
a
2
n
= a
n
a
n1
theo qui np ta nhn thy rng
{a
n
} l dy n iu tng v b chn trn bi c. R rng lim
n
a
n
= c.
(b) T b
2
=
_
c

c >

c = b
1
v b
2
n+1
b
2
n
= c(b
n
b
n1
) theo qui np
ta nhn thy rng dy l n iu tng v b chn trn bi c, suy ra
lim
n
a
n
= c.
2.1.29. S dng qui np chng minh -c rng 0 < a
n
< b, v l dy tng
ngt; Gii hn ca n bng b.

n

C
h
i
152 Ch-ng 2. Dy s thc
2.1.30. Dy l tng ngt v b chn trn bi mt s hu hn, v d 3, chng
minh -c gii hn ca n l
3+

15
3
.
2.1.31. Chng ta c a
1
< a
2
< a
3
. Hn na, ta nhn thy rng vi bt k
n N,
nu a
n
< a
n+1
< a
n+2
th a
n+2
< a
n+3
,
do theo ph-ng php qui np trong cch gii bi ton 2.1.10 ta nhn thy
rng dy {a
n
} tng cht, mt khc n b chn trn bi 4 suy ra lim
n
a
n
= 4.
2.1.32. Theo cch gii ca nhng bi ton tr-c, chng ta c th ch ra rng
dy {a
n
} n iu gim, b chn d-i bi 4, v lim
n
a
n
= 4.
2.1.33. Theo bt ng thc Cauchy, a
n
b
n
. Ta c
a
n+1
=
a
n
+b
n
2
a
n
, n N.
Do dy {a
n
} n iu gim. Mt khc, dy {b
n
} tng v
b
n+1
=
_
b
n
a
n

_
b
2
n
= b
n
, n N
Ngoi ra, b
1
< a
n
, b
n
< a
1
, do c hai dy u hi t. t = lim
n
a
n
v
= lim
n
b
n
. Ta c a
n+1
=
an+bn
2
cho n ta -c =
+
2
, suy ra = .
2.1.34. T 2(a
2
n
+b
2
n
) (a
n
+b
n
)
2
ta c a
n
b
n
, n N Do
a
n+1
=
a
2
n
+b
2
n
a
n
+b
n

a
2
n
+a
n
b
n
a
n
+b
n
= a
n
n N
iu c ngha l dy {a
n
} n iu gim
Hon ton t-ng t ta c dy {b
n
} n iu tng. Ngoi ra, b
1
< a
n
, b
n
<
a
1
. Do c hai dy u hi t.
t = lim
n
a
n
v = lim
n
b
n
. Ta c b
n+1
=
an+bn
2
cho n ta -c
=
+
2
, suy ra = .
2.1.35. Theo bt ng thc lin h gia trung bnh cng v trung bnh nhn
(tc l bt ng thc Cauchy) ta c a
n
b
n
. Ta c
a
n+1
=
a
n
+b
n
2
a
n
, n N

n

C
h
i
2.1. Dy n i u 153
Do dy {a
n
} n iu gim. Mt khc, dy {b
n
} n iu tng bi v
b
n+1
=
2a
n
b
n
a
n
+b
n
b
n
, n N
Hn na, b
1
< a
n
, b
n
< a
1
do c hai dy u hi t.
t = lim
n
a
n
v = lim
n
b
n
. Ta c a
n+1
=
an+bn
2
, cho n ta -c
=
+
2
, do vy = .
Vi nhn xt rng a
n+1
b
n+1
= a
n
b
n
ta suy ra tt c cc phn t ca dy
{a
n
b
n
} u bng a
1
b
1
, do = =

a
1
b
1
.
2.1.36. Ta c a
n+1
=
n+2
2(n+1)
(a
n
+ 1) n N. Suy ra
a
n+1
a
n
=
na
n
+ (n + 2)
2(n + 1)
.
S dng bt ng thc na
n
> n + 2 vi n 1 (c th chng minh bng qui
np) ta thy rng dy l n iu gim, v do n hi t. t = lim
n
a
n
,
t ph-ng trnh a
n+1
=
n+2
2(n+1)
(a
n
+ 1) cho n ta -c = 1.
2.1.37. T bt ng thc a
n+2

1
3
a
n+1
+
2
3
a
n
ta c a
n+2
+
2
3
a
n+1
a
n+1
+
2
3
a
n
. Do dy b
n
= a
n+1
+
2
3
a
n
l dy gim, b chn, v do hi t. t b
l gii hn ca n, chng ta s ch ra rng{a
n
} hi t ti a =
3
5
b. Vi > 0
tu , tn ti n
0
N sao cho

6
> |b
n
b| vi n n
0
. Do ,

6
>

a
n+1
+
2
3
a
n

5
3
a

|a
n+1
a|
2
3
|a
n
a| vi n n
0
.
Do vy |a
n+1
a| <
2
3
|a
n
a| +

6
. Theo qui np ta c
|a
n
0
+k
a|
_
2
3
_
k
|a
n
0
a| +
_
_
2
3
_
k1
+. . . +
2
3
+ 1
_

6

_
2
3
_
k
|a
n
0
a| +
1
2
3
k
1
2
3

6
<
2
3
k
|a
n
0
a| +

2
.
T
_
2
3
_
k
|a
n
0
a| <

2
vi k ln suy ra |a
n
a| < vi n ln.
2.1.38.
(a) b
n
=
_
1 +
1
n
_
n+1
= (1 +
1
n
)a
n
> a
n
.

n

C
h
i
154 Ch-ng 2. Dy s thc
(b) Theo bt ng thc Cauchy G
n+1
< A
n+1
(xem Bi tp 1.2.3) vi a
1
=
1, a
2
= a
3
= . . . = a
n+1
= 1 +
1
n
ta c
n+1

_
1 +
1
n
_
n
< 1 +
1
n + 1
.
Do
_
1 +
1
n
_
n
<
_
1 +
1
n + 1
_
n+1
, n N.
(c) S dng bt ng thc lin h gia trung bnh nhn v trung bnh iu ho
H
n+1
< G
n+1
, n > 1 (xem Bi tp 1.2.3), trong a
1
= 1, a
2
= a
3
=
. . . = a
n+1
= 1 +
1
n1
ta -c
1 +
1
n
<
n+1

_
n
n 1
_
n
,
suy ra b
n
< b
n1
n > 1. Nhn thy rng a
1
a
n
< b
n
b
1
, n N do
hai dy {a
n
} v {b
n
} hi t. Ngoi ra, lim
n
b
n
= lim
n
_
1 +
1
n
_
a
n
=
lim
n
a
n
.
2.1.39.
(a) Theo bt ng thc Cauchy ta c G
n+1
< A
n+1
(xem Bi tp 1.2.3), vi
a
1
= 1, a
2
= a
3
= . . . = a
n+1
= 1 +
x
n
, n N ta nhn thy dy
tng ngt
Nu 0 < x 1, th theo bi tr-c,
_
1 +
x
n
_
n

_
1 +
1
n
_
n
< e.
Nu x > 1 th tn ti s nguyn d-ng n
0
sao cho x n
0
. Do , theo
tnh n iu ca dy
__
1 +
n
0
n
_
n
_
v kt qu ca bi tp tr-cta c
_
1 +
x
n
_
n

_
1 +
n
0
n
_
n
<
_
1 +
n
0
nn
0
_
n
0
n
< e
n
0
.
(b) Hon ton t-ng t cu (a) v ch rng x 0, dy b chn trn, v d
bi 1.

n

C
h
i
2.1. Dy n i u 155
2.1.40. S dng bt ng thc H
n+l+1
< G
n+l+1
, n > 1 (xem Bi tp 1.2.3),
vi a
1
= 1, a
2
= a
3
= . . . = a
n+l+1
= 1 +
x
n
, ta c
n+l+1
_
_
1 +
x
n
_
n+l
> 1 +
x(n + l)
n
2
+nl +x +n
> 1 +
x(n +l)
(n + 1)(n +l)
.
Do , b
n
> b
n+1
, n N.
2.1.41. Theo bt ng thc -c cho trong phn h-ng dn,
a
n+1
a
n
=
1
n
log
n + 1
n
> 0,
b
n+1
b
n
=
1
n + 1
log
n + 1
n
< 0.
D dng thy rng a
1
a
n
< b
n
b
1
n N, suy ra c hai dy hi t ti
cng gii hn.
Ch . K hiu log trong bi chnh l lga vi c s t nhin m ta th-ng k
hiu l ln.
2.1.42. D dng nhn thy tnh n iu v tnh b chn ca dy {a
n
}. T
ng thc a
2
n+1
= a
n
ta thy rng gii hn ca n bng 1. Tip theo ta kim
tra tnh n iu ca dy {c
n
}. u tin gi s rng x > 1 th
c
n
= 2
n
(a
n
1) = 2
n
(a
2
n+1
1) = 2
n
(a
n+1
1)(a
n+1
+ 1)
= 2
n+1
(a
n+1
1)
a
n+1
+ 1
2
> c
n+1
.
iu c ngha l vi x > 1 dy {c n} tng ngt, hon ton t-ng t cho
tr-ng hp 0 < x < 1. Vi x = 1, dy l hng s. Tnh n iu ca dy {d
n
}
chng minh hon ton t-ng t.
Vi x > 1 dy {c
n
} hi t (bi v n n iu gim, b chn d-i bi 0). Mt
khc, vi 0 < x < 1, dy {d
n
} l n iu tng v b chn trn bi 0. T bt
ng thc
d
n
=
c
n
a
n
ko theo c hai dy tin ti cng gii hn vi mi s d-ng x = 1. Nu x = 1,
th c
n
= d
n
= 0.

n

C
h
i
156 Ch-ng 2. Dy s thc
2.2 Gii hn. Tnh cht ca dy hi t
2.2.1.
(a) 1.
(b) 1.
(c) -1.
(d) Ta c
0 <
_

2
3

2
_ _

2
5

2
_
. . .
_

2
2n+1

2
_
< (

2 1)
n
.
Do vy gii hn ca dy bng 0.
(e) u tin ta cn chng minh rng dy a
n
=
n
2
2
n
hi t ti 0. Ta c
a
n+1
= a
n
1
2
(n + 1)
2
n
2
< a
n
,
vi n 3, do dy l n iu gim. R rng n b chn d-i bi 0,
nn n hi t v gii hn g ca n tho mn ph-ng trnh g =
1
2
g, suy
ra g = 0. By gi ta i xc nh gii hn ca dy. t k
n
= [

n], th
th k
n


n < k
n
+ 1, do
0 <
n
2

n
< 2
(k
n
+ 1)
2
2
kn+1
.
Do gii hn ca dy cho bng 0.
(f) t a
n
=
n!
2
n
2
, suy ra a
n+1
= a
n
1
2
(n+1)
2
2n
< a
n
, n N, theo cch gii ca
bi 2.1.3 ta -c g = 0.
(g) t
a
n
=
1

n
_
1

1 +

3
+
1

3 +

5
+. . . +
1

2n 1 +

2n + 1
_
.
Ta c
1

2k1+

2k+1
=

2n1

2n+1
2
do a
n
=

2n+11
2

n
. Vy lim
n
a
n
=

2
2
.

n

C
h
i
2.2. Gii hn. Tnh cht ca dy hi t 157
(h) T bt ng thc
(1 + 2 +. . . +n)
1
n
2
+n

1
n
2
+ 1
+
2
n
2
+ 2
+. . . +
n
n
2
+n
(1 + 2 +. . . +n)
1
n
2
+ 1
s dng nguyn l kp ta thy n c gii hn bng
1
2
.
(i) Nh- phn trn gii hn bng
1
2
.
2.2.2. t a
n
=
n
s
(1+p)
n
, ta c
a
n+1
a
n
=
_
n + 1
n
_
s
1
p + 1
.
Ngoi ra, lim
n
_
n+1
n
_
s
1
p+1
=
1
p+1
. Suy ra dy {a
n
} l n iu gim bt u
t ch s n
0
no y, n cng b chn d-i v d bi 0. Gi gii hn ca n bng
g, g tho mn tnh cht g =
1
p+1
g. Do g = 0.
2.2.3. Ta c
0 < (n + 1)

= n

__
1 +
1
n
_

1
_
< n

__
1 +
1
n
_
1
_
=
1
n
1
.
Do vy gii hn ca dy bng 0.
2.2.4. t =
p
q
, vi p Z v q N. Vi n > q s n! l bi ca , iu
c ngha l n mt lc no cc phn t ca dy u bng 0
2.2.5. Gi s gii hn ca dy tn ti, ta c
0 = lim
n
(sin(n + 2) sin n) = 2 sin 1 lim
n
cos(n + 1),
suy ra lim
n
cos n = 0. T-ng t,
0 = lim
n
(cos(n + 2) cos n) = 2 sin 1 lim
n
sin(n + 1),
V l bi v sin
2
n + cos
2
n = 1. Do gii hn ca {sinn} khng tn ti.
2.2.6. Theo cch chng minh ca bi tr-c.

n

C
h
i
158 Ch-ng 2. Dy s thc
2.2.7. Ta c
lim
n
1
n
_
_
a +
1
n
_
2
+
_
a +
2
n
_
2
+. . . +
_
a +
n 1
n
_
2
_
= lim
n
_
n 1
n
a
2
+
n(n 1)
n
2
+
1 + 2
2
+. . . + (n 1)
2
n
3
_
= a
2
+a +
1
3
.
2.2.8. Ta c
a
n
+a
2
n
+. . . +a
k
n
k = (a
n
1) + (a
2
n
1) +. . . + (a
k
n
1)
Hn na
lim
n
a
l
n
1
a
n
1
= l vi l = 1, 2, . . . , k
do gii hn bng 1 + 2 +. . . +k =
k(k+1)
2
.
2.2.9. S dng ng thc
1
k(k + 1)(k + 2)
=
1
2

1
k

1
k + 1
+
1
2

1
k + 2
, k N
Do o gii hn bng
1
4
.
2.2.10. T
k
3
1
k
3
+ 1
=
(k 1)((k + 1)
2
(k + 1) + 1)
(k + 1)(k
2
k + 1)
ta c
n

k=2
k
3
1
k
3
+ 1
=
2
3
n
2
+n + 1
n
2
+n

n
2
3
.
2.2.11.
1
6
.
2.2.12. T 1
2
k(k+1)
=
(k1)(k+2)
k(k+1)
, ta nhn -c
_
1
2
2.3
__
1
2
3.4
_
. . .
_
1
2
(n + 1)(n + 2)
_
=
1
3
n + 3
n + 1

n
1
3
.

n

C
h
i
2.2. Gii hn. Tnh cht ca dy hi t 159
2.2.13. Ta c
k
3
+ 6k
2
+ 11k + 5 = (k + 1)(k + 2)(k + 3) 1,
Do
lim
n
n

k=1
k
3
+ 6k
2
+ 11k + 5
(k + 3)!
= lim
n
n

k=1
_
1
k!

1
(k + 3)!
_
=
5
3
.
2.2.14. Ta nhn thy rng
x
2
k1
1 x
2
k
=
1
1 x
2
k1

1
1 x
2
k
k = 1, 2, . . . , n.
Do
lim
n
n

k=1
x
2
k1
1 x
2
k
=
_
x
1x
nu |x| < 1,
1
1x
nu |x| > 1.
2.2.15. Vi x = 1,
(1 x)(1 +x)(1 +x
2
) . . . (1 +x
2
n
)
1 x
=
1 x
2
n+1
1 x
suy ra,
a
n
=
n

k=0
(1 +x
2
k
) =
_
1x
2
n+1
1x
nu x = 1,
2
n+1
nu x = 1.
Do
lim
n
a
n
=
_

_
nu x < 1,
0 nu x = 1,
1
1x
nu |x| < 1,
+ nu x 1.
2.2.16. Vi x = 1 ta c
a
n
=
n

k=0
_
1 +
2
x
2
k
+x
2
k
_
=
n

k=0
(x
2
k
+ 1)
2
x
2
k+1
+ 1
=
(x + 1)(x 1)(x + 1)(x
2
+ 1) . . . (x
2
n
+ 1)
(x 1)(x
2
n+1
+ 1)
=
x + 1
x 1

x
2
n+1
1
x
2
n+1
+ 1
.

n

C
h
i
160 Ch-ng 2. Dy s thc
Do
lim
n
a
n
=
_

x+1
x1
nu |x| < 1,
x+1
x1
nu |x| > 1,
0 nu x = 1,
+ nu x = 1.
2.2.17. Vi x = 1, th
1 +x
3
k
+x
2.3
k
=
(1 +x
3
k
+x
2.3
k
)(x
3
k
1)
x
3
k
1
=
x
3
k+1
1
x
3
k
1
Do vy
n

k=1
(1 +x
3
k
+x
2.3
k
) =
x
3
n+1
1
x
3
1
.
Gi g l gii hn ca dy, ta c
g =
_

_
1
1x
3
nu |x| < 1,
+ nu |x| > 1,
1 nu x = 1,
+ nu x = 1.
2.2.18. Ta c k k! = (k + 1)! k!, k N. Do
lim
n
1 1! + 2 2! +. . . +n n!
(n + 1)!
= lim
n
(n + 1)! 1
(n + 1)!
= 1.
2.2.19. Ta ch rng bi ton ch c ngha vi x = 0. Theo 2.2.3, mu thc
n
x
(n 1)
x
dn n 0 nu 0 < x < 1. Ngoi ra, nu x < 0 th mu s cng
dn n 0. Vi x = 1 th mu s bng 1. Do dy phn k n v cc (+
hoc ) vi x 1, x = 0. Vi x > 1 nu t k = [x], th k 1 v
1
_
1
1
n
_
k
1
_
1
1
n
_
x
< 1
_
1
1
n
_
k+1
.
T bt ng thc trn ta nhn thy tn ti hai s v sao cho
< n
_
1
_
1
1
n
_
x
_
< ,
Do
n
x1
< n
x
_
1
_
1
1
n
_
x
_
< n
x1
,

n

C
h
i
2.2. Gii hn. Tnh cht ca dy hi t 161
Do vy nu x 1 < 1999 th dy phn k ti + Nu x 1 > 1999 th dy
hi t ti 0. Tip theo ly x = 2000. ta c
lim
n
n
1999
n
2000
(n 1)
2000
=
1
2000
.
2.2.20. Ta c
a
n
=
_
a
n+1
b
n+1
a
n
b
n
nu a > b,
n+1
n
a nu a = b.
Do lim
n
a
n
= a.
2.2.21. Theo qui np ta c a
n
= (n 1)
2
. Do vy lim
n
a
n
= +.
2.2.22. Theo qui np ta c a
n
=
ab

a
2
+nb
2
. Do vy lim
n
a
n
= 0.
2.2.23. Ta nhn thy rng a
n
= 1
_
1
4
_
n1
Do vy lim
n
a
n
= 1.
2.2.24. D dng nhn thy rng a
n+1
= 1 +b +. . . +b
n1
+b
n
a. Do vy
a
n+1
=
_
1
1b
+
_
a
1
1b
_
b
n
vi b = 1,
n +a vi b = 1.
Do vy nub = 1, a R, dy phn k ti +. Nu b = 1 v a =
1
1b
, dy
hi t ti
1
1b
. Trong tr-ng hp a =
1
1b
v |b| < 1 th dy cng hi t ti
1
1b
. Trong cc tr-ng hp cn li dy l phn k, c ngha l nu b 1 v
a =
1
1b
, th dy khng c gii hn hoc gii hn khng hu hn, nu b > 1 v
a >
1
1b
, th dy phn k ti +. Cui cng nu b > 1 v a <
1
1b
, th dy
phn k ti .
2.2.25. Theo qui np d dng thy cng thc ca cc phn t ca dy tho
mn. Ta gi s rng > . Th =
1+

5
2
v =
1

5
2
. Ngoi ra,

n
_

n

n

1 +

n
.
T lim
n

n
= 0, ta c lim
n
n

a
n
= .

n

C
h
i
162 Ch-ng 2. Dy s thc
2.2.26. u tin ch rng b
n+1
=
an+3bn
4
, suy ra a
n+1
b
n+1
=
1
4
(a
n
b
n
)
iu c ngha l dy {a
n
b
n
} l cp s nhn vi cng bi l
1
4
, do dy
hi t ti 0. V vy ta ch cn ch ra dy {a
n
} hi t . Gi s a b, th dy
{a
n
} n iu tng v a
n
b
n
b.Do n hi t, theo trn suy ra dy {b
n
}
hi t v lim
n
a
n
= lim
n
b
n
. Hon ton t-ng t cho tr-ng hp a > b.
2.2.27. Ta c
a +aa +. . . +
n - s
..
aa . . . a = a(1 + 11 +. . . +
n - s
..
11 . . . 1)
= a(10
n1
+ 2 10
n2
+. . . +n 10
0
)
= a((1 + 10 +. . . + 10
n1
) + (1 + 10 +. . . + 10
n2
)
+. . . + (1 + 10) + 1)
= a
_
10
n
1
9
+
10
n1
1
9
+. . . +
10
2
1
9
10 1
9
_
= a
10(10
n
1) 9n
81
.
Do , gii hn bng
10a
81
.
2.2.28. Ch rng dy {
n

n} vi n > 2 n iu gim, v gii hn ca n


bng 1. D dng kim tra rng
_
n

n 1
_
n
<
_
1
2
_
n
vi n N
Do vy lim
n
(
n

n 1)
n
= 0
2.2.29. Do lim
n
a
n
= 0 nn bt u t ch s n
0
no , |a
n
|
n
<
_
1
2
_
n
. Do
, lim
n
a
n
n
= 0.
2.2.30. t max{a
1
, a
2
, . . . , a
k
} = a
l
. Chia c t v mu cho a
n
l
ta -c
lim
n
p
1
a
n+1
1
+p
2
a
n+1
2
+. . . +p
k
a
n+1
k
p
1
a
n
1
+p
2
a
n
2
+. . . +p
k
a
n
k
= a
l
.

n

C
h
i
2.2. Gii hn. Tnh cht ca dy hi t 163
2.2.31.
(a) Cho > 0 nh sao cho q + < 1. Th tn ti n
0
N sao cho

a
n+1
a
n

< q + vi n n
0
.
Do
|a
n
| < (q +)
nn
0
|a
n
0
|, n n
0
.
Suy ra, lim
n
|a
n
| = 0, tc l, lim
n
a
n
= 0.
(b) Cho > 0 nh sao cho q > 1 Do bt u t mt ch s n
1
no
|a
n
| > (q )
nn
1
|a
n
1
|,, ta li c lim
n
(q )
nn
1
= +. Do ,
lim
n
|a
n
| = +.
2.2.32.
(a) Chn > 0 nh sao cho q + < 1, th th tn ti n
0
N sao cho
|a
n
| < (q +)
n
, n n
0
, suy ra lim
n
a
n
= 0.
(b) Ta c |a
n
| > (q )
n
vi n > n
1
. Nu > 0 nh th q > 1 v suy
ra lim
n
(q )
n
= +, do lim
n
|a
n
| = +.
2.2.33. t a
n
= n

x
n
, ta c
lim
n
a
n+1
a
n
= lim
n
_
n + 1
n
_

x = x, 0 < x < 1.
Suy ra dy hi t v 0 theo bi 2.2.31.
2.2.34. Xt phn t th a
n
ca dy , ta c

a
n+1
a
n

mn
n + 1
x


n
|x|.
Theo bi 2.2.31 ta kt lun rng dy hi t v 0.
2.2.35. Gi s |b
n
| < M vi n N. V lim
n
a
n
= 0 nn vi mi > 0 tn
ti n
0
N sao cho |a
n
| <

M
vi n > n
0
. T suy ra
|a
n
b
n
| < vi n > n
0
.
Do lim
n
a
n
b
n
= 0.

n

C
h
i
164 Ch-ng 2. Dy s thc
2.2.36. Khng gim tng qut ta gi thit rng a b. Xt a < b, chn
> 0 nh sao cho a + < b , theo nh ngha gii hn ca dy suy ra
a
n
< a + < b < b
n
vi n ln. Do max{a
n
, b
n
} = b
n
, v t suy
ra
lim
n
max{a
n
, b
n
} = lim
n
b
n
= b = max{a, b}.
Nu a = b th vi mi > 0 tn ti n
0
sao cho vi mi n > n
0
ta u c
|a
n
a| < v |b
b
b| < , tc l
| max{a
n
, b
n
} a| < .
Tc l ta chng minh -c rng
lim
n
max{a
n
, b
n
} = max{a, b}.
2.2.37. V lim
n
a
n
= 0 vi mi (0, 1) nn
p

1 <
p

1 +a
n
<
p

1 + vi n ln.
T suy ra lim
n
p

1 +a
n
= 1.
2.2.38. t x
n
=
p

1 +a
n
, t li gii bi trn ta -c lim
n
x
n
= 1. T
suy ra
lim
n
p

1 +a
n
1
a
n
= lim
n
x
n
1
x
p
n
1
= lim
n
x
n
1
(x
n
1)(x
p1
n
+. . . + 1)
=
1
p
.
2.2.39. Theo bi 1.2.1 ta c
n
_
p
_
1 +
a
1
+a
2
+. . . +a
p
n
1
_
n
_
p
_
_
1 +
a
1
n
__
1 +
a
2
n
_
. . .
_
1 +
a
p
n
_
1
_
(1)
=
p
_
(n +a
1
)(n +a
2
) . . . (n +a
p
) n.

n

C
h
i
2.2. Gii hn. Tnh cht ca dy hi t 165
Hn na theo bi 1.2.4 ta c
n
_
p
_
_
1 +
a
1
n
_ _
1 +
a
2
n
_
. . .
_
1 +
a
p
n
_
1
_
= n
_
_
_
_
p

_
1 +
a
1
+a
2
+. . . +a
p
n
+

i<j
a
i
a
j
n
2
+. . . +
a
1
a
2
. . . a
p
n
p
1
_
_
_
_
(2)

a
1
+a
2
+. . . +a
p
p
+

i<j
a
i
a
j
np
+. . . +
a
1
a
2
. . . a
p
pn
p1
.
T (1) v (2) cng vi kt qu bi tp trn ta suy ra gii hn cn tm l
a
1
+a
2
+...+ap
p
.
2.2.40. Ch rng
n + 1

n
2
+n + 1

n
2
+ 1
+
1

n
2
+ 2
+. . . +
1

n
2
+n + 1

n + 1

n
2
+ 1
.
S dng nguyn l kp ta suy ra gii hn cn tm l 1.
2.2.41. K hiu a l gi tr ln nht ca cc s a
1
, a
2
, . . . , a
p
, th th
a
n

p

n

a
n
1
+a
n
2
+. . . +a
n
p
p
a.
S dng nguyn l kp ta suy ra
lim
n
n

a
n
1
+a
n
2
+. . . +a
n
p
p
= a = max{a
1
, a
2
, . . . , a
p
}.
2.2.42. V
1
n
_
2 sin
2
n
1999
n + 1
+ cos
2
n
1999
n + 1

n

2.
Suy ra
lim
n
n
_
2 sin
2
n
1999
n + 1
+ cos
2
n
1999
n + 1
= 1.

n

C
h
i
166 Ch-ng 2. Dy s thc
2.2.43. S dng nguyn l kp i vi cc dy -c
1 < (1 +n(1 + cos n))
1
2n+n sinn
< (1 + 2n)
1
2n+n sinn
.
Ta cn ch ra rng
lim
n
(1 + 2n)
1
2n+n sinn
= 1. ()
iu ny -c suy ra t cc bt ng thc kp
1 < (1 +n(1 + cos n))
1
2n+n sinn
< (1 + 2n)
1
n
.
Do gii hn cn tm l 1.
2.2.44. S dng bt ng thc lin h gia trung bnh cng, nhn v iu ho
(xem 1.2.3) vi x > 1 ta -c
1 +
x
2 +x
=
2
1
1+x
+ 1

_
(1 +x)1 =

1 +x
1 +x + 1
2
= 1 +
x
2
.
t x =
k
n
2
, k = 1, 2, . . . , n v thay vo bt ng thc ri cng v vi v ca
cc bt ng thc -c
n

k=1
k
n
2
2 +
k
n
2

k=1
_
_
1 +
k
n
2
1
_

n

k=1
k
2n
2
. ()
Hn na
n

k=1
k
2n
2
=
n(n + 1)
4n
2

n
1
4
.
v
n

k=1
k
n
2
2 +
k
n
2
=
n

k=1
k
2n
2
+k

1
2n
2
+n
n

k=1
k =
n(n + 1)
2(2n
2
+n)

n
1
4
.
T cng vi () v nguyn l kp suy ra
lim
n
n

k=1
_
_
1 +
k
n
2
1
_
=
1
4
.

n

C
h
i
2.2. Gii hn. Tnh cht ca dy hi t 167
2.2.45. Chng ta l lun t-ng t nh- li gii trong bi tr-c. Ly x > 1.
Theo bt ng thc lin h gia trung bnh cng, nhn v iu ho ta c
1 +
x
3 + 2x
=
3
1
1+x
+ 1 + 1

3
_
(1 +x)1.1
1 +x + 1 + 1
3
= 1 +
x
3
.
Vi x =
k
2
n
3
, ta c
n

k=1
k
2
n
3
3 + 2
k
2
n
3

k=1
_
3
_
1 +
k
2
n
3
1
_

k=1
k
2
3n
3
. ()
Hn na
n

k=1
k
2
3n
3
=
n(n + 1)(2n + 1)
18n
3

n
1
9
.
V
n

k=1
k
2
n
3
3 + 2
k
2
n
3
=
n

k=1
k
2
3n
3
+ 2k
2

1
3n
3
+ 2k
2
n

k=1
k
2
=
a(n + 1)(2n + 1)
6(3n
3
+ 2
2
)

n
1
9
.
nn t kt qu trn cng vi () v nh l gii hn ca cc dy b kp, ta c
lim
n
n

k=1
_
3
_
1 +
k
2
n
3
1
_
=
1
9
.
2.2.46. R rng, lim
n
n

a
k
= 1 vi k = 1, 2, ..., p. V vy, ta c
lim
n
_
1
p
p

k=1
n

a
k
_
p
= 1.
2.2.47. Vi n
0
ln v vi n > n
0
, ta c 0 < +
1
n
< +
1
n
0
< 1. V vy
lim
n
n1

k=0
_
+
1
n
_
k
= lim
n
1
_
+
1
n
_
n
1
_
+
1
n
_ =
1
1
.
2.2.48. Bt ng thc hin nhin ng vi x = 1. Gi s x > 1. tnh gii
hn, chng ta p dng qui tc kp cc dy.

n

C
h
i
168 Ch-ng 2. Dy s thc
Chng ta c
0 < (
n

x 1)
2
=
n

x
2
2
n

x + 1.
V vy
(2
n

x 1)
n
< (
n

x
2
)
n
= x
2
. ()
Hn na
(2
n

x 1)
n
= x
2
_
2
n

x

1
n

x
2
_
n
= x
2
_
1 +
_
2
n

x

1
n

x
2
1
__
n
.
S dng bt ng thc Bernoulli, ta c
(2
n

x 1)
n
x
2
_
1 +n
_
2
n

x

1
n

x
2
1
__
()
= x
2
_
1 n
(
n

x 1)
2
n

x
2
_
.
Cng s dng bt ng thc Berrnoulli, ta c
x = (
n

x 1 + 1)
n
1 +n(
n

x 1) > n(
n

x 1).
H qu l
(
n

x 1)
2
<
x
2
n
2
.
V vy, theo () ta c
(2
n

x 1)
n
> x
2
_
1
x
2
n
n

x
2
_
. ( )
Kt hp () v ( ) vi qui tc kp cc dy, ta thy
lim
n
(2
n

x 1)
n
= x
2
.
2.2.49. T-ng t nh- li gii ca cc bi tr-c , chng ta c th thit lp cc
bt ng thc sau
1
(2
n

n 1)
n
n
2
1 n
(
n

n 1)
2
n

n
2
.
T suy ra
lim
n
n
(
n

n 1)
2
n

n
2
= 0.

n

C
h
i
2.2. Gii hn. Tnh cht ca dy hi t 169
kt thc, ch rng vi n 3,
n = (
n

n 1 + 1)
n
>
n(n 1)(n 2)
3!
(
n

n 1)
3
.
V vy
0 n(
n

n 1)
2
n
_
3!
(n 1)(n 2)
_2
3
.
Do ,
lim
n
n(
n

n 1)
2
= 0.
2.2.50.
(a) Ta c
|a
n+k
a
n
| =
arctan(n + 1)
2
n+1
+ +
arctan(n +k)
2
n+k
<

2
_
1
2
n+1
+ +
1
2
n+k
_
<

2
n+1
.
Vi > 0 bt k ly n
0
= [log
2

1]. Khi vi bt k k N v
n > n
0
ta c |a
n+k
a
n
| < . Vy {a
n
} l mt dy Cauchy.
(b) C th ch ra bng qui np rng 4
n
> n
4
vi mi n 5. V vy
|a
n+k
a
n
| <
1
(n + 1)
2
+
1
(n + 2)
2
+ +
1
(n +k)
2
.
H qu l,
|a
n+k
a
n
|
<
1
n(n + 1)
+
1
(n + 1)(n + 2)
+ +
1
(n +k 1)(n +k)
=
1
n

1
n + 1
+
1
n + 1

1
n + 2
+ +
1
n +k 1

1
n +k
=
1
n

1
n +k
<
1
n
<
vi bt k k N v n >
_
1

.
(c) Ta c
|a
2n
a
n
| =
1
2n
+
1
2n 1
+ +
1
n + 1
n
1
2n
=
1
2
.
iu ny chng t a
n
khng l dy Cauchy.

n

C
h
i
170 Ch-ng 2. Dy s thc
(d) Ta c
|a
n+k
a
n
|
=

(1)
n+k1
(n +k)(n +k + 1)
+
(1)
n+k2
(n +k 1)(n +k)
+ +
(1)
n
(n + 1)(n + 2)

1
(n +k)(n +k + 1)
+
1
(n +k 1)(n +k)
+ +
1
(n + 1)(n + 2)
=
1
n +k

1
n +k + 1
+
1
n +k 1

1
n +k
+ +
1
n + 1

1
n + 2
=
1
n + 1

1
n +k + 1
<
1
n + 1
<
vi bt k k N v n > [
1

1].
(e) Ta c
|a
n+k
a
n
| M(|q|
n+k
+|q|
n+k1
+ +|q|
n+1
)
= M
_
|q|
n+1
(1 |q|
k
)
1 |q|
_

M
1 |q|
|q|
n+1
<
vi bt k k N v n > n
0
=
_
ln
(1|q|)
M
ln|q|
1
_
.
(f) Ta c
a
2n
a
n
=
2n
(2n + 1)
2
+
2n 1
(2n)
2
+ +
n + 1
(n + 2)
2
n
2n
(2n + 1)
2

2n
2
(3n)
2
=
2
9
.
Do {a
n
} khng l dy Cauchy.
2.2.51. T iu kin cho ta c
|a
n+k
a
n
| = |a
n+k
a
n+k1
+a
n+k1
a
n+k2
+ +a
n+1
a
n
|
< (|a
n+k1
a
n+k2
| +|a
n+k2
a
n+k3
| + +|a
n
a
n1
|)
< (
k
+
k1
+ +
2
+)|a
n
a
n1
|
(
k
+
k1
+ +
2
+)
n2
|a
2
a
1
|
=

n1
(1
k
)
1
|a
2
a
1
| <

n1
1
|a
2
a
1
|

n

C
h
i
2.2. Gii hn. Tnh cht ca dy hi t 171
Do , vi bt k > 0 cho tr-c, vi n >
_
1 +
ln(
(1)
|a
2
a
1
|
)
ln
_
v vi mi k N
ta c |a
n+k
a
n
| < .
2.2.52. V {S
n
} hi t nn n l dy Cauchy. Chng ta s chng minh {ln
n
}
cng l dy Cauchy. T bt ng thc trong 2.1.4,1 ta c
ln
n+k
ln
n
= ln
_
1 +
1
a
n+k
_
+ + ln
_
1 +
1
a
n+1
_
<
1
a
n+k
+ +
1
a
n+1
<
vi k N v n ln.
2.2.53. T kt qu trong 1.1.23, ta c
R
n+k
R
n
= (R
n+k
R
n+k1
) + (R
n+k1
R
n+k2
) + + (R
n+1
R
n
)
= (1)
n
_
(1)
k1
q
n+k1
q
n+k
+
(1)
k2
q
n+k2
q
n+k1
+
1
q
n+1
q
n+2
+
1
q
n
q
n+1
_
.
Do , v dy {q
n
} n iu nn q
n
n (xem li gii ca bi 1.1.24), ta c
|R
n+k
R
n
|
1
q
n+1
q
n

1
n
2
.
2.2.54. Gi d l cng sai ca cp s cng cho . Tr-c ht chng ta gi s
d = 0. Khi
1
a
k
a
k+1
=
_
1
a
k

1
a
k+1
_
1
d
.
Do
lim
n
_
1
a
1
a
2
+
1
a
2
a
3
+ +
1
a
n
a
n+1
_
=
1
a
1
d
.
Vi d = 0, cp s cng l mt dy hng, v vy
lim
n
_
1
a
1
a
2
+
1
a
2
a
3
+ +
1
a
n
a
n+1
_
= +.
2.2.55. Gi d l cng sai ca cp s cng cho . Tr-c ht chng ta gi s
d = 0. V
1

a
k
+

a
k+1
=

a
k+1

a
k
d
,

n

C
h
i
172 Ch-ng 2. Dy s thc
ta c
lim
n
1

n
_
1

a
1
+

a
2
+
1

a
2
+

a
3
+ +
1

a
n
+

a
n+1
_
=
1

d
.
Vi d = 0, cp s cng l mt dy hng, v vy gii hn bng +.
2.2.56.
(a) Theo bi 2.1.38, ta c
_
1 +
1
n
_
n
< e <
_
1 +
1
n
_
n+1
.
V vy
1 < n(
n

e 1) < n
_
_
1 +
1
n
_
1+
1
n
1
_
. ()
S dng bt ng thc Berrnoulli (xem 1.2.4) chng ta c th chng minh
_
1 +
1
n
_1
n
1 +
1
n
2
.
Do
n
_
_
1 +
1
n
_
1+
1
n
1
_
1 +
1
n
+
1
n
2
.
V vy, t () v qui tc kp cc dy, ta c
lim
n
n(
n

e 1) = 1.
(b) Vi n bt k cho tr-c , ta c
e
1
n
+e
2
n
+ +e
n
n
n
=
(e 1)e
1
n
n(e
1
n
1)
.
Do , p dng cu (a) , ta c
lim
n
e
1
n
+e
2
n
+ +e
n
n
n
= e 1.

n

C
h
i
2.3. nh l Toe plitz, nh l Stolz 173
2.2.57. Ta c a
n+1
a
n
= p(a
n
a
n1
). V vy
a
n
= a + (b a) + (a
3
a
2
) + + (a
n
a
n1
)
a + (b a)(1 p +p
2
+ + (1)
n
p
n2
).
Nu b = a th {a
n
} l dy hng hi t n a. Nu a = b th dy hi t vi iu
kin |p| < 1, v gii hn ca n l a +
ba
1+p
.
2.2.58. Ta thy
c
n+1
=
a
n
+ 2b
n
a
n
+b
n
=
c
n
+ 2
c
n
+ 1
.
V vy
|c
n+1

2| =

2 1
c
n
+ 1
|c
n

2| < (

2 1)|c
n

2| <
1
2
|c
n

2|.
T , qui np ta c
|c
n+1

2| <
1
2
n
|c
1

2|,
iu ny suy ra gii hn ca {c
n
} l

2.
2.3 nh l Toeplitz, nh l Stolz v ng dng
2.3.1. Nu tt c cc s hng ca dy {a
n
} bng a th t (ii) ta c lim
n
b
n
=
a lim
n
c
n,k
= a. V vy, ch cn xt tr-ng hp dy hi t n 0. Khi , vi bt
k m > 1 v n m ta c
|b
n
0| =

k=1
c
n,k
a
k

m1

k=1
|c
n,k
|.|a
k
| +
n

k=m
|c
n,k
|.|a
k
|. ()
T s hi t dn 0 ca {a
n
} dn n vi > 0 cho tr-c, tn ti n
1
tho mn
|a
n
| <

2C
vi n n
1
.
D nhin dy {a
n
} b chn bi D > 0 no . T (i) chng ta suy ra tn ti n
2
sao cho vi n n
2
,
n
1
1

k=1
|c
n,k
| <

2D
.

n

C
h
i
174 Ch-ng 2. Dy s thc
Tip theo, ly m = n
1
trong () ta c
|b
n
| D
n
1
1

k=1
|c
n,k
| +

2C
n

k=n
1
|c
n,k
| <

2
+

2
=
vi mi n max{n
1
, n
2
}. V vy lim
n
b
n
= 0.
2.3.2. S dng nh l Toeplitz vi c
n,k
=
1
n
, k = 1, 2, ..., n.
2.3.3.
(a) Nu c
n,k
khng m th (iii) -c suy ra t (ii).
(b) T iu kin (ii) trong bi 2.3.1 ta suy ra
n

k=1
c
n,k
>
1
2
vi n ln, n > n
0
.
T s phn k ca {a
n
} n + suy ra vi M > 0 cho tr-c, tn ti n
1
sao cho a
n
2M vi mi n > n
1
.
Khng mt tnh tng qut chng ta c th gi s tt c s hng a
n
u
d-ng. t n
2
= max{n
0
, n
1
}. Khi
n

k=1
c
n,k
a
k
=
n
2

k=1
c
n,k
a
k
+
n

k=n
2
+1
c
n,k
a
k

n
2

k=1
c
n,k
a
k
+M M,
v do lim
n
b
n
= +.
2.3.4. y l tr-ng hp c bit ca 2.3.3 vi c
n,k
=
1
n
; k = 1, 2, ..., n.
2.3.5. S dng nh l Toeplitz (2.3.1) vi c
n,k
=
2(nk+1)
n
2
.
2.3.6. S dng bt ng thc lin h gia trung bnh cng, nhn v iu ho
(xem 1.2.3), nh l cc dy b kp v kt qu trong 2.3.2.
2.3.7. p dng bi tr-c cho dy {
a
n+1
an
}.
2.3.8. Nu b = 0 th chng ta ly c
n,k
=
b
nk+1
nb
, ta thy c
n,k
tho mn
iu kin (i) trong 2.3.1. T 2.3.2 ta suy ra iu kin (ii) cng tho mn.Trong
tr-ng hp ny kt qu -c suy ra t nh l Toeplitz. Nu b = 0 th t
c
n,k
=
1+b
nk+1d
n
th ta thu -c
lim
n
a
1
(1 +b
n
) +a
2
(1 +b
n1
) + +a
n
(1 +b
1
)
n
= a.
Do , theo 2.3.2 ta c
lim
n
a
1
b
n
+a
2
b
n1
+ +a
n
b
1
n
= 0.

n

C
h
i
2.3. nh l Toe plitz, nh l Stolz 175
2.3.9. S dng nh l Toeplitz cho dy {
an
bn
} vi c
n,k
=
b
k
b
1
++bn
.
2.3.10. S dng nh l Toeplitz vi c
n,k
=
b
k
b
1
++bn
.
2.3.11. Vi n > 1, chng ta t
a
n
=
x
n
x
n1
y
n
y
n1
, b
n
= y
n
y
n1
v p dng kt qu trong bi tr-c.
2.3.12.
(a) Trong 2.3.10 chng ta t x
n
= 1 +
1

2
+ +
1

n
, y
n
=

n v chng
minh gii hn bng 2.
(b) t
x
n
= a +
a
2
2
+ +
a
n
n
, y
n
=
a
n+1
n
.
Bt u t mt gi tr no ca ch s n dy {y
n
} tng thc s. T
2.2.31 (b) ta thy lim
n
y
n
= +.
V vy
lim
n
n
a
n+1
_
a +
a
2
2
+ +
a
n
n
_
=
1
a 1
.
(c) Chng ta c th p dng nh li Stolz (xem 2.3.11) cho cc dy
x
n
= k! +
(k + 1)!
1!
+ +
(k +n)!
n!
, y
n
= n
k1
.
Ta c
lim
n
x
n
x
n1
y
n
y
n1
= lim
n
(n + 1)(n + 2) (n +k)
n
k+1
(n 1)
k+1
= lim
n
(1 +
1
n
) (1 +
k
n
)
n(1 (1
1
n
)
k+1
)
= lim
n
(1 +
1
n
) (1 +
k
n
)
1 + (1
1
n
) + + (1
1
n
)
k
=
1
k + 1
.

n

C
h
i
176 Ch-ng 2. Dy s thc
(d) t x
n
=
1

n
+ +
1

2n
, y
n
=

n. Khi
lim
n
x
n
x
n1
y
n
y
n1
= lim
n
1

2n
+
1

2n1

1

n1

n 1
= lim
n
(

n +

n 1)
_
1

2n
+
1

2n 1

n 1
_
= lim
n
_
1

2
+
_
n
2n 1

_
n
n 1
+
_
n 1
2n
+
_
n 1
2n 1
1
_
= 2(

2 1).
T , p dng nh l Stolz ta c gii hn l 2(

2 1).
(e) t x
n
= 1
k
+ 2
k
+... +n
k
v y
n
= n
k+1
, ta thy
x
n
x
n1
y
n
y
n1
=
n
k
n
k+1
(n 1)
k+1

n
1
k + 1
.
T y, ta c cc iu kin cn p dng nh l Stolz.
(f) S dng nh l Stolz, ta c
lim
n
1 + 1 a + 2 a
2
+... +n a
n
n a
n+1
=
1
a 1
.
(g) S dng nh l Stolz cho cc dy
x
n
= (k + 1)(1
k
+ 2
k
+ ...n
k
) n
k+1
v y
n
= (k + 1)n
k
.
Khi
x
n
x
n1
y
n
y
n1
=
(k + 1)n
k
n
k+1
+ (n 1)
k+1
(k + 1)[n
k
(n 1)
k
]

n
1
2
.
2.3.13. S dng nh l Stolz trong tr-ng hp
x
n
= a
1
+
a
2

2
+ +
a
n

n
v y
n
=

n.
Ta thy rng
lim
n
1

n
_
a
1
+
a
2

2
+ +
a
n

n
_
= 2a.

n

C
h
i
2.3. nh l Toe plitz, nh l Stolz 177
2.3.14. Trong nh l Stolz chng ta t x
n
= a
n+1
v y
n
= n.
2.3.15. S dng php bin i Toeplitz i vi tr-ng hp {a
n
} vi c
n,k
=
1
2
nk+1
, ta thy
lim
n
_
a
n
1
+
a
n1
2
+
a
1
2
n1
_
= 2a.
2.3.16.
(a) S dng php bin i Toeplitz cho {a
n
} vi
c
n,k
=
1
(n + 1 k)(n + 2 k)
,
ta c th chng minh rng
lim
n
_
a
n
1.2
+
a
n1
2.3
+
a
1
n(n + 1)
_
= a.
(b) T-ng t nh- cu (a), chng ta p dng php bin i Toeplitz cho {a
n
}
vi c
n,k
=
3
2
(1)
nk
2
nk
v chng minh rng gii hn l
2
3
a.
2.3.17. t a
n
=
_
nk
n
_
. p dng kt qu ca bi 2.3.7 , chng ta cn tnh
lim
n
a
n+1
an
. Ta c
_
(n+1)k
n+1
_
_
nk
n
_ =
(nk + 1)(nk + 2)...(nk +k)
(n + 1)(nk n + 1)(nk n + 2)...(nk n +k 1)
.
Do , gii hn bng
k
k
(k1)
k1
.
2.3.18. Ly a
n
l cp s cng vi cng sai d > 0. t
c
n
=
n
n
(a
1
...a
n
)
(a
1
+ +a
n
)
n
.
Khi
c
n+1
c
n
=
(n + 1)a
n+1
a
1
+a
2
+... +a
n+1
_
a
1
+...+an
n
a
1
+...a
n+1
n+1
_
n
=
2a
n+1
a
1
+a
n+1
_
2a
1
+ (n 1)d
2a
1
+nd
_
n

n
2e
1
.
S dng bi 2.3.7, suy ra gii hn bng 2e
1.
Nu d = 0 th gii hn l 1.

n

C
h
i
178 Ch-ng 2. Dy s thc
2.3.19. T b
n
= 2a
n
+ a
n1
, a
n
=
bna
n1
2
v a
n1
=
b
n1
a
n2
2
, ta c
a
n
=
2bnb
n1
+a
n2
2
2
. Thc hin qu trnh ny n 1 ln chng ta thu -c
a
n
=
2
n1
b
n
2
n2
b
n2
+... + (1)
n2
2
1
b
2
+ (1)
n1
a
1
2
n
.
Khi , theo 3.3.16 (b) ta c lim
n
a
n
=
1
3
b.
2.3.20. t c
n
= (a
1
a
2
...a
n
)
nx
. Khi
c
n+1
c
n
= (1 +
1
n
)
nx
(n + 1)
x
a
n+1

n
e
x
a.
V vy, theo 2.3.7, lim
n
n
x
(a
1
a
2
...a
n
)
1
n
= e
x
a.

n

C
h
i
2.3. nh l Toe plitz, nh l Stolz 179
2.3.21.
(a) Chng ta p dng nh l Stolz cho dy x
n
= 1 +
1
2
+... +
1
n
v y
n
= lnn.
iu ny n n
x
n
x
n1
y
n
y
n1
=
1
ln
_
1 +
1
n1
_
n

n
1.
Bi v lim
n
ln(1 +
1
n
)
n
= 1, iu ny dn n bt ng thc (1 +
1
n
)
n
<
e < (1 +
1
n
)
n+1
(xem 2.1.41).
(b) Gii hn l
1
2
(xem li gii cu a).
2.3.22. Chng ta p dng nh l Stolz cho
x
n
=
a
1
1
+
a
2
2
+... +
a
n
n
v y
n
= lnn.
H qu l
x
n
x
n1
y
n
y
n1
=
a
n
ln(1 +
1
n1
)
n

n
a
2.3.23. S dng kt qu trong bi 2.3.7.
(a) 1,
(b) e
2
,
(c) e
2
,
(d) e
3
,
(e) Ta c
lim
n
k

n
n

n!
=
_
e vi k = 1
0 vi k > 1.
2.3.24. S dng nh l Stolz (xem 2.3.11),
lim
n
n

k=1
a
k
k
lnn
= lim
n
a
n+1
ln(1 +
1
n
)
n+1
= a.

n

C
h
i
180 Ch-ng 2. Dy s thc
2.3.25. D dng ch ra rng
a
1
= A
1
, a
2
= 2A
2
A
1
, a
n
= nA
n
(n 1)A
n1
, n 2.
V th
lim
n
n

k=1
a
k
k
lnn
= lim
n
1
2
A
1
+
1
3
A
2
+... +
1
n
A
n1
+A
n
lnn
= A,
y ng thc cui cng -c suy t bi tr-c.
2.3.26. [O. Toeplitz, Prace Matematyczno-Fizyczne, 22(1991), 113-119] Ly
{a
n
} l dy c cc s hng ca n u bng 1. Khi lim
n
a
n
= 1 v b
n
=
n

k=1
c
n,k
a
k
=
n

k=1
c
n,k
. Do 1 = lim
n
b
n
= lim
n
n

k=1
c
n,k
. Vy (ii) -c chng
minh. Ly {a
(k)
n
} l dy m s hng th k bng 1 v cc s hng cn la bng
0. Khi lim
n
a
(k)
n
= 0 v 0 = lim
n
b
n
= lim
n
c
n,k
. Vy (i) cng -c chng
minh. Gi s (iii) khng ng, lc vi bt k C > 0 tn ti n
C
sao cho
n
C

k=1
|c
nc,k
| C. Thc t, vi C > 0 cho tr-c tn ti v s n
c
. Ly n
1
l s
d-ng nh nht tho mn
n
1

k=1
|c
n
1
,k
| > 10
2
. Chng ta t n
1
s hng u tin
ca dy {a
n
} nh- sau
sgn c
n
1
,k
= sgn a
k
v |a
k
| =
1
10
.
Khi
b
n
1
=
n
1

k=1
c
n
1
,k
a
k
=
n
1

k=1
1
10
|c
n
1
,k
| > 10.
Theo (i) tn ti n
0
tho mn
n

k=1
|c
n
1
,k
| < 1 vi n n
0
.
H qu l

n
1

k=1
c
n
1
,k
a
k

<
1
10
vi n n
0
.

n

C
h
i
2.4. i m gii hn. Gii hn tr n v gii hn d-i 181
Ly s nguyn nh nht n
2
tho mn n
2
max{n
0
, n
1
} v
n
2

k=1
|c
n
2
,k
| >
10
4
+ 1 + 10, cc s hng tip theo ca dy {a
n
} -c xc nh bng cch t
sgn c
n
2
,k
= sgn a
k
v |a
k
| =
1
10
2
vi n
1
+ 1 k n
2
.
Khi
b
n
2
=
n
2

k=1
c
n
2
,k
a
k
=
n
1

k=1
c
n
2
,k
a
k
+
n
2

k=n
1
+1
c
n
2
,k
a
k
=
n
1

k=1
c
n
2
,k
a
k
+
1
10
2
n
2

k=n
1
+1
|c
n
2
,k
|.
T suy ra
b
n
2
>
1
10
+
1
10
2
(10
4
+ 1 + 10 1) = 10
2
.
Chng ta xy dng qui np dy {a
n
} vi cc s hng c ch s t n
k1
+ 1 ti
n
k
bng
1
10
k
hoc
1
10
k
, dy bin i b
n
tho mn
b
n
k
> 10
k
vi k = 1, 2, 3...
Khi , dy a
n
hi t n 0 trong khi dy bin i b
n
c mt dy con b
n
k
phn
k. iu ny mu thun vi gi thit. V vy (iii) ng.
2.4 im gii hn. Gii hn trn v gii hn
d-i
2.4.1.
(a) Tr-c ht, chng ta s chng minh cc dy con c cng gi tr gii hn.
Gi s lim
n
a
2k
= a, lim
n
a
2k+1
= b v lim
n
a
3k
= c, Khi lim
n
a
6k
=
a = c v lim
n
a
6k+3
= b = c. T suy ra a = b = c. Vi > 0 cho
tr-c tn ti cc s nguyn d-ng k
1
v k
2
sao cho
|a
2k
a| < , vi mi k > k
1
,
|a
2k1
a| < , vi mi k > k
2
.
V vy |a
n
a| < vi mi n > n
0
= max{2k
1
, 2k
2
+ 1}.

n

C
h
i
182 Ch-ng 2. Dy s thc
(b) Khng. Ly {a
n
} l dy -c xc nh bi a
n
= (1)
n
. Khi lim
n
a
2k
=
1, lim
n
a
2k+1
= 1. Nh-ng lim
n
a
n
khng tn ti.
By gi , ly {a
n
} l dy -c xc nh nh- sau
a
n
=
_
0 nu n = 2
k
, k = 0, 1, 2, ...
1 nu ng-c li.
Khi , lim
k
a
3k
= 1 v lim
k
a
2k+1
= 1, nh-ng lim
k
a
2k
= 0
khng tn ti. Hin nhin, dy {a
n
} phn k.
Cui cng, xt dy sau:
a
n
=
_
0 nu n l s nguyn t,
1 nu n l s hp t.
Vi dy ny chng ta c lim
n
a
3k
= 1 v lim
n
a
2k
= 1, nh-ng lim
n
a
2k+1
khng tn ti, bi v dy {a
2k+1
} cha mt dy con gm cc s c ch s
nguyn t v mt dy con gm cc s c ch s l hp s. (Ch rng
c v s nguyn t. Ng-c li, nu p
1
, p
2
, ...., p
n
l cc s nguyn t,
p
1
< p
2
< ... < p
n
v khng tn ti mt s nguyn t no ln hn p
n
th p
1
.p
2
...p
n
+ 1 > p
n
cng l s nguyn t, bi v n khng c -c s
nguyn t no ngoi n v 1. Vy, iu ny mu thun vi gi thit.)
2.4.2. Khng. Gi {a
n
} l dy -c xc nh bi
a
n
=
_
0 nu n l nguyn t,
1 nu n l hp s.
Khi , mi dy con {a
s.n
}, s > 1, n 2, l dy hng, do n hi t. Dy
{a
n
} phn k (xem li gii ca bi 2.4.1 (b)).
2.4.3. R rng, S
p
S
q
... S
s
S. chng minh bao hm thc ng-c
li, ta gi s x / S
p
S
q
... S
s
. Khi , tn ti cc s d-ng
p
,
q
, ...,
s
v cc s nguyn d-ng n
p
, n
q
, ..., n
s
sao cho
|x a
pn
| >
p
, vi mi n > n
p
,
|x a
qn
| >
q
, vi mi n > n
q
,
...
|x a
sn
| >
s
, vi mi n > n
s
.

n

C
h
i
2.4. i m gii hn. Gii hn tr n v gii hn d-i 183
t = min{
p
,
q
, ...,
s
} v m = max{p
np
, q
nq
, ..., s
ns
}, ta thy |x a
n
|
vi n > m, iu ny dn n x khng l im t ca dy {a
n
}. Vy
S S
p
S
q
... S
s
.
Nu mi dy con {a
pn
}, {a
qn
}, ..., {a
sn
} hi t n a th t S = S
p
S
q
...S
s
ta suy ra {a
n
} hi t n a.
2.4.4. Khng. Ly {a
n
} l dy -c xc nh bi cng thc
a
n
=
_
0 nu n = 2
k
, k = 0, 1, 2...,
1 cc tr-ng hp cn li.
Mi dy con
{a
2k1
}, {a
2(2k1)
}, {a
2
2
(2k1)
}, ..., {a
2
m
(2k1)
}, ...
hi t n 1, trong khi dy {a
n
} phn k.
2.4.5. Gi s dy {a
n
} khng hi t n a. Khi , tn ti > 0 sao cho vi
mi s nguyn d-ng k tn ti n
k
> k tho mn |a
n
k
a| . Nu chng ta
gi s n
k
l s nh nht ca nhng s trn th dy {n
k
} n iu tng. Hn
na, lim
n
n
k
= +. Ta thy dy {a
n
k
} khng cha dy con hi t n a, iu
ny mu thun vi gi thit ca chng ta. V vy {a
n
} hi t n a.
2.4.6.
(a) R rng 1 l im t duy nht ca dy. V vy S ch gm mt im,
S = {1}.
(b) Ta c a
3k
= 0, a
3k+1
= 1, a
3k+2
= 0. V vy, theo bi 2.4.3, tp S cc im
t ca cc dy ny c hai phn t, S = {0, 1}.
(c) Ta c
a
2k
=
1
2
2k
+ 3
v a
2k+1
=
2
2k+2
+ 1
2
2k+1
+ 3
.
V vy S = {0, 2}.
(d) Ta c
a
2k
=
2 ln(6k) + ln(2k)
ln(4k)
v a
2k+1
=
ln(2k + 1)
ln(2(2k + 1))
.
V vy S = {1, 3}.

n

C
h
i
184 Ch-ng 2. Dy s thc
(e)
a
6k
= 1, a
6k+1
= (0, 5)
6k+1
, a
6k+2
= (0, 5)
6k+2
,
a
6k+3
= 1, a
6k+4
= (0, 5)
6k+4
, a
6k+5
= (0, 5)
6k+5
.
V vy S = {1, 0, 1}.
(f)
a
7k
= 0, a
7k+1
=
2
7
, a
7k+2
=
1
7
, a
7k+3
=
4
7
.
a
7k+4
=
4
7
, a
7k+5
=
1
7
, a
7k+6
=
2
7
.
V vy S = {0,
1
7
,
2
7
,
4
7
}.
2.4.7.
(a) Ly =
p
q
, p Z, q N, y p, q nguyn t cng nhau. Khi
a
kq
= 0 v a
kq+l
= kq +
lp
q

_
kp +
_
lp
q
_
+r
_
=
lp
q

_
lp
q
_
,
y l = 1, 2, .., q 1 v r =
lp
q

_
lp
q
_
. Do
S =
_
0,
p
q

_
p
q
_
,
2p
q

_
2p
q
_
, ...,
(q 1)p
q

_
(q 1)p
q
__
.
(b) Chng ta s chng minh mi gi tr thc x [0, 1] l im t ca dy
{n [n]}. Theo bi 1.1.20, tn ti p
n
Z v q
n
N sao cho 0 <

pn
qn
<
1
q
2
n
. T lim
n
q
n
= + suy ra lim
n
(q
n
p
n
) = 0. Ly
x (0, 1) v ly > 0 l s nh nht tho mn 0 < x < x+ < 1.
Gi s n
1
l s ln nht tho mn
0 < q
n
1
p
n
1
<
1
q
n
1
< .
Khi tn ti n
0
N tho mn
n
o
(q
n
1
p
n
1
) (x , x +). ()
(xem li gii ca bi 1.1.21). T (1) suy ra [n
0
q
n
1
n
0
p
n
1
] = 0, hay
t-ng -ng vi n
0
p
n
1
= [n
0
q
n
1
]. Nh- vy n
0
q
n
1
[n
0
q
n
1
] l mt
s hng ca dy cho thuc khong (x, x+), iu ny c ngha x
l mt im t ca dy cho.

n

C
h
i
2.4. i m gii hn. Gii hn tr n v gii hn d-i 185
(c) Tr-c ht , chng ta gi s l mt s hu t trong khong (0, 1). Ly =
p
q
, y p, q nguyn t cng nhau v p < q. Khi a
2kq
= a
2kq+q
= 0,
v
a
2kq+l
= sin
lp
q
vi l = 1, 2, ..., q 1, q + 1, ..., 2q 1.
Do
S =
_
0, sin
p
q
, sin
2p
q
, ..., sin
(q 1)p
q
_
.
Nu Z th dy l mt dy hng. Khi Q\Z, ta c th vit
= [] + ( []) v [] (0, 1).
V vy, sinn = (1)
[]
sin( [])n, v tr-ng hp c th rt ra
t tr-ng hp ni trn.
(d) Ly t [1, 1] l s cho tr-c bt k. Khi , tn ti x R
+
sao cho
sinx = t. Chng ta ch cn xt tr-ng hp > 0, bi v sine l hm
l. V l s v t nn tn ti cc dy cc s nguyn d-ng {p
n
} v {q
n
}
sao cho
x
2
= lim
n
(p
n
q
n

2
).
(Xem li gii ca bi 1.1.21). V vy x = lim
n
(2p
n
q
n
). Do , t
tnh lin tc v tun hon ca hm sine, ta c
t = sin x = lim
n
sin(2p
n
q
n
) = lim
n
sinq
n
.
T suy ra mi s thc trong on [1, 1] u l im t ca dy.
2.4.8. Chng ta s chng minh rng trong bt k khong (a, b) u tn ti t
nht mt s hng ca dy. T lim
n
(
3

n + 1
3

n) = 0 suy ra tn ti n
0
N
sao cho
3

n + 1
3

n < b a, n > n
0
.
Ly m
0
l s nguyn d-ng tho mn
3

m
0
>
3

n
0
a v ly A = {n
N :
3

n
3

m
0
a}. Tp hp A khng trng (chng hn n
0
A) v b
chn trn. t n
1
= max A v n
2
= n
1
+ 1, ta c
3

n
2

3

m
0
> a v
3

n
2
> a +
3

m
0
>
3

n
0
. V vy n
2
> n
0
. Do
3

n
2
<
3

n
1
+ b a
3

m
0
+a +b a, hay t-ng -ng vi a <
3

n
2

m
0
< b.

n

C
h
i
186 Ch-ng 2. Dy s thc
2.4.9. S b chn ca tp tt c cc im t ca dy l hin nhin. Gi S l
tp cc im t ca dy {a
n
}. Nu S hu hn th n ng. Gi s S l tp v
hn v ly s l mt phn t ca n. Gi {s
k
} l dy gm cc thnh phn ca
S -c xc nh nh- sau: vi s
1
l mt s ca S khc s. Chn s
2
thuc S khc
s v tho mn iu kin sau |s
2
s| <
1
2
|s
1
s|, v qui np theo k, ta c
|s
k+1
s| <
1
2
|s
k
s|, s
k+1
= s.
Dy {a
k
} nh- th tho mn iu kin sau
|s
k
s| <
1
2
k1
|s
1
s|, k N.
T s
k
l im t ca dy {a
n
} suy ra tn ti a
n,k
sao cho |a
n
k
s
k
| <
1
2
k1
|s
1

s|. Do
|a
n
k
s| |a
n
k
s
k
| +|s
k
s| <
1
2
k2
|s
1
s|,
iu ny dn n s l mt im t ca dy con {a
n
k
}. V vy s S.
2.4.10. Gi S l tp tt c cc im t ca dy {a
n
}.
(a) Dy {a
n
} b chn. Theo 2. 4. 6, S = {0,
1
7
,
2
7
,
4
7
}. V vy lim
n
a
n
= 0 v
lim
n
a
n
=
4
7
.
(b) Ta c S = {1,
1
2
,
1
2
, 1}, cng vi tnh b chn ca dy ta suy ra lim
n
a
n
=
1 v lim
n
a
n
= 1.
(c) Dy khng b chn v tp cc im t l rng . V vy
lim
n
a
n
= v lim
n
a
n
= +.
(d) Dy khng b chn trn bi v dy con a
2k
= (2k)
2k
tin n v cng. Dy
con vi cc ch s l tin n 0. iu ny chng t
lim
n
a
n
= 0 v lim
n
a
n
= +.
(e) Dy khng b chn bi v a
4k+1
= 4k + 2
n
+ v a
4k+3
= 4k
2
n
. H qu l lim
n
a
n
= v lim
n
a
n
= +.

n

C
h
i
2.4. i m gii hn. Gii hn tr n v gii hn d-i 187
(f) R rng dy b chn. Hn na,
S =
_
e

2
2
, e +

2
2
, e 1, e, e + 1
_
.
Suy ra
lim
n
a
n
= e

2
2
v lim
n
a
n
= e + 1.
(g) lim
n
a
n
= 1 v lim
n
a
n
= 2.
(h) Dy khng b chn trn v a
3k
= 2
3k

n
+. Hn na, S = {1, 1}.
V vy lim
n
a
n
= 1 v lim
n
a
n
= +.
(i) Tr-c ht, chng ta s chng minh rng lim
n
n
lnn
= +. Tht vy, p
dng nh l Stolz (xem 2.3.11), ta c
lim
n
lnn
n
= lim
n
lnn ln(n 1)
n n + 1
= lim
n
ln(1 +
1
n 1
) = 0.
iu ny chng t rng
lim
n
a
2k
= lim
n
ln(2k) 4k
ln2 + ln(2k)
= .
Do , dy {a
n
} khng b chn d-i. Hn na,
lim
n
a
2k+1
= lim
n
ln(2k + 1)
ln2 + ln(2k + 1)
= 1.
Vy lim
n
a
n
= v lim
n
a
n
= 1.
2.4.11. A'p dng bi 2.4.7
(a) lim
n
a
n
= minS = 0 v lim
n
a
n
= max S, y
S =
_
0,
p
q

_
p
q
_
,
2p
q

_
2p
q
_
, ...,
(q 1)p
q

_
(q 1)p
q
__
.
(b) lim
n
a
n
= 0v lim
n
a
n
= 1.

n

C
h
i
188 Ch-ng 2. Dy s thc
(c) lim
n
a
n
= minS v lim
n
a
n
= max S, y S l tp tt c im t ca
dy -c m t trong bi 2.4.7 (c).
(d) lim
n
a
n
= 1 v lim
n
a
n
= 1.
2.4.12.
(a) Nu tp hp S cc im t ca {a
n
} rng th lim
n
a
n
= A. Gi s
S khng rng. Do S ng (xem bi 2.4.9) ta c supS = lim
n
a
n
= L S.
iu ny -c suy ra t nh ngha ca im t, tc l tn ti dy con
{a
n
k
} hi t n L. V vy, vi bt k > 0 tn ti k
0
N sao cho
L < a
n
k
A vi k > k
0
.
Do -c ly bt k nn ta suy ra L A.
(b) Nu dy {a
n
} khng b chn d-i th lim
n
a
n
= A. Do , gi
s dy {a
n
} b chn d-i , tc l, tn ti B R tho mn a
n
B
vi mi n N. Hn na, theo gi s, tn ti mt dy n
k
, n
k
> k,
sao cho a
n
k
A. V vy, theo nh l Bolzano-Weierstrass (xem 2.4.30),
dy {a
n
k
} cha mt dy con hi t. Gi g l gii hn ca n. Khi
B g A. V vy, tp S bao gm tt c cc im t ca dy {a
n
}
khng rng v lim
n
a
n
= inf S g A.
(c) S dng cc l lun trong chng minh ca cu (a).
(d) Phn tch t-ng t nh- trong chng minh cu (b).
2.4.13.
(a) Ly L = lim
n
a
n
. Gi s rng (i) khng tho mn, ng-c li ta c iu phi
chng minh. Khi , tn ti > 0 sao cho vi bt k k N th c n > k
tho mn a
n
L +. Do , theo bi 2. 4.12 (d), lim
n
a
n
L +, iu
ny tri vi gi thit. Gi s (ii) khng tho mn . Khi , tn ti nhiu
> 0 v k N sao cho a
n
L vi mi n > k. Theo 2.3.12 (a), ta c
lim
n
a
n
L , iu ny tri vi gi thit. V vy, ta c L = lim
n
a
n
,
Suy ra (i) v (ii).
By gi chng ta s chng minh iu kin (i) v (ii) suy ra L = lim
n
a
n
.
T (i) suy ra dy {a
n
} b chn trn. Mt khc, t (ii) suy ra tn ti dy

n

C
h
i
2.4. i m gii hn. Gii hn tr n v gii hn d-i 189
con b chn d-i. Theo nh l Bolzano-Weierstrass (xem 2. 4. 30), dy
cha t nht mt dy con hi t. V vy, tp S gm tt c cc im t
ca {a
n
} khng rng. Chng ta s chng minh L = supS. Tht vy,
nu s l mt phn t ca S, theo (i), s L + . T tnh bt k ca
ta c s L. Hn na, t iu kin (ii), ta thy rng vi > 0 bt k
tn ti mt dy con ca dy cho hi t n s tho mn bt ng thc
L s. D nhin s S. Trong tr-ng hp ny s ko theo th hai
cng -c chng minh.
(b) iu ny -c suy ra t-ng t nh- cu (a).
By gi chng ta kho st iu kin cn v cho gii hn trn v gii
hn d-i v hn. Gii hn trn ca {a
n
} l + nu v ch nu dy
khng b chn trn. V vy,
lim
n
a
n
= + nu v ch nu vi mi M R v (1)
vi mi k N tn ti n
k
> k sao cho a
n
k
> M.
Gii hn trn ca {a
n
} l nu v ch nu dy b chn trn , bi L,
v tp cc im t ca n l rng . V vy, c mt s hu hn s hng
ca {a
n
} trong mi khong b chn [M, L]. Cho nn a
n
< M vi tt c
n ln. iu ny suy ra
lim
n
a
n
= nu v ch nu vi mi M R tn ti (2)
k N sao cho vi mi n > k, a
n
< M.
T-ng t ta c
lim
n
a
n
= nu v ch nu vi mi M R v (3)
vi mi k N tn ti n
k
> k sao cho a
n
k
< M,
lim
n
a
n
= + nu v ch nu vi mi M R tn ti (4)
k N sao cho vi mi n
k
> k a
n
> M.
2.4.14. Ta ch chng minh cho bt ng thc (a), v (b) chng minh t-ng
t. (a) hin nhin trong tr-ng hp lim
n
b
n
= + hoc lim
n
a
n
= . Nu

n

C
h
i
190 Ch-ng 2. Dy s thc
lim
n
a
n
= +, th, t vic kt hp iu kin (4) trong li gii ca bi ton
2.4.13 vi bt ng thc a
n
b
n
, ta thu -c lim
n
b
n
= + . Mt cch t-ng
t, nu lim
n
b
n
= , th t vic kt hp iu kin (3) trong li gii ca bi
ton 2.4.13 vi bt ng thc a
n
b
n
, ta thu -c lim
n
a
n
= .
By gi ta gi s c hai gii hn u hu hn v t
lim
n
a
n
= l
1
v lim
n
b
n
= l
2
.
Chng ta mun chng minh l
1
l
2
. Gi s phn chng, l
2
< l
1
. Chn > 0
nh l
2
+ < l
1
. Khi s tn ti c sao cho l
2
+ < c < l
1
.
T (ii) ca bi ton 2.4.13(b), ta c b
n
k
< l
2
+ < c. Mt khc t (i) ta c
c < l
1
< a
n
. Do vy, trong tr-ng hp ring, ta c c < a
n
k
, v t bt
ng thc b
n
k
< a
n
k
ng vi mi n
k
v hn, iu ny tri vi gi thit ca
u bi.
2.4.15. t
lim
n
a
n
= l
1
, lim
n
b
n
= l
2
, lim
n
a
n
= L
1
, lim
n
b
n
= L
2
.
u tin ta s chng minh
lim
n
(a
n
+b
n
) lim
n
a
n
+ lim
n
b
n
. (1)
Gi s l
1
v l
2
hu hn. Khi , theo bi 2.4.13 (b), vi mi > 0 tn ti k
1
sao cho a
n
> l
1
vi n > k
1
, v tn ti k
2
sao chob
n
> l
2
vi n > k
2
.
Do ta c,
a
n
+b
n
> l
1
+l
2
2 vi n > max{k
1
, k
2
}.
Kt hp vi bi ton 2.4.12 (c), dn n lim
n
(a
n
+ b
n
) l
1
+ l
2
2. Cho
0
+
, ta thu -c (1).
Nu l
1
hoc l
2
bng , th bt ng thc (1) l hin nhin. Gi ta s chng
minh mt trong cc gii hn l
1
hoc l
2
bng +, khi s c lim
n
(a
n
+b
n
) =
+.
Gi s l
1
= +, iu ny t-ng -ng vi iu kin (4) trong li gii ca
bi ton 2.4.13, tc l
vi mi m R tn ti k N sao cho a
n
> M nu n > k ()

n

C
h
i
2.4. i m gii hn. Gii hn tr n v gii hn d-i 191
Do l
2
= nn dy {b
n
} b chn d-i. Do , iu kin () -c tho mn
bi {a
n
+b
n
}. Ni cch khc, ta c lim
n
(a
n
+b
n
) = +. Do bt ng thc
(1) -c chng minh.
Chng minh ca cc bt ng thc cn li t-ng t v ta s ch -a ra chng
minh trong tr-ng hp gii hn hu hn. Theo bi 2.4.13, vi mi > 0 s tn
ti mt dy {a
n
} sao cho a
n
k
< l
1
+ v tn ti n
0
tho mn b
n
< L
2
+ khi
n > n
0
. iu c ngha l a
n
k
+ b
n
k
< l
1
+ L
2
+ 2 vi k ln. Do vy,
theo kt qu 2.4.12(b), ta thu -c lim
n
(a
n
+ b
n
) < l
1
+ L
2
+ 2. Do > 0
bt k, nn ta c
lim
n
(a
n
+b
n
) lim
n
a
n
+ lim
n
b
n
. (2)
T-ng t, vi mi > 0 tn ti mt dy {b
n
k
} sao cho b
n
k
> L
2
v tn
ti n
0
tho mn a
n
> l
1
, khi n > n
0
. Do a
n
k
+b
n
k
> l
1
+L
2
2 vi k
ln. Do vy, theo kt qu 2.4.12(c), thu -c lim
n
(a
n
+b
n
) l
1
+L
2
2.
Do c th nh tu nen ta rt ra kt lun
lim
n
(a
n
+b
n
) lim
n
a
n
+ lim
n
b
n
. (3)
Hn na, vi mi > 0 tn ti k
1
tho mn a
n
< L
1
+ khi n > k
1
, v tn
ti k
2
tho mn b
n
< L
2
+, khi n > k
2
. Do vy
a
n
+b
n
< L
1
+L
2
+ 2 vi n > max{k
1
, k
2
}.
Kt hp vi kt qu 2.4.12(a), ta c lim
n
(a
n
+b
n
) L
1
+L
2
+2. Do c th
nh tu nn thu -c
lim
n
(a
n
+b
n
) lim
n
a
n
+ lim
n
b
n
. (4)
By gi chng ta s -a v d v cc dy {a
n
} v {b
n
} sao cho cc bt ng
thc (1)-(4) tho mn. Xt
a
n
=
_

_
0 nu n = 4k,
1 nu n = 4k + 1,
2 nu n = 4k + 2,
1 nu n = 4k + 3,
b
n
=
_

_
2 nu n = 4k,
1 nu n = 4k + 1,
1 nu n = 4k + 2,
0 nu n = 4k + 3,

n

C
h
i
192 Ch-ng 2. Dy s thc
Trong tr-ng hp ny, bt ng thc trong bi ton c dng 0 < 1 < 2 < 3 < 4.
2.4.16. Khng. Ch cn xt dy {a
m
n
}, m = 1, 2, 3, . . . , bi
a
m
n
=
_
1 nu n = m,
0 nu n = m.
Khi ta c
lim
n
(a
1
n
+a
2
n
+. . . ) = 1 > 0 = lim
n
a
1
n
+ lim
n
a
2
n
+. . .
t
a
m
n
=
_
1 nu n = m,
0 nu n = m.
Trong tr-ng hp ny th
lim
n
(a
1
n
+a
2
n
+. . . ) = 1 < 0 = lim
n
a
1
n
+ lim
n
a
2
n
+. . .
2.4.17. t
lim
n
a
n
= l
1
, lim
n
b
n
= l
2
, lim
n
a
n
= L
1
, lim
n
b
n
= L
2
.
Chng ta s chi chng minh bt ng thc
(1) l
1
l
2
lim
n
(a
n
b
n
) l
1
L
2
.
Cch lm t-ng t c th p dng cho cc tr-ng hp khc.
Gi s l
1
v l
2
d-ng. Khi , t kt qu 2.4.13(b), vi mi > 0, s tn ti
n
0
sao cho
a
n
> l
1
, b
n
> l
2
vi n > n
0
.
Tip theo, a
n
b
n
> l
1
l
2
(l
1
+l
2
)+
2
vi nh l
1
> 0 v l
2
> 0. Do
, theo kt qu bi 2.4.12(c), lim
n
(a
n
b
n
) l
1
l
2
(l
1
+l
2
) +
2
. Cho 0
+
,
thu -c
l
1
l
2
lim
n
(a
n
b
n
). (i)
Nu l
1
= 0 hoc l
2
= 0 th bt ng thc (i) l hin nhin. Nu l
1
= + v
l
2
= + th (theo iu kin (4) ca li gii bi ton 2.4.13), vi mi s d-ng
M n nh tr-c, ta c th tm -c n
0
sao cho
a
n
>

M, b
n
>

M, vi n > n
0
.

n

C
h
i
2.4. i m gii hn. Gii hn tr n v gii hn d-i 193
Do a
n
b
n
> M, c ngha l lim
n
(a
n
b
n
) = +.
Gi s mt trong hai gii hn l
1
v l
2
hu hn, gii hn kia hu hn v d-ng.
Khi vi mi 0 < < l
2
v vi mi M > 0, lun tn ti s nguyn d-ng n
0
tho mn vi n > n
0
ta c
b
n
> l
2
, a
n
>
M
l
2

.
Do vy a
n
b
n
> M vi n > n
0
. Cui cng thu -c lim
n
(a
n
b
n
) = +, v bt
ng thc (i) -c chng minh.
By gi ta cn chng minh
lim
n
(a
n
b
n
) l
1
L
2
. (ii)
Nu l
1
v L
2
hu hn, th theo kt qu bi 2.4.13, ta c th tm -c dy {n
k
}
tho mn a
n
k
< l
1
+ v b
n
k
< L
2
+. Do
a
n
k
b
n
k
< l
1
L
2
+(l
1
+L
2
) +
2
Do vy lim
n
(a
n
b
n
) l
1
L
2
+ (l
1
+ L
2
) +
2
. Cho + ta thu -c (ii).
Nu l
1
= + hoc L
2
= +, th bt ng thc (ii) hin nhin.
By gi chng ta s -a v d v cc dy {a
n
} v {b
n
} sao cho cc bt ng
thc u tho mn. Xt
a
n
=
_

_
1 nu n = 4k,
2 nu n = 4k + 1,
3 nu n = 4k + 2,
2 nu n = 4k + 3,
b
n
=
_

_
3 nu n = 4k,
2 nu n = 4k + 1,
2 nu n = 4k + 2,
1 nu n = 4k + 3,
Trong tr-ng hp ny, bt ng thc trong bi ton c dng 1 < 2 < 3 < 6 < 9.
2.4.18. gi s lim
n
a
n
= lim
n
a
n
= g. Khi theo 2.4.13,
(i) Vi mi > 0 tn ti k N tho mn a
n
< g + nu n > k; v

n

C
h
i
194 Ch-ng 2. Dy s thc
(i') Vi mi > 0 tn ti k N tho mn g < a
n
nu n > k.
Do g chnh l gii hn ca dy {a
n
}.
Mt khc, nu lim
n
a
n
= g, th (i) v (ii) trong bi ton 2.4.13(a) v (b) -c
tho mn vi L = g v l = g. Do vy lim
n
a
n
= lim
n
a
n
= g.
Gi s rng lim
n
a
n
= +, Khi khng nh (1) v (4) trong li gii bi
ton 2.4.13 l hin nhin. Nu lim
n
a
n
= lim
n
a
n
= + th iu kin (4)
t-ng -ng vi lim
n
a
n
= +. L lun t-ng t -c p dng cho tr-ng hp
lim
n
a
n
= .
2.4.19. Do 2.4.15, ta c
lim
n
a
n
+ lim
n
b
n
lim
n
(a
n
+b
n
) lim
n
a
n
+ lim
n
b
n
.
Mt khc, theo kt qu tr-c, a = lim
n
a
n
= lim
n
a
n
. Do vy lim
n
(a
n
+b
n
) =
a + lim
n
b
n
. Chng minh cho bt dng th hai hon ton t-ng t nh- trn.
2.4.20. S dng bt ng thc trong bi 2.4.17, chng ta c th p dng
ph-ng php t-ng t nh- trong li gii ca bi ton tr-c.
2.4.21. Chng ta s s dng kt qu trong 2.4.13. t lim
n
a
n
= L. Khi
cc iu kin (i) v (ii) trong 2.4.13 -c tho mn. Nhn c hai v ca bt ng
thc (i) v (ii) vi 1, ta thu -c:
(i) Vi > 0 bt k lun tn ti k N sao cho vi mi n > k ta c L <
a
n
; v
(ii) Vi > 0 bt k v vi mi k N lun tn ti n
k
> k sao cho a
n
k
<
L +.
T 2.4.13(b) ta c
lim
n
(a
n
) = L = lim
n
a
n
.
Chng minh ca bt ng thc th hai ging nh- trn. Trong tr-ng hp gii
hn v hn, cn phi p dng cc khng nh (1)-(4) nu trong li gii bi ton
2.4.13.
2.4.22. Ta s p dng bi ton 2.4.13. t lim
n
a
n
= L. do theo iu kin
(i) v (ii) trong 2.4.13(a), ta c

n

C
h
i
2.4. i m gii hn. Gii hn tr n v gii hn d-i 195
(i) Vi > 0 bt k lun tn ti k N sao cho vi mi n > k ta c a
n
<
L +L
2
; v
(ii) Vi > 0 bt k v vi mi k Nlun tn ti n
k
> k sao cho L
L
2
2
<
a
n
k
.
Gi s L = 0. Khi theo (i),
1
a
n
>
1
L +L
2
=
1
L

L
2
L(L +L
2
)
>
1
L
.
Gi s 0 < <
1
L
. Khi theo (ii)
1
a
n
k
>
1
L
L
2
2
=
1
L
+

L
2
2
L(L
L
2
2
)
>
1
L
+.
Cc iu kin trn dn n (do 2.4.13(b))
lim
n
1
a
n
=
1
L
=
1
lim
n
a
n
.
Gi gi s lim
n
a
n
= 0. Cho M > 0, Theo (i) trong 2.4.13(a), tn ti mt
s t nhin k sao cho a
n
<
1
M
vi n > k. Do
1
an
> M vi n > k, m theo
khng nh (4) ca li gii 2.4.13, th c ngha l lim
n
1
an
= +. Cui cng
gi s lim
n
a
n
= +. Do vi mi > 0 v vi mi k N, lun tn ti
n
k
> k sao cho a
n
k
>
1

(xem khng nh trong li gii 2.4.13(a)). Bt ng


thc trn t-ng -ng vi
1
an
k
< . D nhin suy ra <
1
an
. Do vy c
hai iu kin trong 2.4.13(b) u -c tho mn i vi dy {
1
an
} vi l = 0, c
ngha l lim
n
1
an
= 0. Chng minh cho bt ng thc th nht hon thnh,
bt ng thc th hai chng minh t-ng t.
2.4.23. T gi thit ta rt ra 0 < lim
n
a
n
< +. Kt hp ng thc
lim
n
a
n
lim
n
1
a
n
= 1,
vi cc kt qu tr-c y ta suy ra
lim
n
a
n
=
1
lim
n
1
an
= lim
n
a
n
.
Do vy, theo 2.4.18, dy {a
n
} hi t.

n

C
h
i
196 Ch-ng 2. Dy s thc
2.4.24. Gi s {a
n
} l mt dy sao cho vi mi dy {b
n
} th ng thc u
tin ng. Xt b
n
= a
n
. T 2.4.21 ta suy ra -c
0 = lim
n
(a
n
+ (a
n
)) = lim
n
a
n
+ lim
n
(a
n
) = lim
n
a
n
lim
n
a
n
.
T , theo kt qu trong 2.4.18, ta kt lun -c dy {a
n
} hi t.
2.4.25. Gi s {a
n
} l dy nhn ga tr d-ng sao cho vi mi dy d-ng {b
n
}
th ng thc th nht -c nghim ng. Ly b
n
=
1
an
. Do , theo 2.4.22, ta
c
1 = lim
n
_
a
n
.
1
a
n
_
= lim
n
a
n
. lim
n
_
1
a
n
_
= lim
n
a
n
.
1
lim
n
a
n
.
T suy ra c hai gii hnn trn v d-i ca dy {a
n
} u d-ng v
lim
n
a
n
= lim
n
a
n
. Do dy {a
n
} hi t (theo 2.4.18).
2.4.26. Hin nhin ta c
n

a
n
lim
n
n

a
n
. Ta s chng minh lim
n
n

a
n

lim
n
a
n+1
an
. Nu lim
n
a
n+1
an
= +, th bt ng thc hin nhin ng. Do vy, ta
ga s lim
n
a
n+1
an
= L < +. T vi mi > 0, lun tn ti s k tho mn
a
n+1
a
n
< L + vi n k.
Do vy
a
n
a
k
=
a
n
a
n1
.
a
n1
a
n2

a
k+1
a
k
< (L +)
nk
.
T dn n ,
n

a
n
<
n

a
k
(L +)
k
n
(L +).
Do
n

a
k
(L +)
k
n

n
1, nn
n

a
k
(L +)
k
n
< 1 +,
vi n ln. T nhng kt qu bit ta rt ra
n

a
n
< (1 +)(L +) = L + (L + 1) +
2
vi n ln. Kt hp vi 2.4.12(a), ta c lim
n
n

a
n
L + (L + 1) +
2
. Do
c th nh tu nn lim
n
n

a
n
L = lim
n
a
n+1
an
. chng minh lim
n
a
n+1
an

lim
n
n

a
n
ch cn p dng 2.4.22 v bt ng thc va chng minh trn cho dy
{
1
an
}.

n

C
h
i
2.4. i m gii hn. Gii hn tr n v gii hn d-i 197
2.4.27. u tin ta chng minh lim
n
b
n
lim
n
a
n
. Gi s lim
n
a
n
= L <
+ (nu L = + th bt ng thc l hin nhin). Khi vi > 0, tn ti
k N sao cho a
n
< L + vi n > k. Do vy
b
n
=
a
1
+a
2
+ +a
k
+a
k+1
+ +a
n
n
<
a
1
+a
2
+ +a
k
n

k(L +varepsilon)
n
+L +.
Do
a
1
+a
2
++a
k
n

k(L+)
n

n
0 ,
a
1
+a
2
++a
k
n

k(L+)
n
< T rt ra
b
n
< +L+ vi n ln. Theo kt qu 2.4.12(a), da c th nh tu , ta thu
-c lim
n
b
n
L = lim
n
a
n
. Chng minh cho bt ng thc lim
n
a
n
lim
n
b
n
hon ton t-ng t.
2.4.28.
(a), (b) Ch cn p dng kt qu 2.4.13.
(c) ng thc khng ng. chng t iu , ch cn xt cc dy sau:
a
n
=
_
0 nu n = 2k,
1 nu n = 2k + 1,
b
n
=
_
1 nu n = 2k,
0 nu n = 2k + 1.
Khi
0 = lim
n
min{a
n
, b
n
} = min
_
lim
n
a
n
, lim
n
b
n
_
= 1.
(d) ng thc ny cng khng ng, ta chng t iu ny bng cch xt hai
dy nh- trong cu (c).
2.4.29. Gi s dy {a
n
} tho mn c v hn s n sao cho
vi mi k n, a
k
a
n
. (1)
Cho n
1
l hng t u tin ca cc n tho mn trn, n
2
th hai, v.v. Khi
{a
n
k
} l mt dy con n iu tng ca {a
n
}. Mt khc, nu dy {a
n
} khng
c tnh cht trn, ngha l ch tn ti hu hn n tho mn (1), ta chn s t
nhin m
1
sao cho dy {a
m
1
+n
} khng tho mn (1), Ly m
2
l s t nhin du
tin ln hn m
1
sao cho a
m
2
> a
m
1
. Tip tc qu trnh ny, ta thu -c dy
con{a
mn
} ca {a
n
} l n iu tng.

n

C
h
i
198 Ch-ng 2. Dy s thc
2.4.30. S dng kt qu trn: mt dy c cha mt dy con n iu tng v
b chn th hi t.
2.4.31. Gi s lim
n
a
n+1
an
= +. Khi theo 2.4.14(b),
lim
n
a
1
+ +a
n
+a
n+1
a
n
= +.
Xt
lim
n
a
n+1
a
n
= < +.
Khi , vi mi > 0, tn ti k sao cho
a
n+1
a
n
< + vi n k. (1)
Ni cch khc,
a
n
a
n+1
>
1
+
vi n k. (2)
Do vy, vi k ln ta c
b
n
=
a
1
+ +a
n
+a
n+1
a
n

a
k
+ +a
n
+a
n+1
a
n
=
a
k
a
k+1

a
n2
a
n1
.
a
n1
a
n
+
a
k+1
a
k+2

a
n2
a
n1
.
a
n1
a
n
+ +
a
n2
a
n1
.
a
n1
a
n
+
a
n1
a
n
+ 1 +
a
n+1
a
n

_
1
+
_
nk
+
_
1
+
_
nk1
+ +
1
+
+ 1 +
a
n+1
a
n
.
Nu 0 < < 1, th t bt ng thc trn v 2.4.14(b) dn n lim
n
b
n
= +.
Mt khc, nu 1, th theo 2.4.14 (b) v 2.4.19 ta rt ra
lim
n
b
n
+ lim
n
1
_
1
+
_
nk+1
1
1
+
= +
+
+ 1
. (3)
Trong tr-ng hp = 1 ( > 0 c th ty ) ta thu -c lim
n
b
n
= +. Nu
> 1 th t (3) suy ra
lim
n
b
n
1 + +
1
1
= 2 + ( 1) +
1
1
4.
4 l -c l-ng ti -u bi v kt qu ny t -c i vi dy a
n
= 2
n
, n N.

n

C
h
i
2.5. Cc bi ton hn hp 199
2.5 Cc bi ton hn hp
2.5.1. Gi s lim
n
a
n
= +. t b
n
= [a
n
]. Khi b
n
a
n
< b
n
+ 1. Do
vy
_
1 +
1
b
n
+ 1
_
bn
<
_
1 +
1
a
n
_
an
<
_
1 +
1
b
n
_
bn+1
.
Do theo kt qu trong 2.1.38 v nh lut kp gia dn n
lim
n
_
1 +
1
a
n
_
an
= e
Hn na
lim
n
_
1
1
a
n
_
an
= e
1
bi v
lim
n
_
1
1
a
n
_
an
= lim
n
1
_
1 +
1
an1
_
an
= e
1
.
T dn n
lim
n
_
1 +
1
a
n
_
an
= e, nu {a
n
} phn k ti .
2.5.2. Ta c th p dng cc kt qu bit i vi dy a
n
=
n
x
, x = 0.
2.5.3. Do 2.1.39, 2.1.40 v 2.5.2, ta c (1 +
x
n
)
n
< e
x
< (1 +
x
n
)
l+n
vi
l > x > 0, l N. Do vi mi s d-ng x v vi mi s nguyn d-ng n th
x
n+l
< ln(1 +
x
n
) <
x
n
nu l > x. Ly n = 1 ta c ln(1 + x) < x vi x > 0.
By gi t l = [x] + 1. Khi ta c
ln
_
1 +
x
n
_
>
x
n
2 +
x
n
.
Do ln(1 +x) >
x
2+x
vi x > 0.
Xt f(x) = ln(1 +x)
2x
2+x
, x > 0. Ta c
f

(x) =
x
2
(x + 1)(x + 2)
2
> 0 vi x > 0.
Do vy
f(x) = ln(1 +x)
2x
2 +x
> f(0) = 0 vi x > 0.

n

C
h
i
200 Ch-ng 2. Dy s thc
2.5.4.
(a) Gi s a > 1. t a
n
=
n

a 1. Theo bt ng thc 2.5.3, ta c


2a
n
a
n
+ 2
<
1
n
lna = ln(a
n
+ 1) < a
n
,
Do vy t nh l kp dn n lim
n
n(
n

a 1) = lna vi a > 1 Chng


ta thy ngay khng nh ng vi a = 1. chng minh cho 0 < a < 1,
ch cn p dng llun trn cho
1
a
> 1.
(b) t a
n
=
n

n 1. Khi (a
n
+ 1)
n
= n. Do vy theo 2.5.3 lnn =
nln(a
n
+ 1) < na
n
. iu ny dn n lim
n
na
n
= +.
2.5.5. S dng o hm ta chng minh -c vi x > 1,
x
1+x
ln(1+x)
x. Do lim
n
a
n
= 1, a
n
> 0 bt u t ch s n no . iu dn n
an1
1+an1
lna
n
= ln(1 + (a
n
1)) a
n
1. Chia hai v bt ng thc cho
a
n
1 v p dng nh l kp ta thu -c iu phi chng minh.
2.5.6. Theo nh ngha (xem 2.1.38), ta c e = lim
n
(1 +
1
n
)
n
. Hn na,
_
1 +
1
n
_
n
= 1 +
_
n
1
_
1
n
+
_
n
2
_
1
n
2
+ +
_
n
n
_
1
n
n
= 1 + 1 +
1
2!
_
1
1
n
_
+ +
1
k!
_
1
1
n
_
. . .
_
1
k 1
n
_
+ +
1
n!
_
1
1
n
_
. . .
_
1
n 1
n
_
.
Do
_
1 +
1
n
_
n
< a
n
. (i)
Mt khc
_
1 +
1
n
_
n
> 2 +
1
2!
_
1
1
n
_
+
1
3!
_
1
1
n
__
1
2
n
_
+ +
1
k!
_
1
1
n
_
. . .
_
1
k 1
n
_

n

C
h
i
2.5. Cc bi ton hn hp 201
Cho n tin ra v cng ta thu -c
e a
k
. (ii)
T (i) v (ii) dn n gii hn ca dy {a
n
} bng e. Hn na
a
n+m
a
n
=
1
(n + 1)!
+
1
(n + 2)!
+ +
1
(n +m)!
<
1
(n + 1)!
_
1 +
1
n + 2
+
1
(n + 2)
2
+ +
1
(n + 2)
m1
_
<
1
(n + 1)!
n + 2
n + 1
.
Gi n c nh v cho m , ta thu -c
e a
n

1
(n + 1)!
n + 2
n + 1
.
iu ny kt hp vi (ii) dn n 0 < e a
n
<
1
nn!
.
2.5.7. Ta bit rng (xem 2.5.2) e
x
= lim
n
(1 +
x
n
)
n
, x R. Vi x R c
nh, t a
n
= (1 +
x
1!
+
x
2
2!
+ +
x
n
n!
). Ta c

a
n
(1 +
x
n
)
n

k=2
_
1
_
1
1
n
_

_
1
k 1
n
__
x
k
k!

k=2
_
1
_
1
1
n
_

_
1
k 1
n
__
|x|
k
k!
.
Do 1.2.1,
_
1
1
n
_

_
1
k 1
n
_
1
k1

j=1
j
n
= 1
k(k 1)
2n
vi 2 k n.
Do vy

a
n
(1 +
x
n
)
n

k=2
k(k 1)
2n
|x|
k
k!
=
1
2n
n

k=2
|x|
k
(k 2)!
.
Do lim
n
1
2n
n

k=2
|x|
k
(k2)!
= 0 (d dng t -c khi p dng nh l Stolz, xem
2.3.11), ta c lim
n
a
n
= lim
n
(1 +
x
n
)
n
= e
x
.

n

C
h
i
202 Ch-ng 2. Dy s thc
2.5.8.
(a) T 2.1.38,
1
n+1
< ln(1 +
1
n
) <
1
n
. Vi n > 1 ta c
ln
2n + 1
n
<
1
n
+
1
n + 1
+ +
1
2n
< ln
2n
n 1
.
Do vy kt qu cn chng minh thu -c t tnh lin tc ca hm lga
v p dng nh l kp.
(b) Ta c
1
n + 1
+ +
1
2n
<
1
_
n(n + 1)
+ +
1
_
2n(2n + 1)
<
1
n
+
1
n + 1
+ +
1
2n
.
Do ta thu -c kt qu t (a).
2.5.9. Phn tch t-ng t nh- trong chng minh bi ton 2.5.3, ta thu -c
x
x
2
2
< ln(1 +x) < x vi x > 0. ()
t b
n
lna
n
=
n

k=1
ln
_
1 +
k
n
2
_
. T () suy ra,
k
n
2

k
2
2n
4
< ln
_
1 +
k
n
2
_
<
k
n
2
.
Kt hp vi cc ng thc
n

k=1
k =
n(n + 1)
2
,
n

k=1
k
2
=
n(n + 1)(2n + 1)
6
.
ta suy ra lim
n
b
n
=
1
2
. Cui cng t tnh lin tc ca hm lga suy ra -c
lim
n
a
n
=

e.
2.5.10. Ta c th chng minh bng qui np rng
a
n
= n +n(n 1) + +n(n 1) 2 +n(n 1) 2.1
=
n!
(n 1)!
+
n!
(n 2)!
+ +
n!
1!
+
n!
0!
.

n

C
h
i
2.5. Cc bi ton hn hp 203
Do vy
lim
n
n

k=1
_
1 +
1
a
k
_
= lim
n
_
a
1
+ 1
a
1

a
n
+ 1
a
n
_
= lim
n
a
n
+ 1
n!
= lim
n
_
1 +
1
1!
+ +
1
n!
_
= e,
vi ng thc cui suy ra t 2.5.6.
2.5.11. T 2.5.6,
e = 1 +
1
1!
+ +
1
n!
+

n
nn!
, vi 0 <
n
< 1.
Do vy 0 < n!e [n!e] =
n
n
<
1
n
, dn n iu phi chng minh.
2.5.12. S dng bt ng thc trung bnh lin h gia trung bnh cng v
trung bnh nhn, tnh n iu ca hm lga v bt ng thc chng minh
trong 2.5.3, ta thu -c
1
n
ln

ab ln
1
2
(
n

a +
n

b) = ln
_
1
2
(
n

a 1) +
1
2
(
n

b 1) + 1
_
<
1
2
_
(
n

a 1) + (
n

b 1)
_
.
thu -c kt qu cn chng minh, ta ch cn nhn cc bt ng thc vi n
ri s dng kt qu trong 2.5.4(a).
2.5.13. L-u rng nu lim
n
a
n
n
= a > 0, th lim
n
a
n
= 1.
Gi s {a
n
} v {b
n
} l cc dy sao cho cc s hng u khc 1. T kt qu
2.5.5,
lim
n
nln a
n
n(a
n
1)
= 1. ()
T gi thit lim
n
a
n
n
= a > 0 v t tnh lin tc ca hm lga dn n
lim
n
nln a
n
= lna. Do , t (),
lim
n
(a
n
1) = lim
n
nln a
n
= lna.
L-u rng cc ng thc trn vn ng nu a
n
= 1. Cui cng,
lim
n
nln(pa
n
+qb
n
) = lim
n
n(p(a
n
1) +q(b
n
1)) = lna
p
b
q
.

n

C
h
i
204 Ch-ng 2. Dy s thc
2.5.14. Ta c a
n+1
a
n
=
1
n
(a
n
a
n1
). Tip theo ta c
a
n
= a + (b a) + + (a
n
a
n1
)
= a + (b a)
_
1
1
2!
+
1
3!
+ (1)
n2
1
(n 1)!
_
.
Cui cng t 2.5.7, lim
n
a
n
= b (b a)e
1
.
2.5.15. Xt dy {b
n
}, vi b
n
=
an
n!
, v p dng ph-ng php ging nh- li
gii ca bi ton trn, ta i n kt lun a
n
= n!.
2.5.16. Theo nh- li gii bi 2.5.14, a
n+1
a
n
=
1
2n
(a
n
a
n1
).
Do vy lim
n
a
n
= 2b a 2(b a)e
1
2
.
2.5.17.
(a) Ta c
a
n
= 3
n

k=1
_
1
k

1
k + 1
_
1
(k + 1)!
= 3
n

k=1
k + 1 k
k(k + 1)!
+
n

k=1
1
(k + 1)(k + 1)!
= 3
n

k=1
1
kk!
+
n

k=1
1
(k + 1)!
+
n

k=1
1
(k + 1)(k + 1)!
=
n+1

k=0
1
k!
+
1
(n + 1)(n + 1)!
.
Do vy, theo 2.5.6, ta rt ra lim
n
a
n
= e.
(b) T (a) v 2.5.6,
0 < a
n
e <
1
(n + 1)(n + 1)!
.
Ch . Mt iu rt th v l dy ny tin ti e nhanh hn dy xt trong bi
ton 2.5.6.
2.5.18. T bi 2.5.6 suy ra e = 1 +
1
1!
+ +
1
n!
+ r
n
, vi lim
n
n!r
n
= 0.
Hn na,
1
n + 1
< n!r
n
<
1
n
. ()

n

C
h
i
2.5. Cc bi ton hn hp 205
Do vy,
lim
n
nsin(2n!e) = lim
n
nsin(2n!r
n
)
= lim
n
n2n!r
n
sin(2n!r
n
)
2n!r
n
= lim
n
n2n!r
n
= 2.
ng thc cui cng -c suy ra t ().
2.5.19. Ta s chng minh lim
n
(1
an
n
)
n
= 0. Theo gi thit, vi M > 0 bt
k, ta lun c a
n
> M vi n ln. Do vy
0 < 1
a
n
n
< 1
M
n
.
Tip theo,
0 <
_
1
a
n
n
_
n
<
_
1
M
n
_
n
.
Do vy, theo 2.4.12, 2.4.14 v 2.5.2, ta c
0 < lim
n
_
1
a
n
n
_
n
lim
n
_
1
a
n
n
_
n
e
M
.
Cho M , ta thu -c
0 < lim
n
_
1
a
n
n
_
n
lim
n
_
1
a
n
n
_
n
0.
Do vy lim
n
(1
an
n
)
n
= 0, ta -c iu phi chng minh.
2.5.20. Ta s chng minh lim
n
(1 +
bn
n
)
n
= +. Cho M > 0, ta c b
n
> M
vi n ln. Do vy, cch lm t-ng t nh- bi ton tr-c, ta thu -c
lim
n
_
1 +
b
n
n
_
n
lim
n
_
1 +
M
n
_
n
= e
M
.
Do M c th ln tu nn suy ra lim
n
(1 +
bn
n
)
n
= +.
2.5.21. D dng chng minh -c {a
n
} l dy n iu gim v 0.
(a) Ta c
1
a
n+1

1
an
=
1
1an

n
1. Do vy, theo 2.3.14, lim
n
1
nan
= 1;
(b) Do (a),
lim
n
n(1 na
n
)
lnn
= lim
n
n(
1
nan
1)na
n
lnn
= lim
n
1
an
n
lnn
.

n

C
h
i
206 Ch-ng 2. Dy s thc
S dng nh l Stolz (2.3.11), ta thu -c
lim
n
n(1 na
n
)
lnn
= lim
n
n(
1
a
n+1

1
an
1)
nln(1 +
1
n
)
= lim
n
n(
1
1an
1)
ln(1 +
1
n
)
n
= lim
n
na
n
1 a
n
= 1.
2.5.22. D dng thy rng dy {a
n
} l n iu gim dn v 0. Hn na, p
dng qui tc l'Hpital, ta thu -c
lim
x0
x
2
sin
2
x
x
2
sin
2
x
=
1
3
.
Do vy
lim
n
_
1
a
2
n+1

1
a
2
n
_
=
1
3
.
Theo kt qu bi 2.3.14, suy ra lim
n
na
2
n
= 3.
2.5.23. R rng dy n iu tng. Ta s chng minh n hi t ti +. Ta
c
a
2
n+1
=
_
a
n
+
1
a
1
+ +a
n
_
2
>
_
a
n
+
1
na
n
_
2
> a
2
n
+
2
n
v
a
2
2n
a
2
n
>
2
2n 1
+
2
2n 2
+ +
2
n
>
2n
2n 1
> 1.
Do vy {a
2
n
} khng phi l dy Cauchy. Do dy tng, nn n phi tin ti +.
Hn na,
1
a
n+1
a
n
1 +
1
na
n
. ()
Theo nh l Stolz
lim
n
a
2
n
2 ln n
= lim
n
n(a
2
n+1
a
2
n
)
2 ln(1 +
1
n
)
n
= lim
n
n
2
(a
2
n+1
a
2
n
)
lim
n
n
2
_
2a
n
a
1
+a
2
+ +a
n
+
1
(a
1
+a
2
+ +a
n
)
2
_
= 1,
bi v
0 <
n
(a
1
+a
2
+ +a
n
)
2
<
1
n
;

n

C
h
i
2.5. Cc bi ton hn hp 207
v li p dng nh l Stolz,
lim
n
na
n
a
1
+a
2
+ +a
n
= lim
n
(n + 1)a
n+1
na
n
a
n+1
= lim
n
_
n + 1 n
a
n
a
n+1
_
= 1.
ng thc cui c -c t (). C th ta c
1 n + 1 n
a
n
a
n+1

1 +
n+1
nan
1 +
1
nan
.
Do lim
n
a
n
= +,
lim
n
1 +
n+1
nan
1 +
1
nan
= 1.
2.5.24. T bt ng thc arctan x < x vi x > 0, nn dy n iu gim.
Hn na n b chn d-i bi 0. Do vy dy hi t, gi s ti g, sao cho tho mn
g = arctan g. Do vy g = 0.
2.5.25. L-u rng mi s hng ca dy {a
n
} u thuc vo khong (0, 1).
K hiu x
0
l nghim duy nht ca ph-ng trnh cos x = x. Nu x > x
0
th
cos(cos x) < x. Hm f(x) = cos(cos x) x l n iu gim, bi v f

(x) =
sinxsin(cos x) 1 < 0 vi x R. Do vy, vi x > x
0
th cos(cos x) x <
f(x
0
) = 0. T-ng t nu x < x
0
th cos(cos x) > x.
Gi s a
1
> x
0
. Theo phn trn ta c a
3
= cos(cos a
1
) < a
1
. Do hm s
y = cos(cos x) l n iu tng trong khong (0,

2
), ta thu -c a
5
< a
3
. C
th chng minh bng qui np rng dy {a
2n1
} l n iu gim. Mt khc
a
2
= cos a
1
< cos x
0
= x
0
, dn n a
4
= cos(cos a
2
) > a
2
, v do , {a
2n
}
l dy n iu tng.
L lun t-ng t c th p dng cho tr-ng hp 0 < a
1
< x
0
. Nu a
1
= x
0
, th
tt c cc s hng ca dy {a
n
} bng vi x
0
. Trong tt c cc tr-ng hp th
c hai dy {a
2n1
} v {a
2n
} u tin ti nghim duy nht ca ph-ng trnh
cos(cos x) = x. D dng thy rng x
0
chnh l nghim ca ph-ng trnh trn.
2.5.26. Bng qui np , ta c
a
n
= 1 (1)
n1
sin(sin(. . . sin1) . . . )
. .
(n1) ln
n > 1.

n

C
h
i
208 Ch-ng 2. Dy s thc
Do vy
1
n
n

k=1
a
k
=
n 1
n1

k=1
(1)
k1
sin(sin(. . . sin 1) . . . )
. .
(k1) ln
n
.
Chng ta s chng minh
lim
n
n1

k=1
(1)
k1
sin(sin(. . . sin1) . . . )
. .
k 1 ln
n
= 0. ()
Nu n 1 chn, th
sin1 + sin(sin(. . . sin1) . . . )
. .
(n1) ln
n
<
n1

k=1
(1)
k1
sin(sin(. . . sin1) . . . )
. .
(k1) ln
n
< 0.
Hin nhin vi n 1 l, () cng ng. Cui cng ta c lim
n
1
n
n

k=1
a
k
= 1.
2.5.27. R rng l a
n
(n, n +

2
), n = 1, 2, . . . , v do vy lim
n
a
n
=
+. Hn na,
lim
n
tan
_

2
+n a
n
_
= lim
n
1
tan a
n
= lim
n
1
a
n
= 0.
Do s lin tc ca hm arctan ta c lim
n
(

2
+n a
n
) = 0. Do vy
lim
n
(a
n+1
a
n
)
= lim
n
_

2
+n a
n
(

2
+ (n + 1) a
n+1
)
_
= 0
T suy ra lim
n
(a
n+1
a
n
) = .
2.5.28. Ch rng khng mt tng qut ta c th gi s |a
1
|

2
. C
th l, nu khng ta c th thay |a
2
|

2
. u tin ta xt 0 < a 1 v
0 < a
1


2
. Do vy a
n+1
= a sina
n
< a
n
. iu c ngha l {a
n
} l dy
n iu gim, mt khc n b chn d-i nn suy ra hi t. Gii hn ca dy

n

C
h
i
2.5. Cc bi ton hn hp 209
trng vi im 0 l nghim ca ph-ng trnh x = a sinn, 0 < a 1. Gi s
0 < a

2
v 0 < a
1


2
. Khi ph-ng trnh x = a sinn, 0 < a 1.
s c hai nghim khng m l 0 v x
0
> 0. Nu a
1
< x
0
th dy {a
n
} n
iu tng v b chn trn bi x
0
. C th, a
2
= a sina
1
> a
1
. Tuy nhin,
a
2
= a sina
1
< a sinx
0
= x
0
v, bng qui np, a
n
< a
n+1
< x
0
. T-ng t,
x
0
< a
1


2
dn n a
n
> a
n+1
> x
0
. Do vy lim
n
a
n
= x
0
vi 1 < a

2
.
Nu

2
a < 0, a
1
> 0, th ta xt dy {b
n
} nh- sau: b
1
= a
1
, b
n+1
=
a sinb
n
. Hin nhin, b
n
= (1)
n1
a
n
.
Theo phn trn th trong tr-ng hp 0 < a
1


2
ta c
lim
n
a
n
= 0 nu |a| 1,
lim
n
a
n
= x
0
nu 1 < a

2
,
lim
n
a
n
= 0 khng tn ti nu

2
a < 1.
Nu

2
a
1
< 0, th ta xt dy cho bi b
1
= a
1
, b
n+1
= a sinb
n
, v li p
dng ging nh- trn. Nu a
1
= 0, th tt c s hng ca dy u tin ti 0.
2.5.29. (a) Ta thy a
n
> 0 v a
n+1
= ln(1 + a
n
) < a
n
. Do vy dy s hi
t ti gii hn g tho mn g = ln(1 + g), ngha l g = 0. Ta s chng minh
lim
n
na
n
= 2. S dng o hm, ta thu -c( xem 2.5.3)
2x
2 +x
< ln(1 +x) < x
x
2
2
+
x
3
3
vi x > 0.
T dn n
()
1
a
n

1
a
n
+
1
a
n
(1
1
2
a
n
+
1
3
a
2
n
)
<
1
a
n+1
<
1
a
n
+
1
2
.
t
b
n
=
1
a
n
+
1
a
n
(1
1
2
a
n
+
1
3
a
2
n
)
<
1
a
n+1
ta thy lim
n
b
n
=
1
2
. Cng c hai v ca () thu -c
1
a
1
+
1
a
2
+ +
1
a
n
+b
1
+b
2
+ +b
n
<
1
a
2
+ +
1
a
n
+
1
a
n+1
<
1
a
1
+
1
a
2
+ +
1
a
n
+
n
2
.

n

C
h
i
210 Ch-ng 2. Dy s thc
Do vy,
1
(n + 1)a
1
+
b
1
+b
2
+ +b
n
n + 1
<
1
(n + 1)a
n+1
<
1
(n + 1)a
1
+
n
2(n + 1)
.
Suy ra lim
n
1
(n+1)a
n+1
=
1
2
.
(b) Ta c
(1) lim
n
(na
n
2)
n
lnn
= lim
n
na
n
n
2
an
lnn
chng minh lim
n
n
2
an
ln n
tn ti ta s dng nh l Stolz (2.3.11). Ta c
lim
n
n
2
an
lnn
= lim
n
1
2
a
n+1
+
2
an
ln(1 +
1
n
)
.
Do lim
n
a
n+1
an
= lim
n
ln(1+an)
an
= 1 v lim
n
nln(1 +
1
n
) = 1, ta suy ra
(2) lim
n
n
2
an
lnn
= lim
n
n(2a
n+1
2a
n
+a
n
a
n+1
)
a
2
n
.
Ta cn chng minh lim
n
n(2a
n+1
2an+ana
n+1
)
a
3
n
tn ti. chng minh ta s dng
bt ng thc (c th chng minh d dng bng o hm) nh- sau:
x
x
2
2
+
x
3
3

x
4
4
< ln(1 +x) < x
x
2
2
+
x
3
3
, x > 0.
Do vy
1
6
a
3
n

1
6
a
4
n

1
4
a
5
n
< 2a
n+1
2a
n
+a
n
a
n+1
<
1
6
a
3
n
+
1
3
a
4
n
.
Dn n
lim
n
2a
n+1
a
n
+a
n
a
n+1
a
3
n
=
1
6
.
Kt hp vi (1) v (2) ta thu -c lim
n
n(nan2)
lnn
=
2
3
.
2.5.30. t f(x) = (
1
4
)
x
v F(x) = f(f(x)) x. u tin ta chng minh
F

(x) < 0 vi mi x d-ng. Ta c


F

(x) =
_
1
4
_
(
1
4
)
x
+x
ln
2
4 1.

n

C
h
i
2.5. Cc bi ton hn hp 211
Do vy
F

(x) < 0 nu v chi nu


_
1
4
_
(
1
4
)
x
+x
<
1
ln
2
4
.
D dng kim tra rng hm pha bn tri ca bt ng thc cui t -c gi
tr ln nht bng
1
e ln4
ti x =
ln ln4
ln4
. iu ny dn n F

(x) < 0, ngha l F


gim ngt trong khong (0, +). Hn na, F(
1
2
) = 0. Do F(x) > 0 vi
0 < x <
1
2
v F(x) < 0 vi x >
1
2
. Tip theo ta c
f(f(x)) < x vi x >
1
2
.
Do a
2
= 1 >
1
2
, dn n a
4
= f(f(a
2
)) < a
2
, v bng cch qui np ta thu
-c {a
2n
} l dy gim ngt. Do n tin ti g
1
tho mn f(f(g
1
)) = g
1
.
S hi t ca {a
2n1
} ti g
2
tho mn f(f(g
2
)) = g
2
c th -c thit lp mt
cch t-ng t. C th g
1
= g
2
=
1
2
.
2.5.31. Ch l 0 < a
n
< 2 vi n 2. Nu a
n
> 1, th a
n+1
< 1. t
f(x) = 2
1x
v F(x) = f(f(x)) x. Ta chng minh -c F

(x) < 0 vi
0 < x < 2. Do vy
F(x) < F(1) = 0 vi 1 < x < 2,
F(x) > F(1) = 0 vi 0 < x < 1.
Theo chng minh ca phn trn, ta c nu a
1
< 1, th dy {a
2n
} n iu gim
v dy {a
2n1
} n iu tng, v c hai dy cng tin ti 1.
2.5.32. Ch rng tt c cc s hng ca dy u nm trong khong (1, 2).
Do hm F(x) = 2
x
2
x gim ngt trong khong ny nn F(x) > F(2) = 0
vi x (1, 2). Do vy dy n iu tng v gii hn g ca n tho mn g = 2
g
2
,
hay g = 2.
2.5.33. Ta s dng 2.3.14 cho dy {a
n
+a
n1
} v nhn -c lim
n
an+a
n1
n
=
0. Tip theo ta xt dy b
n
= (1)
n
a
n
. V lim
n
(b
n
b
n2
) = 0, ta c
0 = lim
n
b
n
+b
n1
n
= lim
n
a
n
a
n1
n
.
2.5.34. Theo nh l Stolz (xem 2.3.11),
lim
n
ln
1
an
lnn
= lim
n
ln
1
a
n+1
ln
1
an
ln(n + 1) lnn
= lim
n
nln
a
n+1
an
ln(1 +
1
n
)
n
= lim
n
nln
a
n+1
a
n
.

n

C
h
i
212 Ch-ng 2. Dy s thc
Nu lim
n
n(1
a
n+1
an
) = g l hu hn, th lim
n
(
a
n+1
an
1) = 0). Kt qu cn
chng minh -c suy ra t bt ng thc sau y
n
a
n+1
an
1
1 +
a
n+1
an
1
nln(1 + (
a
n+1
a
n
1)) n(
a
n+1
a
n
1).
Nu g = +, th bt ng thc bn phi ch ra rng
lim
n
nln
a
n+1
an
= , v do , lim
n
ln
1
an
lnn
= +. Cui cng, nu g = ,
th vi mi M > 0 tn ti n
0
tho mn
a
n+1
an
>
M
n
+ 1 vi n > n
0
. T
nln
a
n+1
a
n
> ln(1 +
M
n
)
n

n
M.
V M c th ln tu , nn ta c lim
n
ln
1
an
ln n
= .
2.5.35. Theo nh ngha ca dy,
a
n+1
+b
n+1
= (a
1
+b
1
)(1 (a
n
+b
n
)) + (a
n
+b
n
).
td
n
= a
n
+ b
n
. Khi d
n+1
= d
1
(1 d
n
) + d
n
v bng ph-ng php quy
np ta chng minh -c d
n
= 1 (1 d
1
)
n
. Do ,
a
n
=
a
1
d
1
(1 (1 d
1
)
n
) v b
n
=
b
1
d
1
(1 (1 d
1
)
n
).
V |1 d
1
| < 1, nn
lim
n
a
n
=
a
1
a
1
+b
1
v lim
n
b
n
=
b
1
a
1
+b
1
.
2.5.36. t b
n
= aa
n
. Khi b
n+1
= b
n
(2 b
n
) = (b
n
1)
2
+ 1. T
suy ra b
n+1
1 = (b
n
1)
2
. Hin nhin, dy {a
n
} hi t nu v ch nu
{b
n
1} hi t, ni cch khc, khi |b
1
1| = |aa
1
1| 1. Hn na, nu
a
1
=
2
a
, th lim
n
a
n
= 0, v nu 0 < aa
1
< 2, th lim
n
a
n
=
1
a
.
2.5.37. y l tr-ng hp c bit ca bi 2.5.38.
2.5.38. Ta c th ch ra rng hm f lin tc ti (a, a, ..., a) v f(a, a, ..., a) =
a. Xy dng dy {b
n
} bng cch
b
1
= b
2
= ... = b
k
= min{a
1
, a
2
, ..., a
k
},
b
n
= f(b
n1
, b
n2
, ..., b
nk
) vi n > k.

n

C
h
i
2.5. Cc bi ton hn hp 213
Ch rng nu min{a
1
, a
2
, ..., a
k
} < a, th {b
n
} tng thc s v b chn trn
bi a. Mt khc, nu min{a
1
, a
2
, ..., a
k
} > a, th {b
n
} gim thc s v b chn
d-i bi a. T , trong c hai tr-ng hp, dy {b
n
} hi t v lim
n
b
n
= a.
Hn na, do tnh n iu ca f theo mi bin nn a
n
b
n
vi n N. By
gi ta xy dng dy {c
n
} nh- sau
c
1
= c
2
= ... = c
k
= max{a
1
, a
2
, ..., a
k
},
c
n
= f(c
n1
, c
n2
, ..., c
nk
) vi n > k.
Cng nh- trn, ta ch ra rng lim
n
c
n
= a v c
n
a
n
vi n N. Cui cng,
theo nguyn l kp, lim
n
a
n
= a.
2.5.39. Ta c a
3
= a
2
e
a
2
a
1
, a
4
= a
3
e
a
3
a
2
= a
2
e
a
3
a
1
v, bng cch quy
np, a
n+1
= a
2
e
ana
1
vi n 2. Gi s g l gii hn ca dy. Ta c
e
a
1
a
2
g = e
g
. (star)
Ch rng nu
e
a
1
a
2
= e,th ph-ng trnh () c nghim duy nht g = 1. Nu
e
a
1
a
2
> e th ph-ng trnh ny c hai nghim, v nu 0 <
e
a
1
a
2
< e th n khng
c nghim. Tr-c ht xt tr-ng hp 0 <
e
a
1
e
a
2
< e. Khi dy {a
n
} phn k v
() khng c nghim. Hn na, ta c th chng minh rng dy {a
n
} n iu
tng v do phn k ra +.
By gi ta xt tr-ng hp
e
a
1
a
2
= e. Khi a
2
= e
a
1
1
a
1
v bng quy
np, a
n+1
a
n
. Hn na, nu a
1
1, th cng bng quy np ta c a
n
1.
T , lim
n
a
n
= 1. Nu a
1
> 1, th {a
n
} n iu tng v phn k ra +.
Tip theo, ta xt tr-ng hp
e
a
1
a
2
> e. Khi () c hai nghim l g
1
, g
2
, trong
ta gi s g
1
< g
2
. Gi s rng a
1
< g
1
. Th
e
a
1

e
a
1
a
2
a
1
> 0
hay, ni cch khc, a
2
> a
1
. Dng quy np ta c {a
n
} n iu tng v b chn
trn bi g
1
l gii hn ca n. Nu g
1
< a
1
< g
2
, th {a
n
} n iu gim v b
chn d-i bi g
1
l gii hn ca n. Nu a
1
= g
1
hoc a
1
= g
2
, th dy l hng
s. Cui cng, nu a
1
> g
2
, th dy tng ti +.
2.5.40. (Bi ton ny v li gii ca n da theo Euler trong tr-ng hp tng
qut. Xem [13]). S dng o hm, ta chng minh -c lnx
x
e
vi x > 0.

n

C
h
i
214 Ch-ng 2. Dy s thc
T
an
e
lna
n
= a
n1
lna, n > 1, v do , a
n
a
n1
lna
e
. Vy nu
a > e
1
e
, dy {a
n
} n iu tng. Ta s chng minh trong tr-ng hp ny,
lim
n
a
n
= +. Ta c a
n+1
a
n
= a
an
a
n
. Do khi a > e
1
e
, xt hm
g(x) = a
x
x. Hm ny t gi tr nh nht ti x
0
=
ln(lna)
ln a
< e. iu ny
suy ra a
x
x >
1+ln(lna)
lna
> 0, v h qu l a
n+1
a
n
>
1+ln(lna)
lna
> 0. V
khong cch gia hai s hng lin tip ln hn mt s d-ng nn dy phn k
ra +.
By gi ta xt tr-ng hp 1 < a < e
1
e
. Tr-c ht ta chng minh trong
tr-ng hp ny ph-ng trnh a
x
x = 0 c hai nghim d-ng. o hm ca
hm g(x) = a
x
x trit tiu ti im x
0
> 0 tho mn a
x
0
=
1
ln a
. Hm g t
gi tr nh nht ti x
0
, v g(x
0
) = a
x
0
x
0
=
1
lna
x
0
=
1x
0
ln a
ln a
< 0, v
nu 1 < a < e
1
e
th
1
lna
> e. V g l hm lin tc trn R, n c tnh cht im
trung bnh. Do ph-ng trnh a
x
= x c mt nghim trong khong (0, x
0
) v
mt nghim khc trong khong (x
0
, +). K hiu cc nghim ny l v ,
t-ng ng. Ch g(e) = a
e
e < (e
1
e
)
e
e = 0, e nm gia v .
Nu x > , th a
x
> a

= v g(x) > 0. iu ny c ngha l dy {a


n
}
n iu tng v b chn d-i bi . T lim
n
a
n
= +.
Nu < x < th < a
x
< v g(x) < 0. H qu l, dy {a
n
} b chn
v n iu gim. Do dy hi t ti .
Khi x = hoc x = , ta thu -c dy hng s.
By gi nu 0 < x < , th 1 < a
x
< v g(x) > 0. Do dy {a
n
} tng
ti .
Cui cng, nu a = e
1
e
, th e l nghim duy nhtca ph-ng trnh a
x
= x,
khi hm g t gi tr nh nht l 0 ti e. Do vi 0 < x e, ta c
0 < a
x
e v g(x) g(e) = 0. iu ny dn ti dy {a
n
} n iu tng v
gii hn ca n bng e. Mt khc, nu x > e, dy tng ti v cng.
Ta c th tng kt li kt qu nh- sau:
lim
n
a
n
=
_

_
+ nu a > e
1
e
v x > 0,
+ nu 1 < a < e
1
e
v x > ,
nu 1 < a < e
1
e
v x = ,
nu 1 < a < e
1
e
v 0 < x < ,
+ nu a = e
1
e
v x > e,
6 nu a = e
1
e
v 0 < x e.
.
2.5.41. Bt ng thc c th -c chng minh bng quy np. Ta c lim
n
a
n
=

n

C
h
i
2.5. Cc bi ton hn hp 215
2 (so snh vi li gii ca 2.1.16).
2.5.42. [20] Ch rng
a
n

_
2 +
_
2 +... +

2
. .
n - cn
< 2.
Nhn thy rng nu
1
= 0, th tt c cc s hng ca dy {a
n
} bng 0. Gi s
rng
1
= 0. Ta s chng minhbng quy np rng bt ng thc cho ng.
Vi n = 1, hin nhin. Gi s rng
a
n
=
1
_
2 +
2
_
2 +... +
n

2 = 2 sin
_

4
n

k=1

2
...
k
2
k1
_
.
T
a
2
n+1
2 = 2 sin
_

4
n+1

k=2

2
...
k
2
k2
_
= 2 cos
_

2
+

2
n+1

k=2

2
...
k
2
k1
_
= 2 cos
_

2
n+1

k=2

2
...
k
2
k1
_
= 4 sin
2
_

4
n+1

k=1

2
...
k
2
k1
_
2,
dn ti ng thc cn chng minh. By gi, do tnh lin tc ca hm sinx,
lim
n
a
n
= 2 sin
_

k=1

2
...
k
2
k1
_
.
2.5.43. Dng quy np chng minh rng
arctan
1
2
+... + arctan
1
2n
2
= arctan
n
n + 1
.
V vy lim
n
(arctan
1
2
+... + arctan
1
2n
2
) =

4
.
2.5.44. Ta c
sin
2
(

n
2
+n) = sin
2
(

n
2
+n n) = sin
2

1 +
_
1 +
1
n

n
1.
2.5.45. Ta chng minh bng quy np rng dy n iu tng v b chn trn.
Do n hi t ti g tho mn g =
_
2 +

3 +g v g (2, 3).

n

C
h
i
216 Ch-ng 2. Dy s thc
2.5.46. [13] Ta c
3 =

1 + 2.4 =
_
1 + 2

16 =
_
1 + 2
_
1 + 3

25
=

1 + 2
_
1 + 3
_
1 + 4

36,
v, bng quy np,

_
1 + 2

1 + 3
_
1 +...
_
1 +n
_
(n + 2)
2
= 3. (1)
T
3

_
1 + 2

1 + 3
_
1 +...
_
1 + (n 1)
_
(n + 1). (2)
By gi ta s dng bt ng thc sau:

1 +x

x + 1, x 0, > 1. (3)
T (3) vi x = n v = n + 2,
_
1 +n
_
(n + 2)
2
<
_
(n + 2)
_
(n + 1).
T
_
1 + (n 1)
_
1 +n
_
(n + 2)
2
<
_
1 +

n + 2(n 1)

1 +n
(n + 2)
1
4
_
1 + (n 1)

n + 1,
trong bt ng thc cui cng suy t (3) vi =

n + 2. T (1), lp li
cch lm n ln cho ta
3 (n + 2)
2
n

_
1 + 2

1 + 3
_
1 +...
_
1 + (n 1)
_
(n + 1). (4)
T (2) v (4) cho ta
lim
n

_
1 + 2

1 + 3
_
1 +...
_
1 + (n 1)
_
(n + 1) = 3.

n

C
h
i
2.5. Cc bi ton hn hp 217
2.5.47. Ph-ng trnh x
2
+xa = 0, a > 0, c hai nghim v tho mn
> 0 > . Hn na, ta c
a
n+1
=
a
a
n
1 =
a a
n
a
n
a
n
=
a (1 +)(a
n
) (1 +)
a
n
=
(1 +)(a
n
)
a
n
.
V + = 1, ta nhn thy a
n+1
=
an
an
. Cng nh- th, a
n+1
=

an
an
. Do
a
n+1

a
n+1

=

a
n

a
n

,
v bng quy np
a
n

a
n

= (

)
n1
a
1

a
1

.
V

=

1+
< 1, ta nhn -c lim
n
(

)
n1
= 0, v do lim
n
a
n
= .
2.5.48. Gi s v l cc nghim ca x
2
+ x a = 0, a > 0. Khi
> 0 > . Bng cch lm t-ng t nh- bi trn ta -c
a
n

a
n

= (

)
n1
a
1

a
1

.
Do lim
n
a
n
= .
2.5.49. Vi s nguyn d-ng k bt k, ta c
|a
n+1+k
a
n+1
| = |
1
1 +a
n+k

1
1 +a
n
| =
|a
n+k
a
n
|
(1 +a
n+k
)(1 +a
n
)

1
4
|a
n+k
a
n
|.
By gi, bng quy np ta c
|a
n+1+k
a
n+1
leq(
1
4
)
n
|a
k+1
a
1
|.
Hn na,
|a
k+1
a
1
| |a
k+1
a
k
| +|a
k
a
k1
| +... +|a
2
a
1
|

1
1
1
4
|a
2
a
1
| =
4
3
|a
2
a
1
|.
Do {a
n
} l dy Cauchy. Gii hn ca n l

2.

n

C
h
i
218 Ch-ng 2. Dy s thc
2.5.50. Vi cch lm t-ng t nh- bi ton trn, hy ch ra lim
n
a
n
= 1+

2.
2.5.51. t f(x) =
a
2+x
, x > 0, v F(x) = f(f(x)). Khi F

(x) > 0 vi
x > 0. D dng th rnga
1
< a
3
v a
4
< a
2
. Hn na, v F tng thc s, ta
suy ra dy {a
2n
} gim thc s v dy {a
2n1
} tng thc s. Dy {a
n
} b chn.
Do hai dy con ca n {a
2n
} v {a
2n1
} hi t. Ta c th kim tra chng
c cng mt gii hn l

1 +a 1.
2.5.52. Nu a
1
0, th a
2
= 1 a
1
> 1 v a
3
= a
2

1
2
>
1
2
. Bng quy
np, a
n+1
= a
n

1
2
n1
vi n 2. Suy ra
a
n+1
= (
1
2
n1
+
1
2
n2
+... +
1
2
) +a
2
,
v do lim
n
a
n
= a
1
nu a
1
0. By gi nu a
1
(0, 2), th a
2
[0, 1) v
bng quy np, ta nhn thy a
n+1
[0,
1
2
n1
], iu ny dn n lim
n
a
n
= 0.
Cui cng, nu a
1
2, th a
2
= a
1
1 1. Bng quy np, ta c a
n+1
= a
n

1
2
n1
, v h qu l, cng nh- tr-ng hp u, ta chng minh lim
n
a
n
= a
1
2.
2.5.53.
Ta c
n1

j=1
ja
j
n j
=
a
n 1
+
2a
2
n 2
+
3a
3
n 3
+... +
(n 1)a
n1
1
1
n 1
(
a
1
+
2(n 1)a
2
n 2
+
3(n 1)a
3
n 3
+... +
(n 1)
2
a
n1
1
).
T
j
n 1
n j
= j(
n j
n j
+
j 1
n j
) j(1 +j 1) = j
2
,
ta thu -c
n1

j=1
ja
j
n j

a + 2
2
a
2
+ 3
2
a
3
+... + (n 1)
2
a
n1
n 1
.
Theo kt qu ca bi 2.3.2, ta thu -c
lim
n
a + 2
2
a
2
+ 3
2
a
3
+... + (n 1)
2
a
n1
n 1
= 0.

n

C
h
i
2.5. Cc bi ton hn hp 219
Nhn thy rng
na
n
n

j=1
1
ja
j
=
n

j=1
n
j
a
nj
=
n1

k=0
n
n k
a
k
=
n1

k=0
a
k
+
n1

k=0
ka
k
n k
v s dng (a)
S dng (b) vi a =
1
b
.
2.5.54. V x d-ng, x
x
3
6
< sinx < x, ta c
n

k=1

n +k

n

k=1

3
6(n +k)
3
<
n

k=1
sin

n +k
<
n

k=1

n +k
.
D dng th rng lim
n
n

k=1

3
6(n+k)
3
= 0. Hn na, theo 2.5.8 (a), lim
n
n

k=1

n+k
=
ln2. Vy gii hn l ln2.
2.5.55.
t a
n
=
n

k=1
(1+
k
2
cn
3
). Da vo bt ng thc (xem2.5.3)
x
x+1
< ln(x+1) < x
vi x > 0, ta -c
n

k=1
k
2
cn
3
+k
2
< lna
n
<
n

k=1
k
2
cn
3
.
Do , t cng thc
n

k=1
k
2
=
n(n+1)(2n+1)
6
.
n(n + 1)(2n + 1)
6(cn
3
+n
2
)
< lna
n
<
n(n + 1)(2n + 1)
6cn
3
.
Do lim
n
a
n
= e
1
3c
.
Ta c th chng minh bt ng thc
x
x+1
< ln(x + 1) < x vi x > 0 cng
ng vi 1 < x < 0. Do , nh- chng minh trong phn (a), ta c
lim
n
n

k=1
(1
k
2
cn
3
) = e

1
3c
.

n

C
h
i
220 Ch-ng 2. Dy s thc
2.5.56. V x d-ng, x
x
3
6
< sinx < x
x
3
6
+
x
5
5!
, ta nhn thy

n
3n
n!
n!
(n

n)
n
n

k=1
(1
k
2
6n
3
) <

n
3n
n!
n

k=1
sin
k
n

n
(1)
v

n
3n
n!
n

k=1
sin
k
n

n
<

n
3n
n!
n!
(n

n)
n
n

k=1
_
1
k
2
6n
2
+
k
4
5!n
6
_
. (2)
T (1) v kt qu bit tr-c ta suy ra gii hn ny ln hn hoc bng e

1
18
.
By gi ta chng minh
lim
n
n

k=1
(1
k
2
6n
3
+
k
4
5!n
6
) e

1
18
.
Tht vy,
ln
n

k=1
_
1
k
2
6n
3
+
k
4
5!n
6
_
<
n

k=1
_

k
2
6n
3
+
k
4
5!n
6
_
=
n(n + 1)(2n + 1)
36n
3
+
n(n + 1)(2n + 1)(3n
2
+ 3n 1)
30.5!n
6
.
Cui cng, t (2) v nh l kp ta c
lim
n

n
3n
n!
n

k=1
sin
k
n

n
= e

1
18
.
2.5.57. Tr-c ht ta chng minh a
n
=
n+1
2n
a
n1
+ 1, n 2. Ta c
a
n
=
n

k=0
1
_
n
k
_ =
n1

k=0
k!(n 1 k)!(n k)
(n 1)!n
+ 1
=
n1

k=0
1
_
n1
k
_
n1

k=0
k
n
(n 1 k)!k!
(n 1)!n
+ 1 (1)
= a
n1

n1

k=0
k
n
1
_
n1
k
_ + 1.

n

C
h
i
2.5. Cc bi ton hn hp 221
Hn na,
n1

k=0
k
n
1
_
n1
k
_ =
n1

k=0
n 1 k
n
1
_
n1
k
_ =
n 1
n
a
n1

n1

k=0
k
n
1
_
n1
k
_
V vy
n 1
2n
a
n1
=
n1

k=0
k
n
1
_
n1
k
_.
T (1), ta -ca
n
= a
n1

n1
2n
a
n1
+1 =
n+1
2n
a
n1
+1. Cho nn lim
n
a
n
= 2.
2.5.58. Nu = 0, hin nhin lim
n
a
n
= 0. Nu > 0, th 0 < a
n
<
1
(n1)

nn lim
n
a
n
= 0. By gi ta gii trong tr-ng hp < 0. Lc
a
n
= (1)
n1
(n

1)
_
n
2
)

1
_
. . .
_
_
n
n 1
_

1
_
.
V vy, nu ta chn = 1, ta -c dy phn k a
n
= (1)
n1
. Nu < 1,
ta -c
((
n
p
)

1)((
n
n p
)

1) > (
n
p
1)(
n
n p
1) = 1
vi 1 p < n. Hn na, (
n
n
2
)

1 > 2 1 = 1. V vy
|a
n
| (n

1)(
n
n 1
)

1)
n
+.
Cng nh- vy, ta c nu 1 < < 0, th
|a
n
| (n

1)(
n
n 1
)

1)
n
0.
2.5.59. Ta c (2 +

3)
n
=
n

k=0
_
n
k
_
(

3)
k
2
nk
. Nu nhm cc s hng l v
chn vi nhau t-ng ng, ta c th vit
(2 +

3)
n
= A
n
+

3B
n
v (2

3)
n
= A
n

3B
n
.
T d lim
n
(A
n
+

3B
n
) = + v lim
n
(A
n

3B
n
) = +. V th
lim
n

3B
n
A
n
= 1.

n

C
h
i
222 Ch-ng 2. Dy s thc
V A
n
l cc s nguyn v

3Bn
An
< 1, nn [

3B
n
] = A
n
1 vi n ln. T
suy ra
{

3B
n
}
n
1 hoc {A
n
+

3B
n
} = {

3B
n
}
n
1.
2.5.60. Dy {S
n
} n iu tng. Nu chng b chn trn th s hi t, v do

lim
n
a
n
= lim
n
(S
n
S
n1
) = 0.
Gi s rng lim
n
S
n
= +. T gi thit, S
n+1
a
n+1
+a
n
S
n
a
n
+a
n1
suy
ra S
n
a
n
+a
n1
S
2
a
2
+a
1
. T
a
n
a
n
+
a
n1
S
n

S
2
a
2
+a
1
S
n
.
Cui cng, lim
n
a
n
= 0.
2.5.61. Theo gi thit, vi mi > 0 tn ti mt s nguyn d-ng n
0
tho
mn a
n
< n vi n > n
0
. Do
a
2
1
+a
2
2
+... +a
2
n
n
2
=
a
2
1
+a
2
2
+... +a
2
n
0
n
2
+
a
2
n
0
+1
+... +a
2
n
n
2

a
2
1
+a
2
2
+... +a
2
n
0
n
2
+
n(a
n
0
+1
+ ... +a
n
)
n
2
.
T 2.4.14 v 2.4.19,
lim
n
a
2
1
+a
2
2
+... +a
2
n
n
2
lim
n
a
1
+a
2
+... +a
n
n
.
iu ny hin nhin suy ra lim
n
a
2
1
+a
2
2
+...+a
2
n
n
2
= 0.
2.5.62. Ta s dng nh l Toeplitz (xem 2.3.1). t
A
n
= a
1
+a
2
+... +a
n
, B
n
= b
1
+b
2
+... +b
n
, C
n
= c
1
+c
2
+... +c
n
v
d
n,k
=
a
nk+1
B
k
a
1
B
n
+a
2
B
n1
+... +a
n
B
1
.
By gi ta s chng minh rng cc s d-ng d
n,k
tho mn cc iu kin (i) v
(ii) trong 2.3.1 (cng xem 2.3.3 (a)). Vi k c nh,
d
n,k

a
nk+1
a
1
+a
2
+... + a
nk+1

n
0.

n

C
h
i
2.5. Cc bi ton hn hp 223
R rng
n

k=1
d
n,k
= 1 v nhn thy
c
n
C
n
=
a
1
b
n
+a
2
b
n1
+... +a
n
b
1
a
1
B
n
+a
2
B
n1
+... +a
n
B
1
= d
n,1
b
1
B
1
+d
n,2
b
2
B
2
+... +d
n,n
b
n
B
n
.
Cui cng, theo nh l Toeplitz, lim
n
cn
Cn
= lim
n
bn
Bn
= 0.
2.5.63. Ta bit rng x
x
2
2
< ln(x + 1) < x
x
2
2
+
x
3
3
vi x > 0. t
a
n
= (1 +
1
n
)
n
2
e
n
. Do
1
2
< lna
n
<
1
2
+
1
3n
, dn n lim
n
lna
n
=
1
2
.
Vy gii hn l e

1
2
.
2.5.64. Ta c a
n+1
a
n
>
1
n
2
>
1
n(n1)
=
1
n1
+
1
n
vi n > 1. t
b
n
= a
n

1
n1
. Khi dy {b
n
} n iu tng v b chn trn; n hi t. T
{a
n
} cng hi t.
2.5.65. Theo gi thit a
n+1
2
n

2 a
n
, ta c
a
n+1
2

1
2
n
a
n
2

1
2
n1
.
T dy b
n
= a
n
2

1
2
n1
n iu tng v b chn. Do , n hi t. Hin
nhin, lim
n
b
n
= lim
n
a
n
.
2.5.66. Xt a (l, L). Gi s phn chng rng a khng phi l mt im
gii hn ca {a
n
}. Khi s c mt ln cn ca a ch cha hu hn cc phn
t ca dy. Gi s > 0 l s nh m
l < a < a < a + < L v a
n
[a , a +] vi n > n
1
. ()
Theo gi thit, |a
n+1
a
n
| < vi n > n
2
. Theo 2.4.13 (b), ta bit rng tn
ti a
n
k
tho mn a
n
k
< l + < a vi n
k
> max{n
1
, n
2
}. T a
n
k
+1

a
n
k
+|a
n
k
+1
a
n
k
| < a +. Do t (), a
n
k
+1
< a . V vy, theo 2.4.12
(a), L a < L, mu thun.
2.5.67. Xt a (l, L). Gi s phn chng rng a khng phi l mt im
gii hn ca {a
n
}. Khi s c mt ln cn ca a ch cha hu hn cc phn
t ca dy. Gi s > 0 l s nh m
l < a < a < a + < L v a
n
[a , a +] vi n > n
1
. ()
Theo gi thit,
a
n
a
n+1
<
n
< vi n > n
2
. ()
Theo 2.4.13 (a), c a
n
k
tho mn a
n
k
> L > a. By gi, t , a
n
k
+1
> a+
vi n
k
> max{n
1
, n
2
}. Do t 2.4.12 (c), l a + > a > l, mu thun.

n

C
h
i
224 Ch-ng 2. Dy s thc
2.5.68. Ta s s dng kt qu -c chng minh bi ton trn. T tnh n
iu ca {a
n
},
a
n+1
n + 1 +a
n+1

a
n
n + a
n

a
n
(n + 1 +a
n+1
)(n +a
n
)

1
n
.
Do , theo kt qu ca bi ton trn, tp hp cc im gii hn ca dy cho
l on [l, L], trong
l = lim
n
n
n +a
n
vL = lim
n
n
n +a
n
.
2.5.69. Ch rng

a
2n+1

1
3

=
1
2

a
2n

2
3

=
1
2

1 +a
2n1
2

2
3

=
1
4

a
2n1

1
3

.
Ko theo dy c hai im gii hn l:
1
3
v
2
3
.
2.5.70. Ta bit rng, theo 1.1.14, vi bt k s nguyn d-ng n tn tai mt s
nguyn d-ng q
n
v mt s nguyn p
n
tho mn

2
p
n
q
n

<
1
q
2
n
.
Do |p
n
| < (2 + 1)q
n
, v v th,
|
_
|p
n
| sinp
n
| = |
_
|p
n
| sin(2q
n
p
n
)|

_
|p
n
| sin
1
q
n

2 + 1

q
n
.
V dy {q
n
} khng b chn, n cha mt dy con phn k ra v cng. Do 0
l mt im gii hn ca {a
n
}.
2.5.71. Ta s chng minh c mt dy con {a
n
k
} tho mn
_
n
k
(a
1
+a
n
k
+1
)
(n
k
+ 1)a
n
k
_
n
k
1.
Gi s rng iu kin trn khng ng. Khi tn ti n
0
tho mn
n(a
1
+a
n+1
)
(n + 1)a
n
< 1 n n
0
.

n

C
h
i
2.5. Cc bi ton hn hp 225
T
a
1
n+1
+
a
n+1
n+1
<
an
n
vi n n
0
. Do
a
n
n

a
n1
n 1
<
a
1
n
,
a
n1
n 1

a
n2
n 2
<
a
1
n 2
,
.
.
.
a
n
0
+1
n
0
+ 1

a
n
0
n
0
<
a
1
n
0
+ 1
.
Ly tng tt c cc bt ng thc trn, ta -c
a
n
n
<
a
n
0
n
0
a
1
_
1
n
0
+ 1
+... +
1
n
_
.
Do , theo 2.2.50 (c), lim
n
an
n
= , v l v a
n
> 0.
2.5.72. Bng cch lm t-ng t nh- bi ton trn, ta chng minh tn ti mt
dy con {a
n
k
} tho mn
_
n
k
(a
1
+a
n
k
+p
)
(n
k
+p)a
n
k
_
n
k
1.
2.5.73. Gi s phn chng kt lun trong bi ton khng ng. Khi tn
ti mt s n
0
tho mn vi n n
0
, n
_
1+a
n+1
an
1
_
< 1. Bt ng thc cui
cng c th vit li d-i dng
1
n+1
<
an
n

a
n+1
n+1
. iu ny dn ti (xem li gii
ca 2.5.71)
1
n
0
+ 1
+... +
1
n
<
a
n
0
n
0

a
n
n
.
V lim
n
an
n
= , mu thun vi tnh cht {a
n
} l dy d-ng.
chng minh 1 l hng s c th tt nht, ly a
n
= nln n. Khi
lim
n
_
n
1 + (n + 1) ln(n + 1)
nlnn
n
_
= lim
n
1 + (n + 1) ln(n + 1) nlnn
lnn
= lim
n
1 + ln(n + 1) + ln
_
1 +
1
n
_
n
lnn
= 1.

n

C
h
i
226 Ch-ng 2. Dy s thc
2.5.74. Ch rng a
2
n
= 1 + a
n1
v a
1
= 1. R rng, dy tng thc s.
Ta s chng minh dy b chn trn bi
1
2
(1 +

5) bng ph-ng php quy np.


Tht vy, nu a
n1
<
1
2
(1 +

5), th a
2
n
= 1 + a
n1
<
3
2
+
1
2

5. V vy
a
n
<
_
3
2
+
1
2

5 =
1
2
+
1
2

5, v {a
n
} hi t ti
1
2
(1 +

5).
2.5.75. [20] Hin nhin dy {b
n
} tng thc s. Tr-c ht gi s rng < ln2.
Khi , theo gi thit, c n
0
N tho mn ln(lna
n
) < nln2 nu n n
0
;
hoc, t-ng t-ng, a
n
< e
2
n
nu n n
0
. Ta c
b
n
=
_
a
1
+... +
_
a
n
0
+... +

a
n

a
1
+... +
_
a
n
0
1
+
_
e
2
n
0
+... +

e
2
n

a
1
+... +
_
a
n
0
1
+e
2
n
0
_
1 +... +

1.
Theo bi tr-c,
b
n

_
a
1
+... +

a
n
0
1
+e
2
n
0
1 +

5
2
.
iu ny c ngi l {b
n
} b chn trn v hi t. Gi s > ln2. Theo gi thit,
vi > 0 cho tr-c, tn ti n
0
tho mn ln(lna
n
) > n(+) vi n n
0
. t
+ = ln, ta c a
n
> e

n
vi n n
0
, trong > 2. Do
b
n
=

a
1
+
_
a
2
+... +
_
a
n
0
+... +

a
n
>
_
a
1
+... +
_
a
n
0
1
+e

n
2
nn
0
+1
> e
(

2
)
n
.
Trong tr-ng hp ny, dy {b
n
} phn k ra v cng. Ch rng, nu 0 < a
n

1, th mc d lnlna
n
khng xc nh, dy {b
n
} n iu tng v b chn trn
bi
1+

5
2
, nn n hi t.
2.5.76. [20] Theo gi thit 0 a
n
na
1
nn dy {
an
n
} b chn. K hiu L
l gii hn trn ca n. Khi c mt dy {m
k
} ca cc s nguyn d-ng tho

n

C
h
i
2.5. Cc bi ton hn hp 227
mn lim
n
am
k
m
k
= L. Vi n N c nh tu , ta c th vit m
k
= nl
k
+ r
k
,
trong r
k
{0, 1, ..., n 1}. Do , theo gi thit, a
m
k
l
k
a
n
+a
r
k
. T
a
m
k
m
k

l
k
nl
k
+r
k
a
n
+
a
r
k
m
k
.
Cho k , ta -c
L
a
n
n
, ()
dn ti
lim
n
a
n
n
lim
n
a
n
n
.
Vy {
an
n
} hi t.
2.5.77. Cch lm t-ng t nh- bi ton trn.
2.5.78. [20] Dy {a
n
+1} v {1a
n
} tho mn gi thit ca Bi ton 2.5.76.
T lim
n
an+1
n
v lim
n
1an
n
tn ti v hu hn.
(a) Theo trn, lim
n
an+1
n
= g v lim
n
an
n
= g.
(b) Cc bt ng thc l h qu trc tip t () trong li gii bi 2.5.76.
2.5.79. Ta s chng minh dy {
an
n
} hi t ti A = sup{
an
n
: n N}. Gi s
p l mt s nguyn d-ng c nh tu . Khi
a
n
n
=
a
pln+rn
pl
n
+r
n

a
pln
pl
n
+r
n
,
trong r
n
{0, 1, ..., p 1}. Do vy, theo gi thit, lim
n
an
n

ap
p
. iu ny
dn n lim
n
an
n
lim
p
ap
p
. Vy dy {
an
n
} hi t. Hn na,
am.n
mn

an
n
nn
A lim
n
a
n
n
lim
p
a
p
p
= inf
p
sup
lp
a
l
l
inf
p
sup
mN
a
p.m
pm
inf
p
sup
m
a
m
m
= A.
2.5.80. Tr-c ht ta chng minh tnh b chn ca dy cho. Tht vy, nu
1
a
a
n
, a
n+1
a, th
1
a
a
n+2
=
2
an+a
n+1
a. Do , theo nguyn l -c
pht biu m u trong li gii ca Bi ton 2.1.10. dy {a
n
} b chn. t
l = lim
n
a
n
, L = lim
p
a
n
.

n

C
h
i
228 Ch-ng 2. Dy s thc
Vi mt s > 0 c nh tu tn ti n
1
, n
2
N tho mn
a
n
< L + n > n
1
, (i)
a
n
> l n > n
2
. (ii)
T (i), a
n+2
=
2
an+a
n+1
>
1
L+
, n > n
1
, v d-ng c th nh tu , ta thu
-c l
1
L
. Bng cch t-ng t ta c L
1
l
. Do l =
1
L
. Gi s {n
k
} l dy
cc s nguyn d-ng tho mn lim
k
a
n
k
+2
= L. Ta c th gi s rng cc dy
a
n
k
+1
, a
n
k
v a
n
k
1
hi t ti l
1
, l
2
v l
3
, t-ng ng. Thc ra, nu khc, ta c
th chn dy con tho mn. Theo nh ngha ca {a
n
},
l
1
+l
2
=
2
L
= 2l v l
2
+l
3
=
2
l
1
,
v t l l
1
, l
2
, l
3
L, ta -c l
1
= l
2
= l v l
2
= l
3
= L. T l = L. iu
ny v ng thc l =
1
L
dn n dy {a
n
} hi t ti 1.
2.5.81. V 0 < a
1
b
1
, tn ti [0,

2
) tho mn a
1
= b
1
cos . By gi
ta c th chng minh bng quy np rng, vi = 0,
a
n+1
=
b
1
sin
2
n
tan

2
n
v b
n+1
=
b
1
sin
2
n
sin

2
n
, n N.
V vy lim
n
a
n
= lim
n
b
n
=
b
1
sin

. Nu = 0, tc l a
1
= b
1
, th cc dy
cho {a
n
} v {b
n
} l hng s.
2.5.82. [18] Theo gi thit,
a
k,n
b
k,n
= 1 +
k,n
, trong
k,n
dn ti 0, u i
vi k. Do
n

k=1
a
k,n
=
n

k=1
b
k,n
+
n

k=1

k,n
b
k,n
. ()
T lim
n
n

k=1
b
k,n
tn ti, c mt s M > 0 tho mn

k=1
b
k,n

M, n N.
Hn na, vi bt k > 0, |
k,n
| <

M
vi k = 1, 2, ..., n, n ln. Do ,

k=1

k,n
b
k,n

. Ngha l lim
n
n

k=1

k,n
b
k,n
= 0. Vy, t (),
lim
n
n

k=1
a
k,n
= lim
n
n

k=1
b
k,n
.

n

C
h
i
2.5. Cc bi ton hn hp 229
2.5.83. Ta c
sin
(2k1)a
n
2
(2k1)a
n
2

n
1 u i vi k.
Do , theo bi tr-c,
lim
n
n

k=1
sin
(2k 1)a
n
2
= lim
n
n

k=1
(2k 1)a
n
2
= a.
2.5.84. T bi 2.5.5, nu dy {x
n
} hi t ti 0, th
a
xn
1
xn lna

n
1. iu ny
dn ti
a

k
n
2
1
k
n
2
lna

n
1.
u i vi k. By gi, dng Bi ton 2.5.82 ta -c
lim
n
n

k=1
(a
k
n
2
1) = lim
n
lna
n

k=1
k
n
2
=
1
2
lna.
2.5.85. Nu {x
n
}l mt dy d-ng hi t ti 0, th, theo Bi ton 2.5.3,
ln(1+xn)
xn

n
1. Theo 2.5.82, ta c
lim
n
n

k=1
ln
_
1 +
k
n
2
_
= lim
n
n

k=1
k
n
2
=
1
2
.
V vy lim
n
n

k=1
_
1 +
k
n
2
_
= e
1
2
.
2.5.86. Ta c th chng t rng nu {x
n
} l dy d-ng hi t ti 0 th
(1 +x
n
)
1
p
1
1
p
x
n

n
1. ()
t
c
k,n
=
k
q1
n
p
, k = 1, 2, ..., n.
Khi c
k,n
max{
1
n
,
1
n
q
}, v v vy {c
k,n
} hi t ti 0 u i vi k. Khi t
a
k,n
= (1 +c
k,n
)
1
p
1 v b
k,n
=
1
p
c
k,n
, ri s dng 2.5.82 ta nhn -c
lim
n
n

k=1
_
_
1 +
k
q1
n
q
_1
p
1
_
=
1
p
lim
n
n

k=1
k
q1
n
q
.

n

C
h
i
230 Ch-ng 2. Dy s thc
Theo nh l Stolz (xem 2.3.11)
lim
n
n

k=1
k
q1
n
q
= lim
n
n
q1
n
q
(n 1)
q
= lim
n
n
q1
n
q
n
q
_
1
1
n
_
q
= lim
n
n
q1
n
q
n
q
_
1 q
1
n
+
q(q1)
2
1
n
2

_ =
1
q
.
2.5.87. t a
n
=
a(a+d)(a+nd)
b(b+d)(b+nd)
. Khi
a
n
=
a
b
_
1 +
d
a
_

_
1 +n
d
a
_
_
1 +
d
a
+
_
b
a
1
__ _
1 + 2
d
a
+
_
b
a
1
__

_
1 +n
d
a
+
_
b
a
1
__
.
By gi t x =
b
a
1, khi vi x > 0 v
a
n
=
a
b
1
_
1 +
x
1+
d
a
__
1 +
x
1+2
d
a
_

_
1 +
x
1+n
d
a
_.
V
x
1 +
d
a
+ +
x
1 +n
d
a
<
_
1 +
x
1 +
d
a
_

_
1 +
x
1 +n
d
a
_
,
ta c
a
n
<
a
bx
_
1
1+
d
a
+
1
1+2
d
a
+ +
1
1+n
d
a
_.
Do lim
n
a
n
= 0 v
lim
n
_
1
1 +
d
a
+
1
1 + 2
d
a
+ +
1
1 +n
d
a
_
= +.

n

C
h
i
Ch-ng 3
Chui s thc
3.1 Tng ca chui
3.1.1.
(a) Ta c a
1
= S
1
= 2, a
n
= S
n
S
n1
=
1
n(n1)
, n > 1. Do , chui
cn tm l 2

n=2
1
n(n1)
v tng ca n l S = lim
n
S
n
= 1.
(b) a
n
=
1
2
n
,

n=1
1
2
n+1
= 1.
(c) V a
n
= arctan n arctan(n 1) nn tan a
n
=
1
n
2
n+1
. Do a
n
=
arctan
1
n
2
n+1
v

n=1
arctan
1
n
2
n+1
=

2
.
(d) a
1
= 1, a
n
= (1)
n 2n1
n(n1)
vi n > 1. Tng ca chui l
1 +

n=2
(1)
n
2n 1
n(n 1)
= 0.
3.1.2.
(a) Ta c a
n
=
1
n
2

1
(n+1)
2
. Do S
n
= 1
1
(n+1)
2
v S = lim
n
S
n
= 1.
(b) T-ng t, a
n
=
1
8
_
1
(2n1)
2

1
(2n+1)
2
_
. Suy ra S
n
=
1
8
_
1
1
(2n+1)
2
_
v
S = lim
n
S
n
=
1
8
.
231

n

C
h
i
232 Ch-ng 3. Chui s thc
(c) a
n
=

n+1

n1

n
. Do S
n
=

n+1
v S = lim
n
S
n
= 1.
(d) a
n
=
1
2
_
1
2n1

1
2n+1
_
. Do S = lim
n
S
n
=
1
2
.
(e) a
n
=

n+1

n(n+1)
=
1

n

1

n+1
, S = lim
n
S
n
= 1.
3.1.3.
(a) Ta c
S
n
= ln1 ln4 + ln2 + ln4 ln1 ln7 + ln3 + ln7 ln2
ln10 +... + lnn + ln(3n 2) ln(n 1) ln(3n + 1)
+ ln(n + 1) + ln(3n + 1) lnn ln(3n + 4) = ln
n + 1
3n + 4
.
Do S = ln
1
3
.
(b) S = ln2.
3.1.4.
(a) Ta c
a
n
=
1
n(n + 1)...(n +m)
=
1
m
_
1
n(n + 1)...(n +m1)

1
(n + 1)(n + 2)...(n +m)
_
.
Do S
n
=
1
m
_
1
1.2....m

1
(n+1)(n+2)...(n+m)
_
v S =
1
mm!
.
(b) a
n
=
1
m
_
1
n

1
n+m
_
, S =
1
m
_
1 +
1
2
+... +
1
m
_
.
(c) Ta c
n
2
(n + 1)(n + 2)(n + 3)(n + 4)
=
2
(n + 1)(n + 2)

1
(n + 3)(n + 4)

11
2
_
1
(n + 1)(n + 3)

1
(n + 2)(n + 4)
_
+
11
4
_
1
(n + 1)(n + 4)

1
(n + 2)(n + 3)
_
.
p dng cu (b) thu -c S =
5
36
.

n

C
h
i
3.1. Tng ca chui 233
3.1.5.
(a) Vi n 5, ta c
S
n
= sin

720
+ sin

360
+ sin

120
+ sin

60
+ sin

30
+ sin

6
.
(b) Ch rng 0
lnn
nln n
< 1, n N. Do S = 0.
3.1.6. T a
n
= sin
1
2
n+1
cos
3
2
n+1
=
1
2
(sin
1
2
n1
sin
1
2
n
), suy ra S =
1
2
sin 1.
3.1.7. Ch rng
1
n!(n
4
+n
2
+ 1)
=
1
2
_
n
(n + 1)!((n + 1)n + 1)

n 1
n!(n(n 1) + 1)
+
1
(n + 1)!
_
.
Do
S
n
=
1
2
_
n
(n + 1)!((n + 1)n + 1)
+ 1 +
n

k=0
1
(k + 1)!
_
.
p dng bi 2.5.6 thu -c S = lim
n
S
n
=
1
2
e.
3.1.8. Vi n > 1 ta c a
1
=
1
3
v
a
n
=
1
2
(2n + 1) 1
3.5...(2n + 1)
=
1
2
_
1
3.5...(2n 1)

1
3.5...(2n + 1)
_
.
Do
S
n
=
1
3
+
1
2
_
1
3

1
3.5...(2n + 1)
_
v S = lim
n
S
n
=
1
2
.
3.1.9. Lm t-ng t nh- bi trn, ta c vi n > 1 th
a
n
(a
1
+ 1)(a
2
+ 1)...(a
n
+ 1)
=
a
n
+ 1 1
(a
1
+ 1)(a
2
+ 1)...(a
n
+ 1)
=
1
(a
1
+ 1)(a
2
+ 1)...(a
n1
+ 1)

1
(a
1
+ 1)(a
2
+ 1)...(a
n
+ 1)
Do S
n
= 1
1
(a
1
+1)(a
2
+1)...(an+1)
.

n

C
h
i
234 Ch-ng 3. Chui s thc
3.1.10.
(a) Trong bi trn nu chn a
n
= n 1 th suy ra g = +. Do tng cn
tnh bng 1.
(b) T-ng t, a
n
= 2n 1, g = +, tng cn tnh bng 1.
(c) t a
n
=
1
n
2
. Khi
lim
n
((a
2
+ 1)(a
3
+ 1)...(a
n
+ 1))
= lim
n
(2 1)(2 + 1)
2
2
(3 1)(3 + 1)
3
2
...
(n 1)(n + 1)
n
2
=
1
2
.
T bi 3.1.9 suy ra tng cn tnh bng 1.
3.1.11. T nh ngha suy ra {a
n
} l dy n iu tng ti v cng. Hn na,
ta c th chng minh rng a
2
n
4 = a
2
1
a
2
2
...a
2
n1
(a
2
1
4) bng quy np. Do
lim
n
a
n
a
1
a
2
...a
n1
=
_
a
2
1
4. (1)
Mt khc, vi n > 1 th
1
a
1
a
2
...a
n
=
1
2
_
a
n
a
1
a
2
...a
n1

a
n+1
a
1
a
2
...a
n
_
.
T v t (1) suy ra tng ca chui l
1
a
1
+
1
2
a
2
a
1

1
2
_
a
2
1
4 =
a
1

_
a
2
1
4
2
.
3.1.12. Ta c
1
b
=
1!
b 2

2!
(b 2)b
,
2!
b(b + 1)
=
2!
(b 2)b

3!
(b 2)b(b + 1)
,
.
.
.
n!
b(b + 1)...(b +n 1)
=
n!
(b 2)b(b + 1)...(b +n 2)

(n + 1)!
(b 2)b(b + 1)...(b +n 1)
.

n

C
h
i
3.1. Tng ca chui 235
Ly tng theo v cc ng thc trn thu -c
S
n
=
1
b 2

(n + 1)!
(b 2)b(b + 1)...(b +n 1)
.
T bi 2.5.87 suy ra S = lim
n
S
n
=
1
b2
.
3.1.13. Vi n = 0, 1, ..., t a
n
=
a(a+1)...(a+n)
b(b+1)...(b+n)
, A
n
= a
n
(a + n + 1). Khi
A
n1
A
n
= a
n
(b a 1), n = 0, 1, ..., trong A
1
= a v a
1
= 1.
Ly tng theo v cc ng thc trn t n = 0 n n = N thu -c
a A
N
= A
1
A
N
= (b a 1)
N

n=0
a
n
= (b a 1)S
N+1
,
Do
(b a 1)S
N+1
= a
_
1
(a + 1)...(a +N + 1)
b(b + 1)...(b +N)
_
.
S dng kt qu bi 2.5.87, suy ra lim
N
S
N+1
=
a
ba1
.
3.1.14. T bi trn ta c
1 +

n=1
a(a + 1)...(a +n 1)
b(b + 1)...(b +n 1)
= 1 +
a
b a 1
=
b 1
b a 1
. ()
Trong (), thay a bi a + 1 thu -c
1 +

n=1
(a + 1)...(a +n)
b(b + 1)...(b +n 1)
=
b 1
b a 2
. ()
Ly () tr i () -c

n=1
n
(a + 1)...(a +n 1)
b(b + 1)...(b +n 1)
=
b 1
b a 2

b 1
b a 1
=
b 1
(b a 1)(b a 2)
.
3.1.15. t A
n
=
a
1
a
2
...an
(a
2
+b)(a
3
+b)...(a
n+1
+b)
v S
n
=
n

k=1
A
k
. Khi
A
k
A
k1
=
a
k
a
k+1
+b
, hay t-ng -ng vi A
k
a
k+1
+ A
k
b = A
k1
a
k
. Ly tng theo v cc
ng thc trn t k = 2 n k = n thu -c
A
n
a
n+1
+S
n
b A
1
b = A
1
a
2
. ()

n

C
h
i
236 Ch-ng 3. Chui s thc
Mt khc
0 < A
n
a
n+1
= a
1
a
2
a
3
...a
n+1
(a
2
+b)(a
3
+b)...(a
n+1
+b)
= a
1
1
_
1 +
b
a
2
__
1 +
b
a
3
_
...
_
1 +
b
a
n+1
_.
T bi 1.2.1, suy ra
0 < A
n
a
n+1
<
a
1
b
n+1

k=2
1
a
k
.
Chng t lim
n
A
n
a
n+1
= 0 v t () ta c
lim
n
S
n
=
A
1
(b +a
2
)
b
=
a
1
b
.
3.1.16. T cng thc l-ng gic 4 cos
3
x = cos 3x + 3 cos x, ta c
4 cos
3
x = cos 3x + 3 cos x,
4 cos
3
3x = cos 3
2
x + 3 cos 3x,
4 cos
3
3
2
x = cos 3
3
x + 3 cos 3
2
x,
.
.
.
4 cos
3
3
n
x = cos 3
n+1
x + 3 cos 3
n
x.
Nhn ln l-t cc ng thc trn vi 1,
1
3
,
1
3
2
, ..., (1)
n 1
3
n
sau cng li s
-c
4S
n
= 3 cos x + (1)
n
1
3
n
cos 3
n+1
x.
Do S = lim
n
S
n
=
3
4
cos x.
3.1.17.
(a) T gi thit, ta c
f(x) = af(bx) +cg(x),
af(bx) = a
2
f(b
2
x) +acg(bx),
a
2
f(b
2
x) = a
3
f(b
3
x) +ca
2
g(b
2
x),
.
.
.
a
n1
f(b
n1
x) = a
n
f(b
n
x) +a
n1
cg(b
n1
x).

n

C
h
i
3.1. Tng ca chui 237
Do f(x) = a
n
f(b
n
x) +c (g(x) +ag(bx) +... +a
n1
g(b
n1
x)) . T
gi thit lim
n
a
n
f(b
n
x) = L(x) suy ra

n=0
a
n
g(b
n
x) =
f(x)L(x)
c
.
(b) T-ng t cu (a), ta c
f(x) = af(bx) +cg(x),
a
1
f(b
1
x) = f(x) +a
1
cg(b
1
x),
a
2
f(b
2
x) = a
1
f(b
1
x) +ca
2
g(b
2
x),
.
.
.
a
n
f(b
n
x) = a
1n
f(b
1n
x) +a
n
cg(b
n
x).
T suy ra
af(bx) = a
n
f(b
n
x) c
_
g(x) +a
1
g(b
1
x) +... +a
n
g(b
n
x)
_
.
Do

n=0
1
a
n
g(
x
b
n
) =
M(x) af(bx)
c
.
3.1.18. p dng bi trn vi f(x) = sinx, g(x) = sin
3 x
3
, a = 3, b =
1
3
, c = 4. D thy L(x) = lim
n
3
n
sin
x
3
n
= x, M(x) = lim
n
3
n
sin3
n
x =
0.
3.1.19. p dng bi 3.1.17 vi f(x) = cot x, g(x) = tan x, a = 2, b =
2, c = 1 v
M(x) = lim
n
1
2
n
cot
x
2
n
=
1
x
.
3.1.20. p dng bi 3.1.17 vi
f(x) = arctan x, g(x) = arctan
(1 b)x
1 +bx
2
, a = c = 1,
v s dng cng thc
lim
n
arctan(b
n
x) =
_
0 vi 0 < b < 1,

2
sgn x vi b > 1.
3.1.21. T a
n+1
= a
n
+ a
n1
ta c a
n+1
a
n
= a
2
n
+ a
n1
a
n
vi n 1. Ly
tng theo v cc ng thc thu -c
S
n
= a
n
a
n+1
, n 0. ()

n

C
h
i
238 Ch-ng 3. Chui s thc
C th chng minh bng quy np cc ng thc sau:
a
n
=
1

5
_
_
_
1 +

5
2
_
n+1

_
1

5
2
_
n+1
_
_
, n 0, (i)
a
n1
a
n+1
a
2
n
= (1)
n+1
, n 1. (ii)
Kt hp () vi (ii) -c

S
n
=
n

k=0
(1)
k
S
k
=
n

k=0
(1)
k
a
k
a
k+1
= 1
n

k=1
a
k1
a
k+1
a
2
k
a
k
a
k+1
= 1
n

k=1
_
a
k1
a
k

a
k
a
k+1
_
=
a
n
a
n+1
.
T (i) ta c
lim
n
a
n
a
n+1
= lim
n
1

5
_
_
1+

5
2
_
n+1

_
1

5
2
_
n+1
_
1

5
_
_
1+

5
2
_
n+2

_
1

5
2
_
n+2
_
=
2
1 +

5
.
(iii)
Do

n=0
(1)
n
Sn
=
2
1+

5
.
3.1.22. D dng kim tra rng
(1)
n+1
= a
n+1
a
n+2
a
n
a
n+3
, n 0. ()
Do
S
n
=
n

k=0
(1)
k
a
k
a
k+2
=
n

k=0
a
k+1
a
k+2
a
k
a
k+3
a
k
a
k+2
=
n

k=0
_
a
k+1
a
k

a
k+3
a
k+2
_
= 3 +
a
n+2
a
n+1
+
a
n+3
a
n+2
.
T (iii) trong bi trn suy ra lim
n
S
n
=

5 2.

n

C
h
i
3.1. Tng ca chui 239
3.1.23. S dng ng thc () trong bi trn, ta c
arctan
1
a
2n+1
arctan
1
a
2n+2
= arctan
a
2n+2
a
2n+1
a
2n+1
a
2n+2
+ 1
= arctan
a
2n
a
2n
a
2n+3
= arctan
1
a
2n+3
.
Ly tng theo v cc ng thc trn thu -c
arctan
1
a
1
=
n+1

k=1
arctan
1
a
2k
+ arctan
1
a
2n+3
.
Do

n=1
arctan
1
a
2n
=

4
.
3.1.24.
(a) Vi nhn xt rng arctan
2
n
2
= arctan
1
n1
arctan
1
n+1
, n > 1, ta c
kt lun

n=1
arctan
2
n
2
= arctan 2 + arctan 1 + arctan
1
2
=
3
4
.
(b) Vi n N, arctan
1
n
2
+n+1
= arctan
1
n
arctan
1
n+1
. T suy ra rng

n=1
arctan
1
n
2
+n+1
= arctan 1 =
1
4
.
(c) Vi n > 1, arctan
8n
n
4
2n
2
+5
= arctan
2
(n1)
2
arctan
2
(n+1)
2
, do

n=1
arctan
8n
n
4
2n
2
+ 5
= arctan 2 + arctan 2 + arctan
1
2
=
1
2
+ arctan 2.
3.1.25. S dng cng thc l-ng gic
arctan x arctan y = arctan
x y
1 +xy
d dng chng minh bi ton. Ch rng cc kt qu trong bi 3.1.24 ch l
tr-ng hp ring ca bi ton ny.

n

C
h
i
240 Ch-ng 3. Chui s thc
3.1.26. Gi s chui

n=1
b
n
l mt hon v ca chui

n=1
a
n
. t S
n
=
a
1
+a
2
+... +a
n
, S

n
= b
1
+b
2
+... +b
n
v S = lim
n
S
n
. R rng S

n
S.
Do dy {S

n
} hi t ti gii hn S

S. T-ng t ta cng c S S

. Chng
t chui

n=1
b
n
hi t v c cng tng vi chui

n=1
a
n
.
3.1.27. V
S
2n
=
2n

k=1
1
k
2
=
n

k=1
1
(2k)
2
+
n

k=1
1
(2k 1)
2
v lim
n
S
2n+1
= lim
n
S
2n
nn suy ra

n=1
1
n
2
=

n=1
1
(2n 1)
2
+
1
2
2

n=1
1
n
2
.
3.1.28. [A. M. Yaglom v I. M. Yaglom, Uspehi Mathem. Nauk (N>S.) 8(1953)
s 5(57), trang 181-187 (ting Nga)]
(a) Nu 0 < x <

2
th sin x < x < tan x nn khi cot
2
x <
1
x
2
<
1 +cot
2
x. Chn x =
k
2m+1
vi k = 1, 2, ..., m, sau ly tng t k = 1
n k = m thu -c
m

k=1
cot
2
k
2m+ 1
<
(2m+ 1)
2

2
m

k=1
1
k
2
< m+
m

k=1
cot
2
k
2m+ 1
. (i)
By gi ta s chng minh rng
m

k=1
cot
2
k
2m+ 1
=
m(2m1)
3
. (ii)
Vi 0 < t <

2
, theo cng thc De Moivre ta c
cos nt +i sinnt = (cos t +i sint)
n
= sin
n
t(cot t +i)
n
= sin
n
t
n

k=0
_
n
k
_
i
k
cot
nk
t.
Chn n = 2m+ 1, ng nht phn o ca ng thc trn thu -c
sin(2m+ 1)t = sin
2m+1
tP
m
(cot
2
t), (iii)

n

C
h
i
3.1. Tng ca chui 241
trong
P
m
(x) =
_
2m+ 1
1
_
x
m

_
2m+ 1
3
_
x
m1
+... 1. (iv)
Trong (iii) thay t =
k
2m+1
suy ra P
m
(cot
2 k
2m+1
) = 0. Do x
k
=
cot
2 k
2m+1
, k = 1, 2, ..., m l cc nghim ca a thc P
m
v tng ca
chng l
m

k=1
cot
2
k
2m+ 1
=
_
2m+1
3
_
_
2m+1
1
_ =
m(2m1)
3
. (v)
T (i) v (ii) suy ra
m(2m1)
3
<
(2m + 1)
2

2
m

k=1
1
k
2
< m+
m(2m1)
3
.
Nhn bt ng thc trn vi

2
(2m+1)
2
v cho m thu -c (a).
(b) Ch rng
m

i,j=1
i=j
x
i
x
j
= 2
_
2m+1
5
_
_
2m+1
1
_,
trong x
k
, k = 1, 2, ..., m l cc nghim ca a thc (iv). T v t
(v) suy ra
m

k=1
cot
4
k
2m+ 1
=
_
m(2m1)
3
_
2
2
2m(2m1)(2m2)(2m3)
5!
=
m(2m1)(4m
2
+ 10m9)
45
.
Mt khc, t bt ng thc cot
2
x <
1
x
2
< 1 + cot
2
x (xem (a)) ta c
cot
4
x <
1
x
4
< 1 + 2 cot
2
x + cot
4
x vi 0 < x <

2
. Do
m(2m1)(4m
2
+ 10m9)
45
<
(2m+ 1)
4

4
m

k=1
1
k
4
< m+ 2m
2m1
3
+
m(2m1)(4m
2
+ 10m9)
45
.

n

C
h
i
242 Ch-ng 3. Chui s thc
Vy ta chng minh -c (b).
Ch . Ta c th p dng ph-ng php trn tnh tng ca chui

n=1
1
n
2k
, k N.
(c) T cng thc De Moivre c th chng minh rng vi m = 4n, n N th
cos mt = cos
m
t
_
m
2
_
cos
m2
t sin
2
t +... + sin
m
t,
sinmt =
_
m
1
_
cos
m1
t sint +...
_
m
m1
_
cos t sin
m1
t,
v do
cot mt =
cot
m
t
_
m
2
_
cot
m2
t +...
_
m
m2
_
cot
2
t + 1
_
m
1
_
cot
m1
t
_
m
3
_
cot
m3
t +...
_
m
m1
_
cot t
.
T ng thc cui suy ra
x
k
= cot
4k +
4m
, k = 0, 1, ..., m1,
l cc nghim ca ph-ng trnh
x
m

_
m
1
_
x
m1

_
m
2
_
x
m2
+... +
_
m
m1
_
x + 1 = 0.
Chng t
m1

k=0
cot
4k +
4m
= m. (1)
V m = 4n nn
m1

k=0
cot
4k +
4m
=
2n1

k=0
cot
4k +
4m
+
m1

k=2n
cot
4k +
4m
=
2n1

k=0
cot
4k +
4m

2n

k=1
cot
4k
4m
.
T v t (1) suy ra
cot

4m
cot
3
4m
+ cot
5
4m
cot
7
4m
+... + cot
(2m3)
4m
cot
(2m1)
4m
= m.
(2)

n

C
h
i
3.1. Tng ca chui 243
p dng cng thc l-ng gic
cot cot = tan( )
_
1 +
1
tan tan
_
vo (2) thu -c
m = tan

2m
_
m
2
+
1
tan

4m
tan
3
4m
+... +
1
tan
(2m3)
4m
tan
(2m1)
4m
_
.
Do t bt ng thc
1
x
>
1
tanx
vi 0 < x <

2
suy ra
m < tan

2m
_
m
2
+
1

4m
3
4m
+... +
1
(2m3)
4m
(2m1)
4m
_
. (3)
T-ng t, p dng cng thc l-ng gic
cot cot =
sin( )
sinsin
v bt ng thc
1
x
<
1
sin x
vo (2) thu -c
m = sin

2m
_
1
sin

4m
sin
3
4m
+... +
1
sin
(2m3)
4m
sin
(2m1)
4m
_
> sin

2m
_
1

4m
3
4m
+... +
1
(2m3)
4m
(2m1)
4m
_
.
Kt hp vi (3) ta -c
_
m
tan

2m

m
2
_

2
16m
2
<
1
2.3
+... +
1
(2m3)(2m1)
<

2
16msin

2m
.
Cho m suy ra
1
2.3
+
1
5.7
+... =
1
2

m=0
(1)
m
1
2m+ 1
=

8
.

n

C
h
i
244 Ch-ng 3. Chui s thc
3.1.29. Ta c a
n+1
1 = a
n
(a
n
1). Do
1
a
n+1
1
=
1
an
+
1
an1
. Ly tng
theo v cc ng thc t n = 1 n n = N thu -c
1
a
1
+
1
a
2
+... +
1
a
N
= 1
1
a
N+1
1
. ()
D kim tra rng dy {a
n
} n iu tng n v cng. Do t () suy ra

n=1
1
an
= 1.
3.1.30. T nh ngha dy {a
n
}, ta c
e
a
1
1 = a
1
e
a
2
,
e
a
2
1 = a
2
e
a
3
,
.
.
.
Do
e
a
1
1 = a
1
+a
1
a
2
e
a
3
= ...
= a
1
+a
1
a
2
+... +a
1
a
2
...a
n
+a
1
a
2
...a
n+1
e
a
n+2
.
Mt khc {a
n
} l dy n iu gim hi t v 0 nn
lim
n
(a
1
a
2
...a
n+1
e
a
n+2
) = 0.
T suy ra

n=1
b
n
= e
a
1
1.
3.1.31. Ta c S
n+1
= S
n
+ a
n+1
= S
n
+
1
Sn

2. Xt hm f(x) =
x +
1
x

2, x > 0. Nu dy {S
n
} hi t ti S th f(S) = S. Nghim duy
nht ca ph-ng trnh l
1

2
. V hm x f(f(x)) x n iu gim trn
khong (
1

2
, 1) nn nu x (
1

2
, 1) th
f(f(x)) x < f
_
f
_
1

2
__

2
= 0.
Mt khc hm f n iu gim trn khong (0, 1) nn f(f(x)) >
1

2
vi
x
_
1

2
, 1
_
. Do
1

2
< f(f(x)) < x vi x
_
1

2
, 1
_
.

n

C
h
i
3.1. Tng ca chui 245
Chng t {S
2n1
} l dy n iu gim v b chn d-i nn n hi t v c gii
hn l
1

2
. Hn na lim
n
S
2n
= lim
n
f(S
2n1
) = f
_
1

2
_
=
1

2
. Vy chui
cho hi t v c tng l
1

2
.
3.1.32.
(a) Ch rng
S
2n
= 1
1
2
+
1
3
...
1
2n
= 1 +
1
2
+
1
3
+... +
1
2n

_
1 +
1
2
+
1
3
+... +
1
n
_
=
1
n + 1
+
1
n + 2
+... +
1
2n
.
Do t bi 2.5.8 (a), ta c lim
n
S
2n
= ln2. R rng
lim
n
S
2n+1
= lim
n
_
S
2n
+
1
2n + 1
_
= ln2.
(b) Ta c
2n+1
n(n+1)
=
1
n
+
1
n+1
. Do t (a) suy ra

n=1
(1)
n1
2n + 1
n(n + 1)
=

n=1
(1)
n1
1
n
+

n=1
(1)
n1
1
n + 1
= ln2 (ln2 1) = 1.
(c) Gi S
n
l tng ring th n ca chui -c cho. Khi
S
2n
=
1
x + 2n + 1
+
1
x + 2n + 2
+... +
1
x + 4n 1
+
1
x + 4n
.
Lm t-ng t nh- bi 2.5.8 c th chng minh rng lim
n
S
2n
= ln2. R
rng lim
n
S
2n+1
= ln2.
3.1.33. Ta c
S
2n
= ln
2
1
ln
3
2
+ ln
4
3
ln
5
4
+... + ln
2n
2n 1
ln
2n + 1
2n
= ln
2.4...2n
1.3...(2n 1)
ln
3.5...(2n + 1)
2.4...2n
= ln
_
1
2n + 1
_
(2n)!!
(2n 1)!!
_
2
_
.

n

C
h
i
246 Ch-ng 3. Chui s thc
T cng thc Wallis,

2
= lim
n
1
2n+1
_
(2n)!!
(2n1)!!
_
2
suy ra lim
n
S
2n
= ln

2
.
3.1.34. Ta c

n=1
(1)
n1
ln
_
1
1
(n + 1)
2
_
=

n=1
(1)
n1
_
ln
_
1 +
1
n + 1
_
+ ln
_
1
1
n + 1
__
=

n=2
(1)
n1
ln
_
1 +
1
n
_

n=1
(1)
n1
ln
_
1 +
1
n
_
.
T bi trn suy ra tng ca chui l ln2 2 ln

2
.
3.1.35. Tng ring th n ca chui c th vit d-i dng
S
n
= 1 +
1
2
+... +
1
n
(ln2 ln1 + ln3 ln2
+... + ln(n + 1) lnn) = 1 +
1
2
+... +
1
n
ln(n + 1).
Do t bi 2.1.41 suy ra tng ca chui chnh l hng s Euler .
3.1.36. [20] t F(x) =
_
x
1
f(t)dt. T nh l Taylor suy ra tn ti x
k
, y
k
sao cho k < x
k
< k +
1
2
, k +
1
2
< y
k
< k + 1 tho mn
F
_
k +
1
2
_
F(k) =
1
2
f(k) +
1
8
f

(x
k
),
F
_
k +
1
2
_
+F(k + 1) =
1
2
f(k + 1)
1
8
f

(y
k
).
Ly tng theo v cc ng thc t k = 1 n k = n 1 thu -c
1
2
f(1) +f(2) +f(3) +... +f(n 1) +
1
2
f(n) F(n)
=
1
8
(f

(y
1
) f

(x
1
) +f

(y
2
) f

(x
2
) +... +f

(y
n1
) f

(x
n1
)) .
Gii hn v phi ng thc trn tn ti v f

(y
1
)f

(x
1
)+f

(y
2
)f

(x
2
)+...
l chui an du v cc s hng c tr tuyt i n iu gim v 0.
Nu ly hm f(x) =
1
x
th c th chng minh -c s tn ti ca gii hn
lim
n
_
1 +
1
2
+
1
3
+... +
1
n
lnn
_
.

n

C
h
i
3.1. Tng ca chui 247
(So snh vi 2.1.41 (a)). Ly f(x) = lnx, ta chng minh -c dy
_
lnn!
_
n +
1
2
_
lnn +n
_
hi t. (Theo cng thc Stirling gii hn ca dy l ln

2.)
3.1.37. p dng bi trn vi hm f(x) =
lnx
x
, x > 0, suy ra s tn ti ca
gii hn
lim
n
_
ln1
1
+
ln2
2
+... +
lnn
n

(lnn)
2
2
_
= s.
Do
lim
n
S
2n
= lim
n
_

ln1
1
+
ln2
2
... +
ln2n
2n
_
= lim
n
_

_
ln1
1
+
ln2
2
+... +
ln2n
2n

(ln2n)
2
2
_
+ 2
_
ln2
2
+
ln4
4
+... +
ln2n
2n
_

(ln2n)
2
2
_
= s + lim
n
_
ln1
1
+
ln2
2
+... +
lnn
n

(lnn)
2
2
_
+ lim
n
_
ln2
1
+
ln2
2
+... +
ln2
n
ln2 lnn
_

(ln2)
2
2
= ln2
_

ln2
2
_
,
trong l hng s Euler.
3.1.38. T cng thc Stirling n! =
n

2n(
n
e
)
n
, trong lim
n

n
= 1, ta
c
S
n
=
1
2
ln
(2n + 1)
2n
((2n 1)!!)
2
e
2n
=
1
2
ln
(2n + 1)
2n
2
2n
(n!)
2
((2n)!)
2
e
2n
=
1
2
ln
(2n + 1)
2n
2
2n

2
n
2n(
n
e
)
2n

2
2n
4n(
2n
e
)
4n
e
2n
=
1
2
ln
_
_
2n + 1
2n
_
2n

2
n
2
2
2n
_
.
Do lim
n
S
n
=
1
2
(1 ln2).

n

C
h
i
248 Ch-ng 3. Chui s thc
3.1.39. Gi s c hai gi tr x v y chui cho hi t. t
S
N
(x) =
N

n=1
_
1
(n 1)k + 1
+
1
(n 1)k + 2
+... +
1
nk 1

x
nk
_
.
Khi S
N
(x) S
N
(y) =
yx
k
N

n=1
1
n
. T s hi t ca hai chui suy ra
x = y. By gi, ta s tm gi tr ca x sao cho chui cho hi t. Theo 2.1.41
th dy a
n
= 1 +
1
2
+... +
1
nk
ln(nk) hi t ti hng s Euler . Do
S
N
(k 1) = a
N
+ ln(Nk)
N

n=1
1
nk

N

n=1
k 1
nk
= a
N
+ lnk +
_
lnN
N

n=1
1
n
_
.
T suy ra rng lim
N
S
N
(k 1) = +lnk = lnk. Chng t x = k 1
v tng ca chui bng lnk.
3.1.40. D dng chng minh rng
a
2n
= 3n + 2 vi n = 0, 1, 2, ...,
a
2n1
= 3n + 1 vi n = 1, 2, ...
bng quy np. Do
S
2N
=
2N

n=0
(1)
[
n+1
2
]
1
a
2
n
1
=
N

n=0
(1)
n
a
2
2n
1
+
N

n=1
(1)
n
a
2
2n1
1
=
N

n=0
(1)
n
1
(3n + 1)(3n + 3)
+
N

n=1
(1)
n
1
3n(3n + 2)
()
=
1
3
+
1
2
N

n=1
(1)
n
_
1
3n + 1

1
3n + 3
_
+
1
2
N

n=1
(1)
n
_
1
3n

1
3n + 2
_
=
1
3
+
1
2
N

n=1
(1)
n
_
1
3n

1
3n + 3
_
+
1
2
N

n=1
(1)
n
_
1
3n + 1

1
3n + 2
_
=
1
3

1
6
+
N

n=2
(1)
n
3n
+
(1)
N+1
6(N + 1)
+
1
2
N

n=1
(1)
n
_
1
3n + 1

1
3n + 2
_
.

n

C
h
i
3.1. Tng ca chui 249
Mt khc, theo 3.1.32 (a) ta c
ln2 = lim
n
_
3N

n=1
(1)
n
1
n
_
= lim
n
_
N

n=1
(1)
n
1
3n
_
lim
n
_
N1

n=0
(1)
n
_
1
3n + 1

1
3n + 2
_
_
=
1
3
ln2 lim
n
_
N1

n=0
(1)
n
_
1
3n + 1

1
3n + 2
_
_
.
Suy ra lim
N
N1

n=0
(1)
n
_
1
3n+1

1
3n+2
_
=
2
3
ln2. Kt hp vi () thu -c
lim
N
S
2N
=
1
6

1
3
ln2+
1
3
+
1
3
ln2
1
4
=
1
4
. Hn na lim
N
S
2N+1
= lim
N
S
2N
+
lim
N
(1)
N+1
(3N+4)
2
1
= lim
N
S
2N
, chng t tng ca chui cho bng
1
4
.
3.1.41.
(a) Gi s phn chng rng tng S ca chui cho l mt s hu t, tc l
S =
p
q
. Khi (q 1)!p = q!S =
q

n=1
q!
n!
+

n=q+1
q!
n!
. iu chng t

n=q+1
q!
n!
l mt s nguyn. Mt khc
0 <

n=q+1
q!
n!

1
q + 1
+
1
(q + 1)(q + 2)
+
1
(q + 1)(q + 2)
2
+...
=
q + 2
(q + 1)
2

3
4
,
suy ra mu thun. Vy S l mt s v t.
(b) Lm t-ng t nh- cu (a).
3.1.42. Gi s phn chng rng tng S ca chui cho l mt s hu t, tc
l S =
p
q
. Khi
(q 1)!p = q!S =
q

n=1
q!
n
n!
+

n=q+1
q!
n
n!
.

n

C
h
i
250 Ch-ng 3. Chui s thc
iu chng t

n=q+1
q!n
n!
l mt s nguyn. Mt khc

n=q+1
q!
n
n!

n=q+1
q!
n!
< 1.
Do suy ra mu thun, ta ch cn chng minh rng

n=q+1
q!n
n!
khc 0. Tht
vy

n=q+1
q!
n
n!

1
q + 1

n=q+2
q!
n!

>
1
q + 1

1
q(q + 1)
0.
Chng t S l mt s v t.
3.1.43. Lm t-ng t nh- bi trn.
3.1.44. Gi s phn chng rng

i=1
1
n
i
=
p
q
, p, q N. Theo gi thit c tn
ti mt s nguyn d-ng k sao cho nu i k th
n
i
n
1
n
2
...n
i1
> 3q. Do
k1

i=1
n
1
n
2
...n
k1
q
n
i
+

i=k
n
1
n
2
...n
k1
q
n
i
= pn
1
n
2
...n
k1
.
Mt khc

i=k
n
1
n
2
...n
k1
q
n
i
<
1
3
_
1 +
1
n
k
+
1
n
k
n
k+1
+...
_
< 1,
suy ra mu thun.
3.1.45. Nu tng ca chui l s hu t
p
q
th vi mi s nguyn d-ng k
1
ta
c

k=k
1
1
n
k
=
p
q

k
1
1

k=1
1
n
k
. Do tng

k=k
1
qn
1
n
2
...n
k
1
1
n
k
l mt s nguyn d-ng.
Suy ra

k=k
1
1
n
k

1
qn
1
n
2
...n
k
1
1
. ()
t lim
k
n
k
n
k1
= l > 1 v chn > 0 nh sao cho = l > 1. Khi s
tn ti mt ch s k
0
sao cho nu k > k
0
th
n
k
n
k1
> 1. ()

n

C
h
i
3.1. Tng ca chui 251
V lim
k
n
k
n
1
...n
k1
= +nn tn ti mt s k
1
> k
0
sao cho
n
k
1
n
1
...n
k1
>
q
1
. T
() suy ra

k=k
1
1
n
k

j=0
1

j
n
k
1
=

( 1)n
k
1
<
1
qn
1
n
2
...n
k
1
1
.
iu ny mu thun vi ().
3.1.46. Gi s rng

k=1
1
n
k
=
p
q
, trong p v q l cc s nguyn d-ng. Khi

n
1
n
2
...n
k1

j=0
1
n
k+1

1
q
vi mi k 2. (Xem bi trn.) t a
k
=
2
k

n
k
. Theo gi thit lim
k
a
k
=
+. Ta s ch ra rng tn ti s nguyn d-ng r
1
sao cho a
j
a
r
1
vi j =
1, 2, ..., r
1
1. Thc vy, nu a
1
a
2
th r
1
= 2. Nu a
1
> a
2
th chn r
1
l s nguyn nh nht ln hn 2 sao cho a
1
a
r
1
. Lm t-ng t ta c th tm
-c dy s nguyn r
k
c tnh cht a
j
a
r
k
vi j = 1, 2, ..., r
k
1. Gi r l
s nguyn d-ng nh nht sao cho a
r+j
> q + 1 vi j = 0, 1, 2, ... v a
j
a
r
vi j = 1, 2, ..., r 1. Ch rng n
r
n
r+j
nn a
r
a
2
j
r+j
vi j = 0, 1, 2, ....
Do
n
1
n
2
...n
r1
n
r+j

a
2+2
2
+...+2
r1
r
n
r+j

a
2
j
(2
r
2)
r+j
a
2
r+j
r+j
= a
2
j+1
r+j
< (q + 1)
(j+1)
.
Chng t
n
1
n
2
...n
r1

j=0
1
n
r+j
<

j=0
(q + 1)
(j+1)
=
1
q
,
suy ra mu thun.
3.1.47. V chui

n=1
pn
qn
hi t nn lim
n
pn
qn1
= 0. Do t bt ng thc
cho suy ra
pm
qm1

n=m
pn
qn
. Gi s tp A hu hn. Khi s tn ti mt ch s
m sao cho
S =

n=1
p
n
q
n
=
m1

n=1
p
n
q
n
+
p
m
q
m
1
.

n

C
h
i
252 Ch-ng 3. Chui s thc
Chng t S l s hu t. Gi s rng
S =

n=1
p
n
q
n
=
p
q
Q.
Khi
r
n
=
p
q

n

k=1
p
k
q
k
=

k=n+1
p
k
q
k

p
n+1
q
n+1
1
.
Nhn hai v bt ng thc trn vi b
n
= qq
1
...q
n
thu -c
b
n+1
r
n+1
= b
n
r
n
q
n+1
qq
1
...q
n+1
p
n+1
q
n+1
b
n
r
n
q
n+1
b
n
r
n
(q
n+1
1) = b
n
r
n
.
T suy ra {b
n
r
n
} l mt dy s nguyn d-ng n iu gim. Do bt u
t mt ch s no , cc s hng ca dy u bng nhau. iu mu thun
vi gi thit A hu hn.
3.1.48. R rng, ta c th vit n! = 2
(n)
(n), trong (n) l mt s l, cn
(n) -c tnh bi cng thc (n) = n (n), trong (n) l tng cc ch
s ca s n vit trong h c s 2 (nh l Legendre). Hn na, ta c

k=1
2
n
k
n
k
!
=

n=1

n
2
n
n!
, trong
n
= 1 nu n = n
k
v (n) = 0 nu ng-c li. Gi s
phn chng rng

n=1

n
2
n
n!
=
p
q
, p, q N. Vit q = 2
s
t, trong t l. Chn
N = 2
r
> max{t, 2
s+2
}, khi
(N)
t
N, do 2
s
(N)
p
q
=
(N)
t
p N.
Theo nh l Legendre ta c N! = 2
N1
(N). Nhn hai v ca ng thc
p
q
=
N

n=1

n
2
n
n!
+

n=N+1

n
2
n
n!
vi 2
s
(N) thu -c
2
s
(N)
p
q
= 2
s
(N)
N

n=1

n
2
n
n!
+ 2
s
(N)

n=N+1

n
2
n
n!
. ()
Ch rng
2
s
(N)
N

n=1

n
2
n
n!
= 2
s
N

n=1

n
(N)2
n
2
(n)
(n)
.

n

C
h
i
3.1. Tng ca chui 253
Chng t s hng u v phi ca () l mt s nguyn. Do suy ra mu
thun ta s chng minh rng 0 < 2
s
(N)

n=N+1

n
2
n
n!
< 1. Thc vy
2
s
(N)

n=N+1

n
2
n
n!
= 2
sN+1
N!

n=N+1

n
2
n
n!
= 2
s+2

n=N+1

n
2
nN1
N!
n!
<
2
s+2
N + 1

n=N+1
_
2
N + 2
_
nN1
=
2
s+2
N + 1
N + 2
N
<
2
s+3
N + 1
< 1.
3.2 Chui d-ng
3.2.1.
(a) Ta c
a
n
=

n
2
+ 1
3

n
3
+ 1
=
1

n
2
+ 1 +
3

n
3
+ 1

3n
4
2n
3
+ 3n
2
(n
2
+ 1)
2
+ (n
2
+ 1)
3
_
(n
3
+ 1)
2
+ (n
3
+ 1)
3

n
3
+ 1
.
Do
lim
n
a
n
1
n
=
1
2
.
Theo du hiu so snh chui cho phn k.
(b) V lim
n
n

a
n
= lim
n
_
1
n
n
2
+n+1
_
n
=
1
e
nn chui hi t.
(c) C th chng minh bng quy np rng
(2n 3)!!
(2n 2)!!
>
1
2n 1
vi n > 2.
Do chui phn k theo du hiu so snh.
(d) Chui hi t v lim
n
n

a
n
=
1
e
.

n

C
h
i
254 Ch-ng 3. Chui s thc
(e) 1 cos
1
n
= 2 sin
2 1
2n
<
1
2n
2
, suy ra chui hi t.
(f) V lim
n
n

a
n
= 0 nn chui hi t.
(g) Theo bi 2.5.4 (a), ta c
lim
n
n

a 1
1
n
= lna.
Do chui phn k.
3.2.2.
(a) Chui hi t v
1
n
ln
_
1 +
1
n
_
=
1
n
2
.
(b) S hi t ca dy -c suy t bt ng thc
1

n
ln
n + 1
n 1
<
2

n(n 1)
, vi n > 1.
(c) S dng bt ng thc lnn < n ta nhn -c
1
n
2
lnn
<
1
n(n1)
. Do
chui ang xt l hi t.
(d) Ta c
1
(lnn)
ln n
=
1
n
lnlnn
.
Suy ra chui phn k.
(e) S dng ph-ng php php tnh vi phn, ta c th chng minh rng vi
x ln bt ng thc (lnlnx)
2
< lnx ng. Khi ta c
1
(lnn)
lnln n
=
1
e
(lnlnn)
2
>
1
n
vi n ln, t suy ra chui phn k.
3.2.3. t c
n
=
an
bn
, t gi thit ta -c
c
n+1
=
a
n+1
b
n+1

a
n
b
n
= c
n
, n n
0
,
tc l dy {c
n
} tng n iu vi n n
0
. iu ny chng t rng dy b
chn, tc l tn ti C > 0 sao cho 0 < c
n
< C, n N. T suy ra

n=1
a
n
=

n=1
c
n
b
n
< C

n=1
b
n
, v ta suy ra iu phi chng minh.

n

C
h
i
3.1. Tng ca chui 255
3.2.4.
(a) T bi ton 2.1.38 ta c
a
n+1
a
n
=
_
1 +
1
n
_
n2
e
<
_
n
n + 1
_
2
=
1
(n+1)
2
1
n
2
.
Ta c kt lun rng chui hi t nh s dng du hiu hi t ca bi ton
trn.
(b) T-ng t, s dng 2.1.38 ta c
a
n+1
a
n
=
_
1 +
1
n
_
n
e
>
_
1 +
1
n
_
n
_
1 +
1
n
_
n+1
=
1
(n+1)
1
n
.
Nu chui cho hi t th s dng bi 3.2.3 ta suy ra chui

n=1
1
n
cng
hi t, iu ny l v l, ta suy ra chui ang xt phn k.
3.2.5.
(a) T bi ton 2.5.4 (a), ta c chui

n=1
(
n

a 1)

n=1
1
n

cng hi t
hoc phn k, suy ra chui ang xt s hi t vi > 1 v phn k vi
1.
(b) T li gii bi 2.5.4 (b) ta c lnn < n(
n

n1), do vi n > 3 v > 0


ta -c
1
n

<
_
lnn
n
_

< (
n

n 1)

.
iu ny cho thy rng chui phn k vi 0 < 1, trong tr-ng hp
0 cc s hng trong chui khng tho mn iu kin cn ca chui
hi t. i vi tr-ng hp > 1 s dng bi tp 2.5.5 ta kt lun rng
chui hi t khi v ch khi chui

n=1
_
lnn
n
_

hi t. S hi t ca chui
sau -c suy ra t bi ton 3.2.3 v vi n ln ta c
a
n+1
a
n
=
_
nln(n + 1)
(n + 1) lnn
_

_
n
n + 1
_

n

C
h
i
256 Ch-ng 3. Chui s thc
(c) T bi 2.5.5 ta c chui hi t khi v ch khi chui

n=1
_
ln
_
1 +
1
n
_
n+1
e
_

hi t. S dng bt ng thc
2x
2+x
< ln(1 +x) < x vi x > 0 trong bi
2.5.3 ta suy ra
_
ln
_
1 +
1
n
_
n+1
1
_

<
1
n

, vi > 1
v
_
ln
_
1 +
1
n
_
n+1
1
_

>
1
(2n + 1)

, vi 0 < 1.
T suy ra chui hi t khi > 1 v phn k vi 0 < 1. Hn na
ch rng vi 0 chui khng tho mn iu kin cn ca chui hi
t, suy ra n cng phn k.
(d) D dng kim tra ng thc
lim
n
1 nsin
1
n
1
n
2
=
1
6
.
Do chui hi t khi v ch khi chui

n=1
1
n
2
hi t, v v vy chui s
phn k vi
1
2
v hi t vi >
1
2
.
3.2.6. T 2.5.5 ta -c lim
n
an lna
a
an
1
= 1 v do chui ang xt hi t khi v
ch khi

n=1
a
n
hi t.
3.2.7.
(a) S hi t ca chui

n=1
ln
_
cos
1
n
_
-c suy t ng thc
lim
n
ln
_
cos
1
n
_
1
2n
2
= 1.

n

C
h
i
3.1. Tng ca chui 257
(b) Nu c = 0 th
lim
n
e
alnn+b
c lnn+d
= e
a
c
= 0.
T suy ra chui ang xt hi t.
Nu c = 0 v
a
d
0 th iu kin cn ca chui hi t khng tho mn.
Cn nu c = 0 v
a
d
< 0 th
e
alnn+b
d
= e
b
d
e
a
d
lnn
= e
b
d
n
a
d
.
Do trong tr-ng hp ny chui hi t vi
a
d
< 1 v phn k vi
a
d
1.
(c) Ta c
n
2n
(n +a)
n+b
(n +b)
n+a
=
1
(n +a)
b
_
1 +
a
n
_
n
(n +b)
a
_
1 +
b
n
_
n
.
Tc l chui cho hi t khi v ch khi chui

n=1
1
n
a+b
hi t.
3.2.8. S hi t ca chui

n=1

a
n
a
n+1
-c suy ra trc tip t bt ng
thc

a
n
a
n+1

1
2
(a
n
+ a
n+1
), hn na nu dy {a
n
} n iu gim th

a
n
a
n+1
a
n+1
, t suy ra s hi t ca chui

n=1
a
n
-c suy ra trc tip
t s hi t ca chui

n=1

a
n
a
n+1
. By gi xt dy {a
n
} -c nh ngha
a
n
=
_
1 nu n l,
1
n
4
nu n chn.
Th th
n

k=1

a
k
a
k+1
<
2n

k=1

a
k
a
k+1
=
n

n=1
1
k
2
.
Do chui

n=1

a
n
a
n+1
hi t trong khi

n=1
a
n
phn k.
3.2.9.

n

C
h
i
258 Ch-ng 3. Chui s thc
(a) Tr-c tin ch rng nu dy {a
n
} b chn trn bi mt s M > 0 no
th
a
n
1 +a
n

a
n
1 +M
.
Do chui

n=1
an
1+an
phn k. Mt khc nu dy {a
n
} khng b chn
trn th s tn ti mt dy con {a
n
k
} tin ti v cng. Khi
lim
n
a
n
k
1 +a
n
k
= 1,
v v vy iu kin cn ca chui hi t khng -c tho mn.
(b) Chui

n=1
an
1+nan
c th hi t hoc phn k. Ta s ch ra cc v d
chng t iu ny, xt dy
a
m
=
_
1 nu n = m
2
, m = 1, 2, ...,
1
n
2
ng-c li.
Th th chui

n=1
a
n
phn k, nh-ng
n

k=1
a
k
1 +ka
k
<
n

k=1
1
1 +k
2
+
n

k=1
1
k +k
2
2
n

k=1
1
1 +k
2
.
Trong tr-ng hp ny chui

n=1
an
1+nan
hi t. Nu chn a
n
=
1
n
th c
hai chui

n=1
a
n
v

n=1
an
1+nan
phn k.
(c) S hi t ca chui -c suy ra t bt ng thc
a
n
1 +n
2
a
n

a
n
n
2
a
n
=
1
n
2
.
(d) Nu dy {a
n
} b chn trn bi M th
a
n
1 +a
2
n

a
n
1 +M
2
.
Do trong tr-ng hp ny chui l phn k, nh-ng nu vi a
n
= n
2
th
ta li -c chui

n=1
an
1+a
2
n
hi t.

n

C
h
i
3.1. Tng ca chui 259
3.2.10. Vi mi s nguyn d-ng n v p ta c
a
n+1
S
n+1
+
a
n+2
S
n+2
+... +
a
n+p
S
n+p

n+p

k=n+1
a
k
S
n+p
=
S
n+p
S
n
S
n+p
.
V lim
p
Sn+pSn
Sn+p
= 1 nn dy cc tng ring ca chui

n=1
an
Sn
khng phi l dy
Cauchy, suy ra chui phn k.
Mt khc
a
n
S
2
n

a
n
S
n
S
n1
=
S
n
S
n1
S
n
S
n1
=
1
S
n1

1
S
n
,
v do
n+p

k=n+1
a
k
S
2
k

n+p

k=n+1
_
1
S
k1

1
S
k
_
=
1
S
n

1
S
n+p
<
1
S
n
.
Ta c kt lun chui

n=1
an
S
2
n
hi t theo tiu chun Cauchy.
3.2.11. Ta c
a
n
S
n
S

n1
=
S
n
S
n1
S
n
S

n1
.
Xt p l mt s nguyn d-ng sao cho
1
p
< . Th th vi n ln ta c bt
ng thc
a
n
S
n
S

n1
<
a
n
S
n
S
1
p
n1
.
iu c ngha l c iu phi chng minh ta ch cn xt s hi t ca
chui

n=1
an
SnS
1
p
n
. Ta c bt ng thc
S
n
S
n1
S
n
S
1
p
n1
p
_
_
1
S
1
p
n1

1
S
1
p
n
_
_
,
t-ng -ng
1
S
n1
S
n
p
_
_
1
S
1
p
n1
S
1
p
n
_
_
.

n

C
h
i
260 Ch-ng 3. Chui s thc
S t-ng -ng -c suy ra t vic p dng bt ng thc 1 x
p
p(1 x)
vi 0 < x 1 bng cch t x =
_
S
n1
Sn
_1
p
. T s dng bi tp trn ta suy
ra chui cho hi t vi > 0.
3.2.12. Gi s rng > 1. Th th vi n 2,
a
n
S

a
n
S
n
S
1
n1
.
Ta c th suy ra s hi t ca chui t bi tp 3.2.11. Xt 1, vi n ln
ta c
an
S

an
Sn
. T kt qu ny v s dng bi 3.2.10 ta suy ra chui phn k
vi 1.
3.2.13.
(a) T gi thit suy ra dy {a
n
} n iu gim v 0, hn na
a
n
r
n1
=
r
n1
r
n
r
n1
.
Do vi mi s n v p nguyn d-ng ta c
a
n+1
r
n
+... +
a
n+p
r
n+p1
=
r
n
r
n+1
r
n
+... +
r
n+p1
r
n+p
r
n+p1
>
r
n
r
n+p
r
n
= 1
r
n+p
r
n
.
C nh n ta c gii hn lim
p
_
1
rn+p
rn
_
= 1, s dng tiu chun Cauchy
ta suy ra chui phn k.
(b) Ta c
a
n

r
n1
=
r
n1
r
n

r
n1
=
(

r
n1

r
n
)(

r
n1
+

r
n
)

r
n1
< 2(

r
n1

r
n
).
S dng bt ng thc ny ta suy ra rng dy cc tng ring ca chui

n=1
an

r
n1
l Cauchy, t suy ra chui hi t.

n

C
h
i
3.1. Tng ca chui 261
3.2.14. u tin xt tr-ng hp 1, th th vi n ln ta c
1
r

n1

1
r
n1
.
S dng phn (a) ca bi tp tr-c ta suy ra chui ang xt phn k.
Vi < 1, tn ti s p nguyn d-ng sao cho < 1
1
p
, khi ta c
a
n
r

n1
<
a
n
(r
n1
)
1
1
p
=
r
n1
r
n
r
n1
r
1
p
n1
.
Khi cho x =
_
rn
r
n1
_1
p
trong bt ng thc 1 x
p
p(1 x) vi 0 < x 1,
ta -c
a
n
r

n1
p
_
r
1
p
n1
r
1
p
n
_
.
S dng tiu chun Cauchy suy ra chui trong bi hi t.
3.2.15. Vi 0 < < 1 c
lim
n
a
n+1
ln
2
r
n
a
n+1
r

n
= 0,
s dng bi tp trn ta suy ra chui

n=1
a
n+1
ln
2
r
n
hi t.
3.2.16. Ta bit rng (xem bi tp 2.1.38)
_
1 +
1
n
_
n
< e <
_
1 +
1
n
_
n+1
. ()
Gi thit rng g > 1 v ly > b sao cho g > 1, th th tn ti s
nguyn d-ng n
0
sao cho nln
an
a
n+1
> g vi n n
0
. Theo () ta c
nln
a
n
a
n+1
> g > nln
_
1 +
1
n
_
,
t suy ra
a
n
a
n+1
<
1
(n+1)
g
n
1
g
.

n

C
h
i
262 Ch-ng 3. Chui s thc
Do chui

n=1
a
n
hi t theo du hiu -c chng minh bi tp 3.2.3.
Tr-ng hp g = + ta cng lp lun t-ng t. Chng minh t-ng t cch
trn ta suy ra chui phn k trong tr-ng hp g < 1.
V d sau chng t du hiu -c nu trn khng s dng -c khi g = 1.
Chn a
n
=
1
n
, thy rng g = 1 v

n=1
1
n
phn k. Mt khc khi t a
n
=
1
nln
2
n
ta -c chui

n=1
a
n
hi t (bi 3.2.29), trong tr-ng hp ny g cng bng 1,
tht vy, ch rng
nln
1
nln
2
n
1
(n+1) ln
2
(n+1)
= ln
_
1 +
1
n
_
n
+ 2nln
ln(n + 1)
lnn
.
R rng s hng u ca tng v phi tin v 1, ta cn phi chng minh rng
lim
n
2nln
ln(n+1)
ln n
= 0. Tht vy, iu phi chng minh l ng do
lim
n
_
ln(n + 1)
lnn
_
n
= lim
n
_
1 +
ln
n+1
n
lnn
_
n
= e
0
= 1.
3.2.17.
(a) Ta c
lim
n
nln
a
n
a
n+1
= lim
n
n(

n + 1

n) ln2 = +.
S dng bi tp trn ta suy ra s hi t ca chui -c cho.
(b) T-ng t cu trn ta c
lim
n
nln
a
n
a
n+1
= lim
n
ln
_
1 +
1
n
_
n
ln2 = ln2 < 1.
Do chui phn k theo du hiu Raabe.
(c) T-ng t, t
lim
n
nln
a
n
a
n+1
= ln3 > 1,
suy ra chui hi t.

n

C
h
i
3.1. Tng ca chui 263
(d) Ta c
lim
n
nln
a
n
a
n+1
= lna.
Do chui hi t vi a > e v phn k vi a < e. Khi a = e chui -c
cho s l mt chui iu ho phn k.
(e) Ta c (xem 3.2.16)
lim
n
nln
a
n
a
n+1
= lim
n
nln
ln(n + 1)
lnn
lna = 0.
Do s dng du hiu hi t -c xt bi tr-c ta suy ra chui hi
t vi mi a > 0.
3.2.18. V
lim
n
nln
a
n
a
n+1
= lim
n
nlna

1
n+1
= ln
1
a
,
nn chui hi t khi 0 < a <
1
e
, v phn k vi a >
1
e
(t-ng t bi 3.2.16).
Nu a =
1
e
th (xem 2.1.41) ta c
lim
n
e
1
1+
1
2
+...+
1
n
1
n
= e

,
trong l hng s Euler. S dng du hiu so snh v s phn k ca chui
iu ho

n=1
1
n
suy ra chui phn k khi a =
1
e
.
3.2.19. S dng bt ng thc
x
1+x
ln(1 +x) x vi x > 1 ta c
n
_
an
a
n+1
1
_
1 +
_
an
a
n+1
1
_ nln
_
1 +
a
n
a
n+1
1
_
n
_
a
n
a
n+1
1
_
.
T bt ng thc trn ta thy rng vi nhng r hu hn th hai du hiu
Raabe v du hiu -c nu trong bi 3.2.16 l t-ng -ng, ng thi ta
cng suy ra rng nu lim
n
nln
an
a
n+1
= + th lim
n
n
_
an
a
n+1
1
_
= +.
By gi ta s ch ra rng php ko theo cn li cng ng. Tht vy, nu
lim
n
n
_
an
a
n+1
1
_
= + th vi mi A > 0 tn ti n
0
sao cho bt ng thc
an
a
n+1
1 >
A
n
ng vi mi n > n
0
. Do
nln
a
n
a
n+1
= ln
_
1 +
a
n
a
n+1
1
_
n
> ln
_
1 +
A
n
_
n

n
A.

n

C
h
i
264 Ch-ng 3. Chui s thc
Do A ln tu nn ta suy ra lim
n
nln
an
a
n+1
= +. Lp lun hon ton t-ng
t cho tr-ng hp r = .
3.2.20. V dy {a
n
} n iu tng nn
0 < n
_
1
an
1
a
n+1
1
_
=
1
n
a
n1
a
n
<
1
n
,
S dng du hiu Raabe ta suy ra chui cho phn k.
3.2.21. T nh ngha ca dy ta suy ra
a
n
= a
1
e

n1

k=1
a

k
vi n = 1, 2, ....
Tr-c tin ta s chng t rng chui

n=1
a

n
phn k. Tht vy nu chui hi t
v S th lim
n
a
n
= a
1
e
S
> 0 v do lim
n
a

n
> 0, iu ny mu thun vi
iu kin cn ca chui hi t, suy ra chui

n=1
a

n
phn k, v ta c lim
n
a
n
= 0.
By gi gi thit rng > , ta s chng minh rng trong tr-ng hp ny
chui ang xt s hi t, tc l s phi chng minh rng
a

n
> (n 1) vi n 1. ()
Bt ng thc l hin nhin khi n = 1. Gi thit rng n ng n n, th th
t nh ngha ca dy ta c
a

n+1
= a

n
e
a

n
> a

n
(1 +a

n
) = a

n
+ > n.
Do () ng vi mi n nguyn d-ng. Vi n = 1, bt ng thc trn t-ng
-ng vi
a

n
< ((n 1))

.
Do s dng du hiu so snh ta c chui s hi t khi > . Xt ,
ta c lim
n
a
n
= 0 nn vi n ln, 0 < a
n
< 1, do a

n
a

n
, ng thi
chui

n=1
a

n
hi t suy ra chui

n=1
a

n
hi t.
3.2.22. Ch rng
lim
n
n
_
a
n
a
n+1
1
_
= lim
n
n
n + 1
a = a.

n

C
h
i
3.1. Tng ca chui 265
S dng du hiu Raabe ta suy ra chui hi t khi a > 1 v phn k khi
0 < a < 1.
Tr-ng hp a = 1 chui s tr thnh chui iu ho

n=1
1
n+1
.
3.2.23. T ng thc
lim
n
n
_
a
n
a
n+1
1
_
= lim
n
nb
n+1
(n + 1)a
=
b
a
v du hiu Raabe suy ra chui hi t khi b > a v phn k vi b < a. Tr-ng
hp b = a s hi t ca chui ph thuc vo dy {b
n
}. Tht vy, nu {b
n
} l
dy hng s th chui ca chng ta s l mt chui iu ho

n=1
1
n+1
.
Ta s ch ra rng vi b
n
= a +
2a
ln(n+1)
th chui s hi t. Tht vy, ta c
a
n
=
n!
_
2 +
2
ln2
_ _
3 +
2
ln3
_
...
_
n + 1 +
2
ln(n+1)
_
.
Do
a
n
(n 1) ln(n 1) a
n+1
nlnn
= a
n
_
(n 1) ln(n 1)
(n + 1)nln n
n + 2 +
2
ln(n+2)
_
.
Tnh ton ta -c
lim
n
_
(n 1) ln(n 1)
(n + 1)nln n
n + 2 +
2
ln(n+2)
_
= 1.
Do vi n ln th
a
n
(n 1) ln(n 1) a
n+1
nln n (1 )a
n
> 0.
Vy dy d-ng {a
n
(n 1) ln(n 1)} n iu gim v do hi t. T y
ta suy ra -c s hi t ca chui c s hng tng qut a
n
(n 1) ln(n 1)
a
n+1
nln n . S dng bt ng thc cui cng v du hiu so snh suy ra chui

n=1
a
n
hi t.

n

C
h
i
266 Ch-ng 3. Chui s thc
3.2.24. T gi thit ta c
a
n
((n 1) ln n 1) a
n+1
nlnn = (
n
1)a
n
.
Nu
n
> 1 th
a
n
((n 1) lnn 1) a
n+1
nlnn ( 1)a
n
. ()
Kt hp () vi bt ng thc (n 1) ln(n 1) > (n 1) ln n 1 ta -c
a
n
((n 1) lnn 1) a
n+1
nlnn ( 1)a
n
> 0. ()
iu ny c ngha l dy {a
n
(n 1) ln(n 1)} gim n iu, v v th n
hi t. Do chui c s hng tng qut l a
n
(n 1) ln(n 1) a
n+1
nlnn
hi t. T () suy ra chui

n=1
a
n
hi t.
Nu
n
< 1 th a
n
((n 1) lnn 1) a
n+1
nlnn ( 1)a
n
. Do

a
n
(n 1) ln(n 1) a
n+1
nlnn
_
+ ln
_
1
1
n
_
n1
_
a
n
.
V
lim
n
_
+ ln
_
1
1
n
_
n1
_
= 1 < 0,
thy rng dy {a
n+1
nln n} tng n iu tr mt s n hu hn s hng, do
tn ti s thc d-ng M sao cho a
n+1
nlnn > M. T suy ra a
n+1
>
M
n lnn
,
v ta rt ra kt lun rng chui

n=1
a
n
phn k.
3.2.25. Ta c
lim
n
n
_
a
n
a
n+1
1
_
= lim
n
+
n
n
1
1

n

n
n

= .
Do theo du hiu Raabe ta suy ra s hi t ca chui -c xt vi > 1 v
phn k vi < 1. Trong tr-ng hp = 1 ta suy ra s phn k ca chui t
du hiu -c nu bi trn, v
a
n+1
a
n
= 1
1
n


n
n

= 1
1
n


n
nln n
,
trong
n
=
n lnn
n
1
< 1 vi no .

n

C
h
i
3.1. Tng ca chui 267
3.2.26. Ta s s dng du hiu Gauss trong bi tp trn. C
a
n+1
a
n
=
n
2
+ ( +)n +
n
2
+ (1 +)n +
= 1
1 +
n


n
n
2
.
T suy ra chui -c nu trong bi tp s hi t khi + < v phn k
vi + .
3.2.27. T-ng t chng minh trn ta cng s s dng du hiu Gauss. Ta c
a
n+1
a
n
= 1
p
2
n


n
n
2
.
Do chui phn k nu p 2 v hi t nu p > 2.
3.2.28. Xt S
n
,

S
n
l tng ring th n ca cc chui

n=1
a
n
v

n=1
2
n
a
2
n. Do
vi n 2
k
, ta c
S
n
a
1
+ (a
2
+a
3
) +... + (a
2
k +... +a
2
k+1
1
)
a
1
+ 2a
2
+... + 2
k
a
2
k =

S
k
.
Vi n > 2
k
,
S
n
a
1
+a
2
+ (a
3
+a
4
) +... + (a
2
k1
+1
+... +a
2
k )
a
2
+ 2a
4
+... + 2
k1
a
2
k =
1
2

S
k
.
Do cc dy {S
n
} v {

S
n
} ng thi b chn hoc khng b chn.
3.2.29.
(a) Ta s s dng tiu chun nn ca Cauchy (xem 3.2.28). Khi ta xt chui
sau

n=1
2
n
2
n
(ln2
n
)

n=1
1
(nln2)

,
thy rng n hi t vi > 1 v phn k vi 0 < 1. Nu 0
th s phn k ca chui

n=2
1
(nln n)

-c suy ra trc tip t du hiu so


snh.

n

C
h
i
268 Ch-ng 3. Chui s thc
(b) T ng thc

n=2
2
n
2
n
ln2
n
lnln2
n
=

n=2
1
nln 2 ln(nln2)
v t (a) suy ra chui hi t.
3.2.30. Lp lun t-ng t nh- chng minh cho tiu chun nn ca Cauchy
(3.2.28). Vi n g
k
,
S
n
S
g
k
(a
1
+...a
g
1
1
) + (a
g
1
+... +a
g
2
1
) +... + (a
g
k
+... +a
g
k+1
1
)
(a
1
+...a
g
1
1
) + (g
2
g
1
)a
g
1
+... + (g
k+1
g
k
)a
g
k
.
Vi n > g
k
,
cS
n
cS
g
k
c(a
g
1
+1
+... +a
g
2
) +... +c(a
g
k1
+1
+... +a
g
k
)
c(g
2
g
1
)a
g
2
+... +c(g
k
g
k1
)a
g
k
(g
3
g
2
)a
g
2
+... + (g
k+1
g
k
)a
g
k
.
Cc bt ng thc trn chng minh bi ton.
3.2.31.
(a) Ta p dng nh l Schlomilch (3.2.30) vi g
n
= 3
n
.
(b) S dng nh l Schl

'omilch vi g
n
= n
2
ta -c hai chui

n=1
a
n
v

n=1
(2n + 1)a
n
2 hi t ng bc. V
lim
n
(2n + 1)a
n
2
na
n
2
= 2,
suy ra

n=1
na
n
2 v

n=1
(2n + 1)a
n
2 cng hi t ng bc.
(c) Rt ra t cu (b).
(d) T cu (b) suy ra chui

n=1
1
2

n
v

n=1
n
2
n
hi t ng bc. Ta chng minh
-c chui sau hi t (v d theo du hiu lu tha). chng minh s
hi t hay phn k ca cc chui

n=1
1
2
lnn
,

n=1
1
3
lnn
v

n=1
1
a
lnn
, ta s dng

n

C
h
i
3.1. Tng ca chui 269
nh l nhm cc s hng ca Cauchy hoc du hiu nu trong phn (a).
By gi ta nghin cu dng iu ca chui vi s hng tng qut
1
a
lnlnn
.
Nu a > 1 th s hi t ca chui ang xt s t-ng -ng vi s hi t
ca chui

n=1
3
n
a
lnn
. Ta c th d dng kim tra rng chui phn k,
bng du hiu so snh s m chng hn. iu ny chng t cho ta rng
chui

n=2
1
a
lnlnn
phn k vi a > 1. Ch rng nu 0 < a 1 th chui
ang xt s khng tho mn iu kin cn ca chui hi t.
3.2.32. T bi tp 2.4.13 (a), tn ti mt s > 0 sao cho
(a
n
)
1
lnn
< e
1
, n > k.
Do
1
lnn
lna
n
< 1 , v do a
n
<
1
n
1+
. S dng du hiu so snh ta
suy ra chui

n=1
a
n
hi t.
3.2.33. T-ng t bi trn ta c
a
n

1
n(lnn)
1+
, vi n > k v vi > 0 no .
Do theo bi tp 3.2.29 (a) suy ra chui

n=1
a
n
hi t.
3.2.34.
S
2
n
0
+k
1
S
2
n
01
= (a
2
n
0 +a
2
n
0+1
+... +a
2
n
0
+1
1
)
+ (a
2
n
0
+1
+1
+... +a
2
n
0
+2
1
) +...
+ (a
2
n
0
+k1 +... +a
2
n
0
+k
1
)
2
n
0
a
2
n
0 + 2
n
0
+1
a
2
n
0
+1 +... + 2
n
0
+k1
a
2
n
0
+k1
g(a
n
0
+a
n
0
+1
+... +a
n
0
+k1
).
Do vi k ln
(1 g)
2
n
0
+k
1

n=2
n
0
a
n
g
_
_
2
n
0
1

n=n
0
a
n

2
n
0
+k
1

n=n
0
+k
a
n
_
_
g
2
n
0
1

n=n
0
a
n
.
V vy dy tng ring ca chui

n=1
a
n
b chn, do chui hi t.

n

C
h
i
270 Ch-ng 3. Chui s thc
3.2.35. Gi s chui

n=1
a
n
hi t, th th
lim
n
2n

k=n+1
a
k
= lim
n
2n+1

k=n+1
a
k
= 0.
Do v tnh n iu ca dy {a
n
} ta c
2n

k=n+1
a
k

2n

k=n+1
a
2n
= na
2n
=
1
2
(2na
2n
)
v
2n+1

k=n+1
a
k

2n+1

k=n+1
a
2n+1
=
n + 1
2n + 1
(2n + 1)a
2n+1
.
T suy ra lim
n
na
n
= 0.
t a
n
=
1
n ln(n+1)
. Th th chui c s hng tng qut a
n
s phn k v
lim
n
na
n
= lim
n
1
ln(n + 1)
= 0.
3.2.36. t
a
n
=
_
1
n
vi n = k
2
, k = 1, 2, ...,
1
n
2
trong cc tr-ng hp cn li.
Th th chui

n=1
a
n
s hi t nh-ng gii hn lim
n
a
n
khng tn ti.
3.2.37. Vn ta cn tm l s hi t ca chui

n=1

a
n
. Tht vy, nu chui

n=1

a
n
hi t, ta c th t b
n
=

a
n
.
By gi gi s tn ti dy {b
n
} sao cho c hai chui

n=1
b
n
v

n=1
an
bn
hi t.
Ta suy ra rng

a
n
=
_
b
n

a
n
b
n

1
2
_
b
n
+
a
n
b
n
_
,
v do chui

n=1

a
n
hi t.

n

C
h
i
3.1. Tng ca chui 271
3.2.38. Gi s rng tn ti dy {a
n
} sao cho c hai chui

n=1
a
n
v

n=1
1
n
2
an
cng hi t, t
A =
_
n
s
N :
1
n
s

1
n
2
s
a
ns
_
v A

= N \ A.
Do

nsA
1
ns
< + v do

nsA

1
ns
= + (tt nhin A c th l tp
rng).
By gi ch rng a
ns
>
1
ns
vi n
s
A

, ta suy ra rng chui


n=1
a
n
phn
k, tc l tri vi gi thit.
3.2.39. Ta c

n=1
1
n

1 +a
n+1
a
n
=

n=1
1
na
n
+

n=1
a
n+1
na
n
.
Ta s ch ra rng s hi t ca chui s ko theo s phn k ca chui

n=1
1
nan
.
Theo du hiu nh gi Cauchy, tn ti k N sao cho vi mi n nguyn d-ng,
k+n

i=k+1
a
i+1
ia
i
<
1
4
. Do
n
n+k
k+n

i=k+1
a
i+1
na
i
<
1
4
. Vi n > k ta c
k+n

i=k+1
a
i+1
na
i
<
1
4

k +n
n

1
2
.
T mi lin h gia trung bnh cng v nhn, ta c
n
_
a
k+n+1
a
k+1
<
1
2
, v do a
k+n+1
<
a
k+1
2
n
,
suy ra
1
(k +n + 1)a
k+n+1
>
2
n
(k +n + 1)a
k+1
.
Vy chui

n=1
1
nan
phn k.
3.2.40. Tt nhin chui

n=1
c
n
c th phn k (v d nh- nu a
n
b
n
vi
n N). Tuy vy n hi t. Tht vy, xt chui c cc s hng c dng
1,
1
2
2
,
1
2
2
,
1
2
2
,
1
2
2
,
1
2
2
,
1
7
2
,
1
8
2
,
1
9
2
, ...

n

C
h
i
272 Ch-ng 3. Chui s thc
v
1,
1
2
2
,
1
3
2
,
1
4
2
,
1
5
2
,
1
6
2
,
1
7
2
,
1
8
2
,
1
8
2
, ...,
1
8
2
. .
8
2
+1 ln
, ...
Mi chui trn cha v hn on s hng c tng ln hn 1, do chng phn
k, trong tr-ng hp c
n
=
1
n
2
v do

n=1
c
n
hi t.
3.2.41. Ta s dng tiu chun nn ca Cauchy (3.2.28). S phn k ca chui

n=1
bn
n
t-ng -ng vi s phn k ca chui c s hng tng qut
b
2
n = min
_
a
2
n,
1
nln 2
_
.
Thy rng chui

n=1
b
2
n phn k khi v ch khi chui c ng ca n vi s hng
tng qut
2
n
b
2
2
n
= min
_
2
n
a
2
2
n
,
1
ln2
_
phn k. Ta s ch ra rng chui ny phn k. Tht vy, ni chui

n=1
d
n
phn
k th chui

n=1
min{d
n
, c} vi c > 0 s cng phn k, nu min{d
n
, c} = c
i vi mt s v hn s hng d
n
th s phn k ca chui

n=1
min{d
n
, } -c
suy ra t s phn k ca chui

n=1
d
n
.
3.2.42. Ta c
1
a
n
a
n+1
=
a
n+1
a
n
a
n+1

a
n+1
a
n
a
1
.
T khng nh trn v s hi t ca chui

n=1
(a
n+1
a
n
) ta suy ra s hi t
ca chui cho.
3.2.43. Ta c
1
a
n
a
n+1
=
a
n+1
a
n
a
n+1
.
t b
n
= a
n+1
a
n
v S
n
= b
1
+b
2
+... +b
n
, ta -c
bn
Sn+a
1
=
a
n+1
an
a
n+1
. Do
ta suy ra chui phn k da vo bi tp 3.2.10.

n

C
h
i
3.1. Tng ca chui 273
3.2.44. Nu dy {a
n
} khng b chn th s hi t ca chui -c suy ra t
bi 3.2.11, thy -c iu ny ta c th lp lun t-ng t nh- lp lun trong
bi tp trn. Xt tr-ng hp {a
n
} b chn, ta c
a
n+1
a
n
a
n+1
a

1
a
2
a

1
(a
n+1
a
n
).
Do s hi t ca chui ang xt -c suy ra t s hi t ca chui

n=1
(a
n+1

a
n
).
3.2.45. Ta ch cn ly c
n
=
1
Sn
vi S
n
l tng ring th n ca chui

n=1
a
n
nh- trong bi 3.2.10.
3.2.46. Ta c th t c
n
=
1

r
n1
, vi r
n
= a
n+1
+ a
n+2
+ ..., v t s
dng kt qu ca bi 3.2.13 (b).
3.2.47. Dy {r
n
} l n iu gim, theo bi 3.2.35 ta suy ra lim
n
na
n
= 0,
do
lim
n
na
n
= lim
n
n(r
n1
r
n
) = lim
n
((n 1)r
n1
nr
n
+r
n1
) = 0.
3.2.48.
(a) V lim
n
a
n
= +, a
n
> 2 vi n ln. S hi t ca chui -c suy ra
t bt ng thc
1
a
n
n
<
1
2
n
vi n ln.
(b) Nh- trong (a), n c th -c chn ln sao cho
1
a
ln
n
n
<
1
3
lnn
. Vy, theo
3.2.17(c),chui hi t.
(c) Chui c th hi t hoc phn k, ph thuc vo dy {a
n
}. Nu a
n
=
lnn, n 2, th chui

n=1
1
a
lnlnn
n
phn k(xem 3.2.2(e)). Mt khc, nu
a
n
= n, th n > e
e
,
1
a
lnln n
n
=
1
e
lnln nlnn
n
<
1
n

vi > 1.
Trong tr-ng hp ny, chui ang xt hi t.
3.2.49. Chui phn k v iu kin cn a
n
0 cho s hi t khng -c tho
mn( xem 2.5.25).

n

C
h
i
274 Ch-ng 3. Chui s thc
3.2.50. Tr-c ht, gi s p = 0. Khi , theo 2.5.22, ta c lim
n

na
n
=

3,
v v th chui phn k. By gi, gi s p > 0. Khi lim
n
a
n
= 0. T
lim
n
a
n+1
a
n
= lim
n
sina
n
a
n

1
n
p
= 0.
Chui hi t theo tiu chun t s.
3.2.51. Quan st rng a
n
(n, n + /2). T
1
a
2
n
<
1
n
2

2
v v vy
chui

n=1
1
a
2
n
hi t.
3.2.52. t b
n
=

a
n
; khi b
n
(n, n + /2) . T chui

n=1
1
an
=

n=1
1
b
2
n
hi t (xem li gii ca bi ton tr-c).
3.2.53. Chuiphn k do lim
n
na
n
= 2 (xem 2.5.29).
3.2.54. n gin, ta -a ra k hiu sau:
L
n
= a
1
+a
2
+... +a
2n1
v M
n
= a
2
+a
4
+... +a
2n
Do tnh n iu ca {a
n
},
L
n
M
n
v L
n
a
1
M
n
. ()
T 2M
n
= M
n
+M
n
M
n
+L
n
a
1
=

n=2
a
n
. Vy
lim
n
M
n
= +. ()
Kt hp () v (), ta c
L
n
M
n
1 =
L
n
M
n
M
n

a
1
M
n
0(n )

n

C
h
i
3.1. Tng ca chui 275
3.2.55. T nh ngha ca k
n
ta c 0 S
kn
n <
1
kn
. Bit rng lim
n
(S
kn

lnk
n
) = , y l hng s Euler ( xem 2.1.41), nn
lim
n
(n lnk
n
) = lim
n
(n + 1 lnk
n+1
) = ,
t suy ra
lim
n
_
1 ln
k
n+1
k
n
_
= 0,
Vy
lim
n
k
n+1
k
n
= e.
3.2.56.
(a) [A. J. Kempner, Amer. Math. Monthly 23(1914), 48-50] Mt s c k ch
s s hng A c th -c vit d-i dng
10
k1
a
1
+ 10
k2
a
2
+... +a
k
y 0 < a
i
9, i = 1, 2, ..., k.
Vi k cho tr-c, tn ti 9
k
s c k ch s trong A, v mi s ln hn 10
k1
.
V vy

nA
1
n
<

k=1
9
k
10
k1
= 90.
(b) Nh- trong (a), ta c

nA
1
n

<

k=1
9
k
10
(k1)
.
V vy nu > log
10
9, th chui

nA
1
n

hi t. Ngoi ra, t

nA
1
n

>

k=1
9
k
(10
k
1)

>

k=1
9
k
10
k
chui

nA
1
n

phn k nu log
10
9.
Nhn x t. Gi A
k
l tp con ca cc s nguyn d-ng khng cha ch
s k trong khai trin thp phn ca chng. Theo cng cch ny, ta c th
ch ra chui

nA
1
n

hi t nu > log
10
9.

n

C
h
i
276 Ch-ng 3. Chui s thc
3.2.57. Gi s rng < g < 1 v ly > 0 nh g + < 1. Khi
, vi n ln,ln
1
an
< (g +) lnn v a
n
>
1
n
g+
. V vy chui phn k. Nu
g = th vi n ln, ln
1
an
< 1 lnn. Vy a
n
> n v chui phn k.
Chng minh t-ng t cho g > 1. Chng ta xt hai chui:

n=1
1
n
v

n=2
1
nln
2
n
.
Chui u tin phn k v chui th hai hi t mc d g = 1.
3.2.58. S t-ng -ng ca cc tiu chun ny -c cho trong li gii ca
bi tp 3.2.19. Do 2.5.34, nu tiu chun Raabe kt lun -c th tiu chun
ca bi ton tr-c cng vy. ch ra iu ng-c li khng ng chng ta xt
chui vi s hng a
n
xc nh bi a
2n1
=
1
n
2
, a
2n
=
1
4n
2
.
3.2.59. Cho b
n
=
_
2 +
_
2 +... +

2
. .
n - cn
, c b
n
= 2 cos

2
n+1
(so snh vi
2.5.41). Do nh ngha ca {a
n
}, ta c a
2
n
= 2 b
n1
, v do vy a
n
=
2 sin

2
n+1
<

2
n
. V th chui trong bi ra hi t.
3.2.60. Gi s K l mt s d-ng sao cho
(a
1
a
n
) + (a
2
a
n
) +... + (a
n1
a
n
) K vi n N
T vi mi n N ta c a
1
+ a
2
+ ... + a
n
na
n
K. Chn m N bt
k. V dy {a
n
} n iu v hi t v 0 nn tn ti n
0
N sao cho :
a
n

1
2
a
m
vi n n
0
. ()
Ta c :
a
1
+... +a
m
ma
n
+a
m+1
+... +a
n
(n m)a
n
K.
Li do s n iu ca {a
n
},
a
m+1
+... +a
n
(n m)a
n
v a
1
+... +a
m
ma
m
.
V th m(a
m
a
n
) = ma
m
ma
n
= a
1
+ a
2
+... +a
m
ma
n
K. T
y v t () suy ra
1
2
ma
m
m(a
m
a
n
) K. Cui cng,
S
m
= a
1
+a
2
+... +a
m
= S
m
ma
m
+ma
m
K +ma
m
3K.
3.2.61. T quan h
a
n
= a
n+1
+a
n+2
+a
n+3
+..., a
n+1
= a
n+2
+a
n+3
+...
suy ra a
n+1
=
1
2
a
n
. S dng ph-ng php quy np ta c a
n
=
1
2
n
, n N.

n

C
h
i
3.1. Tng ca chui 277
3.2.62. [20] Cho r
n,k
= a
n
+ a
n+1
+ ... +a
n+k
, n = 1, 2, ..., k = 0, 1, 2, ...,
v cho lim
k
r
n,k
= r
n
, n = 1, 2, .... Gi thit rng s (0, S) v a
n
1
l s
hng u tin ca dy {a
n
} tho mn a
n
1
< s. Ng-c li tn ti k
1
sao
cho r
n
1
,k
1
< s r
n
1
,k
1
+1
, hoc r
n
1
s. Trong tr-ng hp th hai, ta c
s a
n
1
1
r
n
1
s v do vy r
n
1
= s. Trong tr-ng hp u ta tm a
n
2
l
s hng u tho mn n
2
> n
1
+k
1
, r
n
1
,k
1
+a
n
2
< s. Ng-c li tn ti k
2
sao
cho r
n
1
,k
1
+ r
n
2
,k
2
< s r
n
1
,k
1
+ r
n
2
,k
2
+1
, hoc r
n
1
,k
1
+ r
n
2
= s. Ta lp li
qu trnh ny n khi tr-ng hp u xut hin ti mi b-c th kt lun rng
s = r
n
1
,k
1
+r
n
2
,k
2
+....
3.2.63. [20] Gi s ng-c li, s c k N sao cho a
k
= 2p +

n=k+1
a
n
,
y p > 0. Lc a
k
p = p +

n=k+1
a
n
=

n=1

n
a
n
, y
n
nhn
gi tr 0 hoc 1. Bi tnh n iu ca {a
n
},
n
= 0 vi n k. Do
a
k
p =

n=1

n
a
n

n=k+1
a
n
= a
k
2p, mu thun!
3.2.64. Theo nh l Stolz (xem 2.3.11), ta c
lim
n
a
1
S
1
1
+a
2
S
1
2
+... +a
n
S
1
n
lnS
n
= lim
n
a
n
S
1
n
ln(1 a
n
S
1
n
)
= 1.
ng thc cui suy ra t ,chng hn 2.5.5.
3.2.65. t a
n
= 1, n N.
3.2.66. V
a
1
+a
2
+...+an
n
>
a
1
n
nn chui
n

i=1
a
1
+a
2
+...+an
n
hi t vi mi dy
d-ng {a
n
}. S hi t ny c lp vi s hi t ca chui
n

i=1
a
n
.
3.2.67. Vi gi thit,
a
2
a
1
,
2
n

k=2
n1
+1
a
k

1
2
n1
2
n1

k=1
a
k
.
V vy
2
n

k=1
a
k
2
_
1 +
1
2
__
1 +
1
2
2
_
+... +
_
1 +
1
2
n1
_
a
1
.

n

C
h
i
278 Ch-ng 3. Chui s thc
Hn na,
_
1 +
1
2
__
1 +
1
2
2
_
+... +
_
1 +
1
2
n1
_
= e
n1

k=1
ln
(
1+
1
2
k
)
e
n1

k=1
1
2
k
< e.
3.2.68. t c
n
=
(n+1)
n
n
n1
= n
_
n+1
n
_
n
, n N. Th
c
1
....c
n
= (n + 1)
n
v c
n
< ne ()
Dng bt ng thc lin h gia trung bnh cng v trung bnh nhn ta -c
n

a
1
... a
n
=
1
n + 1
n

a
1
c
1
...a
n
c
n

a
1
c
1
+... +a
n
c
n
n(n + 1)
.
V vy
N

n=1
n

a
1
... a
n

N

i=1
a
1
c
1
+... +a
n
c
n
n(n + 1)
= a
1
c
1
_
1
1 2
+
1
2 3
+... +
1
N (N + 1)
_
+
a
2
c
2
_
1
2 3
+... +
1
N (N + 1)
_
+... +a
N
c
N
1
N(N + 1)
< a
1
c
1
+a
2
c
2
1
2
+a
3
c
3
1
3
+... +a
N
c
N
1
N
2a
1
+ea
2
+... +ea
N
.
Bt ng thc cui cng c -c do (). Cho N , ta c bt ng thc cn
chng minh.
3.2.69. Vit c
n
d-i dng
c
n
=
(n + 1)
n
... (n +k 1)
n
(n +k)
n
n
n1
... (n +k 2)
n1
(n +k 1)
n1
=
_
n +k
n
_
n
n(n + 1) ... (n +k 1),
ta -c c
1
... c
n
= (n +1)
n
... (n +k)
n
. V vy, nh- trong li gii ca bi

n

C
h
i
3.1. Tng ca chui 279
ton tr-c, ta -c
N

n=1
n

a
1
... a
n

N

n=1
a
1
c
1
+... +a
n
c
n
n(n + 1) ... (n +k)
= a
1
c
1
(
1
1 2... (1 +k)
+... +
1
N (N + 1)... (N +k)
)
+a
2
c
2
(
1
2 3... (2 +k)
+... +
1
N (N + 1)... (N +k)
)
+... +a
N
c
N
1
N(N + 1)...(N +k)
<
1
k
_
1
k!
a
1
c
1
+
1
2 3 .... (1 +k)
a
2
c
2
+... +
1
N(N + 1) ... (N +k 1)
a
N
c
N
_
.
Bt ng thc cui cng -c suy ra t bi tp 3.1.4 (a). V
1
l(l + 1) ... (l +k 1)
c
l
=
_
l +k
l
_
l
,
cho N , ta -c bt ng thc cn chng minh.
3.2.70. t T
n
= a
1
+a
2
+... +a
n
v S
n
l tng ring th n ca chui cn
tm. Khi
S
N
=
1
a
1
+
N

n=2
n
2
(T
n
T
n1
)
T
2
n

1
a
1
+
N

n=2
n
2
(T
n
T
n1
)
T
n
T
n1
=
1
a
1
+
N

n=2
n
2
T
n1

n=2
n
2
T
n
=
1
a
1
+
N1

n=1
(n + 1)
2
T
n

n=2
n
2
T
n

5
a
1
+
N1

n=2
2n
T
n
+
N1

n=2
1
T
n

5
a
1
+
N

n=1
2n
T
n
+
N

n=1
1
T
n
.
Hn na theo bt ng thc Cauchy (xem 1.2.12),
_
N

n=1
n
T
n
_
2

n=1
n
2
a
n
T
2
n
N

n=1
1
a
n
S
N
M

n

C
h
i
280 Ch-ng 3. Chui s thc
vi M =

n=1
1
an
. V vy
N

n=1
n
T
n

_
S
N

M.
Nn, S
N

5
a
1
+ 2

S
N

M +M, v
S
N

_

M +
_
2M +
5
a
1
_
2
.
3.2.71. Theo bt ng thc trung bnh iu ho (xem 1.2.3)
2
k

n=2
k1
+1
1
nan(n1)a
n1
2
k1

2
k1
2
k

n=2
k1
+1
[na
n
(n 1)a
n1
]
=
2
k1
2
k
a
2
k 2
k1
a
2
k1

1
2a
2
k
.
V vy
2
k

n=2
k1
+1
1
na
n
(n 1)a
n1

2
k
4a
2
k
.
Nn
S
2
k
k

l=1
2
l
4a
2
l
.
S phn k ca chui suy -c t nh l nn ca Cauchy (xem 3.2.28).
3.2.72. Ta s ch ra rng chui

n=1
1
pn
phn k. Nu hi t th tn ti n sao
cho

m=n+1
1
pm
<
1
2
. t a = p
1
p
2
... p
n
. Khi s 1 + ka vi k N c th
vit thnh tch cc s nguyn t. S phn tch ny khng cha s no trong cc
s p
1
, ..., p
n
. V vy

k=1
1
1 +ka
<

l=1
_

m=n+1
1
p
m
_
l
<

l=1
_
1
2
_
l
= 1,
mu thun.

n

C
h
i
3.1. Tng ca chui 281
3.2.73. Suy ra t bi 3.2.71 v 3.2.72.
3.2.74. Ta c
lim
n

k=2
1
k
n+1

k=2
1
k
n
= lim
n
1
2
n+1
_
1 +
2
n+1
3
n+1
+
2
n+1
4
n+1
+...
_
1
2
n
_
1 +
2
n
3
n
+
2
n
4
n
+...
_ =
1
2
,
v tng trong ngoc hi t v 1 khi n dn ti v cng. Do
2
n+1
3
n+1
+
2
n+1
4
n+1
+... = 2
n+1

k=3
1
k
n+1
.
Hn na,

k=3
1
k
n+1
=
1
3
n+1
+

k=2
1
(2k)
n+1
+

k=2
1
(2k + 1)
n+1

1
3
n+1
+ 2

k=2
1
(2k)
n+1
=
1
3
n+1
+
1
2
2n+1
+
1
2
n

k=3
1
k
n+1
.
Nn
2
n+1

k=3
1
k
n+1

_
2
3
_
n+1
+
1
2
n
_
1
1
2
n
_ ,
v v vy
2
n+1

k=3
1
k
n+1

n
0.
3.2.75. Tr-c ht gi thit rng chui

n=1
a
n
hi t. Khi s hi t ca
chui cho -c suy ra t bt ng thc
1
T

1
a

1
. Nu chui

n=1
a
n
phn k
th tn ti dy tng cht n
m
cc s nguyn d-ng sao cho S
nm1
m < S
nm
.
Khi
T
nm
= S
1
+... +S
nm
S
n
1
+... +S
nm
>
m(m+ 1)
2
.
V vy

n=n
2
a
n
T

n
=

m=2
m
m+1
1

k=nm
a
k
T

m=2
S
n
m+1
1
S
nm1
T

nm
<

m=2
1
T

nm
<

m=2
1
_
m
2
+m
2
_

n

C
h
i
282 Ch-ng 3. Chui s thc
V vy chui cho hi t nu >
1
2
. Chui ny c th phn k nu
1
2
.
V d chn a
n
= 1, n N.
3.2.76. Theo bi 3.2.35, lim
n
n
an
= 0. Ly 0 < K < 1. Khi tn ti n
0
sao
cho n Ka
n
vi mi n n
0
. V vy
ln
k
a
n
a
n
ln
k
_
1
K
_
ln
k
n
a
n
.
V vy s hi t ca chui

n=1
ln
k
an
an
suy ra s hi t ca chui

n=1
ln
k
n
an
.
chng minh ta t
I
1
= {n N : a
n
n
k+2
} v I
2
= N\I
1
.
Khi vi n I
1
ta c lna
n
(k + 2) lnn v v vy t s hi t ca chui

nI
1
ln
k
n
an
suy ra s hi t ca chui

nI
1
ln
k
an
an
. Hn na, khi n ln thuc I
2
,
ln
k
a
n
a
n
<
a
k
k+1
n
a
n
<
1
n
k+2
k+1
.
V vy,

nI
2
ln
k
an
an
< v
k+2
k+1
> 1.
3.2.77. Tr-c ht gi thit rng
f((n))((n + 1) (n))
f(n)
q < 1.
Khi theo (1) trong bi tr-c,
S
(n)1
<
(1)1

k=1
f(k) +qS
n1
.
V vy (n) > n, (1 q)S
n1
<
(1)1

k=1
f(k). Suy ra chui

n=1
f(n) hi t.
S dng bt ng thc (2) trong bi tr-c v chng minh t-ng t ta -c phn
th hai ca mnh .
3.2.78. S dng kt qu ca bi tr-c vi (n) = 2n.
3.2.79. S dng kt qu bi 3.2.78 vi (n) = 2
n
.

n

C
h
i
3.1. Tng ca chui 283
3.2.80. S dng kt qu bi 3.2.77 t-ng ng vi
(n) = 3
n
, (n) = n
2
, v (n) = n
3
.
3.2.81.
(1) Ta c a
n
b
n
a
n+1
b
n+1
ca
n+1
. V vy {a
n
b
n
} l dy s d-ng gim
v vy hi t. V vy chui

n=1
(a
n
b
n
a
n+1
b
n+1
) hi t. S hi t ca
chui

n=1
a
n
suy ra t tiu chun so snh.
(2) Ta c
a
n+1
a
n

1
b
n+1
1
bn
V vy s hi t ca chui

n=1
a
n
suy -c t tiu chun ch ra trong
bi 3.2.3.
3.2.82. c tiu chun D'Alembert (tiu chun t s) ta chn b
n
= 1 vi mi
n = 1, 2, .... Nu chn b
n
= n vi n = 1, 2, ... ta c tiu chun Raabe. Chn
b
n
= nlnn vi n = 2, 3, ... ta c tiu chun Bertrand.
3.2.83. [J. Tong, Amer. Math. Monthly, 101(1994), 450-452]
(1) t S
n
=

n=1
a
n
v t
b
n
=
S
n

k=1
a
k
a
n
=
r
n
a
n
.
Tt nhin b
n
> 0 vi n N. Hn na
b
n
a
n
a
n+1
b
n+1
=
r
n
a
n+1

r
n+1
a
n+1
=
a
n+1
a
n+1
= 1.
(2) Trong tr-ng hp ny t
b
n
=
n

k=1
a
k
a
n
=
S
n
a
n
.

n

C
h
i
284 Ch-ng 3. Chui s thc
Khi chui

n=1
1
bn
phn k (xem bi 3.2.10). Hn na
b
n
a
n
a
n+1
b
n+1
=
S
n
a
n+1

S
n+1
a
n+1
=
a
n+1
a
n+1
= 1.
3.2.84.
(a) S dng tiu chun t s cho mi chui:

n=1
a
kn
,

n=0
a
1+kn
, ...,

n=0
a
(k1)+kn
.
(b) Ch cn p dng tiu chun Raabe (xem 3.2.19) cho mi chui trn.
3.2.85. Theo gi thit tn ti hng s d-ng K sao cho

n
K
1
lnn
, n 2.
Ta nh ngha 2 tp cc s nguyn d-ng N
1
v N
2
nh- sau:
N
1
=
_
n : a
n

1
n
2
_
v N
2
= N\N
1
.
Vi n N
1
ln ta c,
a
1n
n
a
1
K
lnn
n
= a
ln
n
e
K
lnn
n
=
_
e
K
n
_
ln
1
an
lnn

e
2K
n
2
. (1)
Hn na vi n N
2
ln ta c,
a
1n
n
a
n
a

K
lnn
n
=
_
1
a
n
_ K
lnn
n
2K
lnn
= e
2K
. (2)
Kt hp (1), (2) cng vi s hi t ca chui

n=2
a
n
, ta -c

nN
1
a
1n
n
< + v

nN
2
a
1n
n
< +.

n

C
h
i
3.3. Du hi u tch phn 285
3.3 Du hiu tch phn
3.3.1. Vi k 1 x k, k 2 ta c f(x) f(k). Mt khc vi
k x k + 1 th f(x) f(k). V vy
_
k+1
k
f(x)dx f(k)
_
k
k1
f(x)dx, k = 2, 3, ...
Cng theo v ca cc bt ng thc trn t k = 2 ti k = n ta -c
_
n+1
2
f(x)dx f(2) +f(3) +... +f(n)
_
n
1
f(x)dx.
Tiu chun tch phn -c chng minh.
3.3.2. Ch rng
f

f
d-ng v n iu tng v vy theo tiu chun tch phn
s hi t ca chui cho t-ng -ng vi tnh b chn ca dy
__
n
1
f

(x)dx
_
v
_
_
n
1
f

(x)
f(x)
dx
_
. V
_
n
1
f

(x)dx = f(n) f(1) v


_
n
1
f

(x)
f(x)
dx = lnf(n) lnf(1),
nn hai dy cng b chn hoc cng khng b chn.
3.3.3. Ta c S
N
I
N
(S
N+1
I
N+1
) =
_
N+1
N
f(x)dx f(N + 1) 0.
Hn na, f(n)
_
n
n1
f(x)dx f(n 1) vi n = 2, 3, ..., N. Cng cc bt
ng thc trn t n = 2 n n = N ta -c S
N
f(1) I
N
S
N
f(N).
V vy 0 < f(N) S
N
I
N
f(1), l iu phi chng minh.
3.3.4. S hi t ca dy cho -c suy t bi ton tr-c. By gi ta cn ch
ra rng gii hn ca dy thuc (0, 1).
(a) V f(x) =
1
x
l hm tng cht trn khong (0, +), S
N
I
N
< S
2
I
2
<
f(1) = 1 vi N > 2 v
f(2) +f(3) +... +f(N 1) +f(N)
> f(2) +f(3) +... +f(N 1) >
_
N
2
f(x)dx,
hoc t-ng -ng, S
N
f(1) > I
N
I
2
. Cui cng,
0 < 1 I
2
lim
N
(S
N
I
N
) S
2
I
2
< 1.
(Xem 2.1.41 v 3.1.36).

n

C
h
i
286 Ch-ng 3. Chui s thc
(b) Chng minh t-ng t nh- (a).
3.3.5.
(a) S hi t ca chui

n=2
1
n(lnn)

t-ng -ng vi tnh b chn ca dy


_
n
2
1
x(lnx)

dx. Vi = 1,
_
n
2
1
x(lnx)

dx =
(lnn)
+1
+ 1

(ln2)
+1
+ 1
.
V vy chui hi t nu > 1 v phn k nu 0 < < 1. R rng
nu 0 th chui phn k. Cui cng, nu = 1 th
_
n
2
1
xln x
dx =
ln(lnn) ln(ln2). V vy, dy
_
n
2
1
x lnx
dx khng b chn v do chui
phn k.
(b) Trong tr-ng hp ny ta c
_
n
3
1
xlnxln(lnx)
dx = ln(ln(lnn)) ln(ln(ln3)).
V vy theo tiu chun tch phn chui phn k.
3.3.6.
(a) Ta c
N

n=1
a
n+1
S
n
lnS
n
=
N

n=1
S
n+1
S
n
S
n
lnS
n

n=1
_
S
n+1
Sn
1
xlnx
dx
= lnlnS
N+1
lnlnS
1

N
.
(b) T-ng t cu (a) ta c
N

n=2
a
n
S
n
ln
2
S
n
=
N

n=2
S
n
S
n1
S
n
ln
2
S
n

n=2
_
Sn
S
n1
1
xln
2
x
dx
=
1
lnS
N
+
1
lnS
1

N
.
3.3.7. Nu

(x)f((x))
f(x)
q < 1, x > x
0
,

n

C
h
i
3.3. Du hi u tch phn 287
th
_
(x)
(x
0
)
f(t)dt =
_
x
x
0

(t)f((t))dt q
_
x
x
0
f(t)dt.
V vy
(1 q)
_
(x)
(x
0
)
f(t)dt q
_
_
x
x
0
f(t)dt
_
(x)
(x
0
)
f(t)dt
_
= q
_
_
(x
0
)
x
0
f(t)dt
_
(x)
x
f(t)dt
_
q
_
(x
0
)
x
0
f(t)dt.
V vy bng tiu chun tch phn chui

n=1
f(n) hi t. By gi nu

(x)f((x))
f(x)
1 vi mi x > x
0
,
th
_
(x)
(x
0
)
f(t)dt
_
x
x
0
f(t)dt. Cho nn
_
(x)
x
f(t)dt
_
(x
0
)
x
0
f(t)dt.
Hn na, v vi n bt k tn tik
n
N sao cho n < (n) < n +k
n
, ta c
I
n
+k
n
I
n
=
_
n+kn
n
f(t)dt
_
(n)
n
f(t)dt
_
(x
0
)
x
0
f(t)dt.
Cho nn dy {I
n
} khng phi l dy Cauchy nn n khng b chn. Theo tiu
chun tch phn chui phn k.
3.3.8.
(a) Nu lim
x
_
g(x)
f

(x)
f(x)
g

(x)
_
> 0 th tn ti x
0
v > 0 sao cho
g(x)
f

(x)
f(x)
g

(x) vi mi x x
0
.
V vy (g(x)f(x))

f(x), x x
0
. Cho nn vi n ln ta c
_
n
x
0
f(x)dx
1

_
n
x
0
(f(x)g(x)

dx
=
1

(g(x
0
)f(x
0
) g(n)f(n))
1

g(x
0
)f(x
0
).
Theo tiu chun tch phn chui hi t.

n

C
h
i
288 Ch-ng 3. Chui s thc
(b) Ging nh- (a) ta c (g(x)f(x))

0 vi mi x x
0
v vy gf l hm
tng trn [x
0
, ) nn g(x)f(x) g(x
0
)f(x
0
) nu x x
0
. iu ny c
ngha l f(x)
f(x
0
)g(x
0
)
g(x)
vi mi x > x
0
. Do dy
_
n
1
f(x)dx khng
b chn v theo gi thit dy
_
n
1
1
g(x)
dx khng b chn.
3.3.9. S dng kt qu ca bi tr-c vi g(x) = x.
3.3.10. Trong bi 3.3.8 ly g(x) = xlnx.
3.3.11. (a) t
g(x) =
_

x
f(t)dt
f(x)
.
Khi g(x)
f

(x)
f(x)
g

(x) = 1 > 0.
(b) t
g(x) =
_
x
1/2
f(t)dt
f(x)
.
th
_
n
1
1
g(x)
dx = ln
_
n
1/2
f(t)dtln
_
1
1/2
f(t)dt, iu c ngha l dy
_
n
1
1
g(x)
dx
b chn. Hn na,
g(x)
f

(x)
f(x)
g

(x) = 1 < 0.
3.3.12. Ta s dng tiu chun -c chng minh trong bi 3.3.9. Ly
f(x) = (lnx)
(lnx)

, x > 1, ta -c
x
f

(x)
f(x)
= (lnx)
1
( lnlnx + 1).
Nu 1 th lim
x
(lnx)
1
( lnlnx + 1) = +v v vy chui hi t. Mt
khc nu 0 < 1, th ta c lim
x
(lnx)
1
( lnlnx + 1) = 0, iu c
ngha l chui phn k.
3.3.13. t
f(x) =
1
x
1+
1
lnlnx
lnx
, x > e.
Ta c th chng minh -c lim
x
_
x
f

(x)
f(x)
_
= 1. V vy khng th p dng
tiu chun trong bi 3.3.9. Ta s p dng tiu chun trong bi 3.3.10. Vi x
ln th
_

(x)
f(x)

1
x
_
xlnx =
lnx
lnlnx

lnx
(lnlnx)
2
+ 1 > 2
v lim
x
_
ln x
lnlnx

lnx
(lnlnx)
2
_
= +.

n

C
h
i
3.3. Du hi u tch phn 289
3.3.14. Ta c (
n+1

n
)
1

n+1
f(
n+1
)

_

n+1
n
1
tf(f)
dt. V vy

n=1
_
1

n

n+1
_
1
f(
n+1
)

_

1
1
tf(t)
dt < .
Ta chng minh -c rng chui

n=1
_
1
n

n+1
_
1
f(
n+1
)
hi t. K hiu {S
n
}
v {S

n
} t-ng ng l dy tng ring ca chui cho trong bi ton v chui
trn. Th
S
N
S

N
=
N

n=1
_
1

n

n+1
__
1
f(
n
)

1
f(
n+1
)
_

n=1
_
1
f(
n
)

1
f(
n+1
)
_
<
1
f(
1
)
.
Vy chui cho hi t.
3.3.15. Vi hm n iu f,
f(
n+1
)(
n+1

n
)
_

n+1
n
f(t)dt f(
n
)(
n+1

n
). ()
(a) Vi bt ng thc bn tri v gi thit ta c,
M

n=1
f(
n+1
)
_

1
f(t)dt < .
(b) T bt ng thc bn phi trong () suy ra chui

n=1
f(
n
) hi t.
3.3.16. Tr-c ht ta gi thit rng chui

n=1
1
f(n)
hi t. Khi , bng tiu
chun tch phn, tch phn suy rng
_

1
1
f(t)
dt hi t. Tch phn tng phn v
i bin ta -c
_

1
1
f(t)
dt = lim
t
t
f(t)

1
f(t)
+
_

1
tf

(t)
f
2
(t)
dt
= lim
t
t
f(t)

1
f(t)
+
_

f(1)
f
1
(t)
t
2
dt. ()

n

C
h
i
290 Ch-ng 3. Chui s thc
Ta s ch ra rng
lim
t
t
f(t)
= 0 ()
S hi t ca tch phn suy rng suy ra lim
t
_
2t
t
1
f(x)
dx = 0. V
1
2
2t
f(2t)
=
1
f(2t)
_
2t
t
dx <
_
2t
t
1
f(x)
dx
_

1
f
1
(t)
t
2
dt, ng thc () ng. V vy tch phn
_

1
f
1
(t)
t
2
dt hi t.
Hn na, ta c

n=1
f
1
(n)
(n + 1)
2

n=1
_
n+1
n
f
1
(t)
t
2
dt =
_

1
f
1
(t)
t
2
dt < ,
t suy ra chui

n=1
f
1
(n)
(n+1)
2
hi t. Hin nhin l chui

n=1
f
1
(n)
n
2
cng hi t.
chng minh iu ng-c li, gi thit l chui

n=1
f
1
(n)
n
2
hi t. Bng
cch t-ng t ta c th ch ra rng tch phn
_

f(1)
f
1
(t)
t
2
dt hi t, v v vy tch
phn
_

1
1
f(t)
dt cng hi t. Bng tiu chun tch phn chui

n=1
1
f(n)
hi t.
3.3.17. Tr-c ht ta thy rng hm nh ngha -c trn ton khong
[e, ). Khi (x) = 1 vi mi x [e, e
e
), (x) = 2 vi mi x [e
e
, e
e
e
).
n gin ta t e
1
= e v e
k
= e
e
k1
vi k > 1. V vy ta c
(x) = k vi x
_
e
k
, e
k+1
_
.
t
f(x) =
1
x(ln
1
x)(ln
2
x) ... (ln
(x)
x)
;
th
f(x) =
1
x(ln
1
x)(ln
2
x)...(ln
k
x)
vi x
_
e
k
, e
k+1
_
.
By gi bng tiu chun tch phn chui cho hi t v vi n > e
k
,
I
n
=
_
n
e
f(x)dx
_
e
k
e
f(x)dx =
_
e
2
e
1
xlnx
dx +
_
e
3
e
2
1
x(lnx)(ln
2
x)
dx
+... +
_
e
k
e
k1
1
x(lnx)(ln
2
x) ... (ln
k1
x)
dx = k 1.

n

C
h
i
3.4. Hi t tuy t i. nh l Le ibniz 291
3.4 Hi t tuyt i. nh l Leibniz
3.4.1.
(a) Ta c
lim
n
n

an
n + 1

n
= |a|.
V vy chui hi t tuyt i nu |a| < 1 v phn k nu |a| > 1. Nu
|a| = 1 th chui phn k v
lim
n

an
n + 1

n
= lim
n
1
(1 +
1
n
)
n
=
1
e
.
(b) t f(x) =
(lnx)
a
x
vi x > 0. Khi f

(x) =
(lnx)
a1
(alnx)
x
2
< 0 vi x >
max{1, e
a
}. V vy theo tiu chun Leibniz chui hi t vi mi a R.
By gi ta kim tra xem liu chui c hi t tuyt i hay khng tc l
chui

n=2
(lnn)
a
n
hi t? Dng tiu chun Cauchy (xem 3.2.28), s hi t
ca chui t-ng -ng vi s hi t ca chui

n=2
n
a
(ln2)
a
. V vy chui
cho hi t tuyt i nu a < 1.
(c) Nu a > 0 th chui hi t theo tiu chun Leibniz. Nu a < 0 th

n=1
(1)
n
sin
a
n
=

n=1
(1)
n+1
sin
|a|
n
.
Li p dng tiu chun Leibniz ta thy rng chui hi t vi mi a R.
Chui khng hi t tuyt i nu a = 0 v
lim
n
sin
|a|
n
1
n
= |a|.
(d) Chui hi t khi v ch khi 1
a
2
4a8
a
2
+6a16
< 1, tc l
nu a [4,
4
5
) [3, ). R rng chui hi t tuyt i
nu a
_
4,
4
5
_
(3, ).
(e) V
lim
n
n

n
n
a
n
2

= 0 nu |a| > 1,

n

C
h
i
292 Ch-ng 3. Chui s thc
nn chui hi t tuyt i nu |a| > 1. Nu |a| 1 th iu kin cn
chui hi t khng tho mn v lim
n
n
n
|a|
n
2
= +.
(f) Thy rng
lim
n
(lnn)
ln n
n
a
= lim
n
n
ln lnna
= +.
V vy iu kin cn chui hi t khng -c tho mn.
3.4.2. Nu |a| < 1 th vi n ln,

a
n1
na
n1
+ lnn

< |a|
n1
.
V vy chui hi t tuyt i. Nu |a| 1 th
a
n1
na
n1
+ lnn
=
1
2
_
1
1 +
lnn
na
n1
_
.
V vy vi n ln cc s hng ca chui d-ng v bng tiu chun so snh s
hi t ca chui suy -c t s hi t ca chui

n=1
1
n
.
3.4.3. Tr-c ht gi thit rng a
n
> 0 vi mi n N. Dng o hm ta c
th ch ra rng sinx > x
x
3
6
vi x > 0. V vy 1
sinan
an
<
1
6
a
2
n
. V a
2
n
< a
n
vi n ln chui

n=1
a
2
n
tc l chui cho hi t. Nu b qua gi thit a
n
> 0
th chui c th hi t hay phn k. Thc ra ta ly a
n
= (1)
n 1
n

vi > 0.
Khi chui

n=1
_
1
sinan
an
_
phn k nu 0 <
1
2
v hi t nu >
1
2
.
3.4.4. Khng, ta ch ra phn th d sau:
a
n
=
(1)
n
n
+
1
nlnn
, b
n
=
(1)
n
n
, n 2.
3.4.5. Ta c a
n
= p
n
q
n
v |a
n
| = p
n
+q
n
. Ch rng p
n
v q
n
khng m.
V vy hai chui

n=1
p
n
v

n=1
q
n
phn k, v

n=1
a
n
hi t v

n=1
|a
n
| phn k.
3.4.6. t S
n
= a
1
+... +a
n
. Theo bi tr-c ta c
lim
n
P
n
Q
n
= lim
n
_
1 +
S
n
Q
n
_
= 1.

n

C
h
i
3.4. Hi t tuy t i. nh l Le ibniz 293
3.4.7. Chui khng hi t tuyt i. Ta s ch ra rng n hi t ( hi t c
iu kin ). Ta nhm cc s hng cng du v -c

n=1
(1)
[
n
3
]
n
=
3
2
+

n=1
(1)
n
_
1
3n
+
1
3n + 1
+
1
3n + 2
_
.
V vy theo nh l Leibniz chui hi t.
3.4.8. R rng chui hi t tuyt i nu a > 1 v phn k nu a 0.
Ta s ch ra rng nu 0 < a 1 th chui hi t c iu kin. Thy rng
s hng u tin ca chui l m, nm s hng tip theo d-ng, v.v. By gi
nhm cc s hng cng du ta -c chui an du sau

n=1
(1)
n
A
n
, trong
A
n
=
(n+1)
2
1

k=n
2
1
k
a
. Hn na vi a = 1,
A
n
<
1
n
2a
+
_
(n+1)
2
n
2
1
t
a
dt =
1
n
2a
+
1
1 a
((n + 1)
22a
n
22a
).
V vy (xem 2.2.3), lim
n
A
n
= 0 nu
1
2
< a < 1. Vi a = 1 ta c
2
n+1
< A
n
<
2n+1
n
2
, v v vy, lim
n
A
n
= 0 vi
1
2
< a 1. Ta s ch ra rng vi a nh- trn
th dy {A
n
} n iu gim. Thc vy,
A
n
A
n+1
=
(n+1)
2
1

k=n
2
1
k
a

(n+2)
2
1

k=(n+1)
2
1
k
a
=
(n+1)
2
1

k=n
2
1
k
a

(n+1)
2
+1

=n
2
1
(k

+ 2n + 1)
a
=
(n+1)
2
1

k=n
2
_
1
k
a

1
(k

+ 2n + 1)
a
_

1
((n + 2)
2
2)
a

1
((n + 2)
2
1)
a
=
2n

k=0
_
1
(n
2
+k)
a

1
((n + 1)
2
+k)
a
_

1
((n + 2)
2
2)
a

1
((n + 2)
2
1)
a
> (2n + 1)
_
1
(n
2
+ 2n)
a

1
((n + 1)
2
+ 2n)
a
_

1
((n + 2)
2
1)
a
>
1
((n + 2)
2
1)
a
,

n

C
h
i
294 Ch-ng 3. Chui s thc
trong bt ng thc cui cng suy ra t tnh n iu ca hm
g(x) =
1
(n
2
+x)
a

1
((n + 1)
2
+x)
a
trn on [0, 2n]. V vy, vi n ln,
A
n
A
n+1
> (2n + 1)
_
1
(n
2
+ 2n)
a

1
((n + 1)
2
+ 2n)
a
_

2
(n + 1)
2a
=
2
n
2a
_
_
n +
1
2
_
_
_
1 +
2
n
_
a

_
1 +
4
n
+
1
n
2
_
a
_
_
1 +
1
n
_
2a
_
n
2a
(2a 1) > 0,
v (1 +x)
a
> 1 ax v (1 +x)
a
< 1 ax+
a(a+1)
2
x
2
vi a, x > 0 (hai bt
ng thc ny c th chng minh bng o hm). V vy dng nh l Leibniz,
chui

n=1
(1)
n
A
n
hi t nu
1
2
< a 1.
Nu 0 < a
1
2
, th v A
n
> (2n + 1)
1
(n
2
+2n)
a
, iu kin cn chui

n=1
(1)
n
A
n
hi t khng tho mn.
3.4.9. Ging nh- li gii ca bi 3.4.7 v 3.4.8, ta nhm cc s hng cng du
v vit li chui d-i dng sau:

n=1
(1)
n1
_
1
[e
n1
] + 1
+... +
1
[e
n
]
_
.
Ta cng thy rng
1
[e
n1
] + 1
+... +
1
[e
n
]
>
[e
n
] [e
n1
]
[e
n
]
= 1
[e
n1
]
[e
n
]
.
Hn na v
lim
n
_
1
[e
n1
]
[e
n
]
_
= 1
1
e
,
nn iu kin cn chui hi t khng -c tho mn v do chui phn k.
3.4.10.
(a) C th vit chui d-i dng

n=0
(1)
n
A
n
, trong A
n
=
2
n+1
1

k=2
n
1
k
.
V A
n
> 2
n 1
2
n+1
1

n
1
2
, nn chui phn k.

n

C
h
i
3.4. Hi t tuy t i. nh l Le ibniz 295
(b) Cng ging (a), ta vit chui d-i dng

n=1
(1)
n
A
n
, trong A
n
=
2
n+1
1

k=2
n
1
k lnk
.
Hn na,
0 < A
n
< 2
n
1
2
n
ln2
n
.
Suy ra lim
n
A
n
= 0. Ta s ch ra rng {A
n
} n iu gim. Thc vy,
A
n+1
=
2
n+1
1

k=2
n+1
1
k lnk
=
2
n+1
1

l=0
1
(2
n+1
+l) ln(2
n+1
+l)
=
2
n
1

l=0
_
1
(2
n+1
+ 2l) ln(2
n+1
+ 2l)
+
1
(2
n+1
+ 2l + 1) ln(2
n+1
+ 2l + 1)
_
<
2
n
1

l=0
1
(2
n+1
+ 2l) ln(2
n+1
+ 2l)
<
2
n
1

l=0
1
(2
n
+l) ln(2
n
+l)
= A
n
.
3.4.11.
(1)
n

n
(1)
n
+

n
sin
1

n
= (1)
n
_
1
(1)
n
(1)
n
+

n
_
sin
1

n
= (1)
n
sin
1

n
+
(1)
n
n 1
sin
1

n
n 1
sin
1

n
.
Theo tiu chun Leibniz c hai chui

n=2
(1)
n
sin
1

n
v

n=2
(1)
n
n 1
sin
1

n
hi t. Nh-ng chui

n=2

n
n 1
sin
1

n
phn k v v vy chui cho phn k.
3.4.12.
(a) Chui hi t tuyt i (xem 3.2.1 (f)).

n

C
h
i
296 Ch-ng 3. Chui s thc
(b) T tiu chun Leibniz chui hi t. Chui hi t c iu kin (xem 3.2.1
(g)).
(c) R rng dy {
n

n}, n 3 n iu gim v vy chui hi t. Hn na n


khng hi t tuyt i (xem 3.2.5 (b)).
(d) S hi t ca chui suy ra t tnh n iu ca dy
__
1 +
1
n
_
n
_
v dy c
gii hn l e (xem 2.1.38). chng minh chui khng hi t tuyt i
ta dng bt ng thc
ln(1 +x) < x
1
2
x
2
+
1
3
x
3
, x > 0,
vi x =
1
n
, v ta c
_
1 +
1
n
_
n
< e
1
1
2n
+
1
3n
2
. V vy
e
_
1 +
1
n
_
n
> e
_
1 e

1
2n
+
1
3n
2
_
> e
_
1 e

1
4n
_
vi n > 1.
T 2.5.4 (a) suy ra vi n ln
4n
_
1 e

1
4n
_
>
1
2
.
V vy chui

n=1
_
e
_
1 +
1
n
_
n
_
phn k.
(e) S hi t ca chui suy ra t tnh n iu ca chui
_
_
1 +
1
n
_
n+1
_
v t
gii hn e ca chui (xem 2.1.38). Theo bi 3.2.5 (c), chui khng hi t
tuyt i.
3.4.13.
(a) Hm
f(x) =
(lnx)
a
x
b
, x (e
1
b
, +).
n iu gim ti 0 khi x . V vy theo tiu chun Leibniz chui
hi t. Ta chng minh rng vi b > 1 chui hi t tuyt i. Dng nh
l Cauchy (xem 3.2.28) ta ch cn chng minh s hi t ca chui

n=1
2
n
n
a
2
nb
.
By gi dng tiu chun cn chui hi t nu b > 1 v phn k nu
0 < b < 1. R rng nu b = 1 chui phn k.

n

C
h
i
3.4. Hi t tuy t i. nh l Le ibniz 297
(b) Ch rng
(lnn)
ln n
n
b
=
e
(lnn)(lnln n)
n
b
=
n
lnlnn
n
b
.
V vy iu kin cn ca hi t khng -c tho mn.
3.4.14. Do tnh n iu ca dy {a
n
} ta c
r
2n
= (a
2n+1
a
2n+2
) + (a
2n+3
a
2n+4
) +... > 0,
r
2n+1
= (a
2n+2
+a
2n+3
) + (a
2n+4
+a
2n+5
) +... < 0
v
r
2n
= a
2n+1
+ (a
2n+2
+a
2n+3
) +... < a
2n+1
,
r
2n+1
= a
2n+2
+ (a
2n+3
+a
2n+4
) +... < a
2n+2
3.4.15. Ch rng
n

k=1
(a
k
+a
k+1
) 2
n

k=1
a
k
= a
n+1
a
1

n
a
1
.
3.4.16. Ch rng
n

k=1
(aa
k
+ba
k+1
+ca
k+2
) (a +b +c)
n

k=1
a
k
= b(a
n+1
a
1
) +c(a
n+1
+a
n+2
a
1
a
2
)
n
ba
1
c(a
1
+ a
2
).
3.4.17. Theo gi thit tn ti cc hng s d-ng c v C sao cho vi n ln,
c < |a
n
| C. V vy,

1
a
n+1

1
a
n

1
c
2
|a
n+1
a
n
|,
|a
n+1
a
n
| C
2

1
a
n+1

1
a
n

.
S dng tiu chun so snh ta suy ra iu phi chng minh.
3.4.18. K hiu S
n
v

S
n
t-ng ng l cc tng ring th n ca cc chui

n=1
a
n
v

n=1
n(a
n
a
n+1
). Th,

S
n
=
n

k=1
k(a
k
a
k+1
) =
n

k=1
ka
k

n

k=1
(k + 1)a
k+1
+
n

k=1
a
k+1
= (n + 1)a
n+1
+S
n+1
,
t suy ra iu phi chng minh.

n

C
h
i
298 Ch-ng 3. Chui s thc
3.4.19. S dng tiu chun Leibniz suy ra chui hi t.
3.4.20. Nu |a| < 1 th chui hi t tuyt i. Thc vy, v | sinx| |x|,

n! sina sin
a
2
... sin
a
n

|a|
n
.
Xt tr-ng hp |a| 1. Trong tr-ng hp ny chui phn k v iu kin
cn khng tho mn. Tht vy, vi a c nh tn ti n
0
sao cho
|a|
n
0
1.
Khi , t C = (n
0
1)!

sin a sin
a
2
... sin
1
n
0
1

v dng bt ng thc
sinx
x
> 1
x
2
6
, x > 0, ta -c

n! sina sin
a
2
... sin
a
n

= Cn
0
... nsin
|a|
n
0
... sin
|a|
n
Cn
0
... nsin
1
n
0
... nsin
1
n
= C
n

k=n
0
_
1
1
6k
2
_
C
n

k=n
0
_
1
1
k
2
_
= C
(n
0
1)(n + 1)
n
0
n

n
C
n
0
1
n
0
> 0.
3.4.21. Theo 2.5.4 (a),
lim
n
n

a
n

b+
n

c
2
1
n
= lim
n
_
n

a 1
1
n

b 1 +
n

c 1
2
1
n
_
= lna
1
2
(lnb + lnc) = ln
a

bc
.
V vy nu a >

bc, th bt u t ch s n no cc s hng ca chui d-ng


v theo tiu chun so snh chui phn k. Nu a <

bc th cc s hng ca
chui m v n cng phn k. Vi a =

bc ta c

n=1
_
n

a
n

b +
n

c
2
_
=
1
2

n=1
_
2n

b
2n

c
_
2
.
V
lim
n
_
2n

b 1
2n

c + 1
1
2n
_
2
= (lnb lnc)
2
,
nn s hi t ca chui suy ra t s hi t ca chui

n=1
1
n
2
.

n

C
h
i
3.4. Hi t tuy t i. nh l Le ibniz 299
3.4.22.
(a) Theo 1.1.14, tn ti dy s nguyn {p
n
} v dy s nguyn d-ng {q
n
} sao
cho


p
n
q
n

<
1
q
2
n
.
V vy | cos p
n
| = | cos(q
n
p
n
)| > cos
1
qn
= 1 2 sin
2 1
2qn
> 1
1
2q
2
n
.
Do
(| cos p
n
|)
pn
>
_
1
1
2q
2
n
_
pn
> 1
p
n
q
n
1
2q
n
.
iu ny chng t rng dy con {(cos p
n
)
pn
} ca dy {cos
n
n} khng
hi t v 0 nn iu kin cn khng -c tho mn.
(b) Theo bi 1.1.22 ta bit rng dy {p
n
} v {q
n
} trong (a) c th chn sao cho
mi s hng ca {q
n
} l. Khi dng bt ng thc

2

p
n
q
n

<
1
q
2
n
ta -c | sinp
n
| = | cos(

2
q
n
p
n
)| > cos
1
qn
> 1
1
2q
2
n
. V vy ging
nh- (a) chui (sinp
n
)
pn
khng hi t v 0 v do chui phn k.
3.4.23.
(a) Theo gi thit (xem 2.4.13 (b)), tn ti n
0
v sao cho
n
_
a
n
a
n+1
1
_
> > 0 vi n n
0
.
V vy
a
n+1
an
<
n
n+
< 1, iu chng t rng bt u t ch s n
0
dy
{a
n
} n iu gim. Ta s ch ra rng lim
n
a
n
= 0. Theo trn ta c
a
n+1
=
a
n+1
a
n

a
n
a
n1
...
a
n
0
+1
a
n
0
a
n
0
<
n(n 1)...n
0
( +n)...( +n
0
)
a
n
0
.
By gi ch cn chng minh rng lim
n
n(n1)...n
0
(+n)...(+n
0
)
= 0. Thc vy,
lim
n
n(n 1)...n
0
( +n)...( +n
0
)
= lim
n
1
_
1 +

n
_
...
_
1 +

n
0
_ = 0,

n

C
h
i
300 Ch-ng 3. Chui s thc
v (xem 1.2.1)
_
1 +

n
0
_
...
_
1 +

n
_
> 1 +

n
0
+... +

n

n
.
V vy theo tiu chun Leibniz chui

n=1
(1)
n
a
n
hi t.
(b) Theo gi thit n
_
an
a
n+1
1
_
0, dy {a
n
} n iu tng v v vy chui

n=1
(1)
n
a
n
phn k, v iu kin cn ca hi t khng -c tho mn.
3.4.24. Theo gi thit, lim
n
n
_
an
a
n+1
1
_
= . Vi = 0 ta c th p dng
tiu chun chng minh trong bi tr-c. Vi = 0 iu kin cn hi t
khng -c tho mn. Tht vy, ta c
1
a
n
=
1
a
1

a
1
a
2

a
2
a
3
...
a
n1
a
n
=
1
a
1
_
1 +

1
1
1+
__
1 +

2
2
1+
_
...
_
1 +

n1
(n 1)
1+
_
.
Hn na tn ti sao cho |
n
| . V vy
a
n

a
1
_
1 +

1
1+
_ _
1 +

2
1+
_
...
_
1 +

(n1)
1+
_
a
1
e
A
,
trong A =

n=1
1
n
1+
.
3.4.25. Theo bi 2.5.34, s tn ti gii hn lim
n
nln
an
a
n+1
t-ng -ng vi
s tn ti gii hn lim
n
n
_
an
a
n+1
1
_
v hai gii hn ny bng nhau. t
a
n
=
n!e
n
n
n+p
. Khi lim
n
nln
an
a
n+1
= p
1
2
. V vy theo 3.4.23 chui hi t nu
p >
1
2
v phn k nu p <
1
2
. Trong tr-ng hp p =
1
2
iu kin cn chui
hi t khng -c tho mn v theo cng thc Stirling lim
n
a
n
=

2 .
3.4.26. t S
n
= a
1
+a
2
+... +a
n
. Dng php bin i Abel ta -c
a
1
p
1
+a
2
p
2
+... +a
n
p
n
=
n1

k=1
S
k
(p
k
p
k+1
) +S
n
p
n
,

n

C
h
i
3.4. Hi t tuy t i. nh l Le ibniz 301
v ta c
a
1
p
1
+a
2
p
2
+... +a
n
p
n
p
n
= S
n

n1

k=1
S
k
p
k+1
p
k
p
n
.
By gi ch cn p dng nh l Toeplitz (xem 2.3.1).
3.4.27. Dng kt qu trong bi trn vi chui

n=1
a
n
b
n
v chn p
n
=
1
an
.
3.4.28. y l mt tr-ng hp c bit ca bi tr-c.
3.4.29. Nu chui khng hi t tuyt i, th chui con tt c cc s hng
d-ng v chui con tt c cc s hng m phn k (xem 3.4.5).
3.4.30. [20] Khng, ta s ch ra v d chng t iu ny:
Xt chui hi t tuyt i

n=1
b
n
, t
a
1
= b
1
, a
2
= a
3
=
b
2
2!
, a
4
= a
5
= ... = a
9
=
b
3
3!
, ...,
a
1!+2!+...+(n1)!+1
= a
1!+2!+...+(n1)!+2
= ... = a
1!+2!+...+(n1)!+n!
=
b
n
n!
, ...
Khi chui

n=1
a
n
hi t c iu kin. Nh-ng vi mi k 1 v l 2 th
chui con
a
k
+a
k+l
+a
k+2l
+...
hi t. Thc vy, vi n l c
n!
l
s hng dng
bn
n!
. Nhm nhng s hng ny
li, ta -c chui hi t
C
0
+
1
l

n=n
0
b
n
, C
0
l mt hng s no .
3.4.31. Xt chui
1 +
1
2
3

2
+
1
2
3

1
3

2
+... +
1
n
3

n
+... +
1
n
3

n
. .
n ln

1
3

n
+...

n

C
h
i
302 Ch-ng 3. Chui s thc
3.4.32. C. Xt chui
1 +
1
2 ln2
+
1
2 ln2

1
ln2
+... +
1
nlnn
+... +
1
nlnn
. .
n ln

1
lnn
+...
Khi
N
2
+3N2
2

n=1
a
k
n
=
_

_
1 +
N

n=2
_
1
n
k1
ln
k
n
+
1
ln
k
n
_
nu k chn.
1 +
N

n=2
_
1
n
k1
ln
k
n

1
ln
k
n
_
nu k l.
Theo nh l Cauchy (xem 3.2.28) chui

n=2
1
ln
k
n
phn k vi mi k N. Mt khc, chui

n=2
1
n
k1
ln
k
n
hi t vi k 2.
3.4.33. [20] Gi thit ng-c li lim
n

1
+
2
+...+n
n
= 2 > 0. Khi , theo
2.4.13 (b), tn ti n
0
sao cho vi n > n
0
,

1
+
2
+... +
n
> n ()
t E
n
=
1
+
2
+... +
n
. Tnh tng tng phn ta nhn -c

1
a
1
+
1
a
2
+... +
n
a
n
=
n1

k=1
E
k
(a
k
a
k+1
)E
n
a
n
.
V vy theo () ta c,

1
a
1
+
2
a
2
+... +
n
a
n
>
n
0

k=1
E
k
(a
k
a
k+1
) +
n1

k=n
0
+1
k(a
k
a
k+1
) +na
n
= hng s +
n

k=n
0
+2
a
k
,
iu ny l v l.
3.4.34. [20] t E
n
=
1
+
2
+... +
n
, n N. Dy {E
n
} c tnh cht l
gia hai phn t tri du c mt phn t trit tiu. Xt hai tr-ng hp:
(1) S phn t trit tiu ca {E
n
} l hu hn,

n

C
h
i
3.4. Hi t tuy t i. nh l Le ibniz 303
(2) S phn t trit tiu ca {E
n
} l v hn.
(1) chnh l mt tr-ng hp c bit ca 3.2.35. Trong tr-ng hp (2), theo tiu
chun Cauchy, vi mi > 0 tn ti mt n
0
sao cho nu n > m > n
0
, th
>

k=m+1

k
a
k

k=m+1
((E
k
E
m
) (E
k1
E
m
))a
k

k=m+1
(E
k
E
m
)(a
k
a
k+1
) + (E
n
E
m
)a
n+1

.
()
Gi s E
m
= 0 v cc phn t E
m+1
, E
m+2
, ...E
n
cng du, khi t () v
s n iu ca dy {a
n
} ta c
| E
n
a
n
|< , n m+ 1
3.4.35. Vic chng minh t-ng t nh- 3.4.33. t E
n
= p
1
b
1
+... +p
n
b
n
v
gi s rng lim
n
p
1
b
1
+...+pnbn
n
= 2 > 0. Khi vi n > n
0
ta c p
1
b
1
+... +
p
n
b
n
> n, v do
b
1
+... +b
n
=
1
p
1
(p
1
b
1
) +... +
1
p
1
(p
n
b
n
)
=
n1

k=1
E
k
_
1
p
k

1
p
k+1
_
+E
n
1
p
n
> hng s
n

k=n
0
2
1
p
k
,
iu ny l v l.
3.4.36. Tr-c ht chng ta chng minh nu p = q th chui cho hi t. Ta
c
S
lp
=
_
1 +
1
2
+... +
1
p
_

_
1
p + 1
+... +
1
2p
_
+... + (1)
l+1
_
1
(l 1)p + 1
+... +
1
lp
_
.
V S
lp
l tng ring ca mt chui an du. Theo tiu chun Leibniz tn ti gii
hn lim
l
S
lp
. R rng, mi tng ring c dng S
lp+k
, k = 1, 2,...,p 1, tin ti
cng mt gii hn khi l .
Gi s chui (ca chng ta) (ban u) hi t. Khi theo 3.4.34,
lim
n
np nq
np +nq
=
p q
p +q
= 0,
iu ny chng t p = q.

n

C
h
i
304 Ch-ng 3. Chui s thc
3.4.37. Ta nhn thy nu iu kin (i)-(iii) tho mn th vi mi dy hi t
{a
n
}, dy chuyn v {b
n
} -c xc nh. Vic chng minh c nhiu cch nh-
trong li gii bi ton 2.3.1 v 2.3.26.
3.5 Tiu chun Dirichlet v tiu chun Abel
3.5.1.
(a) V
sin
2
n
n
=
1
2n
(1 cos(2n)), ta xt cc chui

n=1
(1)
n
1
n
v

n=1
(1)
n
1
n
cos(2n).
Theo tiu chun Leibniz, chui th nht hi t. Chui th hai cng hi
t theo tiu chun Dirichlet (xem [12], trang 105). Tht vy, t cng thc
c th -c chng minh bng qui np sau:
n

k=1
cos ka =
sin
na
2
cos
(n+1)a
2
sin
a
2
vi a = 2l, l Z, (1)
ta nhn -c

k=1
(1)
k
cos(2k)

k=1
cos(( 2)k)

sin
(2)n
2
cos
(n+1)(2)
2
cos 1

1
cos 1
.
V dy cc tng ring ca chui

n=1
(1)
n
cos(2n) l b chn. Hn na
dy
_
1
n
_
n iu tin ti 0. Nh- vy chui

n=1
(1)
n 1
n
cos(2n) hi t.
(b) Dy
a
n
=
1 +
1
2
+... +
1
n
n
cc gi tr trung bnh ca {
1
n
} hi t ti 0 (xem 2.3.2). D dng kim tra
rng dy {a
n
} n iu gim. Theo cng thc c th chng minh bng

n

C
h
i
3.5. Ti u chun Dirichle t v ti u chun Abe l 305
qui np sau:
n

k=1
sinka =
sin
na
2
sin
(n+1)a
2
sin
a
2
vi a = 2l, l Z, (2)
ta c

k=1
sink

sin
n
2
sin
n+1
2
sin
1
2

1
sin
1
2
.
Nh- vy chui hi t theo tiu chun Dirichlet.
(c) Ta thy rng
cos
_

n
2
n + 1
_
= cos
_
n
n
n + 1
_
= (1)
n
cos
_


n + 1
_
= (1)
n+1
cos

n + 1
.
nh- vy chui cho c th vit d-i dng

n=2
(1)
n+1
cos

n+1
ln
2
n
.
Cc chui trn hi t theo tiu chun Abel (xem [12], trang 106), v chui

n=2
(1)
n+1 1
ln
2
n
hi t (theo tiu chun Leibnitz) v dy {cos

1+n
} n
iu v b chn.
(d) Ta c
sin
n
4
n
a
+ sin
n
4
=
sin
n
4
n
a
_
1
sin
n
4
n
a
1 +
sin
n
4
n
a
_
.
Chui

n=1
sin
n
4
n
a
, a > 0,
hi t theo tiu chun Dirichlet. Xt chui vi cc phn t d-ng

n=1
sin
2 n
4
n
2
a
1 +
sin
n
4
n
a
.

n

C
h
i
306 Ch-ng 3. Chui s thc
Tn ti cc hng s c
a
v C
a
sao cho
c
a
1
n
2a
<
sin
2 n
4
n
2a
1 +
sin
n
4
n
a
< C
a
1
n
2
a
, n = 4k, k N.
Nh- vy, chui hi t vi a >
1
2
v phn k vi 0 < a
1
2
.
3.5.2. Ta c
N

n=2
sin(n +
1
n
)
lnlnn
=
N

n=2
sin ncos
1
n
lnlnn
+
N

n=2
cos nsin
1
n
lnlnn
Theo cng thc (2) trong bi gii 3.5.1(b) v theo tiu chun Dirichlet ta thy
rng chui

n=2
sinn
lnlnn
hi t. V dy
_
cos
1
n
_
n iu v b chn nn chui
N

n=2
sinncos
1
n
ln lnn
hi t theo tiu chun Abel. Nh- vy t cng thc (1) trong li
gii 3.5.1(a) v tiu chun Dirichlet suy ra chui
N

n=2
cos n sin
1
n
lnlnn
hi t.
3.5.3. (a) Ta c
2

k=1
sin(k
2
a) sin(ka)

k=1
[cos(k(k 1)a) cos(k(k + 1)a)]

=| 1 cos(n(n + 1)a) | 2
Nh- vy chui cho hi t theo tiu chun Dirichlet.
(b) T-ng t (a), p dng tiu chun Dirichlet.
3.5.4. T cng thc
cos nsin(na)
n
=
1
2
sin(n(a + 1))
n
+
1
2
sin(n(a 1))
n
cc chui u hi t theo tiu chun Dirichlet (s dng cng thc (2) trong bi
gii 3.5.1(b)).
3.5.5. Nu a = k, k Z, th mi s hng ca chui u bng 0. Nu a = k
th theo bt ng thc | sinx | sin
2
x =
1
2
(1 cos 2x), ta c
N

n=1
| sin(na) |
n

1
2
N

n=1
1
n

1
2
N

n=1
cos(2na)
n
.
Do vy trong tr-ng hp ny chui hi t khng tuyt i.

n

C
h
i
3.5. Ti u chun Dirichle t v ti u chun Abe l 307
3.5.6. Tr-c ht gi s rng 0 < a < , v t m =
_

a
_
. Khi , vi n
ln,

k=1
sin(ak)
k

k=1

sin(ak)
k

k=m+1
sin(ak)
k

.
V | sint| < |t| vi t = 0,
m

k=1

sin(ak)
k

<
m

k=1
ka
k
= ma

. ()
Hn na, t (2) trong bi gii 3.5.1(b) v t bt ng thc sin t >
2

t, 0 < t <

2
ta c

k=m+1
sin(ak)
k

<
1
(m+ 1)| sin
a
2
|
<
1
a

a
=

. ()
Kt hp () v () chng ta thy rng bt ng thc tho mn vi a (0, ).
Do hm sin l hm l nn n cng tho mn vi a (, 0). Hn na sink =
0 v hm sin l tun hon nn bt ng thc tho mn vi mi a R.
3.5.7. Chui cho hi t theo tiu chun Abel v chui

n=1
(1)
n 1

n
hi t
v {arctg n} l dy n iu tng v b chn.
3.5.8. Theo tiu chun Abel chui cho hi t. Tht vy, chui

n=1
(1)
n 1
n
hi t v dy {
n

lnx} b chn, gim cht vi x > e v tng cht vi 1 < x < e.


3.5.9.
(a) Tr-c ht ta thy rng chui

n=1
an
bn
hi t theo tiu chun Abel. Hn na
v chui

n=1
a
n
hi t nn dy {r
n
}, vi r
n
=

k=n
a
k
, dn ti 0. V vy,
vi p n,

k=n
a
k
b
k

k=n
r
k
r
k+1
b
k

k=n
r
k
b
k

k=n
r
k+1
b
k

r
n
b
n
+
p

k=n+1
r
k
_
1
b
k

1
b
k1
_

r
p+1
b
p


_
1
b
n
+
1
b
n

1
b
p
+
1
b
p
_
=
2
b
n
,

n

C
h
i
308 Ch-ng 3. Chui s thc
trong
n
= sup
kn
|r
k
|. V vy,

k=n
a
k
b
k

2
n
1
b
n
= o
_
1
b
n
_
.
(b) Xem 3.4.26.
3.5.10. Nhn xt rng

k=0
(k + 1)c
n+k
=

k=1
k
n +k 1
(n +k 1)c
n+k1
.
Theo tiu chun Abel chui

k=0
(k + 1)c
n+k
hi t vi mi n N. t r
n
=
nc
n
+ (n + 1)c
n+1
+..., ta c
t
n
=

k=0
(k + 1)c
n+k
=

k=n
(k n + 1)c
k
=

k=n
kc
k
(n 1)

k=n
1
k
kc
k
= r
n
(n 1)

k=n
1
k
(r
k
r
k+1
)
=
1
n
r
n
+ (n 1)

k=n+1
_
1
k 1

1
k
_
r
k
.
Do
|t
n
|
1
n
|r
n
| + sup
kn+1
|r
k
|(n 1)

k=n+1
_
1
k 1

1
k
_

1
n
|r
n
| + sup
kn+1
|r
k
|
n 1
n
.
Kt hp vi lim
n
r
n
= 0 ta c lim
n
t
n
= 0.
3.5.11. Tnh tng tng phn,
S
n
=
n

i=1
a
i
b
k
i
=
n1

i=1
A
i
(b
k
i
b
k
i+1
) +A
n
b
k
n
,

n

C
h
i
3.5. Ti u chun Dirichle t v ti u chun Abe l 309
vi A
n
l tng ring th n ca chui

n=1
a
n
. Vi > 0 cho tr-c tn ti n
0
sao
cho |b
i
| < vi i n
0
. Nh- vy, nu m > n n
0
v |A
n
| L, th
|S
m
S
n
| =

m1

i=n
A
i
(b
k
i
b
k
i+1
) A
n
b
k
n
+A
m
b
k
m

m1

i=n
|A
i
||b
k
i
b
k
i+1
| +|A
n
b
k
n
| +|A
m
b
k
m
|
L
_
m1

i=n
|b
i
b
i+1
||b
k1
i
+b
k2
i
b
i+1
+... +b
k1
i+1
| +|b
k
n
| +|b
k
m
|
_
= L
_
k
k1
m1

i=n
|b
i
b
i+1
| + 2
k
_
.
V vy chui

n=1
a
n
b
n
hi t theo tiu chun Cauchy.
3.5.12. Tnh tng tng phn
S
n
=
n

i=1
a
i
b
i
=
n1

i=1
A
i
(b
i
b
i+1
) +A
n
b
n
, ()
Trong A
n
l tng ring th n ca

n=1
a
n
. V chui

n=1
(b
n
b
n+1
)
hi t tuyt i v dy {A
n
} b chn nn chui

n=1
A
n
(b
n
b
n+1
) hi t
tuyt i. S hi t ca chui

n=1
(b
n
b
n+1
) chng t rng lim
n
b
n
tn ti v
(b
1
b
2
) + (b
2
b
3
) +... + (b
n1
b
n
) = b
1
b
n
. Do lim
n
A
n
b
n
cng tn
ti v theo () ta c chui

n=1
a
n
b
n
hi t.
3.5.13. Vi 0 x < 1 dy {x
n
} n iu gim v b chn v vy c th p
dng tiu chun Abel. Vi 1 < x < 0 c hai dy {x
2n
} v {x
2n1
} l n
iu v b chn. V vy,

n=1
a
2n
x
2n
v

n=1
a
2n1
x
2n1
hi t. S hi t ca hai
chui ny xut pht t ng thc

n=1
a
n
x
n
=

n=1
a
2n
x
2n
+

n=1
a
2n1
x
2n1

n

C
h
i
310 Ch-ng 3. Chui s thc
3.5.14. Ta thy nu x > x
0
th

n=1
a
n
n
x
=

n=1
a
n
n
x
0
.
1
n
xx
0
.
Nh- vy ta c th p dng tiu chun Abel.
3.5.15. Ta c

n=1
n!a
n
x(x + 1)...(x +n)
=

n=1
a
n
n
x

n!n
x
x(x + 1)...(x +n)
Ch rng vi n ln tt c cc s
n!n
x
x(x+1)...(x+n)
cng du. Ta s ch ra rng
chng to nn mt dy n iu. Nhn xt rng t l ca cc phn t th n+1
v n l
(n + 1)(
n+1
n
)
x
x +n + 1
=
e
(x+1) ln(1+
1
n
)
1 +
x+1
n
.
t R
n
= e
(x+1) ln(1+
1
n
)
1
x+1
n
. T kt qu trong 2.5.7 ta thy rng
R
n
= (x + 1)
_
ln
_
1 +
1
n
_

1
n
_
+
1
2!
(x + 1)
2
ln
2
_
1 +
1
n
_
+
1
3!
(x + 1)
3
ln
3
_
1 +
1
n
_
+...
=
1
n
2
_

x + 1
2
+
1
2
(x + 1)
2
+O
_
1
n
__
+
1
3!
(x + 1)
3
ln
3
_
1 +
1
n
_
+...,
Trong O(a
n
), biu th biu thc phn d- chia cho a
n
, b chn khi n .
iu ny cho thy rng vi n ln, R
n
d-ng nu x(x + 1) > 0 v m nu
x(x + 1) < 0. Do , vi mi n ln t l ca hai phn t lin tip cu dy
_
n!n
x
x(x+1)...(x+n)
_
cng ln hn 1 hoc nh hn 1. Chng ta s ch ra rng dy
ny hi t vi x = 0, 1, 2, .... Mun vy ta vit
n!n
x
x(x + 1)...(x +n)
=
1
x
n
x +n
n1

k=1
_
1 +
1
k
_
x
1 +
x
k
.
Tr-c ht gi s x > 1. Vi mi x chng ta c (1 +
1
k
)
x
> 1 +
x
k
. Do ,
ln
n1

k=1
_
1 +
1
k
_
x
1 +
x
k
=
n1

k=1
_
xln
_
1 +
1
k
_
ln
_
1 +
x
k
_
_
,

n

C
h
i
3.5. Ti u chun Dirichle t v ti u chun Abe l 311
trong mi s hng ca tng u d-ng. Hn na,
lim
k
xln
_
1 +
1
k
_
ln
_
1 +
x
k
_
1
k
2
=
x(x 1)
2
.
Dn n kt qu l tn ti gii hn
lim
k
ln
n1

k=1
_
1 +
1
k
_
x
1 +
x
k
do chui

k=1
1
k
2
hi t. Nh- vy dy ang xt hi t vi x > 1.
By gi gi s rng x (0, 1). Khi vi mi x ta c
_
1 +
1
k
_
x
< 1 +
x
k
nn ta c th p dng cc lp lun nh- trn vi dy c cc phn t
ln
n1

k=1
_
1 +
1
k
_
x
1 +
x
k
.
Cui cng, xt tr-ng hp x < 0, x = 1, 2, 3, ... . Chn k
0
l mt s
d-ng sao cho 1 +
x
k
> 0 vi k k
0
. ch ra rng dy sau hi t
n1

k=k
0
_
1 +
1
k
_
x
1 +
x
k
ta ch rng
_
1 +
1
k
_
x
< 1 +
x
k
vi k k
0
v x l nh- trong tr-ng hp x > 1.
3.5.16. Theo tiu chun Abel vi |x| < 1 s hi t ca chui

n=1
a
n
x
n
ch ra
rng

n=1
a
n
x
2n
hi t (xem 3.5.13). V {
1
1x
2n
} l n iu v b chn, t ng
thc

n=1
a
n
x
n
1 x
n
=

n=1
_
a
n
x
n
1
1 x
2n
+a
n
x
2n
1
1 x
2n
_
v tiu chun Abel ta c chui

n=1
a
n
x
n
1x
n
hi t.

n

C
h
i
312 Ch-ng 3. Chui s thc
3.5.17. [20]. Gi s

n=1
b
n
l mt chui hi t c iu kin. t F(x) = 2
x
2
v nh ngha chui mi

n=1
a
n
bng cch t
a
1
= a
2
=
b
1
2
, a
k
=
b
m
F(m) F(m1)
vi F(m1) < k F(m).
Chui ny cng hi t c iu kin. By gi chng ta s ch ra rng mi chui
con c dng
a
k
+a
kl
+a
kl
2 +... ()
hi t. Tr-c ht ch rng vi mt s nguyn d-ng n bt k tn ti duy nht
mt t
m
, t
m
=
_
log
l
F(m)
k
_
, sao cho
kl
tm
F(m) < kl
tm+1
.
Theo nh ngha ca t
m
, bt u t ch s m no chui con () c t
m
t
m1
s hng c dng
bm
F(m)F(m1)
. Nhm cc s hng ny li ta chuyn () thnh
chui
c
0
+

m=n
1
t
m
t
m1
F(m) F(m1)
b
m
,
trong c
0
l mt hng s no . Chui ny hi t theo tiu chun Abel v dy
vi cc phn t
c
m
=
t
m
t
m1
F(m) F(m1)
l n iu gim. Tht vy
c
m
>
(2m1) log
l
2 1
2
m
2
2
(m1)
2
v c
m+1
<
(2m+ 1) log
l
2 + 1
2
(m+1)
2
2
m
2
.
Do vi m ln ta c c
m+1
< c
m
v
lim
m
(2m+ 1) log
l
2 + 1
(2m1) log
l
2 1

2
m
2
2
(m1)
2
2
(m+1)
2
2
m
2
= 0

n

C
h
i
3.6. Tch Cauchy ca cc chui v hn 313
3.6 Tch Cauchy ca cc chui v hn
3.6.1. Gi s chui

n=0
a
n
hi t tuyt i. K hiu A
n
, B
n
v C
n
l cc tng
ring th n t-ng ng ca

n=0
a
n
,

n=0
b
n
v

n=0
c
n
. Khi
C
n
= a
0
b
0
+ (a
0
b
1
+a
1
b
0
) +... + (a
0
b
n
+a
1
b
n1
+... +a
n
b
0
)
= a
0
B
n
+a
1
B
n1
+... +a
n
B
0
.
Nu vit
B = B
n
+r
n
, vi lim
n
r
n
= 0.
Th
C
n
= BA
n
(a
0
r
n
+a
1
r
n1
+... +a
n
r
0
).
By gi ta s ch ra rng
lim
n
(a
0
r
n
+a
1
r
n1
+... +a
n
r
0
) = 0. ()
Mun vy, chn mt gi tr > 0 tu v m, M nh- sau
|r
n
| m vi n 0, M =

n=0
|a
n
|.
Khi tn ti k N v l N sao cho nu n k th |r
n
| <

2M
v nu
n l + 1 th |a
l+1
| +... +|a
n
| <

2M
. Nh- vy vi n l +k chng ta c
|a
0
r
n
+a
1
r
n1
+... +a
n
r
0
|
(|a
0
||r
n
| +... +|a
l
||r
nl
|) + (|a
l+1
||r
nl1
| +... +|a
n
||r
0
|)
< (|a
0
| +|a
1
| +... +|a
l
|)

2M
+ (|a
l+1
| +... +|a
n
|)m
< M

2M
+

2m
m =
T y () -c chng minh.
Theo nhng suy lun trn ta thy rng nu hai chui cho hi t tuyt
i th tch Cauchy ca chng cng hi t tuyt i.
3.6.2.

n

C
h
i
314 Ch-ng 3. Chui s thc
(a) Theo nh l Mertens nu |x| < 1 th tch Cauchy ca chui

n=0
x
n
vi
chnh n s hi t. Hn na,
c
n
= x
n
+xx
n1
+... +x
n
= (n + 1)x
n
.
Do

n=1
nx
n1
=
_
1
1 x
_
2
.
(b)
1
1x

1
1y
.
(c) Chui cho l tch Cauchy ca hai chui

n=1
1
n(n+1)
v

n=1
1
n!
. Tng ca
chui th nht l 1 (xem 3.1.4(b)) v tng ca chui th hai l e 1 (xem
2.5.6). Do tng ca chui cho l e 1 theo nh l Mertens.
3.6.3.
(a) Ta c
c
n
=
n

k=0
2
k
k!

1
2
nk
(n k)!
=
1
n!
n

k=0
_
n
k
_
2
k
1
2
nk
=
1
n!
_
2 +
1
2
_
n
.
Theo 2.5.7 tng ca tch Cauchy l e
5
2
.
(b) Tch Cauchy l chui

n=1
1
3
n+1
n

n=1
(3)
k
k
.
Theo 3.1.32(a) tng ca n l
1
2
ln2.

n

C
h
i
3.6. Tch Cauchy ca cc chui v hn 315
(c) Ta c
c
2n+1
= x
2n+1
2n+1

k=0
(1)
k
(k + 1)(2n + 1 k + 1)
= x
2n+1
_
n

k=0
(1)
k
(k + 1)(2n + 1 k + 1)
+
2n+1

k=n+1
(1)
k
(k + 1)(2n + 1 k + 1)
_
= x
2n+1
_
n

k=0
(1)
k
(k + 1)(2n + 1 k + 1)

=0
(1)
k

(k

+ 1)(2n + 1 k

+ 1)
_
= 0
Hn na, v c
2n+1
= 0 ta c
c
2n
= x
2n
2n

k=0
(1)
2nk
(k + 1)(2n k + 1)
= x
2n
_
2n1

k=0
(1)
k
(k + 1)(2n 1 k + 1)
+
2n1

k=0
(1)
k
(k + 1) + (2n + 1)
_
= x
2n
(0 + (n) + (2n + 1)) = (n + 1)x
2n
.
Cui cng theo 3.6.2(a),

n=0
(n + 1)x
2n
=
1
(1 x
2
)
2
.
3.6.4. Ta c th thy rng chui

n=0
A
n
x
n
l tch Cauchy ca

n=0
x
n
v

n=0
a
n
x
n
, do n hi t vi |x| < 1 vi tng l
1
1x

n=0
a
n
x
n
.
3.6.5. chng minh ng thc cho ta cn bng cc h s ca x
n
trong
cng thc (1 +x)
n
(1 +x)
n
= (1 +x)
2n
. Nh- vy,
c
n
= (1)
n
x
2n
1
(n!)
2
n

k=0
_
n
k
_
2
= (1)
n
x
2n
1
(n!)
2
_
2n
n
_

n

C
h
i
316 Ch-ng 3. Chui s thc
3.6.6. T quan h
c
n
=
_
1
a
1 3 ... (2n 1)
2 4 ... 2n
+
1
2
1
a + 2
1 3 ... (2n 3)
2 4 ... (2n 2)
+... +
1
a + 2n
1 3 ... (2n 1)
2 4 ... (2n)
_
x
n
,
ta chng minh -c ng thc
1
a
1 3 ... (2n 1)
2 4 ... 2n
+
1
2
1
a + 2
1 3 ... (2n 3)
2 4 ... (2n 2)
+... +
1
a + 2n
1 3 ... (2n 1)
2 4 ... (2n)
=
(a + 1)(a + 3)...(a + (2n 1))
a(a + 2)(a + 4)...(a + 2n)
.
dn n iu chng ta phn tch v phi ca biu din trn thnh
(a + 1)(a + 3)...(a + (2n 1))
a(a + 2)(a + 4)...(a + 2n)
=

0
a
+

1
a + 2
+... +

n
a + 2n
.
Nhn c hai v vi a(a +2)(a +4)...(a +2n) v thay th a = 0, a = 2, ... ,
a = 2k, ... , a = 2n, ta c

0
=
(2n 1)!!
(2n)!!
,

1
=
1(2n 3)!!
2(2n 2)!!
, ...,

k
=
(2k + 1)(2k + 3)...(1)1 3...(2(n k) 1)
2k((2k + 2)...(2) 2 4...(2(n k))
=
(2k 1)!!(2(n k) 1)!!
(2k)!!(2(n k))!!
, ...,

n
=
(2n 1)!!
2n!!
,
v ng thc -c chng minh.
3.6.7. K hiu A
n
, B
n
, C
n
l tng ring th n t-ng ng ca

n=0
a
n
,

n=0
b
n
,
v

n=0
c
n
. D dng kim tra -c rng
C
n
= a
0
B
n
+a
1
B
n1
+... +a
n
B
0
.

n

C
h
i
3.6. Tch Cauchy ca cc chui v hn 317
Do vy
C
0
+C
1
+... +C
n
= A
0
B
n
+A
1
B
n1
+... +A
n
B
0
.
Chia hai v ca ng thc cui cng cho n + 1, s dng 2.3.2 v 2.3.8 ta nhn
-c C = AB.
3.6.8. Gi

n=1
c
n
l tch Cauchy ca

n=1
(1)
n1 1
n
vi chnh n. Khi
c
n
= (1)
n1
_
1
1 n
+
1
2(n 1)
+... +
1
k(n k + 1)
+... +
1
n 1
_
.
V
1
k(n k + 1)
=
1
n + 1
_
1
k
+
1
n k + 1
_
vi k = 1, 2, ..., n
ta c th vit
c
n
= (1)
n1
2
n + 1
_
1 +
1
2
+
1
3
+... +
1
n
_
.
Ta bit rng

n=1
(1)
n1 1
n
= ln2 (xem 3.1.32(a)) v chui

n=1
(1)
n1 2
n+1
(1 +
1
2
+... +
1
n
) hi t (xem 3.4.19). Nh- vy theo kt qu ca
bi ton tr-c,

n=1
(1)
n1
2
n + 1
_
1 +
1
2
+... +
1
n
_
= (ln2)
2
.
3.6.9. Nu

n=1
c
n
l tch Cauchy ca chui

n=1
(1)
n1 1

n
vi chnh n, khi

c
n
= (1)
n1
_
1
1

n
+... +
1

n k + 1
+... +
1

n 1
_
.
V mi s hng trong du ngoc ln hn
1
n
, ta thy rng |c
n
| > 1 vi n > 1. Do
vy chui

n=1
c
n
phn k.
3.6.10. Ta c
c
n
= a
0
b
n
+a
1
b
n1
+... +a
n
b
0
> a
0
b
n
,
do vy nu chui

n=0
b
n
phn k th chui tch Cauchy

n=0
c
n
cng phn k.

n

C
h
i
318 Ch-ng 3. Chui s thc
3.6.11. Khng. Xt hai chui phn k sau
1

n=1
_
3
2
_
n
v 1 +

n=1
_
3
2
_
n1
_
2
n
+
1
2
n+1
_
.
khi
c
n
= a
o
b
n
+b
0
a
n
+
n1

k=1
a
k
b
nk
,
trong a
0
= b
0
= 1, a
n
= (
3
2
)
n
, b
n
= (
3
2
)
n1
(2
n
+
1
2
n+1
). Do
c
n
=
_
3
2
_
n1
_
2
n
+
1
2
n+1
_

_
3
2
_
n

_
3
2
_
n1 n1

k=1
_
2
nk
+
1
2
nk+1
_
=
_
3
4
_
n
3.6.12. Gi A
n
, B
n
, C
n
l tng ting th n ca cc chui

n=0
a
n
,

n=0
b
n
v

n=0
c
n
t-ng ng. Khi ,
C
n
= a
0
B
n
+a
1
B
n1
+... +a
n
B
0
.
suy ra,
n

k=1
a
k
(b
n
+b
n1
+... +b
nk+1
)
= a
1
(B
n
B
n1
) +a
2
(B
n
B
n2
) +... +a
n
(B
n
B
0
)
= B
n
(A
n
a
0
) a
1
B
n1
a
2
B
n2
... a
n
B
0
= B
n
A
n
C
n
.
3.6.13. Gi

n=0
c
n
l tch Cauchy ca chui

n=0
(1)
n
a
n
vi

n=0
(1)
n
b
n
. Khi

c
n
= (1)
n
(a
0
b
n
+a
1
b
n1
+... +a
n
b
0
).
Tr-c ht gi s chui

n=0
c
n
hi t. Khi lim
n
c
n
= 0. V cc dy {a
n
} v
{b
n
} n iu ta c
|c
n
| b
n
(a
0
+... +a
n
) v |c
n
| a
n
(b
0
+... +b
n
)

n

C
h
i
3.6. Tch Cauchy ca cc chui v hn 319
Nh- vy
lim
n
a
n
(b
0
+b
1
+... +b
n
) = 0 v lim
n
b
n
(a
0
+a
1
+... +a
n
) = 0.
Gi s hai ng thc trn tho mn. Khi da vo cc bi ton tr-c ta c
th ch ra rng
lim
n
n

k=1
(1)
k
a
k
((1)
n
b
n
+ (1)
n1
b
n1
+... + (1)
nk+1
b
nk+1
) = 0.
Ch rng
|(1)
n
b
n
+ (1)
n1
b
n1
+... + (1)
nk+1
b
nk+1
| b
nk+1
,
Do vy

k=1
(1)
k
a
k
((1)
n
b
n
+ (1)
n1
b
n1
+... + (1)
nk+1
b
nk+1
)

k=1
a
k
b
nk+1
.
By gi ta s ch ra rng lim
n
n

k=1
a
k
b
nk+1
= 0. Tht vy,
0 <
2n

k=1
a
k
b
2nk+1
(a
1
+... +a
n
)b
n
+ (b
1
+... +b
n
)a
n
,
iu ny chng t rng lim
n
2n

k=1
a
k
b
2nk+1
= 0. T-ng t ta cng c th ch ra
rng lim
n
2n1

k=1
a
k
b
2nk
= 0, t dn ti iu phi chng minh.
3.6.14. Tr-c ht ta nhn thy rng ch cn xt tr-ng hp v khng
v-t qu 1. Ta s ch ra rng
lim
n
1
n

_
1 +
1
2

+... +
1
n

_
= 0
nu v ch nu + > 1. Theo nh l Stolz (xem 2.3.11)
lim
n
1
n

_
1 +
1
2

+... +
1
n

_
= lim
n
1
n

(n

(n 1)

)
= lim
n
1
n
+
(1 (1
1
n
)

)
.

n

C
h
i
320 Ch-ng 3. Chui s thc
Theo qui tc L'Hpital
lim
x+
1
x
+
(1 (1
1
x
)

)
= lim
t0
+
t
+
1 (1 t)

= lim
t0
+
( +)t
+1
(1 t)
1
Do
lim
n
1
n
+
(1 (1
1
n
)

)
=
_

_
0 nu + > 1,
1

nu + = 1,
+ nu + < 1.
T bi ton trn ta suy ra iu phi chng minh.
3.6.15. Gi s chui

n=0
a
n
b
n
hi t. Theo kt qu bi 3.6.13 c th ch ra
rng
lim
n
a
n
(b
0
+b
1
+.. +b
n
) = 0 v lim
n
b
n
(a
0
+a
1
+.. +a
n
) = 0.
Vi mt gi tr > 0 tu cho tr-c tn ti mt gi tr k
0
N sao cho
a
k
0
+1
b
k
0
+1
+a
k
0
+2
b
k
0
+2
+... <

2
. Nh- vy vi n > k
0
,
a
n
(b
1
+... +b
n
) < a
n
(b
1
+... +b
k
0
) +

2
.
Mt khc, v lim
n
a
n
= 0, tn ti mt gi tr n
1
> k
0
sao cho
a
n
<

2(b
0
+... +b
k
0
)
nu n > n
1
.
iu ch ra rng a
n
(b
0
+ ... + b
n
) < vi n > n
1
. T chng ta chng
minh -c lim
n
a
n
(b
0
+... +b
n
) = 0.
By gi gi s tch Cauchy hi t. Theo 3.6.13 lim
n
a
n
(b
0
+ ... + b
n
) = 0
T suy ra vi n ln,
(n + 1)a
n
b
n
< a
n
(b
0
+... +b
n
) < 1
v do
(a
n
b
n
)
1+
<
_
1
n + 1
_
1+
.

n

C
h
i
3.7. Sp x p li chui. Chui k p 321
3.7 Sp xp li chui. Chui kp
3.7.1. Gi S
n
= a
1
+a
2
+... +a
n
l tng ring th n ca

n=1
a
n
. Khi
b
1
+b
2
+... +b
n
= S
mn
vi n 1
V mi dy con ca dy u hi t ti cng mt gii hn, lim
n
S
mn
= lim
n
S
n
.
3.7.2. K hiu {T
n
} l dy ca cc tng ring ca chui -c sp li. Khi
T
3n
=
_
1
1
2
_

1
4
+
_
1
3

1
6
_

1
8
+...
+
_
1
2n 1

1
4n 2
_

1
4n
=
1
2

1
4
+
1
6

1
8
+... +
1
4n 2

1
4n
=
1
2
_
1
1
2
+
1
3

1
4
+... +
1
2n 1

1
2n
_
.
Do theo 3.1.32(a), ta c lim
n
T
3n
=
1
2
ln2. Hin nhin rng lim
n
T
3n
=
lim
n
T
3n+1
= lim
n
T
3n+2
, suy ra
1
1
2

1
4
+
1
3

1
6

1
8
+
1
5
... =
1
2
ln2
3.7.3. Gi {T
n
} l dy tng ring ca chui -c sp li. t f(n) = 1 +
1
2
+
1
3
+
1
4
+... +
1
n1
+
1
n
. Khi
T
+
= 1 +
1
3
+... +
1
2 1

1
2

1
4
...
1
2
= f(2 1)
1
2
f( 1)
1
2
f() = f(2)
1
2
f()
1
2
f().
Chng minh bng qui np
T
n(+)
= f(2n)
1
2
f(n)
1
2
f(n).

n

C
h
i
322 Ch-ng 3. Chui s thc
Nh- ch ra, ng thc ng vi n = 1. Nu n ng vi n N, th
T
(n+1)(+)
= f(2n)
1
2
f(n)
1
2
f(n) +
1
2n + 1
+
1
2n + 3
+... +
1
2(n + 1) 1

1
2n + 2

1
2n + 4
...
1
2(n + 1)
= f(2n)
1
2
f(n)
1
2
f(n) +f(2(n + 1) 1)

1
2
f((n + 1) 1) f(2n) +
1
2
f(n)
1
2
f((n + 1))) +
1
2
f(n)
= f(2(n + 1))
1
2
f((n + 1))
1
2
f((n + 1))
Do , theo 2.1.41
lim
n
T
n(+)
= lim
n
_
f(2n) ln(2n)
1
2
f(n)
_
+
1
2
ln(n)
1
2
f(n) +
1
2
f(n)
_
+ lim
n
_
ln(2n)
1
2
(ln(n) + ln(n))
_
= lim
n
ln
2n
_
n
2

= ln2 +
1
2
ln

.
R rng, vi k = 1, 2, 3, ..., ( +) 1 ta c lim
n
T
n(+)+k
= lim
n
T
n(+)
.
Nh- vy tng ca chui l ln2 +
1
2
ln

.
3.7.4. Kt qu nhn -c chnh l mt tr-ng hp c bit ( = 1 v = 4)
ca bi ton trn.
3.7.5. C th p dng kt qu bi ton 3.7.3 vi = 4 v = 1.
3.7.6. Xt chui
1
1
2
+
1
3
+
1
5

1
4
+
1
7
+
1
9
+
1
11

1
6
+... (1)
nhn -c bng cch sp xp li cc s hng ca chui

n=1
(1)
n1
n
theo n,
n = 1, 2, 3, ..., cc s hng d-ng theo sau mt s hng m. Nhm cc s hng
ca chui (1) theo cch sau:
_
1
1
2
_
+
_
1
3
+
1
5

1
4
_
+
_
1
7
+
1
9
+
1
11

1
6
_
+...,

n

C
h
i
3.7. Sp x p li chui. Chui k p 323
ta c

n=1
_
1
n
2
n + 1
+
1
n
2
n + 3
+... +
1
n
2
+n 1

1
2n
_
. (2)
Gi S
n
v T
n
l tng ring th n ca chui (1) v (2) t-ng ng. Khi
T
n
= S(n+1)n
2
+n
>
n

n=1
_
k
k
2
+k 1

1
2k
_
>
1
4
n

k=1
1
k

n
+.
3.7.7. Nhm cc s hng cu chui ta vit li d-i dng

n=1
_
1

4n 3
+
1

4n 1

2n
_
.
Hn na,
1

4n 3
+
1

4n 1

2n
=
_
(4n 1)2n +
_
(4n 3)2n
_
(4n 3)(4n 1)

4n 3

4n 1

2n
>
2

2n

4n 1

4n 1

2n
>
2

2n

4n

4n 1

2n
=
2

4n 1
.
Do lim
n
S
3n
= +, vi {S
n
} l dy tng ring ca chui sp li. Nh-
vy chui phn k.
3.7.8. Gi s chui

n=1
a
n
hi t tuyt i, gi S
n
l tng ring th n ca
chui v t S = lim
n
S
n
. K hiu {T
n
} l dy cc tng ring ca chui
sp li. T s hi t tuyt i ca

n=1
a
n
, vi > 0 cho tr-c, tn ti n N
sao cho
|a
n+1
| +|a
n+2
| +... < . (1)
Gi m l s ln mi s hng a
1
, a
2
,...,a
n
xut hin trong T
m
. Khi , theo
(1)
|S T
m
| |S S
n
| +|S
n
T
m
| < 2.

n

C
h
i
324 Ch-ng 3. Chui s thc
3.7.9. [4] Tr-c ht gi s rng l > 0 v t n = d +u, d > u, sau sp
li chui

n=1
(1)
n1
f(n). Tng ring th n ca chui mi l
T
n
= T
d+u
= (f(1) f(2) +f(3) ... f(2u))
+ (f(2u + 1) +f(2u + 3) +... +f(2d 1)).
Tng ny c cc s hng m cha u v mi s hng cn li cha d trong i s
l d-ng. Tng ca nhm th hai cha d u s hng, v nh- vy tng ny
s nm trong khong (d u)f(2u) v (d u)f(2d). V tng trong ngoc th
nht hi t ti S khi u , s thay i trong tng bng vi gii hn ca tng
trong ngoc th hai. t (u) = d u. Khi
(u)f(2d) < f(2u + 1) +f(2u + 3) +... +f(2d 1) < (u)f(2u), (1)
v s n iu ca dy {nf(n)} ch ra rng
u
u +(u)
<
f(2u + 2(u))
f(2u)
< 1. (2)
Chn (u) sao cho
lim
u
(u)f(2u) = l. (3)
(v d c th chn (u) = l
_
1
f(2u)
_
) khi lim
u
(u)
u
= 0 v
l = lim
u
1
2
(u)
u
2uf(2u) v lim
u
2uf(2u) = +.
Nh- vy (2) ch ra rng lim
u
f(2u+2(u))
f(2u)
= 1. T (1) v s dng nguyn l kp
ta suy ra
lim
u
(f(2u + 1) +f(2u + 3) +... +f(2d 1)) = l.
Nh- vy ta chng minh -c rng lim
u
T
2u+(u)
= S +l.
By gi ch rng nu 2u +(u) < k < 2(u + 1) +(u + 1) th
0 T
k
T
2u+(u)
+f(2u + 2) T
2u+2+(u+1)
T
2u+(u)
+f(2u + 2).
V f(2u + 2) 0 khi u ta thy rng lim
k
T
k
= S +l.
Trong tr-ng hp l < 0 ta c th i vai tr ca d v u v thc hin t-ng
t.

n

C
h
i
3.7. Sp x p li chui. Chui k p 325
3.7.10. Cho > 0, t mt ch s n
0
no ta c
g
n
< f(n) <
g +
n
. (1)
Xt chui nhn -c sau khi thay i thnh phn th n c dng (xem li gii
bi 3.7.9)
T
n
= T
d+u
= (f(1) f(2) +f(3) ... f(2u)) +
+ (f(2u + 1) +f(2u + 3) +... +f(2d 1)).
Hn na gi s rng s d cc s hng d-ng tho mn lim
n
d
u
= k. Khi
trong tr-ng hp d > u,
1
2u + 1
+
1
2u + 3
+... +
1
2d 1
=
=
_
1 +
1
2
+... +
1
2u + 1
+... +
1
2d 1
ln(2d 1)
_

_
1 +
1
2
+... +
1
2u 1
ln(2u 1)
_

_
1
2u
+
1
2u + 2
+... +
1
2d 2
_
ln
2u 1
2d 1
.
Theo bi 2.1.41, hai biu thc trong ngoc n tin n hng s Euler . Ta
ch ra rng (bi 2.5.8 (a)) thnh phn th ba tin n
1
2
lnk. V vy
lim
n
_
1
2u + 1
+
1
2u + 3
+... +
1
2d 1
_
=
1
2
lnk.
T suy ra (1) cho ta
lim
n
(f(2u + 1) +f(2u + 3) +... +f(2d 1)) =
1
2
g lnk.
V vy tng S s sai khc mt l-ng
1
2
g lnk so vi chui cho. Lp lun t-ng
t cho tr-ng hp d < u.
3.7.11. S dng kt qu ca bi 3.7.9 vi (u) = l[(2u)
p
].
3.7.12. S dng kt qu ca bi 3.7.10 vi lim
n
d
u
= .

n

C
h
i
326 Ch-ng 3. Chui s thc
3.7.13. Khng. Tht vy, xt chui

k=1
a
n
k
l mt chui nhn -c t vic
i ch chui phn k

k=1
a
n
. Tnh n iu ca dy {a
n
} chng t rng
a
n
1
+a
n
2
+... +a
nm
a
1
+a
2
+... +a
m
.
V vy khng th tng phn k ca chui.
3.7.14. [20] Chn mt dy con {a
rn
} ca dy {a
n
} sao cho
a
rn
< min
_
2
n
, Q
n
Q
n1
_
, n = 1, 2, ..., vi Q
0
= 0.
Khi
a
r
1
+a
r
2
+... +a
rn
Q
n
v a
r
1
+a
r
2
+... +a
rn
< 1.
Do v lim
n
Q
n
= + nn dy {Q
n
(a
r
1
+ a
r
2
+ ... + a
rn
)} cng phn
k v v hn. By gi ta cng cc s hng khng xut hin trong dy con {a
rn
}
ca chui

n
a
n
vo tng a
r
1
+a
r
2
+... +a
rn
theo cch d-i y sao cho
a
1
+a
2
+... +a
r
1
1
+a
r
1
+a
r
1
+1
+... +a
i
+a
r
k
+... +a
rn
Q
n
,
trong a
i
l s hng thch hp cui cng. iu c ngha l nu ta thm mt
s hng khng xut hin trong dy con {a
rn
} c ch s ln hn i th bt ng
thc trn s khng cn ng na.
3.7.15. [W.Sierpi nski, Bull. Intern. Acad. Sci. Cracovie, 1911, 149-158] Hai
chui

n=1
p
n
v

n=1
q
n
l hai chui con b ca chui hi t c iu kin

n
a
n
cha mi s hng khng m v m lin tip t-ng ng. l s thc cho tr-c.
V chui

n
p
n
phn k ti + nn tn ti mt ch s k
1
sao cho
p
1
+p
2
+... +p
k
1
> .
Ta chn -c mt ch s n
1
nh nht sao cho
p
1
+p
2
+... +p
k
1
+q
1
+q
2
+... +q
n
1
< .
Xt mt ch s k
2
sao cho
p
1
+p
2
+... +p
k
1
+q
1
+q
2
+... +q
n
1
+p
k
1
+1
+p
k
1
+2
+... +p
k
2
> .

n

C
h
i
3.7. Sp x p li chui. Chui k p 327
v mt ch s n
2
nh nht sao cho
p
1
+p
2
+... +p
k
1
+q
1
+q
2
+... +q
n
1
+p
k
1
+1
+... +p
k
2
+q
n
1
+1
+... +q
n
2
< .
Tip tc qu trnh trn ta tm -c hai dy k
1
, k
2
, ... v n
1
, n
2
, ... v php i
ch t-ng ng i vi chui ang xt. t S
n
l tng ring th n ca chui.
Khi
S
n
vi n < k
1
nh-ng S
n
vi k
1
n < k
1
+n
1
.
Hn na
S
n
vi k
m
+n
m
n < k
m+1
+n
m
,
S
n
vi k
m+1
+n
m
n < k
m+1
+n
m+1
,
vi m = 1, 2, .... T cch xc nh dy {k
m
} v {n
m
} ta c
|S
k
m+1
1+nm
| < p
k
m+1
,
|S
k
m+1
+nm
| < p
k
m+1
,
|S
k
m+1
+nm+l
| < p
k
m+1
, vi l = 1, 2, ..., n
m+1
n
m
1,
|S
k
m+1
+n
m+1
| < |q
n
m+1
|,
|S
k
m+1
+n
m+1
+l
| < |q
n
m+1
|, vi l = 1, 2, ..., k
m+2
k
m+1
1,
V lim
n
p
n
= lim
n
q
n
= 0 suy ra lim
n
S
n
= .
3.7.16. K hiu {S
m
}, {T
m
} l cc dy tng ring ca

n=1
a
n
v

k=1
a
n
k
t-ng
ng. V {n
k
k} l dy b chn nn tn ti l N sao cho k l n
k
k +l
vi mi k N. Nu m > l v n
k
m l th k l n
k
m l. V vy
k m, v t suy ra
{1, 2, ..., ml} {n
1
, n
2
, ..., n
m
}. (1)
Tht vy, nu s l mt s nguyn d-ng khng ln hn m l th tn ti duy
nht mt s nguyn d-ng k N sao cho s = n
k
. iu cho ta khng nh
k mhay ni cch khc s {n
1
, n
2
, ..., n
m
}. Theo (1) ta thy rng cc phn
t a
n
, n = 1, 2, ..., ml u xut hin trong T
m
. Mt khc, nu k m th
n
k
k + l m + l, v ta c th suy ra rng mi s hng a
n
1
, a
n
2
, ..., a
nm
u xut hin trong S
m+l
. V vy
|S
m
T
m
| |a
ml+1
| +... +|a
m+l
|, vi m > l.

n

C
h
i
328 Ch-ng 3. Chui s thc
Suy ra lim
m
S
m
= lim
m
T
m
.
Nu dy {n
k
k} khng b chn th cc bi tp 3.7.2 - 3.7.6 cho ta cc v
d v vic khi i ch mt chui c th lm cho n phn k hoc thay i tng
ca n. Ta -a ra mt v d khng lm thay i tng ca mt chui khi i
ch cc phn t ca chng. Ta xy dng dy {n
k
} bng cch i ch cc v
tr ng vi cc ch s nguyn d-ng
n(n+1)
2
v
n(n+3)
2
, cn li gi nguyn. V
n(n+3)
2

n(n+1)
2
= n ta c dy {n
k
k} khng b chn. Hn na
T
m
S
m
=
_
0, nu m =
n(n+3)
2
,
a
n(n+3)/2
a
n(n+1)/2
, nu
n(n+1)
2
m <
n(n+3)
2
.
3.7.17. [R. P. Agnew, Proc. Amer. Math. Soc. 6(1995), 563-564] Ta s dng
nh Toeplitz (xem 3.4.37). t S
m
=
m

k=1
a
k
v T
m
=
m

k=1
a
n
k
. Gi thit
rng m ln sao cho 1 {n
1
, n
2
, ..., n
m
}, v sp xp cc phn t ca tp
{n
1
, n
2
, ..., n
m
} to thnh mt dy tng
1, 2, 3, ...,
0,m
,
1,m
+ 1,
1,m
+ 2, ...,
1,m
,

2,m
+ 1,
2,m
+ 2, ...,
2,m
, ...,
jm,m
+ 1,
jm,m
+ 2, ...,
jm,m
,
trong
0 <
0,m
<
1,m
<
1,m
, <
2,m
< ..., <
jm,m
.
Do tng ring T
m
ca dy va nhn -c s -c vit l
T
m
= S

0
,m
+ (S

1
,m
S

1
,m
) +... + (S

jm
,m
S

jm
,m
).
Suy ra T
m
=
m

k=1
c
m,k
S
k
, trong
c
m,k
=
_

_
1 nu k =
l,m
, l = 0, 1, ..., j
m
,
1 nu k =
l,m
, l = 0, 1, ..., j
m
,
0 tri li.
V lim
n

0,m
= +, lim
n
c
m,k
= 0 vi mi N. Hn na

k=1
c
m,k
= 1 vi
m = 1, 2, ..., v

k=1
|c
m,k
| = 2B
m
1, trong B
m
l s cac khi s nguyn
d-ng khng giao nhau trong tp {n
1
, n
2
, ..., n
m
}. Cui cng s dng nh l
Toeplitz ta c lim
n
T
m
= lim
n
S
m
khi v ch khi tn ti s N sao cho B
m
N
vi mi m N.

n

C
h
i
3.7. Sp x p li chui. Chui k p 329
3.7.18. Gi s rng chui

k=1
c
n
l hi t tuyt i v c tng l S. Khi
vi mi > 0 tn ti k
0
sao cho
|c
1
+c
2
+... +c
k
0
S| <

2
v

l=k
0
+1
|c
l
| <

2
.
Vi m, n ln sao cho vi mi l {1, 2, ..., k
0
} tn ti i, k, i {1, 2, ..., m},
k {1, 2, ..., n}, sao cho c
l
= a
i,k
. Th th
|S
m,n
S| < |c
1
+c
2
+... +c
k
0
S| +

l=k
0
+1
|c
l
| < .
V vy chui kp S hi t. T-ng t nh- vy ta chng minh -c tnh hi t
tuyt i ca chui kp.
3.7.19. t
S

i,k=1
|a
i,k
|, T

n=1
|c
n
|,
S

m,n
=
m

i=1
n

k=1
|a
i,k
|, T

n
=
n

k=1
|c
n
|,
Cho tr-c cc s thc d-ng v s nguyn d-ng l N bt k. Xt cc ch s
m, n ln sao cho mi s hng ca T

l
u thuc S

m,n
v |S

m,n
| < .
Th th T

l
S

m,n
< S

+ , c ngha l chui

n=1
c
n
hi t tuyt i. t
tng ring th n v tng ca chui

n=1
c
n
ln l-t l T
n
v T. chng minh
ng thc

i,k=1
a
i,k
=

n=1
c
n
,
ta c nh > 0 v xt l ln sao cho
|T

l
T

| <

2
v |T
l
T| <

2
.
Nu S
m,n
=
m

i=1
n

k=1
a
i,k
v nu n, m ln sao cho moi s hng ca T
l
u
nm trong S
m,n
th ta c
|S
m,n
T| < |T T
l
| +|T

l
| < .

n

C
h
i
330 Ch-ng 3. Chui s thc
3.7.20. y l h qu ca hai bi tp trn.
3.7.21. Khng mt tng qut ta c th gi s rng chui lp

i=1
_

k=1
|a
i,k
|
_
hi t. t

k=1
|a
i,k
| =
i
v

i=1

i
= , th th mi chui

k=1
a
i,k
, i = 1, 2, ...
u hi t v

k=1
a
i,k

= |S
i
|
i
. T kt qu ny v s hi t ca chui

i=1

i
ta suy ra s hi t tuyt i ca chui

i=1
S
i
, v suy ra

i=1
S
i
=

i=1
_

k=1
a
i,k
_
.
3.7.22. t

i,k=1
a
i,k
= S,

i,k=1
|a
i,k
| = S

v t S
m,n
=
m

i=1
n

k=1
a
i,k
v
S

m,n
=
m

i=1
n

k=1
|a
i,k
|. Ta s ch ra rng chui lp

i=1
_

k=1
|a
i,k
|
_
hi t v S

. Tht vy, vi mi > 0, tn ti s n


0
sao cho S

< S

m,N
< S

vi m, n > n
0
. C nh m, dy {S

m,n
} s tng n iu v b chn, v vy n
hi t v lim
n
S

m,n
= S

m
. T bi tp trn ta c mt chui lp hi t tuyt i
th hi t, v vy chui

k=1
a
i,k
hi t vi mi i ti S
i
, tc l vi mi > 0 tn
ti m
1
sao cho
|(S
1
+S
2
+... +S
m
) S| < vi m > m
1
.
T gi thit rng chui kp hi t tuyt i ta c
|S
m,n
S| <

2
v |S

m,n
S

| <

2
vi m, n > m
1
.
Suy ra vi m > m
1
ta c
|(S
1
+S
2
+... +S
m
) S| =

i=1

k=1
a
i,k
S

|S
m,n
S| +

i=1

k=n+1
a
i,k

|S
m,n
S| +|S

m,n
S

| < .
Chng minh t-ng t i vi cc tr-ng hp cn li.

n

C
h
i
3.7. Sp x p li chui. Chui k p 331
3.7.23. Ch rng chui

n=1
(a
n,1
+a
n1,2
+a
n2,3
+... +a
1,n
) l mt chui
kp -c sp. Nu mt trong cc chui

i,k=1
|a
i,k
|,

n=1
(|a
n,1
| +|a
n1,2
| +|a
n2,3
| +... +|a
1,n
|)
hi t th ta nhn -c iu phi chng minh t bi tp 3.7.18, 3.7.19 v 3.7.22.
V th ta ch cn chng minh rng t tnh hi t tuyt i ca mt trong cc
chui lp ta s suy ra tnh hi t tuyt i ca chui kp -c sp. Ta gi thit
rng chui

i=1

k=1
|a
i,k
| hi t ti S

. Xt dy {c
n
} gm cc phn t ca ma
trn v hn (a
i,k
)
i,k=1,2,...
. Th th vi l N tn ti hai s m, n ln sao cho
|c
1
| +|c
2
| +... +|c
l
|
m

i=1
n

k=1
|a
i,k
| S

.
V vy chui

i=1
c
n
hi t tuyt i, tc l chui kp hi t tuyt i (xem bi
tp 3.7.18).
3.7.24. V
m

k=0
_
m
k
_
= 2
m
ta c
m

n,k=0
k+n=m
1
n!k!
=
2
m
m!
. V vy
m

n,k=0
k+n=m
1
n!k!(n +k + 1)
=
2
m
(m+ 1)!
.
S dng kt qu bi 3.7.23 ta suy ra

n,k=0
1
n!k!(n +k + 1)
=

m=0
m

n,k=0
n+k=m
1
n!k!(n +k + 1)
=

m=0
2
m
(m+ 1)!
=
1
2

m=0
2
m+1
(m+ 1)!
=
1
2
(e
2
1),
ng thc cui cng suy ra t bi 2.5.7.

n

C
h
i
332 Ch-ng 3. Chui s thc
3.7.25. T bi tp 3.7.23 ta c

n,k=1
1
nk(n +k + 2)
=

n=0
1
n

k=1
1
n + 2
_
1
k

1
n +k + 2
_
=

n=1
1
n(n + 2)
_
1 +
1
2
+
1
3
+... +
1
n + 2
_
=
1
2

n=1
_
1
n

1
n(n + 2)
__
1 +
1
2
+
1
3
+... +
1
n + 2
_
=
1
2
_
1 +
1
2
+
1
3
+
1
2
_
1 +
1
2
+
1
3
+
1
4
_
+
1
3
_
1
4
+
1
5
_
+
1
4
_
1
5
+
1
6
_
+...
_
=
7
4
.
3.7.26. T bi 3.7.23 ta c

n,k=0
n!k!
(n +k + 2)!
=

k=0
k!
k + 1

n=0
_
n!
(n +k + 1)!

(n + 1)!
(n +k + 2)!
_
=

k=0
k!
k + 1
0!
(k + 1)!
=

k=0
1
(k + 1)
2
.
T bi 3.1.28 ta suy ra iu phi chng minh.
3.7.27. Ch rng tng ca mi hng trong ma trn l hu hn. Tht vy
tng ca hng u tin l x, hng th hai l x(1x) , hng th ba l x(1x)
2
,
v.v. Hn na
x +x(1 x) +x(1 x)
2
+... = 1.
Mt khc tng cc ct ch c th bng 1 hoc -1 lin tip, v th chui lp s hi
t. Theo bi 3.7.23 ta suy ra chui lp khng th hi t tuyt i.
3.7.28.
(a) T s hi t tuyt i ca chui

i=0
x
i
v

k=0
y
k
ta suy ra -c s hi
t tuyt i ca chui lp

i=0
_

k=0
x
i
y
k
_
, bi v

i=0
_

k=0
|x
i
y
k
|
_
=

i=0
|x
i
|
_
1
1|y|
_
=
1
(1|x|)(1|y|)
. T suy ra s hi t tuyt i ca
chui kp cho.

n

C
h
i
3.7. Sp x p li chui. Chui k p 333
(b) T chui lp ta c th suy ra chui ca ta hi t khi v ch khi >
1, > 1.
(c) Xt cc s hng sao cho i +k = n ta c

i,k=1
1
(i +k)
p
=

n=2
(n 1)
1
n
p
.
V vy chui kp hi t khi v ch khi p > 2 v phn k trong tr-ng hp
p 2.
3.7.29.
(a) Ta ch cn tnh tng ca chui lp. Ta c

i=2
_

k=2
1
(p +i)
k
_
=

i=2
_
1
p +i

1
p +i 1
_
=
1
p + 1
.
(b) T-ng t (a) ta i tnh tng ca chui lp

k=1
_

i=2
1
(2k)
i
_
=

k=2
1
(2k)(2k 1)
=

k=2
_
1
2k 1

1
2k
_
=

k=2
(1)
k1
1
k
= ln2.
ng thc cn li -c suy ra t 3.1.32(a).
(c) T-ng t (b) ta c

i=1
_

k=1
1
(4i 1)
2k
_
=

i=1
1
(4i)(4i 2)
=
1
4
ln2.
3.7.30. V S
m,n
=

i=1

k=1
a
i,k
= b
m,n
ta thy rng
a
1,1
= S
1,1
= b
1,1
,
a
1,n
= S
1,n
S
1,n1
= b
1,n
b
1,n1
, n > 1,
a
m,1
= S
m,1
S
m1,1
= b
m1,1
b
m,1
, m > 1.
T-ng t vi m, n > 1 ta c
a
m,n
= S
m,n
S
m1,n
(S
m,n1
S
m1,n1
)
= b
m,n
b
m1,n
(b
m,n1
b
m1,n1
), n, m > 1.

n

C
h
i
334 Ch-ng 3. Chui s thc
3.7.31. Ta c S
m,n
= (1)
m+n
(
1
2
m
+
1
2
n
). V vy vi > 0 tn ti n
0
sao
cho vi m, n > n
0
ta c |S
m,n
| < , tc l chui kp s hi t v 0. Tuy nhin
c hai chui lp u phn k. Tht vy, ta c
n

k=1
ai, k = S
i,n
S
i1,n
= (1)
i+n
3
2
i
+ (1)
i+n
1
2
n1
,
cho ta thy rng mi chui

k=1
a
i,k
, i N u phn k.
3.7.32. Ta c

i=1
_

k=1
|x|
i
k
_
=

i=1
|x|
i
1 |x|
i
.
Ta c th th rng v phi ca ng thc trn l hi t, iu ny c ngha chui
lp s hi t tuyt i, do d theo 3.7.23 th

i,k=1
x
ik
=

k=1
x
k
1 x
k
.
Chn cc cp (i, k) c cng tch ik ta c

i,k=1
x
ik
=

n=1
(n)x
n
,
do s cc bi ca n bng s cc cp (i, k) vi ik = n. Hn na vi n = 2, 3, ...
ta c
S
n,n
S
n1,n1
=
= x
n
+x
2n
+... +x
(n1)n
+x
n
2
+x
n(n1)
+... +x
n2
+x
n
= 2
x
n
x
n
2
1 x
n
+x
n
2
.
Hin nhin ta c S
1,1
= x = 2
xx
1x
+x. Do ta tnh -c
S
n,n
= (S
n,n
S
n1,n1
) + (S
n1,n1
S
n2,n2
) +... +S
1,1
,
ta thy rng

k=1
x
k
1 x
k
= 2

n=1
x
n
x
n
2
1 x
n
+

n=1
x
n
2
.

n

C
h
i
3.7. Sp x p li chui. Chui k p 335
3.7.33. T bi tp trn ta ch ra -c rng chui lp hi t tuyt i, do
ng thc u tin -c suy ra trc tip t 3.7.23. chng minh ng thc
cn li ta xt mt s sp xp li ca chui kp -c trnh by trong bi 3.7.32.
3.7.34.
(a) T 3.7.23 ta c

p=2
S
p
=

n=2
1
2
n
+

n=2
1
3
n
+... =

k=2
1
k(k 1)
= 1.
(b) Cng nh- cu (a) ta c

p=2
(1)
p
S
p
=

n=2
1
k(k + 1)
=
1
2
.
3.7.35. t B l tp cc s t nhin khng phi l lu tha ca mt s no
th th
A = {k
n
: n N, n 2, k B}.
V
1
n1
=

j=1
1
n
j
, n 2, s dng kt qu ca bi 3.7.23 v 3.7.34 ta c

nA
1
n 1
=

nA

j=1
1
n
j
=

kB

n=2

j=1
1
k
nj
=

kB

j=1

n=2
1
k
nj
=

k=2

n=2
1
k
n
= 1.
3.7.36. [G. T. Williams, Amer. Math. Monthly, 60 (1953), 19-25] V tri ca
ng thc bng
lim
N
N

j=1
N

k=1
_
1
k
2
1
j
2n2
+
1
k
4
1
j
2n4
+... +
1
k
2n2
1
j
2
_
. ()
Tnh ton ta -c
lim
N
N

j=1
_
_
_
N

k=1
k=j
j
22n
k
22n
k
2
j
2
+ (n 1)
1
j
2n
_
_
_. ()

n

C
h
i
336 Ch-ng 3. Chui s thc
Ch rng
N

j=1
N

k=1
k=j
j
22n
k
22n
k
2
j
2
=
N

j=1
N

k=1
k=j
j
22n
k
2
j
2

N

j=1
N

k=1
k=j
k
22n
k
2
j
2
=
N

j=1
N

k=1
k=j
1
j
2n2
1
k
2
j
2
+
N

j=1
N

k=1
k=j
1
k
2n2
1
j
2
k
2
=
N

j=1
1
j
2n2
N

k=1
k=j
1
k
2
j
2
.
Do
lim
N
N

j=1
_
_
_
N

k=1
k=j
j
22n
k
22n
k
2
j
2
+ (n 1)
1
j
2n
_
_
_
( )
= lim
N
_
_
_
2
N

j=1
1
j
2n2
N

k=1
k=j
1
k
2
j
2
+ (n 1)
N

j=1
1
j
2n
_
_
_
.
By gi nhn xt rng
2j
N

k=1
k=j
1
k
2
j
2
=
N

k=1
k=j
1
k j

N

k=1
k=j
1
k +j
=
j1

k=1
1
k j
+
N

k=j+1
1
k j

N

k=1
1
k +j
+
1
2j
=
j1

k=1
1
k
+
Nj

k=1
1
k

N+j

k=j+1
1
k
+
1
2j
=
N+j

k=1
1
k
+
1
j
+
Nj

k=1
1
k
+
1
2j
=
3
2j

_
1
N j + 1
+
1
N j + 2
+... +
1
N +j
_
.

n

C
h
i
3.7. Sp x p li chui. Chui k p 337
T s dng ( ) ta nhn -c
N

j=1
_
_
_
N

k=1
k=j
j
22n
k
22n
k
2
j
2
+ (n 1)
1
j
2n
_
_
_
=
_
n +
1
2
_
N

j=1
1
j
2n

N

j=1
1
j
2n1
_
1
N j + 1
+... +
1
N +j
_
.
Hn na v 0 <
1
Nj+1
+... +
1
N+j
<
2j
Nj+1
ta thy rng
0 <
N

j=1
1
j
2n1
_
1
N j + 1
+... +
1
N +j
_
< 2
N

j=1
1
j
2n2
1
N j + 1
2
N

j=1
1
j
1
N j + 1
=
2
N + 1
N

j=1
_
1
j
+
1
N j + 1
_
=
4
N + 1
N

j=1
1
j

4
N + 1
( + ln(N + 1)),
trong l hng s Euler (xem 2.1.41). Cui cng theo () ta c
lim
N
N

j=1
N

k=1
_
1
k
2
1
j
2n2
+
1
k
4
1
j
2n4
+... +
1
k
2n2
1
j
2
_
= lim
N
_
n +
1
2
_
N

j=1
1
j
2n
=
_
n +
1
2
_
(2n).
3.7.37. Thay gi tr n = 2 trong bi ton trn ta -c
(2)(2) =
_
2 +
1
2
_
(4).
V (2) =

2
6
(xem 3.1.28(a)), ta -c (xem 3.1.28(b))
(4) =

n=1
1
n
4
=

4
90
.

n

C
h
i
338 Ch-ng 3. Chui s thc
T-ng t khi cho n = 3 ta -c
(6) =

n=1
1
n
6
=

6
945
.
V t-ng t c
(8) =

n=1
1
n
8
=

8
9450
.
3.8 Tch v hn
3.8.1.
(a) Ta c
P
n
=
n

k=2
_
1
1
k
2
_
=
n

k=2
(k 1)(k + 1)
k
2
=
n + 1
2n

n
1
2
.
(b)
n

k=2
(k 1)(k
2
+k + 1
(k + 1)(k
2
k + 1
=
=
n

k=2
(k 1)(k + 1)
2
(k + 1) + 1
(k + 1)(k
2
k + 1
=
2(n
2
+n + 1)
3n(n + 1)

n
2
3
.
(c) Vi x = 0 gi tr ca tch l 1. Nu x = 2
m
(

2
+k), th cos

2
m
= 0 v
sin

2
n
= 0 do
n

k=1
cos
x
2
k
=
n

k=1
1
2
sin
x
2
k1
sin
x
2
n
=
sin x
2
n
sin
x
2
n

n
sinx
x
.
(d) S dng cc cng thc
sinh(2x) = 2 sinh xcosh x v lim
x0
sinhx
x
= 1,
t-ng t cu trn ta c

n=1
cosh
x
2
n
=
_
sinhx
x
nu x = 0,
1 nu x = 0,

n

C
h
i
3.8. Tch v hn 339
(e) Ta c
n

k=0
_
1 +x
2
k
_
=
n

k=0
1 x
2
k+1
1 x
2
k
=
1 x
2
n+1
1 x

n
1
1 x
.
(f)
n

k=1
_
1 +
1
k(k + 2)
_
=
n

k=1
(k + 1)
2
k(k + 2)
=
2(n + 1)
n + 2

n
2.
(g) V
n

k=1
a
(1)
k
k
= a
n

k=1
(1)
k
k
,
s dng tnh lin tc ca hm lu tha v 3.1.32(a) ta suy ra rng

n=1
a
(1)
n
n
= a
ln 2
.
(h)
n

k=1
e
1
k
1 +
1
k
=
e
n

k=1
1
k
n + 1
= e
n

k=1
1
k
ln n

n
n + 1
.
Do s dng bi 2.1.41, ta c

n=1
e
1
n
1 +
1
n
= e

,
trong l hng s Euler.
(i) Ta c
P
n
=
n

k=1
(3k)
2
(3k 1)(3k + 1)
=
n

k=1
(3k)
3
(3k 1)3k(3k + 1)
=
3
3n
(n!)
3
(3n + 1)!
.
S dng cng thc Stirling
n! =
n

2n
_
n
e
_
n
, vi lim
n

n
= 1,
ta nhn -c
lim
n
P
n
=
3
3n
(2)
3/2
n
3n+3/2
e
3n
(2)
1/2
(3n + 1)
3n+1+1/2
e
3n1
= 2e lim
n
_
3n
3n + 1
_
3n
_
n
3n + 1
_
3/2
=
2
3

3
.

n

C
h
i
340 Ch-ng 3. Chui s thc
3.8.2.
(a)
P
2n
=
2n

k=2
_
1 +
(1)
k
k
_
=
3
2

2
3

5
4

4
5
...
_
1 +
1
2n
_
= 1 +
1
2n

n
1,
P
2n1
=
3
2

2
3

5
4

4
5
...
2n 1
2n 2
2n 2
2n 1
= 1.
(b) Ta c
P
n
=
n

k=1
_
1 +
1
k
_
= 2
3
2

4
3
...
n + 1
n
= n + 1
n
,
do tch

n=1
_
1 +
1
n
_
phn k.
(c) Tch

n=1
_
1
1
n
_
phn k, tht vy,
P
n
=
n

k=1
_
1
1
k
_
=
1
2

2
3

3
4
...
n 1
n
=
1
n

n
0.
3.8.3. Tr-c ht ch rng i vi cc s hng khng m a
n
ta c
a
1
+a
2
+... +a
n
(1 +a
1
)(1 +a
2
)...(1 +a
n
). (1)
Hn na s dng bt ng thc e
x
1 +x, x 0 ta -c
(1 +a
1
)(1 +a
2
)...(1 +a
n
) e
a
1
+a
2
+...+an
. (2)
T cc bt ng thc (1) v (2) v tnh lin tc ca hm lu tha ta suy ra s
hi t ca tch

n=1
(1 +a
n
) l t-ng -ng vi s hi t ca chui

n=1
a
n
.
3.8.4. Gi thit rng chui

n=1
a
n
hi t, tc l vi N ln th

n=N
a
n

1
2
.
T 1.2.1 ta c

k=N
(1 a
k
) 1
i

k=N
a
k
>
1
2
.

n

C
h
i
3.8. Tch v hn 341
V P
n
=
n

k=1
(1 a
k
) = P
N1
n

k=N
(1 a
k
) nn c th suy ra rng dy
_
P
N
P
N1
_
n iu gim b chn d-i, tc l n hi t, gi thit rng chui hi t v
P, th th P [
1
2
, 1]. T suy ra lim
n
P
n
= P
N1
P = 0. chng minh
iu cn li ta gi thit

n=1
a
n
phn k. Nu dy {a
n
} khng hi t v 0 th
dy {1 a
n
} khng hi t v 1 v khi iu kin cn cho tnh hi t ca tch

n=1
(1a
n
) khng -c tho mn. V th ta gi thit rn lim
n
a
n
= 0, v ng
thi 0 a
n
< 1 vi n bt u t mt ch s N no . T cng thc (xem 2.5.7)
e
x
= 1 x +
_
x
2
2!

x
3
3!
_
+
_
x
4
4!

x
5
5!
_
+...,
ta -c 1 x e
x
vi 0 x < 1, v cc s hng trong cc du ngoc n l
d-ng. T suy ra
0
n

k=N
(1 a
k
) e

k=N
a
k
, n N,
v -ng nhin lim
n
n

k=N
(1 a
k
) = 0. T ta kt lun rng

n=1
(1 a
n
)
phn k.
3.8.5. Ch rng
2n

k=1
(1 +a
k
) =
n

k=1
(1 +a
2k1
)(1 +a
2k
)
=
n

k=1
_
1 +
1

k
+
1
k
__
1
1

k
_
=
n

k=1
_
1
1
k

k
_
.
Do s dng 3.8.4 ta suy ra tch hi t.
3.8.6.
(a) V cos
1
n
= 1
_
1 cos
1
n
_
v 1 = 1 cos
1
n
> 0, n N, s dng kt
qu ca bi 3.8.4 suy ra s hi t ca tch s -c suy ra t s hi t ca
chui

n=1
_
1 cos
1
n
_
(xem 3.2.1.(e)).
(b) T-ng t cu (a) ta c s hi t ca tch -c suy ra t s hi t ca chui

n=1
_
1 nsin
1
n
_
(xem 3.2.1.(d)).

n

C
h
i
342 Ch-ng 3. Chui s thc
(c) Ta c
tan
_

4
+
1
n
_
=
1 + tan
1
n
1 tan
1
n
= 1 +
2 tan
1
n
1 tan
1
n
.
V
2tan
1
n
1tan
1
n
> 0 vi n 2 v
lim
n
2 tan
1
n
1tan
1
n
1
n
= 2,
theo cu 3.8.3 ta suy ra tch phn k.
(d) Ch rng lim
n
1nln(1+
1
n
)
1
n
=
1
2
v s dng kt qu ca bi 3.8.4 ta suy
ra tch ang xt l hi t.
(e) S phn k ca tch -c suy ra t tnh phn k ca chui

n=1
(
n

n 1)
(xem 3.2.5(a)).
(f) V lim
n
n
2

n = 1, s dng kt qu bi 2.5.5 ta suy ra rng lim


n
1
n
2
lnn
n
2
n1
=
1. Do s hi t ca tch -c suy t s hi t ca chui

n=2
ln n
n
2
.
3.8.7. T gi thit ta suy ra chui

n=1
a
n
hi t v khng mt tng qut ta
c th gi thit rng |a
n
| < 1. V
lim
n
a
n
ln(a
n
+ 1)
a
2
n
=
1
2
(1)
v chui

n=1
a
n
hi t, nn s hi t ca chui

n=1
a
2
n
t-ng -ng vi s hi t
ca chui

n=1
ln(1 +a
n
), tc l t-ng -ng vi s hi t ca tch

n=1
(1 +a
n
).
Ch rng nu

n=1
a
2
n
hi t th theo (1), vi n ln ta c
a
n
ln(1 +a
n
) >
1
4
a
2
n
.
Do chui

n=1
ln(1 +a
n
) phn k ti , tc l

n=1
(1 +a
n
) phn k ti 0.

n

C
h
i
3.8. Tch v hn 343
3.8.8. Kt qu -c suy ra t 3.8.7.
3.8.9. S dng 3.8.7 va 3.8.8.
3.8.10. S dng ng thc
lim
n
| ln(1 +a
n
) a
n
+
1
2
a
2
n
|
|a
n
|
3
=
1
3
v tin hnh nh- cch gii ca bi 3.8.7.
3.8.11. Khng. Tht vy, s dng kt qu bi tr-c ta thy rng tch -c
nu trong phn gi s hi t khi >
1
3
. Mt khc cc chui

1
2

+
_
1
2

+
1
2
2
_

1
3

+
_
1
3

+
1
3
2
_

1
4

+...
v
_

1
2

_
2
+
_
1
2

+
1
2
2
_
2
+
_

1
3

_
2
+
_
1
3

+
1
3
2
_
2
+
_

1
4

_
2
+...
cng hi t khi
1
2
.
3.8.12. Ch rng nu lim
n
a
n
= 0 th
lim
n
| ln(1 +a
n
) a
n
+
1
2
a
2
n

1
3
a
3
n
+... +
(1)
k
k
a
k
n
|
|a
n
|
k+1
=
1
k + 1
.
3.8.13. S dng cng thc Taylor
ln(1 +a
n
) = a
n

1
2(1 +
n
)
2
a
2
n
= a
n

n
a
2
n
,
trong
2
9
<
n
< 2 nu |a
n
| <
1
2
. Do vi n
1
, n
2
ln v n
1
< n
2
th
n
2

n=n
1
ln(1 +a
n
) =
n
2

n=n
1

n
2

n=n
1
a
2
n
, trong
_
2
9
, 2
_
.
T suy ra chui

n=1
a
n
hi t theo tiu chun Cauchy.

n

C
h
i
344 Ch-ng 3. Chui s thc
3.8.14. Nu cc tch

n=1
(1+a
n
) v

n=1
(1a
n
) cng hi t th tch

n=1
(1a
2
n
)
cng hi t. T suy ra chui

n=1
a
2
n
hi t . iu phi chng minh -c suy
ra t bi tp trn.
3.8.15. C, tht vy v dy {a
n
} n iu gim v 1 nn ta vit -c a
n
=
1 +
n
trong {
n
} l mt dy n iu gim dn v 0. S hi t ca tch
ang xt t-ng -ng vi s hi t ca chui

n=1
(1)
n1
ln(1 +
n
).
R rng chui ang xt l hi t theo tiu chun Leibniz v chui an du.
3.8.16.
(a) V lim
n
(a
n
+ b
n
) = 1 + 1 = 2 nn tch ang xt khng tho mn iu
kin cn ca tnh hi t.
(b) Tnh hi t ca tch

n=1
a
2
n
-c suy ra t s hi t ca chui

n=1
lna
2
n
.
(c), (d) S hi t ca cc tch -c suy ra t s hi t ca cc chui

n=1
ln(a
n
b
n
) =

n=1
lna
n
+

n=1
lnb
n
v

n=1
ln(
a
n
b
n
) =

n=1
lna
n

n=1
lnb
n
.
3.8.17. Gi s ta c chui

n=1
x
2
n
hi t , khi lim
n
x
n
= 0, tnh hi t ca
hai tch v hn -c suy ra t 3.8.4 v t ng thc
lim
n
1 cos x
n
x
2
n
=
1
2
, v lim
n
1
sinxn
xn
x
2
n
=
1
6
.
By gi gi thit rng mt trong hai tch hi t, th th lim
n
x
n
= 0 v tnh hi
t ca

n=1
x
2
n
cng -c suy t cc ng thc k trn.

n

C
h
i
3.8. Tch v hn 345
3.8.18. Ch rng
a
1
n

k=2
_
1 +
a
k
S
k1
_
= a
1
n

k=2
S
k
S
k1
= S
n
.
3.8.19. Xem bi tp 3.1.9.
3.8.20. Xem bi tp 3.1.9.
3.8.21. S dng bi tp trn vi a
n
= x
n
.
3.8.22. Gi s rng tch

n=1
a
n
hi t, tc l lim
n
P
n
= P = 0, trong
P
n
=
n

k=1
a
k
. T iu ny ta suy ra tn ti mt s > 0 sao cho |P
n
| > vi
n N. Dy hi t {P
n
} l mt dy Cauchy, do vi mi > 0 tn ti s t
nhin n
0
sao cho |P
n+k
P
n1
| < vi n > n
0
v k N. Th th

P
n+k
P
n1
1

<

|P
n1
|
, vi n n
0
.
Gi thit rng vi mi > 0 tn ti s t nhin n
0
sao cho
|a
n
a
n+1
... a
n+k
1| < ()
vi n n
0
v k N. Chon =
1
2
ta c
1
2
<
P
n1
P
n
0
<
3
2
vi n > n
0
. ()
Trong () thay bng
2
3|Pn
0
ta tm -c mt s t nhin n
1
sao cho

P
n+k
P
n1
1

<
2
3|P
n
0
|
vi n n
1
, k N.
Do vi n > max{n
0
, n
1
} ta -c
|P
n+k
P
n1
| <
2
3

P
n1
P
n
0

< .
iu ny c ngha l dy {P
n
} l dy Cauchy, hn na t () ta suy ra gii
hn ca n khc 0.

n

C
h
i
346 Ch-ng 3. Chui s thc
3.8.23. Ta c
2n

k=1
(1 +x
k
) =
2n

k=1
1 x
2k
1 x
k
=
2n

k=1
(1 x
2k
)
2n

k=1
(1 x
k
)
=
2n

k=1
(1 x
2k
)
n

k=1
(1 x
2k
)
n

k=1
(1 x
2k1
)
=
2n

k=n+1
(1 x
2k
)
n

k=1
(1 x
2k1
)
.
S dng tiu chun Cauchy (xem bi 3.8.22) ta suy ra iu phi chng minh.
3.8.24. y l mt h qu ca 3.8.3.
3.8.25. Ch rng vi a
1
, a
2
, ..., a
n
R th
|(1 +a
1
)(1 +a
2
)...(1 +a
n
) 1| (1 +|a
1
|)(1 +|a
2
|)...(1 +|a
n
|) 1
v s dng tiu chun Cauchy (xem 3.8.22).
3.8.26. t P
n
= (1 +a
1
)(1 +a
2
)...(1 +a
n
), n N. Th th P
n
P
n1
=
P
n1
a
n
v
P
n
= P
1
+ (P
2
P
1
) +... + (P
n
P
n1
)
= P
1
+P
1
a
2
+P
2
a
3
... +P
n1
a
n
.
Cho nn
P
n
= (1 +a
1
) +a
2
(1 +a
1
) +a
3
(1 +a
1
)(1 +a
2
)
+... +a
n
(1 +a
1
)(1 +a
2
)...(1 +a
n1
),
hay l t-ng ng
P
n
= (1 +a
1
) + (a
2
+a
1
a
2
) + (a
3
+a
1
a
3
+a
2
a
3
+a
1
a
2
a
3
)
+... + (a
n
+a
1
a
n
+a
2
a
n
+...a
n1
a
n
+... +a
n2
a
n1
a
n
+ ... +a
1
a
2
...a
n1
a
n
.
Ch rng t s hi t ca tch

n=1
(1 +a
n
) ta ko theo s hi t tuyt i ca
chui 1 + a
1
+

n=2
a
n
(1 + a
1
)(1 + a
2
)...(1 + a
n1
). Chui ny to thnh t

n

C
h
i
3.8. Tch v hn 347
vic sp xp chui kp c cc s hng thuc mt ma trn v hn
_
_
_
_
a
1
a
2
a
3
a
4
...
a
1
a
2
a
1
a
3
a
2
a
3
a
1
a
4
...
a
1
a
2
a
3
a
1
a
2
a
4
a
1
a
3
a
4
a
2
a
3
a
4
...
. . . . . . . . . . . . . . . . . . . . . . . . . . . . . . . . . .
_
_
_
_
T 3.7.18 ta suy ra chui kp hi t tuyt i v t 3.7.22 ta -c chui lp
ang xt l hi t, v t chng minh -c ng thc trong bi.
3.8.27. T s hi t tuyt i ca chui

n=1
a
n
ta suy ra chui

n=1
a
n
x hi t
tuyt i vi mi x R. S dng kt qu bi trn ta suy ra iu phi chng
minh.
3.8.28. Hin nhin vi |q| < 1 v x R th tch

n=1
(1 + q
n
x) hi t tuyt
i. Trong bi trn chn a
n
= q
n
ta -c hm f(x) =

n=1
(1 + q
n
x) =
1 +A
1
x+A
2
x
2
+.... By gi ch rng f(x) = (1 +qx)f(qx) v ng nht
h s ta -c
A
1
=
q
1 q
v A
n
= A
n1
q
n
1 q
n
vi n = 2, 3....
Cui cng ta ch -c ra rng (s dng quy np)
A
n
=
q
n(n+1)
2
(1 q)(1 q
2
) ... (1 q
n
)
.
3.8.29. t f(x) =

n=1
(1 + q
2n1
x) v ch rng (1 +qx)f(q
2
x) = f(x)
ta bin lun nh- cu 3.8.28.
3.8.30. Ta c

n=1
(1 +a
n
x)
_
1 +
a
n
x
_
=
_
1 +

k=1
A
k
x
k
__
1 +

k=1
A
k
x
k
_
= 1 +

k=1
A
k
_
x
k
+
1
x
k
_
+

k=1
A
k
x
k

k=1
A
k
x
k
.

n

C
h
i
348 Ch-ng 3. Chui s thc
T s hi t tuyt i ca

k=1
A
k
x
k
v

k=1
A
k
x
k
ta suy ra s hi t ca tch
Cauchy ca chng (xem bi gii ca 3.6.1). Ch rng tch Cauchy ny to
thnh mt chui kp t-ng ng vi ma trn v hn
_
_
_
_
A
1
A
1
A
2
A
2
A
3
A
3
...
A
2
A
1
_
x +
1
x
_
A
3
A
2
_
x +
1
x
_
A
4
A
3
_
x +
1
x
_
...
A
3
A
1
_
x
2
+
1
x
2
_
A
4
A
2
_
x
2
+
1
x
2
_
A
5
A
3
_
x
2
+
1
x
2
_
...
. . . . . . . . . . . . . . . . . . . . . . . . . . . . . . . . . . . . . . . . . . . . . . . . . . . .
_
_
_
_
Do t 3.7.18 v 3.7.22 ta suy ra

k=1
A
k
x
k

k=1
A
k
x
k
= (A
1
A
1
+A
2
A
2
+A
3
A
3
+...) + (A
2
A
1
+A
3
A
2
+...)
_
x +
1
x
_
+ (A
3
A
1
+A
4
A
2
+...)
_
x
2
+
1
x
2
_
+...
3.8.31. [4] T 3.8.30 ta c

n=1
(1 +q
2n1
x)
_
1 +
q
2n1
x
_
= B
0
+

n=1
B
n
_
x
n
+
1
x
n
_
.
t
F(x) =

n=1
(1 +q
2n1
x)
_
1 +
q
2n1
x
_
v s dng ng thc qxF(q
2
x) = F(x) ta -c
B
1
= B
0
q, B
n
= B
n1
q
2n1
,
s dng quy np suy ra
B
n
= B
0
q
n
2
, n = 1, 2, ....
Khi
F(x) = B
0
_
1 +

n=1
q
n
2
_
x
n
+
1
x
n
_
_
.
xc nh B
0
ta s dng kt qu bi 3.8.29 v 3.8.30. t P
n
=
n

k=1
(1q
2k
)
v P =

n=1
(1 q
2n
), th th
B
0
q
n
2
= B
n
= A
n
+A
1
A
n+1
+... =
q
n
2
P
n
+
q
(n+1)
2
+1
P
n
P
n+1
+...,

n

C
h
i
3.8. Tch v hn 349
ta -c
P
n
B
0
1 <
q
2n
P
2
+
q
4n
P
4
+....
Cho n ta -c B
0
=
1
P
.
3.8.32. S dng kt qu bi 3.8.30 vi
(a) x = 1.
(b) x = 1.
(c) x = q.
3.8.33. Ch rng vi n > 1 ta c
a
n
=
1
2
_
n1

k=1
x k
x +k

n

k=1
x k
x +k
_
.
V th
S
n
=
n

k=1
a
k
=
1
1 +x
+
n

k=2
a
k
=
1
2

1
2
n

k=1
x k
x +k
.
Nu x l mt s nguyn d-ng th vi n ln ta c S
n
=
1
2
. Ta phi chng
minh rng vi x = 1, 2, ... th lim
n
S
n
=
1
2
bng cch nhn xt rng vi k
ln th

xk
x+k

= 1
2x
x+k
. Do s dng kt qu bi 3.8.4
lim
n
n

k=1

x k
x +k

= 0.
ta -c iu phi chng minh.
3.8.34. Gi thit rng

n=1
(1 +ca
n
) hi t vi c = c
0
v c = c
1
m c
0
= c
1
.
Khi cc tch

n=1
(1 +c
1
a
n
)
c
0
c
1
v

n=1
(1 +c
1
a
n
)
c
0
c
1
1 +c
0
a
n
cng hi t. Hn na
(1 +c
1
a
n
)
c
0
c
1
1 +c
0
a
n
= 1 +
c
0
(c
0
c
1
)
2
a
2
n
(1 +
n
),

n

C
h
i
350 Ch-ng 3. Chui s thc
trong
n
0 khi n . Do s dng bi 3.8.3 v 3.8.4 ta suy ra chui

n=1
a
2
n
hi t. Thm na khi s dng kt qu bi 3.8.13 ta -c chui

n=1
a
n
cng hi t. T suy ra vi c R bt k c hai chui

n=1
(ca
n
)
2
v chui

n=1
ca
n
hi t. Ta suy ra iu phi chng minh t bi 3.8.7.
3.8.35. R rng chui

n=1
a
n
n

k=0
(x
2
k
2
) hi t ti 0 vi x l s nguyn
d-ng. Gi s n hi t ti mt gi tr x
0
khng nguyn d-ng. Vi x R ta
xt dy {b
n
} m
b
n
=
n

k=0
(x
2
k
2
)
n

k=0
(x
2
0
k
2
)
.
Khi
b
n
=
n

k=0
x
2
k
2
x
2
0
k
2
=
n

k=0
_
1 +
x
2
x
2
0
x
2
0
k
2
_
.
T suy ra bt u t mt ch s no dy s n iu. Hn na v tch
n

k=0
x
2
k
2
x
2
0
k
2
hi t nn dy {b
n
} b chn, ta cng c

n=1
a
n
n

k=0
(x
2
k
2
) =

n=1
a
n
n

k=0
(x
2
0
k
2
)b
n
Theo tiu chun Abel chui ang xt hi t vi mi x R.
3.8.36.
(a) Ta c
_
1
1
p
x
n
_
1
= 1 +
n

k=1
1
p
kx
n
.
Nhn N ng thc u li vi nhau -c
N

n=1
_
1
1
p
x
n
_
1
= 1 +

k=1
1
k
x
=
p
N

k=1
1
k
x
+

k=p
N
+1
1
k
x
, (i)

n

C
h
i
3.8. Tch v hn 351
trong

l k hiu tng trn cc s t nhin c cc -c s ch l cc


s nguyn t p
1
, p
2
, ..., p
N
. Khi
0 <
N

n=1
_
1
1
p
x
n
_
1

p
N

k=1
1
k
x
=

k=p
N
+1
1
k
x
<

k=p
N
+1
1
k
x
.
V lim
N

k=p
N
+1
1
k
x
= 0 ta -c

n=1
_
1
1
p
x
n
_
1
=

n=1
1
n
x
.
(b) T (i) trong phn (a) ta suy ra
N

n=1
_
1
1
p
x
n
_
1
>
p
N

k=1
1
k
.
T s phn k ca chui

n=1
1
n
ta suy ra tch

n=1
_
1
1
pn
_
phn k v 0, tc l t-ng -ng vi s phn k ca tch

n=1
1
pn
(xem 3.8.4).
3.8.37. [18]
(a) Dng cng thc De Moivre cos mt + i sinmt = (cos t + i sint)
m
cho
m = 2n + 1 ta c
sin(2n + 1)t = (2n + 1) cos
2n
t sint
_
2n + 1
3
_
cos
2n2
t sin
3
t
+... + (1)
n
sin
2n+1
t.
Hay l ta vit -c thnh
sin(2n + 1)t = sin tW(sin
2
t), (1)
trong W(u) l a thc bc n. V hm v tri ca ng thc bng
0 ti cc im t
k
=
k
2n+1
, k = 1, 2, ..., n u thuc vo khong
_
0,

2
_
ta

n

C
h
i
352 Ch-ng 3. Chui s thc
suy ra a thc W(u) bng 0 ti cc im u
k
= sin
2
t
k
, k = 1, 2, ..., n,
v ta c
W(u) = A
n

k=1
_
1
u
sin
2
t
k
_
.
Do vy s dng (1) ta -c
sin(2n + 1)t = Asint
n

k=1
_
1
sin
2
t
sin
2 k
2n+1
_
. (2)
Ta cn phi i tm A. Ta c A = lim
t0
sin(2n+1)t
sint
= 2n + 1, thay gi tr A
vo (2) v chn t =
x
2n+1
ta -c
sin x = (2n + 1) sin
x
2n + 1
n

k=1
_
1
sin
2 x
2n+1
sin
2 k
2n+1
_
. (3)
i vi x R v m N no cho tr-c sao cho |x| < (m+1) ta ly
n ln hn m, theo (3) ta -c
sinx = P
m,n
Q
m,n
, (4)
trong
P
m,n
= (2n + 1) sin
x
2n + 1
m

k=1
_
1
sin
2 x
2n+1
sin
2 k
2n+1
_
,
Q
m,n
=
n

k=m+1
_
1
sin
2 x
2n+1
sin
2 k
2n+1
_
.
Cho n -c
lim
n
P
m,n
= x
m

k=1
_
1
x
2
k
2

2
_
. (5)
T (4) vi x = k ta -c lim
n
Q
m,n
= Q
m
. nh gi Q
m
ta ch
rng t gi thit trn,
0 <
|x|
2n + 1
<
k
2n + 1

n
2n + 1
<

2
, vi k = m+ 1, ..., n.

n

C
h
i
3.8. Tch v hn 353
S dng cc bt ng thc
2

u < sinu < u, 0 < u <



2
, ta thy rng
n

k=m+1
_
1
x
2
4k
2
_
< Q
m,n
< 1. V tch

n=1
_
1
x
2
4n
2
_
hi t ta c
n

k=m+1
_
1
x
2
4k
2
_
Q
m,n
1.
T suy ra
lim
m
Q
m
= 1. (6)
Cui cng ng thc cn tm -c suy ra t (4), (5) v (6).
(b) S dng (a) v ch rng sin 2x = 2 sin xcos x.
3.8.38. Thay x =

2
trong biu thc nu trong 3.8.37(a).
3.8.39.
(a) S hi t ca tch -c xt s t-ng -ng vi s hi t ca chui

n=1
_
ln
_
1 +
x
n
_

x
n
_
,
cn s hi t tuyt i ca chui -c suy t ng thc
lim
n

ln
_
1 +
x
n
_

x
n

x
2
n
2
=
1
2
.
(b) Ta c
_
1 +
1
n
_
x
1 +
x
n
= 1 +
x(x 1)
2n
2
+o
_
1
n
2
_
.
T 3.8.3 ta suy ra tch hi t tuyt i.
3.8.40. R rng tch

n=1
(1 + a
n
), a
n
> 1 hi t khi v ch khi chui

n=1
ln(1 +a
n
) hi t. Hn na nu P l gi tr ca tch v S l tng ca chui
th P = e
S
.

n

C
h
i
354 Ti li u tham kho
Gi s rng tch hi t tuyt i, t ng thc
lim
n
| ln(1 +a
n
)|
|a
n
|
= 1, (v lim
n
a
n
= 0). (1)
chui

n=1
ln(1 + a
n
) hi t tuyt i. Hin nhin l ( t 3.7.8) ts c mi thay
i ca chui u hi t v mt tng. Cui cng s dng ch t u chng
minh ta suy ra mi thay i cc phn t ca tch u khng lm thay i gi
tr ca tch.
By gi gi thit rng gi tr ca tch

n=1
(1 +a
n
) khng ph thuc vo trt
t ca cc t nhn t ca tch, iu ny c ngha l tng ca chui

n=1
ln(1+a
n
)
cng c lp vi trt t cc s hng ca mnh. S dng nh l Riemann ta suy
ra chui hi t tuyt i, tc l t (1) ta suy ra chui

n=1
|1 + a
n
| hi t, ta c
iu phi chng minh.
3.8.41. [20] t R
n
=
3
2

5
4

7
6
...
2n+1
2n
=
(2n+1)!!
(2n)!!
. Ta c
_
1 +
1
2
__
1 +
1
4
_
...
_
1 +
1
2
_
= R

,
_
1
1
3
__
1
1
5
_
...
_
1
1
2 + 1
_
=
1
R

.
Th th tch ring th + ca tch ang xt s bng
Rn
R
n
. S dng cng thc
Wallis (xem 3.8.38) ta -c
lim
n
(2n + 1)!!
(2n)!!

n
=
2

,
v t ta -c
lim
n
R
n
R
n
=
_

.
3.8.42. Nu tch

n=1
(1 + a
n
) hi t nh-ng khng hi t tuyt i th chui

n=1
ln(1 + a
n
) hi t c iu kin (xem 3.8.40). T nh l Riemann ta suy ra
cc s hng ca chui c th sp li to thnh mt chui mi hi t c tng
l mt s bt k cho tr-c S hoc phn k (ti ). iu phi chng minh
-c suy t biu thc P = e
S
(xem 3.8.40).

n

C
h
i
Ti liu tham kho
[1] J. Bana s, S. We drychowicz, Zbir zada n z analizy matematycznej, Win-
dawnictwa Naukowo - Techniczne, Warszawa,1994.
[2] W. I. Bernuk, I. K. Zuk, O. W. Melnikov, Sbornik olimpiadnych zadac
po matematike, Narodnaja Asveta, Minsk , 1980.
[3] P. Biler, A. Witkowski, Problems in Mathematical Analysis, Marcel
Dekker, Inc, New York and Base, 1990.
[4] T. J. Bromwich, An Introduction to the Theory of Infinte Series, Macmil-
lan and Co., ltd., London ,1949.
[5] R. B. Burckel, An Introduction to Classical Complex Analysis, Academic
Press, New York and San Francisco, 1979.
[6] B. Demidovic, Sbornik zadac i upraznenij po matemaiceskomu analizu,
Nauka, Moskva, 1969.
[7] A. J. Dorogovcev, Matematiceskij analiz. Spravocnoe posobe, Vyscaja
Skola, Kiev, 1985.
[8] A. J. Dorogovcev, Matematiceskij analiz. Sbornik zadac, Vyscaja Skola,
Kiev, 1987.
[9] G. M. Fichtenholz, Differential - und Integralrechnung, I,II,III, V.E.B.
Deutscher Verlag Wiss. , Berlin, 1966-1968.
[10] G. H. Hardy, A Course of Pure Mathematics , Cambrige University Press,
Cambirige, 1946.
[11] G. H. Hardy, J. E. Littlewood, G.Polya, Inequalities , Cambrige University
Press, Cambirige, 1967.
355

n

C
h
i
356 Ti li u tham kho
[12] G. Klambauer, Mathematical Analysis, Marcel Dekker, Inc., New York,
1975.
[13] G. Klambauer, Problems and Propositions in Analysis, Marcel Dekker,
Inc., New York and Basel, 1979.
[14] K. Knopp, Theorie und Anwendung der Unendhichen Reihen, Springer-
Verlag, Berlin and Heidelberg, 1947.
[15] L. D. Kudriavtsev, A. D. Kutasov, V. I. Chejlov, M. I. Shabunin, Problems
de An a Matem atico. L

imite, Continuidad, Derivabilidad, Mir, Moskva,


1989.
[16] K. Kuratowski, Introduction to Calculus, Pergamon Press, Oxford - Ei-
denburg - New York, Polish Scientific Publishers, Warsaw, 1969.
[17] D. S. Mitrinovi c, Elemetary Inequalities, P. Noordhoff Ltd., Gronigen,
1964.
[18] D. S. Mitrinovi c, D. D. Adamovi c, Nizovi i Redovi. Definicije, stavovi.
zadaci, problemi (Serbo - Croatian), Naucna Knijga, Belgrade, 1971.
[19] A. Ostrowski, Aufgabensammlung zur Infinitesimalrechnung, Band I:
Funktionen einer Variablen, Birkhauser Verlag, Basel und Stuttgart,
1964.
[20] G. P olia, G. Szego, Problems and theorems in analysis I, Spriger - Verlag,
Berlin Heidelberg New York, 1978.
[21] Ya. I. Rivikind, Zadaci pr matematiceskomu analizu, Vysejsaja Skola,
Minsk, 1973.
[22] W.I. Rozhkov, V.D. Kurdevanidze, N. G. Pafionov, Sbornik zadac matem-
aticeskich olimpiad, Izdat. Univ. Druzhby Narodov, Maskva, 1987.
[23] W. Rudin, Principle of Mathematical Analysis, McGraw - Hill Book
Company, New York, 1964.
[24] W. A. Sadownicij, A. S. Pdkolzin, Zadaci studenceskich olimpiad po
matematike, Nauka, Moskva, 1978.
[25] W. Sierpi nski, Arytmetyka teoratyczna, PWN, Warszawa, 1959.
[26] W. Sierpi nski, Dzialania niesko nczone, Czytelnik, Warszawa, 1948.

n

C
h
i
Ti li u tham kho 357
[27] H. Silverman, Complex variables, Houghton Mifflin Company, Boston,
1975.
[28] G. A. Tonojan, W. N. Sergeev, Studencceskije oimpiady, Erevanskogo
Universiteta, Erivan, 1985.

Potrebbero piacerti anche